1. Trang chủ
  2. » Giáo Dục - Đào Tạo

Bất đẳng thức hiện đại

357 905 0

Đang tải... (xem toàn văn)

Tài liệu hạn chế xem trước, để xem đầy đủ mời bạn chọn Tải xuống

THÔNG TIN TÀI LIỆU

http://boxtailieu.net http://boxtailieu.net Chuyên đề Bất đẳng thức hiện đại Võ Quốc Bá Cẩn-Phạm Thị Hằng http://boxtailieu.net ii http://boxtailieu.net Mục lục Lời nói đầu v 1 Tìm tòi một số kỹ thuật giải toán 1.1 Đại lượng (a b)(b c)(c a) . . . . . . . . . . 1.2 Những kiểu lời giải đặc biệt bằng AM-GM . . . . 1.3 Kỹ thuật pqr . . . . . . . . . . . . . . . . . . . . 1.3.1 Lời nói đầu . . . . . . . . . . . . . . . . . 1.3.2 Những đẳng thức cần nhớ . . . . . . . . . 1.3.3 Bất đẳng thức Schur . . . . . . . . . . . . 1.3.4 Đại lượng (a b)2 (b c)2 (c a)2 . . . . . 1.3.5 Làm mạnh hơn nữa . . . . . . . . . . . . 1.3.6 pqr hoán vị . . . . . . . . . . . . . . . . . 1.4 The CYH techniques . . . . . . . . . . . . . . . . 1.4.1 Lời nói đầu . . . . . . . . . . . . . . . . . 1.4.2 Bất đẳng thức Cauchy Schwarz và Holder. 1.4.3 Một số kỹ thuật cần chú ý . . . . . . . . . 1.5 The Hyberbolic functional technique . . . . . . . 1.5.1 Lời nói đầu . . . . . . . . . . . . . . . . . 1.5.2 Một số ví dụ mở đầu . . . . . . . . . . . . 1.5.3 Đặt vấn đề . . . . . . . . . . . . . . . . . 1.5.4 Giải quyết vấn đề . . . . . . . . . . . . . 1.5.5 Một số mở rộng . . . . . . . . . . . . . . . 1.6 Các dạng tổng bình phương . . . . . . . . . . . . 1.7 Hàm lồi, hàm bậc nhất . . . . . . . . . . . . . . . 1.8 Quy nạp . . . . . . . . . . . . . . . . . . . . . . . 2 Sáng tạo bất đẳng thức . . . . . . . . . . . . . . . . . . . . . . . . . . . . . . . . . . . . . . . . . . . . . . . . . . . . . . . . . . . . . . . . . . . . . . . . . . . . . . . . . . . . . . . . . . . . . . . . . . . . . . . . . . . . . . . . . . . . . . . . . . . . . . . . . . . . . . . . . . . . . . . . . . . . . . . . . . . . . . . . . . . . . . . . . . . . . . . . . . . . . . . . . . . . . . . . . . . . . . . . . . . . . . . . . . . . . . . . . . . . . . . . . . . . . . . . . . . . . . . . . . . . . . . . . . . . . . . . . . . . . . . . 1 1 12 22 22 23 23 28 42 55 70 70 70 72 143 143 143 146 152 164 179 186 196 201 A Một số bất đẳng thức thông dụng 343 A.1 Bất đẳng thức trung bình cộng-trung bình nhân-trung bình điều hòa (AM-GM-HM) . . . . . . . . . . . . . . . . . . . . . . . . . . . . . . . 343 iii http://boxtailieu.net iv MỤC LỤC A.2 A.3 A.4 A.5 A.6 Bất Bất Bất Bất Bất đẳng đẳng đẳng đẳng đẳng thức thức thức thức thức AM-GM suy rộng . . . . . . trung bình lũy thừa . . . . . trung bình lũy thừa suy rộng Bernoulli . . . . . . . . . . . Cauchy Schwarz . . . . . . . A.7 Bất đẳng thức Holder . . . A.8 Bất đẳng thức Minkowski . A.9 Bất đẳng thức Chebyshev . A.10 Khai triển Abel . . . . . . . A.11 Bất đẳng thức Maclaurin . A.12 Bất đẳng thức Schur . . . . A.13 Hàm lồi, hàm lõm . . . . . A.14 Bất đẳng thức Jensen . . . A.15 Tổng, tích hoán vị-đối xứng . . . . . . . . . . . . . . . . . . . . . . . . . . . . . . . . . . . . . . . . . . . . . . . . . . . . . . . . . . . . . . . . . . . . . . . . . . . . . . . . . . . . . . . . . . . . . . . . . . . . . . . . . . . . . . . . . . . . . . . . . . . . . . . . . . . . . . . . . . . . . . . . . . . . . . . . . . . . 343 343 344 344 344 . . . . . . . . . . . . . . . . . . . . . . . . . . . . . . . . . . . . . . . . . . . . . . . . . . . . . . . . . . . . . . . . . . . . . . . . . . . . . . . . . . . . . . . . . . . . . . . . . . . . . . . . . . . . . . . . . . . . . . . . . . . . . . . . . . . . . . . 344 345 345 345 345 346 346 346 346 http://boxtailieu.net Lời nói đầu Bất đẳng thức là một trong những vấn đề hay và khó nhất của chương trình toán phổ thông bởi nó có mặt trên hầu khắp các lĩnh vực của toán học và nó đòi hòi chúng ta phải có một vốn kiến thức tương đối vững vàng trên tất cả các lĩnh vực. Mỗi người chúng ta, đặc biệt là các bạn yêu toán, dù ít dù nhiều thì cũng đã từng đau đầu trước một bất đẳng thức khó và cũng đã từng có được một cảm giác tự hào phấn khích mà mình chứng minh được bất đẳng thức đó. Nhằm “kích hoạt” niềm say mê bất đẳng thức trong các bạn, chúng tôi thực hiện quyển sách “Chuyên đề bất đẳng thức hiện đại”. Sách gồm 2 chương. Chương I chúng tôi xin được giới thiệu đến các bạn những kỹ thuật (xin chỉ gọi là kỹ thuật) mà chúng tôi tìm tòi tích lũy được trong suốt thời gian học tập của mình. Do tất cả các kỹ thuật mà chúng tôi đề cập ở đây đều có mỗi liên hệ khăng khít với nhau (cái này bổ trợ cái kia và ngược lại) nên chúng tôi xin được phép trình bày theo kiểu từng bài chuyên đề nhỏ, mỗi chuyên đề là một kỹ thuật. Tuy nhiên, lĩnh vực bất đẳng thức hiện nay rất phát triển (phát triển nhất của toán học sơ cấp hiện nay), cho nên chúng tôi không thể đề cập hết các kỹ thuật (phương pháp) được, các kỹ thuật (phương pháp) đã từng xuất hiện ở các sách, chúng tôi sẽ không nhắc lại ở đây, các bạn có thể tìm đọc chúng dựa vào các tài liệu mà chúng tôi đặt ở phần tài liệu tham khảo. Về các kỹ thuật mà chúng tôi sẽ giới thiệu trong sách, hầu hết chúng là những kỹ thuật mạnh và được dùng để giải những bài toán khó (đến rất khó) nên đôi khi (việc giải các bài toán khó) thì có thể gặp phải những tính toán, biến đổi phức tạp, đây là điều không thể tránh khỏi. Nhưng các bạn hãy yên tâm, vì các bài toán xuất hiện trong các kỳ thi học giỏi (quốc gia, olypimpic 30/4, thậm chí thi toán quốc tế) thường chỉ là những bài rất đơn giản, bình thường nên việc sử dụng các kỹ thuật này rất nhẹ nhàng và đơn giản. Chẳng hạn như bài toán thi IMO 2006 sau Bài toán 0.1 Tìm hằng số nhỏ nhất sao cho bất đẳng thức sau đúng với các số thực a; b; c ab(a2 b2 ) + bc(b2 c2 ) + ca(c2 a2 ) k(a2 + b2 + c2 )2 : Lời giải của đáp án là một lời giải rất dài và phức tạp (sử dụng bất đẳng thức AMGM), đòi hỏi người làm phải “rất khéo léo”, nhưng với lời giải bằng kỹ thuật “đánh v http://boxtailieu.net vi LỜI NÓI ĐẦU giá các bất đẳng thức hoán vị”, chúng ta chỉ nhận được một lời giải ngắn gọn 1/3 so với lời giải gốc ban đầu. Chương II của sách là tuyển tập những bài toán mà chúng tôi (theo quan niệm của bản thân) là hay và rất khó. Chúng tôi chủ yếu tuyển chọn những bài bất đẳng thức chứa căn hoặc những bài “không mẫu mực” vì chúng ta không thể dùng những biến đổi thông thường để giải chúng và như thế thì mới thúc đẩy chúng ta sáng tạo được. Trong chương này, phần lớn chúng tôi đều giải bằng cách sử dụng bất đẳng thức Cauchy Schwarz-Holder (CYH techniques) và bất đẳng thức Schur (bậc 3, bậc 4). Thực tế là đối với một số bài toán thì không chỉ có một lời giải duy nhất mà còn có nhiều lời giải khác nữa, nhưng ở đây chúng tôi chọn lời giải bằng các bất đẳng thức trên, vì chúng tôi muốn các bạn “hòa nhập” vào quan điểm của chúng tôi là “Cái đơn giản nhất là cái mạnh nhất!” Trong chương này, có một số bài toán khó, lời giải mà chúng tôi tìm được rất phức tạp, chúng tôi rất mong các bạn sẽ suy nghĩ về chúng và tìm được một lời giải đơn giản hơn. Chúng tôi thực hiện quyển sách này với mong muốn cung cấp thêm cho các bạn thêm một nguồn bài tập (khó) về bất đẳng thức để có thể luyện tập thêm kĩ năng giải toán của mình. Mặc dù đã rất cố gắng nhưng không có điều gì là tuyệt đối cả, nên khó tránh khỏi những thiếu sót, sai lầm. Mong các bạn thông cảm và góp ý cho chúng tôi để có thể quyển sách có thể được chỉnh sửa và hoàn thiện hơn. Xin chân thành cảm ơn. Xin gửi tặng quyển sách này đến người con gái tôi yêu quý nhất, bạn Phạm Thị Hằng, học sinh chuyên toán K34, trường THPT Chuyên Phan Bội Châu, thành phố Vinh, tỉnh Nghệ An. Võ Quốc Bá Cẩn SV lớp YY0647A1, trường ĐHYD Cần Thơ Số nhà C65 khu dân cư Phú An, phường Phú Thứ, quận Cái Răng, tp. Cần Thơ E-mail: can_hang2007@yahoo.com http://boxtailieu.net Chương 1 Tìm tòi một số kỹ thuật giải toán 1.1 Đại lượng (a b)(b c)(c a) Với những bất đẳng thức hoán vị vòng quanh, việc xử lý chúng khó hơn các bất đẳng thức đối xứng rất nhiều. Tuy nhiên, một điểm đáng chú ý ở các dạng bất đẳng thức này, chúng ta có thể biến đổi chúng thành dạng "bán đối xứng" như sau Đặt f (a; b; c) chính là biểu thức hoán vị vòng quanh ở đề bài, ta có thể viết lại f (a; b; c) như sau 1 1 f (a; b; c) = [f (a; b; c) + f (c; b; a)] + [f (a; b; c) f (c; b; a)] 2 2 Khi đó, có một điểm đáng chú ý là f (a; b; c) + f (c; b; a) là một biểu thức đối xứng theo a; b; c và f (a; b; c) f (c; b; a), ta có thể tách ra một đại lượng khá đặc biệt là (a b)(b c)(c a): Từ đó, việc đánh giá bài toán trở nên đơn giản hơn nhiều. Sau đây là một vài ví dụ Ví dụ 1.1 Cho các số dương a; b; c: Chứng minh rằng ab bc ca + 2 + 2 3a2 + b2 3b + c2 3c + a2 3 : 4 (Dương Đức Lâm) Lời giải. Bất đẳng thức tương đương với X (a cyc b)(3a b) 3a2 + b2 0 1 http://boxtailieu.net 2 CHƯƠNG 1. TÌM TÒI MỘT SỐ KỸ THUẬT GIẢI TOÁN , X (a b) cyc , X (a cyc X a2 b2 a2 + b2 cyc a+b a2 + b2 2(3a b) 3a2 + b2 Y a2 2 b) (3a2 2ab + 3b2 ) (a2 + b2 )(3a2 + b2 ) b2 a2 + b2 cyc Sử dụng bất đẳng thức AM-GM, ta có X (a cyc 2 b) (3a2 2ab + 3b2 ) (a2 + b2 )(3a2 + b2 ) Nên ta chỉ cần chứng minh v uY u (a 3 3t cyc v uY u (a 3 3t cyc 2 b) (3a2 2ab + 3b2 ) (a2 + b2 )(3a2 + b2 ) 2 b) (3a2 2ab + 3b2 ) (a2 + b2 )(3a2 + b2 ) Y a2 b2 a2 + b2 cyc 2 b) (3a2 2ab + 3b2 ) Y (a2 b2 )3 (a2 + b2 )(3a2 + b2 ) (a2 + b2 )3 cyc cyc Y Y , 27 (3a2 2ab + 3b2 )(a2 + b2 )2 (a b)(a + b)3 (3a2 + b2 ) Y (a , 27 cyc cyc Bất đẳng thức này được chứng minh nếu ta chứng minh được bất đẳng thức sau với mọi x; y > 0 3(3x2 2xy + 3y 2 )(x2 + y 2 )2 jx yj (x + y)3 (3x2 + y 2 ) Theo bất đẳng thức Cauchy Schwarz, ta có x2 + y 2 1 (x + y)2 2 Nên ta chỉ cần chứng minh 3(3x2 2xy + 3y 2 )(x2 + y 2 ) 2 x2 y 2 (3x2 + y 2 ) Bất đẳng thức này hiển nhiên đúng do x2 + y 2 x2 y2 và 3(3x2 2xy + 3y 2 ) 2(3x2 + y 2 ) = 3x2 6xy + 7y 2 = 3(x Bất đẳng thức được chứng minh xong. Đẳng thức xảy ra khi và chỉ khi a = b = c: http://boxtailieu.net y)2 + 4y 2 0: 1.1. ĐẠI LƯỢNG (A B)(B C)(C A) 3 Ví dụ 1.2 Cho a; b; c là độ dài ba cạnh của một tam giác nhọn. Chứng minh rằng a2 b3 c3 a3 + 2 + 2 2 2 +b b +c c + a2 a2 b2 c2 + + : a+b b+c c+a (Võ Quốc Bá Cẩn) Lời giải. Trước hết, ta hãy chú ý rằng X b3 cyc a3 a2 + b2 X (b = a)(a2 + ab + b2 ) X (a = a2 + b2 cyc cyc P X ab(b b) + cyc a)(a2 + c2 )(b2 + c2 ) ab(b a2 a) + b2 ) cyc = (a2 + b2 )(b2 + c2 )(c2 + a2 ) ! ! P 2 2 P P a b ab(b a) + abc c3 (a cyc = cyc (a2 + b2 )(b2 + c2 )(c2 + a2 ) (a b)(b c)(c a) P a2 b2 + abc cyc = X a2 X b2 = (a a+b cyc P ! a cyc (a2 + b2 )(b2 + c2 )(c2 + a2 ) cyc b) cyc b) = 0 Từ đó, ta có thể viết lại bất đẳng thức như sau X a3 + b3 cyc , X cyc a2 + b2 X a2 + b2 cyc ab(a b)2 (a + b)(a2 + b2 ) X b3 a+b (a cyc b)(b a2 c)(c a3 X a2 b2 + + b2 a+b cyc a) P 2 2 a b + abc cyc (a2 + b2 )(b2 + c2 )(c2 + a2 ) P cyc ! a Sử dụng bất đẳng thức AM-GM, ta có X cyc ab(a b)2 (a + b)(a2 + b2 ) s 33 a2 b2 c2 (a b)2 (b c)2 (c a)2 (a + b)(b + c)(c + a)(a2 + b2 )(b2 + c2 )(c2 + a2 ) http://boxtailieu.net 4 CHƯƠNG 1. TÌM TÒI MỘT SỐ KỸ THUẬT GIẢI TOÁN Ta cần chứng minh s 33 a2 b2 c2 (a b)2 (b c)2 (c a)2 (a + b)(b + c)(c + a)(a2 + b2 )(b2 + c2 )(c2 + a2 ) (a b)(b c)(c P a) a2 b2 + abc cyc ! a cyc (a2 + b2 )(b2 + c2 )(c2 + a2 ) , P 27a2 b2 c2 (a b)2 (b c)2 (c a)2 (a + b)(b + c)(c + a)(a2 + b2 )(b2 + c2 )(c2 + a2 ) (a 3 3 b) (b 3 c) (c a) P 2 2 a b + abc cyc P a cyc (a2 + b2 )3 (b2 + c2 )3 (c2 + a2 )3 !3 ,27a2 b2 c2 (a2 + b2 )2 (b2 + c2 )2 (c2 + a2 )2 2 (a 2 b )(b 2 2 2 2 c )(c a ) X X 2 2 a b + abc cyc cyc !3 a Do a; b; c là độ dài 3 cạnh của một tam giác nhọn nên ta dễ dàng chứng minh được a2 b2 c2 (a2 b2 )(b2 c2 )(c2 Ngoài ra, ta cũng có 2 2 2 2 2 X 2 (a + b )(b + c )(c + a ) = 2 a cyc 8 9 X cyc ! 2 a ! a2 ) X 2 2 a b cyc X cyc ! 2 2 a b a2 b2 c2 ! v !3 u X 8u t3 2 2 a b 9 cyc v P 13 u 0P 2 2 a b + abc a u u 8 t @ cyc cyc A 3 9 2 2 2 2 2 2 2 2 2 2 ) (a + b ) (b + c ) (c + a ) 8 27 X cyc 2 2 a b + abc X cyc !3 a Nhân tương ứng vế với vế các bất đẳng thức này, ta thu được bất đẳng thức ở trên. Đẳng thức xảy ra khi và chỉ khi a = b = c hoặc a = b; c = 0 và các hoán vị. http://boxtailieu.net 1.1. ĐẠI LƯỢNG (A B)(B C)(C A) 5 Ví dụ 1.3 Cho các số không âm a; b; c; không có 2 số nào cùng bằng 0: Chứng minh rằng p 3(a2 + b2 + c2 ) b3 c3 a3 + 2 + 2 : 2 2 2 2 a +b b +c c +a 2 (Võ Quốc Bá Cẩn) Lời giải. Viết lại bất đẳng thức như sau s X X a3 + b3 a+b 3 a2 2 + b2 a 2 cyc cyc , X (a cyc (a b)(b P cyc c)(c cyc X b3 cyc (a b)2 r P P 3 a2 + a cyc P a) cyc a2 b2 + abc P a nên ta chỉ cần chứng minh được b)2 (a + b) 2(a2 + b2 ) X (a cyc , X (a b)2 P + 2 a (a b)(b c)(c a) P a2 b2 + abc cyc 1 a+b+c a+b a2 + b2 b)2 cyc b)(b c)(c a) P 2 2 a b + abc cyc b)2 P ! a cyc (a2 + b2 )(b2 + c2 )(c2 + a2 ) (a a cyc (a2 + b2 )(b2 + c2 )(c2 + a2 ) cyc X ! P cyc 2(a , ! a cyc (a2 + b2 )(b2 + c2 )(c2 + a2 ) a3 + b2 a2 cyc X (a cyc cyc a+ cyc + r P Do 3 a2 X b)2 (a + b) 2(a2 + b2 ) X 2ab + ac + bc a2 + b2 2(a b)(b c)(c a) P cyc ! a P a2 b2 + abc cyc (a2 + b2 )(b2 + c2 )(c2 + a2 ) http://boxtailieu.net P cyc ! a 6 CHƯƠNG 1. TÌM TÒI MỘT SỐ KỸ THUẬT GIẢI TOÁN Sử dụng bất đẳng thức AM-GM, ta có X b)2 2ab + ac + bc a2 + b2 (a b)2 (b (a cyc s 33 c)2 (c a)2 (2ab + ac + bc)(2bc + ab + ac)(2ac + bc + ba) (a2 + b2 )(b2 + c2 )(c2 + a2 ) Ta phải chứng minh s 33 (a 2(a b)2 (b c)2 (c b)(b a)2 (2ab + ac + bc)(2bc + ab + ac)(2ac + bc + ba) (a2 + b2 )(b2 + c2 )(c2 + a2 ) ! ! P P 2 2 P a) a a b + abc a c)(c cyc cyc cyc (a2 + b2 )(b2 + c2 )(c2 + a2 ) " #" # Y Y 2 2 2 , 27 (2ab + ac + bc) (a + b ) cyc " cyc Y 8 (a cyc # b) X cyc !3 a X 2 cyc Y (a2 + b2 )2 cyc 64 81 X X !2 a2 cyc X cyc !3 a !3 ab cyc và a b + abc cyc Vì Y (2ab + ac + bc) 2 2 X a2 b2 cyc !2 nên ta chỉ cần chứng minh được 16 3 X cyc !3 ab X cyc " 2 a !2 Y (a cyc X cyc # b) 2 2 a b !2 X cyc !3 a X cyc http://boxtailieu.net 2 2 a b + abc X cyc !3 a 1.1. ĐẠI LƯỢNG (A Bây giờ, chú ý rằng !2 X 2 2 8 a b X cyc X = 8 B)(B a b cyc X = A !2 ab cyc 2 2 !2 2 2 a b C)(C X X 3 7 a b + abc 2 2 a b + 2abc X a 0 + 12abc X a2 b2 !2 " cyc cyc a !3 a cyc ! X cyc ! X 2 2 cyc cyc abc A) 3 X 2 2 a b + abc cyc X !3 a cyc trong đó A=5 X a2 b2 cyc !2 cyc ! X ! a cyc X + 3a2 b2 c2 cyc !2 a Ta còn phải chứng minh X 2 ! ab cyc X 2 a cyc # Y (a b) cyc X cyc !3 a Chuẩn hóa cho a + b + c = 1: Đặt q = ab + bc + ca; r = abc thì ta có p (a b)2 (b c)2 (c a)2 (a b)(b c)(c a) p = q 2 4q 3 + 2(9q 2)r 27r2 Ta phải chứng minh Nếu 9q 2q(1 2 thì p 2q)2 Do 2q)2 2(1 Nếu 9q p q2 2q)2 2q(1 2 thì p q2 4q 3 q2 4q 3 + 2(9q p 1 + 2(9q 4q = 2)r 4q 3 + 2(9q 27r2 2)r p 1 27r2 4q = r 1 2 2 4 (1 27 r 4 (1 27 2)r 27r2 h q 2(1 2q)2 1 + [2(1 4 3q)3 3q)3 http://boxtailieu.net p 1 4q 4q)2 + 1] 0 1 (27r 27 i 9q + 2)2 0 8 CHƯƠNG 1. TÌM TÒI MỘT SỐ KỸ THUẬT GIẢI TOÁN )2q(1 2q)2 p q2 2q(1 2q)2 2q(1 2q)2 4q 3 + 2(9q 2)r 27r2 r p 4 2 (1 3q)3 = 2q(1 2q)2 (1 3q) 3(1 3q) 27 9 2 8 46 (1 3q) = (9q 2)(81q 2 63q + 13) + > 0: 9 729 729 Bất đẳng thức được chứng minh xong. Đẳng thức xảy ra khi và chỉ khi a = b = c: Ví dụ 1.4 Cho các số dương a; b; c thỏa mãn giác. Xác định hằng số k nhỏ nhất sao cho a b c + + 2 2 b+c c+a a + b2 k 1 1 1 a; b; c là độ dài 3 cạnh của một tam c a b + + b c a a b c + 2+ 2 2 b c a : (Võ Quốc Bá Cẩn) Lời giải. Cho a = b = c, khi đó bất đẳng thức trở thành 3 a+1 9k a a 1 = 3(a + 1) 3 ,k 1 3(a + 1) 1 Cho a ! +1, ta được k 3 . Ta sẽ chứng minh đây chính là giá trị mà ta cần tìm, tức là a b c 1 a b c a b c + + + + + 2+ 2 b + c2 c + a2 a + b2 3 b c a b2 c a X b X a X a2 X a + + 3 , 3 2 b bc cyc a b + c2 cyc cyc cyc Do P cyc a b+c2 P cyc a c2 nên ta chỉ cần chứng minh được X a2 cyc , b3 + X a X b + bc cyc a2 cyc X a2 cyc b3 + X a bc cyc 2 3 X a c2 cyc X b a2 cyc 0 Đặt x = a1 ; y = 1b ; z = 1c , khi đó x; y; z là độ dài 3 cạnh của một tam giác. Bất đẳng thức trở thành X y 3 X yz X x2 + 2 0 2 x x y cyc cyc cyc http://boxtailieu.net 1.1. ĐẠI LƯỢNG (A , X B)(B 2y 2 x y3 +y x2 cyc , X cyc , X (x + A) X yz cyc 2 x x+2 x cyc 2(x X x2 X y2 cyc y)(y y cyc z)(z x) P ! 0 x cyc xyz 2(x 2 y) (2y + zx) 2x2 y cyc 9 X y z + x2 2xy y)2 (x C)(C y)(y z)(z x) P x cyc xyz Sử dụng bất đẳng thức AM-GM, ta có X (x 2 2 y) (2y + zx) 2x2 y cyc rQ (x 33 y)2 cyc Q (2x2 + yz) cyc 2xyz Ta cần chứng minh rQ (x 33 Q y)2 cyc (2x2 + yz) cyc 2(x y)(y x) P x cyc 2xyz Y , 27 (2x2 + yz) z)(z xyz 64(x y)(y z)(z x) cyc X cyc !3 x Để chứng minh bất đẳng thức này, trước hết ta sẽ chứng minh Y 9 (2x2 + yz) cyc X cyc !3 x X cyc xy ! Do tính thuần nhất, ta có thể chuẩn hóa cho x+y+z = 1. Đặt q = xy+yz+zx; r = xyz, khi đó ta có 31 q 14 và Y (2x2 + yz) = 27r2 + 2(1 9q)r + 4q 3 cyc Bất đẳng thức trở thành 243r2 + 18(1 9q)r + 36q 3 http://boxtailieu.net q 0 10 CHƯƠNG 1. TÌM TÒI MỘT SỐ KỸ THUẬT GIẢI TOÁN 5q 1 1 , 18 Đây là một hàm lõm theo r và với chú ý rằng r 243r2 + 18(1 9q)r + 36q 3 q 5q 1 18 243 = 1 (16q 4 ta có 2 1)(1 + (1 8q)(5q 3q)2 0 1) + 36q 3 q Tiếp theo, sử dụng bất đẳng thức trên, ta chỉ cần chứng minh ! ! X X 3 x xy 64(x y)(y z)(z x) cyc cyc Đặt x = m + n; y = n + p; z = p + m (m; n; p > 0), bất đẳng thức này tương đương với ! ! X X X 2 3 m m +3 mn 32(m n)(n p)(m p) cyc cyc cyc Từ đây, giả sử p = minfm; n; pg, và đặt m = p + u; n = p + v (u; v X m = 3p + u + v u + v 0), ta có cyc X m2 + 3 cyc X mn = 12p2 + 8(u + v)p + u2 + 3uv + v 2 u2 + 3uv + v 2 cyc (m n)(n p)(m p) = uv(u v) Nên ta chỉ cần chứng minh 3(u + v)(u2 + 3uv + v 2 ) , 3u3 , 3u u 32uv(u 20u2 v + 44uv 2 + 3v 3 10 v 3 2 + v) 0 32 2 uv + 3v 3 3 0: hiển nhiên đúng. Vậy ta có đpcm. Ví dụ 1.5 Cho các số không âm a; b; c; không có 2 số nào đồng thời bằng 0: Chứng minh rằng (a b)(13a + 5b) (b + a2 + b2 c)(13b + 5c) (c + b2 + c2 a)(13c + 5a) c2 + a2 0: (Võ Quốc Bá Cẩn) 1 Đây chính là bất đẳng thức Schur bậc 3 http://boxtailieu.net 1.1. ĐẠI LƯỢNG (A B)(B C)(C A) 11 Lời giải. Bất đẳng thức tương đương với X 4(a b)2 + 9(a2 a2 + b2 cyc ,4 ,4 X (a cyc a2 X (a b)2 a2 + b2 cyc 2 b) + b2 9 b2 ) X b2 cyc 2 0 a2 a2 + b2 2 9(a b )(b2 c2 )(c2 a2 ) 2 (a + b2 )(b2 + c2 )(c2 + a2 ) Theo bất đẳng thức AM-GM, 4 X (a cyc b)2 2 a + b2 s 12 3 (a b)2 (b c)2 (c a)2 (a2 + b2 )(b2 + c2 )(c2 + a2 ) Ta cần chứng minh s (a b)2 (b c)2 (c a)2 43 2 (a + b2 )(b2 + c2 )(c2 + a2 ) 3(a2 b2 )(b2 c2 )(c2 a2 ) (a2 + b2 )(b2 + c2 )(c2 + a2 ) Bất đẳng thức này là hệ quả của bất đẳng thức sau với mọi x > y 4(x2 + y 2 )2 Nếu x 6y thì x4 Nếu x , x4 3(x2 y 2 )(x + y)2 6x3 y + 8x2 y 2 + 6xy 3 + 7y 4 6x3 y + 8x2 y 2 + 6xy 3 + 7y 4 = x3 (x 0 0 6y) + 8x2 y 2 + 6xy 3 + 7y 4 0 3y)2 + xy 2 (6y 0: 6y; ta có x4 6x3 y + 8x2 y 2 + 6xy 3 + 7y 4 = x2 (x x) + 7y 4 Vậy ta có đpcm. Đẳng thức xảy ra khi a = b = c: Ví dụ 1.6 Cho các số không âm a; b; c; không có 2 số nào đồng thời bằng 0: Chứng minh rằng ab bc ca 3 : + 2 + 2 2 2 2 2 a + 4b b + 4c c + 4a 5 Ví dụ 1.7 Cho các số không âm a; b; c; không có 2 số nào đồng thời bằng 0: Chứng minh rằng (a b)(3a b) (b c)(3b c) (c a)(3c a) + 2 + 2 3a2 + 2ab + 3b2 3b + 2bc + 3c2 3c + 2ca + 3a2 0: (Thomas Mildorf) http://boxtailieu.net 12 1.2 CHƯƠNG 1. TÌM TÒI MỘT SỐ KỸ THUẬT GIẢI TOÁN Những kiểu lời giải đặc biệt bằng AM-GM Ví dụ 1.8 Cho các số không âm a; b; c thỏa a + b + c = 3: Chứng minh rằng r r r 3 a3 b3 c3 + + : 2 2 2 2 2 2 a + 3b b + 3c c + 3a 2 (Phan Thành Việt) Lời giải. Sử dụng bất đẳng thức AM-GM, ta có r X X a3 a2 p = 6 a2 + 3b2 4a(a + b + c) 3(a2 + 3b2 ) cyc cyc 6 X cyc = 6 X cyc a2 4a(a + b + c) + 3(a2 + 3b2 ) a2 7a2 + 9b2 + 4ab + 4ca Mặt khác, theo bất đẳng thức Cauchy Schwarz thì !" # X X a2 2 2 2 (c + 2a) (7a + 9b + 4ab + 4ca) 7a2 + 9b2 + 4ab + 4ca cyc cyc " #2 !2 X X X 2 a(c + 2a) = 2 a + ab cyc cyc cyc Nên ta chỉ cần chứng minh được 8 2 X a2 + cyc , X cyc a4 + X cyc X !2 X ab a2 b2 + 3 cyc (c + 2a)2 (7a2 + 9b2 + 4ab + 4ca) cyc X cyc a3 b 3 X ab3 cyc Giả sử a = min fa; b; cg ; đặt b = a + x; c = a + y (x; y thành 6(x2 xy + y 2 )a2 + (4x3 + 9x2 y Ta có 4x3 + 9x2 y 2abc X a 0) thì bất đẳng thức trở 9xy 2 + 4y 3 )a + x4 + 3x3 y + x2 y 2 9 9xy 2 + 4y 3 = 4x3 + y(2x 4 http://boxtailieu.net 0 cyc 7 y)2 + y 3 4 3xy 3 + y 4 0 0 1.2. NHỮNG KIỂU LỜI GIẢI ĐẶC BIỆT BẰNG AM-GM x4 + 3x3 y + x2 y 2 3xy 3 + y 4 = 13 2 3 x2 + xy 2 y2 3 + x2 y 2 4 0: Nên bất đẳng thức trên hiển nhiên đúng. Vậy ta có đpcm. Đẳng thức xảy ra khi va chỉ khi a = b = c = 1: Ví dụ 1.9 Cho các số không âm a; b; c; không có 2 số nào đồng thời bằng 0: Chứng minh rằng 5(a + b + c) p p p 4a2 + bc + 4b2 + ca + 4c2 + ab : 2 (Võ Quốc Bá Cẩn) Lời giải. Không mất tính tổng quát, ta có thể giả sử a thức AM-GM, ta có p 2 4a2 + bc p 2 4b2 + ca )2 Xp cyc c, khi đó theo bất đẳng 2a + c + c(b 2a) 4a2 + bc = 4a + c + 2a + c 2a + c 2b + c + 4b2 + ca c(a 2b) = 4b + c + 2b + c 2b + c p 2 4c2 + ab 4a2 + bc b b+ c 2(ab + 4c2 ) + 2 2b + c 5 2(ab + 4c2 ) 4a + 5b + c + +c 2 2b + c b 2a a 2b + 2a + c 2b + c Ta cần chứng minh 5 a+ c 2 , 2(ab + 4c2 ) +c 2b + c c(2a + 10b 11c) 2(2b + c) , 2a + 10b 11c 2(2b + c) , c b 2a a 2b + 2a + c 2b + c b 2a a 2b + 2a + c 2b + c b 2a a 2b + 2a + c 2b + c 14b 11c 2a b + 2(2b + c) 2a + c 0: hiển nhiên đúng vì a b c: Đẳng thức xảy ra khi và chỉ khi a = b; c = 0 hoặc các hoán vị tương ứng. http://boxtailieu.net 14 CHƯƠNG 1. TÌM TÒI MỘT SỐ KỸ THUẬT GIẢI TOÁN Ví dụ 1.10 Cho các số không âm a; b; c; không có 2 số nào đồng thời bằng 0: Chứng minh rằng p p p a a2 + 3bc + b b2 + 3ca + c c2 + 3ab 2(ab + bc + ca): (Vasile Cirtoaje) Lời giải. Sử dụng bất đẳng thức AM-GM, ta có X a(b + c)(a2 + 3bc) X p p a a2 + 3bc = 2 cyc (b + c) a + 3bc cyc 2 X a(b + c)(a2 + 3bc) a2 + 3bc + (b + c)2 cyc Do đó, ta chỉ cần chứng minh được ,2 , 2 X a(b + c)(a2 + 3bc) a2 + 3bc + (b + c)2 cyc X X a(b + c)(a2 b2 c2 + bc) s + 5bc cyc , , , X X a(b + c)(a2 + 3bc) a2 + 3bc + (b + c)2 cyc 2 X a3 (b + c) 0 X ab(a2 X ca(c2 cyc cyc cyc b2 ) s + 5bc X ab(a2 b2 ) s + 5bc , 5abc cyc a(b + c) cyc (s = a2 + b2 + c2 ) a(b3 + c3 ) s + 5bc cyc ab cyc 0 a2 ) s + 5bc X ab(a2 b2 ) s + 5ca X (a b)(a2 b2 ) (s + 5bc)(s + 5ca) cyc 0 0 0: hiển nhiên đúng. Đẳng thức xảy ra khi và chỉ khi a = b = c hoặc a = b; c = 0 hoặc các hoán vị tương ứng. Nhận xét 1 Chúng ta cũng có một cách khác để giải bài toán này như sau Viết lại bất đẳng thức như sau X p X X a a2 + 3bc a 2 ab a2 cyc cyc http://boxtailieu.net cyc 1.2. NHỮNG KIỂU LỜI GIẢI ĐẶC BIỆT BẰNG AM-GM , 3abc X cyc a+ p 1 a2 + 3bc 2 X ab cyc X 15 a2 cyc Sử dụng bất đẳng thức Cauchy Schwarz, ta có X cyc a+ p 1 a2 + 3bc P a+ cyc 9 Pp v u u P a + t3 a2 + 3bc cyc cyc 3 !2 = P a P cyc a 9 v u u P a + t4 cyc 9 P cyc a2 + 3 P cyc ! bc cyc Vậy nên ta chỉ cần chứng minh được 9abc P a cyc , X 2 X ab cyc a3 + 3abc cyc X a2 cyc X ab(a + b): cyc Đây chính là bất đẳng thức Schur bậc 3 nên ta có đpcm. Ví dụ 1.11 Cho các số không âm a; b; c, không có 2 sốn ào đồng thời bằng 0: Chứng minh rằng b+c c+a a+b p +p +p 3: 3ab + c2 3bc + a2 3ca + b2 (Michael Rozenberg) Lời giải. Sử dụng bất đẳng thức AM-GM, ta có X cyc p a+b 3ab + c2 = X cyc X cyc Ta cần chứng minh X cyc 6(a + b)(a + b + c) p 2(a + b + c) 3 3ab + c2 12(a + b)(a + b + c) 4(a + b + c)2 + 9(3ab + c2 ) 12(a + b)(a + b + c) 4(a + b + c)2 + 9(3ab + c2 ) http://boxtailieu.net 3 16 CHƯƠNG 1. TÌM TÒI MỘT SỐ KỸ THUẬT GIẢI TOÁN , X 8a2 + 8b2 , cyc X (a 11ab + 4c(a + b) 4s2 + 27ab + 9c2 c)(16a cyc , X cyc (b 13c2 0 11b + 13c) + (b c)(16b 4s2 + 27ab + 9c2 (s = a + b + c) 11a + 13c) 16c 11a + 13b 16b 11a + 13c 2 2 4s + 27ab + 9c 4s2 + 27ca + 9b2 X , x(b c)2 (4s2 + 27bc + 9a2 ) 0 0 0 c) cyc trong đó x = 99a2 150(b + c)a + 48b2 + 87bc + 48c2 + 4s2 và y; z tương tự. Đặt t = b+c 2 , ta có x = 99a2 = 99a2 99a2 150(b + c)a + 48b2 + 87bc + 48c2 + 4s2 9 300at + 4(a + 2t)2 + 183t2 + (b c)2 4 300at + 4(a + 2t)2 + 183t2 = 103a2 284at + 199t2 1 = [(103a 142t)2 + 333t2 ] 103 0 Tương tự, ta có y; z 0. Bất đẳng thức được chứng minh. Đẳng thức xảy ra khi và chỉ khi a = b = c. Ví dụ 1.12 Cho các số không âm a; b; c thỏa a + b + c = 1: Chứng minh rằng r r r a b c + + 2: 2a2 + bc 2b2 + ca 2c2 + ab (Võ Quốc Bá Cẩn) Lời giải. Sử dụng bất đẳng thức AM-GM, ta có Xr X a a p = 2 2a + bc a(a + b + c) (2a2 + bc) cyc cyc X a 2 a(a + b + c) + 2a2 + bc cyc X a = 2 2 + ab + bc + ca 3a cyc http://boxtailieu.net 1.2. NHỮNG KIỂU LỜI GIẢI ĐẶC BIỆT BẰNG AM-GM 17 Đặt q = ab + bc + ca; r = abc, ta cần chứng minh X cyc , , 9abc X X a(3b2 + q)(3c2 + q) X a2 (b + c) + q 2 cyc , 9qr + 3q(q q2 (3a2 + q)(3b2 + q)(3c2 + q) X a 27a2 b2 c2 + 9q cyc 3r) + q 2 , [q 2 Chú ý rằng r 1 cyc ab + 3q cyc a 3a2 + q a2 b2 + 3q 2 cyc 27r2 + 9q(q 2 4q 3 + 2(9q X 2)r 2r) + 3q 2 (1 27r2 ] + 4r X a2 + q 3 cyc 2q) + q 3 0 0 và 4q 3 + 2(9q 27r2 = (a 2)r b)2 (b c)2 (c a)2 0: Nên bất đẳng thức đã cho hiển nhiên đúng. Đẳng thức xảy ra khi và chỉ khi (a; b; c) = 1 1 2; 2; 0 : Nhận xét 2 Từ bài toán này, ta suy ra kết quả rất khó sau, hiện chỉ mới nhận được một lời giải của chúng tôi trên mathlinks r r r p a b c 2: + + 2 a2 + bc b2 + ca c2 + ab Ví dụ 1.13 Cho các số không âm a; b; c thỏa mãn a + b + c = 1: Chứng minh rằng p p p a + b2 + b + c2 + c + a2 2: (Phan Thành Nam) Lời giải. Ta có p p p a + b2 + b + c2 + c + a2 X p , a + b2 b 1 cyc , X cyc p a a + b2 + b 1 http://boxtailieu.net 2 18 CHƯƠNG 1. TÌM TÒI MỘT SỐ KỸ THUẬT GIẢI TOÁN Theo bất đẳng thức AM-GM, X X a p = a + b2 + b cyc cyc 2(a+b) a+b2 2(a+b) X = 2a2 cyc Do đó, chỉ cần chứng minh được X cyc ,4 ,4 X a4 b2 + 3 cyc X a3 b2 c 19 X 4 2 a b 4 ! a b c cyc X X a2 b3 c + 16 3a b c cyc 1 X a4 bc 12a2 b2 c2 3 2 2 2 2 a b c a b c ! + 15 X 4 a bc cyc cyc X 2 3 cyc cyc Đẳng thức xảy ra khi và chỉ khi a = b = c = vị tương ứng. cyc 1 3 3a2 b2 c2 : cyc hoặc a = 1; b = c = 0 hoặc các hoán Ví dụ 1.14 Cho các số không âm a; b; c; không có 2 số nào đồng thời bằng 0: Chứng minh rằng 1 1 1 4 p +p +p : a+b+c 4a2 + bc 4b2 + ca 4c2 + ab (Phạm Kim Hùng) Lời giải. Sử dụng bất đẳng thức AM-GM, ta có X cyc p 1 4a2 + bc = X cyc 2 X cyc = 4 X cyc 3 2a + 3 2a 1 2b + 12 b + 12 c p + 12 c 4a2 + bc 3 2a + 12 b + 12 c 4a2 + bc + 3 2a + 12 b + 12 c 3a + b + c 16a2 + 4bc + (3a + b + c)2 http://boxtailieu.net 2 ! a b c cyc 0 Bằng cách sử dụng bất đẳng thức AM-GM, ta thấy ngay được X X X X X X a4 b2 a2 b3 c; a3 b2 c a2 b2 c2 ; a4 bc a2 b3 c; a4 bc cyc 0 cyc cyc ! +b a(a + b) + 5ab + 4b2 + ca X +3 2 2 2 a bc +b cyc 2 3 a (a+b)2 +a+b2 cyc 2(a+b) a(a + b) 2a2 + 5ab + 4b2 + ca cyc cyc + X X a p 1.2. NHỮNG KIỂU LỜI GIẢI ĐẶC BIỆT BẰNG AM-GM 19 Cuối cùng, ta chỉ cần chứng minh X cyc , , X cyc X cyc ,2 , , 1 a+b+c 3a + b + c 16a2 + 4bc + (3a + b + c)2 (3a + b + c)(a + b + c) 16a2 + 4bc + (3a + b + c)2 3(3a + b + c)(a + b + c) 16a2 + 4bc + (3a + b + c)2 1 0 X b2 + c2 8a2 + 3a(b + c) 16a2 + 4bc + (3a + b + c)2 cyc X (c + 4a)(c cyc 1 16a2 0 a) (b + 4a)(a b) + 4bc + (3a + b + c)2 0 , X (c + 4a)(c a) 16a2 + 4bc + (3a + b + c)2 X (b + 4a)(a b) 16a2 + 4bc + (3a + b + c)2 0 , X 16b2 (a + 4b)(a b) + 4ca + (3b + c + a)2 X (b + 4a)(a b) 16a2 + 4bc + (3a + b + c)2 0 X cyc cyc ,7 X (a b)2 (7a2 + 7b2 cyc [16a2 cyc cyc (a b)2 (7a2 + 7b2 c2 + 34ab 6ca 6bc) + 4bc + (3a + b + c)2 ][16b2 + 4ca + (3b + c + a)2 ] c2 + 34ab 6ca 0 6bc)[16c2 + 4ab + (3c + a + b)2 ] cyc , Giả sử a b X Sc (a b)2 0 cyc c; khi đó dễ thấy Sb ; Sc a2 [16b2 + ca + (3b + c + a)2 ] 0. Ta có b2 [16a2 + bc + (3a + b + c)2 ] (7b2 + 7c2 a2 + 34bc 6ca 6ab) + (7c2 + 7a2 = 6(a b)2 + 28c(a + b) 0 ) a2 Sb + b2 Sa 0 http://boxtailieu.net b2 + 34ca 6ab 6bc) 0 20 CHƯƠNG 1. TÌM TÒI MỘT SỐ KỸ THUẬT GIẢI TOÁN Suy ra X Sc (a b)2 c)2 + Sb (c Sa (b a)2 Sa (b cyc = (b c)2 b2 (a2 Sb + b2 Sa ) c)2 + a2 Sb (b b2 c)2 0: Bất đẳng thức được chứng minh. Đẳng thức xảy ra khi và chỉ khi a = b; c = 0 hoặc các hoán vị tương ứng. Ví dụ 1.15 Cho các số không âm a; b; c thỏa mãn a + b + c = 1: Chứng minh rằng r r r p (b c)2 (c a)2 (a b)2 a+ + b+ + c+ 3: 8 8 8 (Võ Quốc Bá Cẩn) Lời giải. Sử dụng bất đẳng thức AM-GM, ta có r p p X 16a + 2(b 1 X 3(a + 1) (b c)2 p = a+ 8 a+1 4 3 cyc cyc = = c)2 1 X 3(a + 1)2 + 16a + 2(b c)2 p a+1 8 3 cyc " # X 16a + 2(b c)2 1 p 12 + a+1 8 3 cyc " # X 8a + (b c)2 1 p 6+ a+1 4 3 cyc Ta cần chứng minh X 8a + (b c)2 a+1 , X [8a + (b X , 2 a3 cyc Do X c)2 ](b + 1)(c + 1) ab(a + b) + cyc X bc(b cyc X ab(a + b) 6 6(a + 1)(b + 1)(c + 1) X c)2 + 4 ab + 18abc cyc 6abc cyc http://boxtailieu.net 2 1.2. NHỮNG KIỂU LỜI GIẢI ĐẶC BIỆT BẰNG AM-GM Nên ta chỉ cần chứng minh được X X 2 a3 + 12abc + bc(b cyc X c)2 + 4 ab cyc X X , 2 a3 + 12abc + bc(b cyc X c) + 4 ab X X , 2 a3 + 12abc + bc(b X ab(a + b) c)2 ab(a + b) 6abc cyc , X 3 a (b + c) + 6 cyc ! X X 2 a2 a2 cyc ! X a cyc X 2 X 6abc cyc X 2 !2 cyc cyc ,2 ,2 c)2 cyc cyc X cyc X X , 2 a3 + 12abc + bc(b cyc 2 cyc 2 cyc 21 ab(a ! X ! a cyc b)2 cyc ! a cyc 2 2 a b cyc X ab(a b)2 cyc X 8abc a: cyc Bất đẳng thức cuối hiển nhiên đúng theo bất đẳng thức AM-GM nên ta có đpcm. Đẳng thức xảy ra khi và chỉ khi a = b = c = 31 : Ví dụ 1.16 Cho các số không âm a; b; c; không có 2 số nào đồng thời bằng 0: Chứng minh rằng a b c 3 p : +p +p 2 2 2 2 ab + 3c bc + 3a ca + 3b (Vasile Cirtoaje) Lời giải. Sử dụng bất đẳng thức AM-GM, ta có X cyc p a ab + 3c2 = = Ta cần chứng minh X X a(c + a) a(c + a) p 2 2 + ab + 3c2 2 (c + a) cyc (c + a) ab + 3c cyc X a(c + a) 2 2 + ab + 2ca + 4c2 a cyc X 2 a(c + a) 2 + ab + 2ca + 4c2 a cyc http://boxtailieu.net 3 2 22 CHƯƠNG 1. TÌM TÒI MỘT SỐ KỸ THUẬT GIẢI TOÁN Bằng biến đổi tương đương, ta thấy bất đẳng thức này tương đương với X X X X X 4 ab5 + 12 a2 b4 + 16 a4 b2 24 a3 b3 + 18 a4 bc cyc cyc cyc X +39 a2 b3 c X 17 a3 b2 c cyc Ta có 144a2 b2 c2 24 cyc X cyc 0 cyc X a3 b3 = 12 a2 b2 (a cyc 17 cyc Sử dụng bất đẳng thức AM-GM, ta có X 4 ab5 b)2 0 cyc Nên ta chỉ cần chứng minh được X X X X 4 ab5 + 4 a4 b2 + 18 a4 bc + 39 a2 b3 c cyc cyc cyc X X 12 a2 b4 + 12 a4 b2 cyc cyc X a3 b2 c + 144a2 b2 c2 cyc 12a2 b2 c2 cyc X 4 a4 b2 12a2 b2 c2 cyc X a4 bc 3a2 b2 c2 cyc X 17 a4 bc cyc X 39 a2 b3 c 17 X a3 b2 c cyc 117a2 b2 c2 : cyc Cộng tương ứng vế với vế các bất đẳng thức trên, ta suy ra đpcm. Đẳng thức xảy ra khi và chỉ khi a = b = c: 1.3 1.3.1 Kỹ thuật pqr Lời nói đầu Kỹ thuật pqr là một trong những kỹ thuật hay, hữu ích và hiệu quả nhất đối với bất đẳng thức 3 biến. Phần lớn các bài toán trong sách, chúng tôi đều chọn kỹ thuật pqr để giải, vì vậy, việc hệ thống lại một số kiến thức cần biết về chúng là không thể thiếu được. Bạn sẽ thấy được những điều ngạc nhiên, lạ lẫm khi mà không chỉ có những bài toán đối xứng mới có thể được giải quyết theo pqr mà thậm chí cả những bài toán dạng hoán vị vòng quanh, ta cũng có thể dùng nó để giải trong khi những phương pháp, những kỹ thuật khác lại không đủ khả năng để thực hiện điều này. Sau khi xin được bắt đầu bài viết của chúng tôi "Kỹ thuật pqr". http://boxtailieu.net 1.3. KỸ THUẬT P QR 1.3.2 23 Những đẳng thức cần nhớ Với 3 biến bất kì a; b; c; ta đặt p = a+b+c; q = ab+bc+ca; r = abc (p2 Khi đó, chúng ta có những đẳng thức sau a2 + b2 + c2 a3 + b3 + c3 ab(a + b) + bc(b + c) + ca(c + a) (a + b)(b + c)(c + a) a4 + b4 + c4 a2 b2 + b2 c2 + c2 a2 a3 (b + c) + b3 (c + a) + c3 (a + b) a3 (b2 + c2 ) + b3 (c2 + a2 ) + c3 (a2 + b2 ) a4 (b + c) + b4 (c + a) + c4 (a + b) a5 + b5 + c5 = = = = = = = = = = p2 p3 pq pq p4 q2 p2 q pq 2 qp3 p5 3q; q 2 2q 3pq + 3r 3r r 4p2 q + 2q 2 + 4pr 2pr 2q 2 pr (2p2 + q)r 3pq 2 + (5q p2 )r 5p3 q + 5pq 2 + 5(p2 3pr): q)r Còn rất nhiều những đẳng thức khác nữa, các bạn hãy tự xây dựng cho mình thêm nhé, chúng sẽ rất có ứng dụng về sau. 1.3.3 Bất đẳng thức Schur Định lý 1.1 (Bất đẳng thức Schur) Cho các số không âm a; b; c: Khi đó, với mọi r > 0; ta có bất đẳng thức sau ar (a b)(a c) + br (b c)(b a) + cr (c a)(c b) 0 Đẳng thức xảy ra khi và chỉ khi a = b = c hoặc a = b; c = 0 hoặc các hoán vị tương ứng. Chứng minh. Do tính đối xứng, giả sử a b c: Khi đó, ta viết bất đẳng thức lại như sau (a b)[ar (a c) br (b c)] + cr (a c)(b c) 0 Ta có a c b c 0; ar br Nên bất đẳng thức đúng. Bất đẳng thức Schur được chứng minh. Chúng ta có 2 trường hợp đặc biệt thường hay được ứng dụng để giải toán là r = 1 và r = 2: Khi đó, chúng ta được những bất đẳng thức tương ứng là Hệ quả 1.1 (Bất đẳng thức Schur bậc 3) Cho các số không âm a; b; c: Khi đó, bất đẳng thức sau đúng a3 + b3 + c3 + 3abc ab(a + b) + bc(b + c) + ca(c + a) http://boxtailieu.net 24 CHƯƠNG 1. TÌM TÒI MỘT SỐ KỸ THUẬT GIẢI TOÁN , abc (a + b c)(b + c a)(c + a b): Đẳng thức xảy ra khi và chỉ khi a = b = c hoặc a = b; c = 0 hoặc các hoán vị tương ứng. Hệ quả 1.2 (Bất đẳng thức Schur bậc 4) Cho các số không âm a; b; c: Khi đó, bất đẳng thức sau đúng a4 + b4 + c4 + abc(a + b + c) a3 (b + c) + b3 (c + a) + c3 (a + b): Đẳng thức xảy ra khi và chỉ khi a = b = c hoặc a = b; c = 0 hoặc các hoán vị tương ứng. Dạng pqr tương ứng của 2 bất đẳng thức trên là 9 (4q r p2 ) p(4q r p2 )(p2 6p q) Nhưng do 4q p2 có thể không dương mà r thì luôn luôn không âm nên chúng ta hay dùng cả 2 bất đẳng thức trên ở dạng sau (sẽ rất hiệu quả) r r max 0; max 0; p2 ) p(4q (4q 9 p2 )(p2 6p q) Đôi khi bạn sẽ gặp phải trường hợp giả thiết bài toán là a; b; c là độ dài 3 cạnh của một tam giác (khi đó ta có 4q p2 ), khi đó ta thấy a + b c; b + c a; c + a b là những số không âm, vậy nên theo bất đẳng thức Schur, ta có X (b + c a)[(b + c a) (c + a b)][(b + c a) (a + b c)] 0 cyc , X (b + c ,r Tương tự, ta có a)(a b)(a c) 0 cyc X (b + c p(5q p2 ) 18 a)2 (a b)(a c) cyc http://boxtailieu.net 0 1.3. KỸ THUẬT P QR 25 p4 ,r 7p2 q + 13q 2 9p Vậy chúng ta có các đánh giá p(5q p2 ) p4 ; 18 min 7p2 q + 13q 2 9p r p2 )(p2 6p (4q max 0; q) p(4q ; p2 ) 9 : Chúng ta thường dùng bất đẳng thức Schur để giải bất đẳng thức trong trường hợp bất đẳng thức có những đẳng thức tại các điểm a = b = c hoặc a = b; c = 0 hoặc trong trường hợp a; b; c là độ dài 3 cạnh tam giác thì là a = 2; b = c = 1: Ví dụ 1.17 Cho các số không âm a; b; c thỏa mãn ab + bc + ca = 3: Chứng minh rằng a3 + b3 + c3 + 7abc 10: (Vasile Cirtoaje) Lời giải. Bất đẳng thức tương đương với 10r + p3 Nếu p 10 0 2 3 thì ta có p3 p Nếu 2 3 9p p p 9p 10 3p 10 p 6 3 10 > 0 3 thì theo bất đẳng thức Schur bậc 3, ta có r p2 ) p(12 9 Do đó 10r + p3 9p 10 30 p2 10(p(12 9 Mà 3p 30 p2 ) + p3 p 2 3 2 9p 10 = 1 (p 9 p 3 2 3 = 18 3)(30 p2 3p) p 6 3 > 0: Nên bất đẳng thức cần chứng minh đúng. Đẳng thức xảy ra khi và chỉ khi a = b = c = 1: Ví dụ 1.18 Cho các số dương a; b; c thỏa mãn a + b + c = 3: Chứng minh rằng abc + 12 ab + bc + ca 5: (Vasile Cirtoaje) http://boxtailieu.net 26 CHƯƠNG 1. TÌM TÒI MỘT SỐ KỸ THUẬT GIẢI TOÁN Lời giải. Bất đẳng thức tương đương với r+ 12 q 5 0 Sử dụng bất đẳng thức Schur bậc 3, ta có 4q r 9 3 Do đó r+ 12 q 5 4q 9 3 12 q + 3)2 4(q 5= 0: 3q Bất đẳng thức được chứng minh. Đẳng thức xảy ra khi và chỉ khi a = b = c = 1: Ví dụ 1.19 Cho các số không âm a; b; c; không có 2 số nào đồng thời bằng 0 thỏa mãn a2 + b2 + c2 = 1: Chứng minh b2 a3 + 2 bc + c2 c b3 + 2 ca + a2 a p c3 ab + b2 2: (Võ Quốc Bá Cẩn) Lời giải. Bất đẳng thức tương đương X a3 (b + c) cyc , , Ta có p b3 + c3 X a3 (b + c) +b+c b3 + c3 cyc X a3 cyc X cyc ! a2 X cyc a2 2 2 X a+ 2 X P ! P 9 a3 cyc P P 2 a2 ab cyc cyc 2 2 9 a2 X a+ cyc cyc Nên ta chỉ cần chứng minh p cyc 1 ab + b2 1 ab + b2 2 P ab cyc a+ cyc http://boxtailieu.net p 2 p 2 1.3. KỸ THUẬT P QR 27 Đặt p = a + b + c; q = ab + bc + ca; r = abc ) q = 1 p 2 2 Nếu p p 1 p 2 2 p p 7p2 + 6 2p p 5 + 27r = Bất đẳng thức tương đương 5 + 27r p(4q p2 ) 9 2 theo bất dẳng thức Schur, ta có r p 7p2 + 6 2p p2 1 2 : 0 = p(p2 2) ; 9 do đó p 1 p 2 p 7p2 + 6 2p 2 p 1 p 2 (5 p2 ) 0 2 5 + 3p(p2 2) p Nếu 2 p thì bất đẳng thức trên hiển nhiên đúng. Vậy ta có đpcm. Đẳng thức xảy ra khia = b = p12 ; c = 0 và các hoán vị tương ứng. Ví dụ 1.20 Cho các số không âm a; b; c; không có 2 số nào đồng thời bằng 0 thỏa mãn a2 + b2 + c2 = 3: Chứng minh rằng p 2 b+c 1 p 2 c+a p 1 2 a+b p 1 3 2 1 : (Phạm Kim Hùng) Lời giải. Chú ý là các biểu thức trong các dấu ngoặc đều dương và 2+ p b+c 2+ p c+a 2+ p p p 2 2 2+1 a+b 3 Nên p p 2+1 2 2 3 Y cyc 2 p b+c " 1 = Từ giả thiết ta suy ra p2 Y 2+ p b+c cyc (4 #" Y cyc 2 p b+c b c)(4 c a)(4 a p (a + b)(b + c)(c + a) 1 b) 2q = 3: Do đó theo bất đẳng thức Schur bậc 3, ta có r p2 ) p(4q 9 = p(p2 6) 9 http://boxtailieu.net # 28 CHƯƠNG 1. TÌM TÒI MỘT SỐ KỸ THUẬT GIẢI TOÁN Khi đó, ta có (4 b c)(4 c a)(4 a p (a + b)(b + c)(c + a) b) = = 4(4 p)2 + (4 p)q + r p pq r (4 p)(p2 8p + 29) + 2r p 2[p(p2 3) 2r] (4 2 p)(p2 8p + 29) + 2p(p9 r h i 2 2 p(p2 3) 2p(p9 6) 6) 561p + 108p2 7p3 p = f (p) 3 2p(7p2 15) p p 3; 3 nên f (p) f (3) = 2 2; do đó Dễ thấy f (p) là hàm nghịch biến trên 1044 = p p 2 2 2+1 3 Y cyc ) Y cyc p 2 b+c p 1 2 b+c p 2 2 1 p 3 2 1 : Vậy ta có đpcm. Đẳng thức xảy ra khi a = b = c = 1: 1.3.4 Đại lượng (a b)2 (b c)2 (c a)2 Đối với những bất đẳng thức rất chặt và đẳng thức xảy ra tại những điểm không đặc biệt như bất đăng thức Schur (chẳng hạn đẳng thức xảy ra tại a = 3; b = 2; c = 2) thì việc sử dụng bất đẳng thức Schur để giải chúng là điều hiển nhiên không thực hiện được, do đó chúng ta cần tìm một đánh giá khác phù hợp hơn và hiệu quả hơn để giải chúng. Đại lượng P = (a b)2 (b c)2 (c a)2 0 là đại lượng trung gian khác mà chúng ta chọn ở đây. Tại sao ta lại chọn nó? Vì hầu hết các bất đẳng thức đối xứng đều xảy ra đẳng thức khi có ít nhất 2 biến bằng nhau mà biểu thức P để xảy ra dấu đẳng thức, ta cũng chỉ cần a = b hoặc b = c hoặc c = a là đủ, cho nên ta có thể thấy P rất chặt. Vì vậy, ta sẽ khai thác xem P có ứng dụng gì không? Khai triển ra theo pqr ta được P = p2 q 2 4q 3 + 2p(9q 2p2 )r 27r2 0 Ta hãy xem đây là một tam thức bậc 2 theo r, khi đó giải ra ta có nghiệm p p p(9q 2p2 ) 2(p2 3q) p2 3q p(9q 2p2 ) + 2(p2 3q) p2 r 27 27 http://boxtailieu.net 3q 1.3. KỸ THUẬT P QR 29 Đến đây, có lẽ các bạn vẫn chưa thấy được gì ngoài sự cồng kềnh của bất đẳng thức trên. Đừng vội nản lòng bạn ạ, biết đâu sẽ có một phép màu nào đấy. Và thực sự là như vậy, ta hãy đặt p p p 2 p2 3q p + p2 3q ; v0 = u0 = p3 p3 2 p p 3q p + 2 p2 3q u1 = ; v0 = 3 3 Khi đó, ta thu được một điều đặc biệt là 8 < 2u0 + v0 = 2u1 + v1 = p u2 + 2u0 v0 = u21 + 2u1 v1 = q : 02 u0 v0 r u21 v1 Ngoài ra, trong trường hợp a; b; c là các số không âm, ta thấy u0 ; u1 ; v1 là những số không âm và v0 0 nếu 4q p2 và v0 0 nếu p2 4q: Như vậy, ta thu được một kết quả hết sức đặc biệt sau khi chứng minh một bất đẳng thức. Khi đưa bất đẳng thức đó về dạng pqr có dạng f (r) 0 thì 1) Nếu f (r) là hàm đồng biến, khi đó ta chỉ cần chứng minh f (u20 v0 ) 0 tức là ta chỉ cần xét nó trong trường hợp có 2 biến bằng nhau là đủ. Nếu bất đẳng thức yêu cầu chứng minh với các số không âm thì ta chỉ cần chứng minh f max 0; u20 v0 0 tức là ta chỉ cần chứng minh nó đúng trong trường hợp có 2 biến bằng nhau và trong trường hợp p2 4q thì f (0) 0: 2) Nếu f (r) là hàm nghịch biến, khi đó ta chỉ cần chứng minh f (u21 v1 ) 0 tức là ta chỉ cần xét nó trong trường hợp có 2 biến bằng nhau là đủ. 3) Nếu f (r) là hàm lõm (f 00 (x) 0), khi đó ta chỉ cần chứng minh min f (u21 v1 ); f (u20 v0 ) 0 tức là ta chỉ cần xét nó trong trường hợp có 2 biến bằng nhau là đủ. Nếu bất đẳng thức yêu cầu chứng minh với các số không âm thì ta chỉ cần chứng minh min f (u21 v1 ); f max 0; u20 v0 0 tức là ta chỉ cần chứng minh nó đúng trong trường hợp có 2 biến bằng nhau và trong trường hợp p2 4q thì f (0) 0: http://boxtailieu.net 30 CHƯƠNG 1. TÌM TÒI MỘT SỐ KỸ THUẬT GIẢI TOÁN Ví dụ 1.21 Cho các số dương a; b; c: Chứng minh rằng r b2 c2 3 5 a5 + b5 + c5 a2 + + : b+c c+a a+b 2 3 (Michael Rozenberg) Lời giải. Sử dụng bất đẳng thức Cauchy Schwarz, ta có X a2 b+c cyc X = cyc " P P a2 + b + c 2a b+c #2 = X (b + c a)2 b+c cyc a)2 (b + c cyc a)2 (b + c)(b + c cyc Chuẩn hóa cho a + b + c = 1 và đặt q = ab + bc + ca; r = abc: Ta có X X X X X (b + c a)2 = (1 2a)2 = 3 4 a + 4 a2 = 3 cyc X cyc (b + c)(b + c a)2 = cyc cyc X (1 cyc 2a)2 = 3 a)(1 5 cyc = X a5 X = cyc a3 cyc ! X X a + 8 a2 cyc 2 + 8(1 X a2 cyc 2q) ! 4(1 X a cyc = (1 3q + 3r)(1 2q) (q 2 = 5(1 q)r + 1 5q + 5q 2 cyc 3q + 3r) = 2(1 ! 8q cyc X a2 b2 cyc ! 2r) + qr 2q X 4 a3 cyc 6r) X + abc ab cyc Ta cần chứng minh (3 8q)2 1 2q 6r , f (r) = 81(1 2q 3 r 6r)5 [(5 5 5(1 5q + 5q 2 q)r + 1 3 5q + 5q 2 ] 5q)r + 1 (3 8q)10 Ta có f 0 (r) = 405(1 Nếu 1 2q 6r)4 [36(q 1)r + (1 4q)(7q 5)] 4q thì ta có 36(q 1)r + (1 4q)(7q 5) (1 4q)(7q http://boxtailieu.net 5) 0 0 1.3. KỸ THUẬT P QR Nếu 4q 36(q 31 (4q 1)(1 q) ; 6 1 thì theo bất đẳng thức Schur bậc 4, r 1)r + (1 Vậy nên f 0 (r) thức 4q)(7q 5) 6(q 1)(4q = (1 3q)(2q ta có 1)(1 q) + (1 4q)(7q 1)(4q 1) 0 5) 0; tức f (r) nghịch biến, từ đó ta suy ra để chứng minh bất đẳng " P P 2 (b + c a) cyc (b + c)(b + c #2 a)2 cyc 3 2 r 5 a5 + b5 + c5 3 ta chỉ cần xét nó trong 2 trường hợp sau là đủ: b = 0; c = 1 hoặc b = c = 1: Trường hợp 1. b = c = 1; khi đó bất đẳng thức trở thành r 5 (4 4a + 3a2 )2 5 a + 2 3 2 3 4 4a + 2a + a 3 , g(a) = (4 4a + 3a2 )10 (a5 + 2)(4 4a + 2a2 + a3 )5 Ta có g 0 (a) = 10(a 81 1)3 (7a5 + 5a4 + 16)(4 4a + 3a2 )9 (a5 + 2)2 (4 4a + 2a2 + a3 )6 g 0 (a) = 0 , a = 1 Từ đây, ta dễ dàng kiểm tra được g(a) g(1) = 81 Trường hợp 2. b = 0; c = 1; khi đó bất đẳng thức trở thành r 5 (3a2 2a + 3)2 5 a + 1 3 2 (a + 1)(a + 1) 3 , h(a) = Ta có h0 (a) = (a2 (3a2 2a + 3)10 + 1)5 (a + 1)5 (a5 + 1) 81 5(a 1)k(a)(3a2 2a + 3)9 (a2 + 1)6 (a + 1)5 (a5 + 1)2 với k(a) = 7a6 4a5 + 7a4 12a3 + 7a2 http://boxtailieu.net 4a + 7 32 CHƯƠNG 1. TÌM TÒI MỘT SỐ KỸ THUẬT GIẢI TOÁN Ta sẽ chứng minh k(a) > 0 , 7 a3 + Đặt t = a + 1 a 1 a3 4 a2 + 1 a2 +7 a+ 1 a 12 > 0 2 thì bất đẳng thức trở thành 7(t3 3t) 4(t2 2) + 7t 12 > 0 , 7t3 > 4t2 + 14t + 4 14 4 4 , 3 + 2 + p 9r + 2q 9r + 2q do đó ta chỉ cần chứng minh 2(1 q)[6q + 2q 2 3q 3 + (q 2 + 7q)r 5r + q 2q 2 , g(r) = 5(34 + 25q)r2 5r2 ] 3q 2 + 14q 10q(2 + 6q + 5q 2 )r + 12q (36 + 25q)r 10q 2 + 19q 3 + 6q 4 0 Ta có 0 g = 5q(408 60q + 276q 2 + 399q 3 150q 4 125q 5 ) 0 nên hiển nhiên g(r) 0: Do đó f 0 (r) 0 nên f (r) nghịch biến, như vậy ta chỉ cần xét bất đẳng thức trong trường hợp có 2 biến bằng nhau là đủ. Cho y = z = 1; bất đẳng thức trở thành s r p 3 18x [2(x + 1)2 + 4] 3 3(2x + 2)(x + 3)2 p +4 8 4(2x + 1) + x+2 2(x + 1)3 + 8x 2(x + 2)(x + 1)2 r r p 2(4x2 + 19x + 4) (x2 + 2x + 3)3 3 6(x + 3)2 p ,8 +8 x+2 x3 + 3x2 + 7x + 1 x+2 s p p 2(4x2 + 19x + 4) (x2 + 2x + 3)3 (x + 2) 3 6 , h(x) = + (x + 3)2 (x + 3)4 (x3 + 3x2 + 7x + 1) 8 http://boxtailieu.net 1.3. KỸ THUẬT P QR 35 Ta có h0 (x) = = p (x2 + 2x + 3)(x2 1)(7x3 + 37x2 + 103x + 105) (x 1)(8x + 41) p p + (x + 3)3 8x2 + 38x + 8 2(x + 3)3 (x + 2)(x3 + 3x2 + 7x + 1)3 "p # (x2 + 2x + 3)(x + 1)(7x3 + 37x2 + 103x + 105) 2(8x + 41) x 1 p p 2(x + 3)3 8x2 + 38x + 8 (x + 2)(x3 + 3x2 + 7x + 1)3 Ta chứng minh p (x2 + 2x + 3)(x + 1)(7x3 + 37x2 + 103x + 105) p (x + 2)(x3 + 3x2 + 7x + 1)3 p 2(8x + 41) 8x2 + 38x + 8 Dễ thấy p p (x2 + 2x + 3) (x + 2)(x3 + 3x2 + 7x + 1) p 8 2 2 p > p 2 2 8x + 38x + 8 3 8x + 38x + 8 Nên ta chỉ cần chứng minh 4(x + 1)(7x3 + 37x2 + 103x + 105) 3(x3 + 3x2 + 7x + 1) , 4x4 19x3 + 23x2 53x + 297 8x + 41 0: Bất đẳng thức cuối hiển nhiên đúng, do đó h0 (x) = 0 , x = 1: Từ đây bằng cách lập p bảng biến thiên, ta thấy h(x) h(1) = 3 8 6 : Vậy ta có đpcm. Đẳng thức xảy ra khi và chỉ khi x = y = z; tức ABC đều. Khai thác thêm nữa (với a; b; c không âm), ta có p 27r p(9q 2p2 ) + 2(p2 3q) p2 3q p 2(p2 3q) p2 32 q p p2 3q 2 = p(9q 2p ) + p p2 32 q h i 2 (p2 3q) p2 32 q + p2 (p2 3q) p(9q 2p2 ) + p p2 32 q = 27q 2 (p2 q) 2p(2p2 3q) Và ta thu được r q 2 (p2 2p(2p2 q) 3q) http://boxtailieu.net 36 CHƯƠNG 1. TÌM TÒI MỘT SỐ KỸ THUẬT GIẢI TOÁN Có thể thấy được bất đẳng thức này chặt hơn bất đẳng thức đã biết sau (mà ta vẫn hay dùng) q2 r 3p Tương tự, ta cũng có 27r p(9q 2p2 ) 2(p2 p 3q) p2 3q p 3q)(p 2q)p p2 3q = p(9q 2p ) p(p2 2q) 2(p2 3q) (p2 2q)2 + p2 (p2 p(9q 2p2 ) p(p2 2q) 2 4 2 2 (4q p )(4p 10p q + 3q ) = p(p2 2q) 2 2(p 2 2 3q) Và ta cũng thu được r max 0; (4q p2 )(4p4 p(p2 10p2 q + 3q 2 ) 2q) Có thể thấy bất đẳng thức này chặt hơn 2 bất đẳng thức Schur bậc 3 và bậc 4. Ví dụ 1.23 Cho các số dương a; b; c thỏa mãn a + b + c = 1: Chứng minh rằng p p p a(a + bc) b(b + ca) c(c + ab) + + b + ca c + ab a + bc 1 p : 2 abc Lời giải. Sử dụng bất đẳng thức Cauchy Schwarz, ta có " X cyc p a(a + bc) b + ca #2 " #2 p a(a + b)(a + c) = (b + c)(b + a) cyc " #2 p X a(a + c) p = (b + c) a + b cyc " # ! X Xa+c a (a + b)(b + c) b+c cyc cyc P 2 P ! a + ab Xa+c cyc cyc = (a + b)(b + c)(c + a) cyc b + c X http://boxtailieu.net 1.3. KỸ THUẬT P QR 37 Lại có Xa+c cyc = b+c X1 X b b X 1 = b+c b + c cyc b + c cyc cyc !2 P a X 1 cyc P 2 P b+c a + ab cyc cyc Nên ta chỉ cần chứng minh được P P 2 P 6 6X 1 6 6 (a + b)(b + c)(c + a) 6 cyc b + c 4 a2 + cyc ab cyc , 1 q q r 1+q q r cyc !2 3 7 7 P 2 P 7 7 a + ab 7 5 cyc cyc a cyc 1 1 1 4abc 1 4r q 4(1 q 2 ) q r , 4 q r r 4(1 q 2 ) q , 3 q r r Ta có 4(1 q q2 ) r q r 4(1 q q2 ) q 2 (1 q) 2(2 3q) 2 q q 2 (1 q) 2(2 3q) 8(1 q )(2 3q) 2(2 3q) q(4 7q + q 2 ) q(1 q) 2(2 3q)(3 5q + 4q 2 ) = (1 q)(4 7q + q 2 ) q(1 3q)(5 7q) = 3 3: (1 q)(4 7q + q 2 ) = Bất đẳng thức được chứng minh xong. Đẳng thức xảy ra khi và chỉ khi a = b = c = 31 : Ví dụ 1.24 Cho các số dương a; b; c: Chứng minh rằng r r r 2 2 2 2 2 2 a+b+c 3 b + c 3 c + a 3 a + b p + + : 3 2 2 2 a + bc b + ca c + ab abc (Phạm Hữu Đức) http://boxtailieu.net 38 CHƯƠNG 1. TÌM TÒI MỘT SỐ KỸ THUẬT GIẢI TOÁN Lời giải. Sử dụng bất đẳng thức Holder, ta có X cyc r 3 b2 + c2 a2 + bc !3 " 3 X # X (b2 + c2 ) cyc cyc 1 a2 + bc ! X =6 cyc Ta cần chứng minh X 6 2 a cyc ! X cyc P !3 a cyc P , 1 2 a + bc 6 a2 , !3 P a cyc P a2 ! X cyc cyc +6 X cyc cyc a2 P !3 a cyc abc abc + bc a2 a3 + bc 6 X a cyc Sử dụng bất đẳng thức Cauchy Schwarz, ta có X cyc a3 a2 + bc P P a2 cyc !2 a(a2 + bc) cyc P a2 cyc = P !2 a3 + 3abc cyc Vậy nên ta chỉ cần chứng minh được P !3 a cyc P a2 cyc , 6 +P P cyc a3 2 a !2 + 3abc 6 p2 2q + 6(p2 2q)2 p3 3pq + 6r 6p Chuẩn hóa cho p = 1; khi đó bất đẳng thức trở thành 1 1 2q + 6(1 2q)2 1 3q + 6r a cyc cyc p3 X 6 http://boxtailieu.net 0 a2 ! X cyc 1 a2 + bc ! 1.3. KỸ THUẬT P QR 39 Ta có 1 1 2q + 6(1 2q)2 1 3q + 6r 1 1 = = 2q + 2q)2 6(1 1 3q + 3q 2 (1 q) 2 3q 2 1 6(1 2q) (2 3q) + 1 2q 2 9q + 12q 2 3q 3 14 99q + 264q 2 315q 3 + 144q 4 (1 2q)(2 9q + 12q 2 3q 3 ) Lại có 14 99q + 264q 2 315q 3 + 144q 4 , (2 6(1 9q + 12q 2 )(1 9q + 12q 2 2q)(2 3q)2 3q 3 ) 0: Bất đẳng thức cuối hiển nhiên đúng. Do đó ta có đpcm. Đẳng thức xảy ra khi và chỉ khi a = b = c: Ví dụ 1.25 Cho các số dương a; b; c thỏa mãn a + b + c = 1: Chứng minh rằng 1 1 1 + + + 48(ab + bc + ca) a b c 25: Lời giải 1. Bất đẳng thức tương đương với q + 48q r 25 0 Đây là một hàm nghịch biến theo r nên ta chỉ cần chứng minh bất đẳng thức trong 1 trường hợp có 2 biến bằng nhau là đủ, giả sử a = b ) a 2a; khi đó bất 2; c = 1 đẳng thức trở thành 2 1 + + 48(a2 + 2ac) 25 a c , 2 1 + + 48[a2 + 2a(1 a 1 2a , 2(1 2a)] 3a)2 (1 4a)2 a(1 2a) 25 0: Hiển nhiên đúng. Vậy ta có đpcm. Đẳng thức xảy ra khi và chỉ khi a = b = c = hoặc a = 12 ; b = c = 14 hoặc các hoán vị tương ứng. http://boxtailieu.net 1 3 40 CHƯƠNG 1. TÌM TÒI MỘT SỐ KỸ THUẬT GIẢI TOÁN Lời giải 2. Ta có 27r 9q = p 3q) 1 2 + 2(1 9q 2+ 2(1 3q) 1 1 h 3q) 1 (1 9q = 3q 12 5 q 12 5 q 2+ p 2 12 5 q 12 5 q 1 1 +1 3q 3q i 27q 2 (7 16q) 5(5 12q) )r q 2 (7 5(5 16q) 12q) Do đó q + 48q r 25 5(5 q(7 12q) + 48q 16q) 25 = (1 3q)(5 16q)2 q(7 16q) 0: Bất đẳng thức được chứng minh. Ví dụ 1.26 Cho các số không âm a; b; c: Chứng minh rằng s (a2 + b2 )(a2 + c2 ) + (a + b)(a + c) s (b2 + c2 )(b2 + a2 ) + (b + c)(b + a) s (c2 + a2 )(c2 + b2 ) (c + a)(c + b) a + b + c: (Võ Quốc Bá Cẩn) Lời giải. Đặt a = x2 ; b = y 2 ; c = z 2 (x; y; z có 0): Sử dụng bất đẳng thức Holder, ta (x4 + y 4 )(x + y)2 ) )VT = X cyc s x4 + y 4 x2 + y 2 (x2 + y 2 )3 (x2 + y 2 )2 (x + y)2 (x4 + y 4 )(x4 + z 4 ) (x2 + y 2 )(x2 + z 2 ) X (x2 + y 2 )(x2 + z 2 ) cyc http://boxtailieu.net (x + y)(x + z) 1.3. KỸ THUẬT P QR 41 Do đó X (x2 + y 2 )(x2 + z 2 ) VT (x + y)(x + z) cyc = X (x + y)(x + z) 2 cyc = X X [(x + y)2 = cyc X xy(z + x) (x + y)(x + z) 2 cyc X zx(x + y) + x+y " X z+x cyc X xy(x + y + 2z) cyc X 2 x+y cyc 2xy][(x + z)2 (x + y)(x + z) X + X 2xz] X 4x2 yz (x + y)(x + z) cyc 4x2 yz (x + y)(x + z) cyc x = (x + y)(x + z) 2 xy + 4xyz (x + y)(x + z) cyc cyc cyc ! P 2 P 4xyz x + xy X X cyc cyc = x2 + xy (x + y)(y + z)(z + x) cyc cyc Ta cần chứng minh X P 4xyz xy cyc x2 + cyc P xy cyc ! (x + y)(y + z)(z + x) X 1 x + y cyc # 0 Chuẩn hóa cho p = 1; bất đẳng thức trở thành q(q r) , q2 4r(1 (4 Ta có q2 (4 3q)r q2 (4 3q) q) 3q)r q 2 (1 q) q 3 (1 = 2(2 3q) 2(2 3q) 3q) 0: Bất đẳng thức được chứng minh xong. Đẳng thức xảy ra khi và chỉ khi a = b = c hoặc a > 0; b = c ! 0 hoặc các hoán vị tương ứng. Ví dụ 1.27 Cho các số dương a; b; c thỏa mãn a + b + c = 3: Chứng minh rằng 8 1 1 1 + + a b c +9 10(a2 + b2 + c2 ): (Vasile Cirtoaje) Lời giải. Bất đẳng thức tương đương với 8q + 20q r 81 http://boxtailieu.net 42 CHƯƠNG 1. TÌM TÒI MỘT SỐ KỸ THUẬT GIẢI TOÁN Đây là một hàm nghịch biến theo r nên ta chỉ cần xét bất đẳng thức trong trường hợp có 2 biến bằng nhau là đủ. Giả sử a = b ) a 32 và c = 3 2a; khi đó bất đẳng thức trở thành 16 8 + + 9 20a2 + 10(3 2a)2 a 3 2a 3(2a , 1)2 (10a2 25a + 16) a(3 2a) 0: Hiển nhiên đúng. Đẳng thức xảy ra khi và chỉ khi a = b = 21 ; c = 2 hoặc các hoán vị tương ứng. 1.3.5 Làm mạnh hơn nữa Đối với các bài toán thông thường, chúng ta có thể làm theo cách trên để giải, nhưng đối với những bài toán có chứa căn thức, lũy thừa tổng quát,... Rõ ràng các cách trên là bất khả thi. Do đó, chúng ta cần làm mạnh kỹ thuật của chúng ta hơn nữa để làm sao nó có thể giải quyết được các dạng toán đó. Chúng ta xuất phát từ bổ đề sau đây Bổ đề 1.1 Cho các số không âm a; b; c thỏa a b c; không có 2 số nào đồng thời bằng 0; ta cố định a + b + c = p; abc = r: Khi đó tồn tại 2 số không âm a0 a1 sao cho a 2 [a0 ; a1 ]. Ngoài ra, nếu a = a0 thì b = c và nếu a = a1 thì a = c. Chứng minh. Theo bất đẳng thức AM-GM, p3 27r. Từ giả thiết, ta có b + c = p a; bc = ar nên theo định lý Viet, b; c là các nghiệm của phương trình f (x) = x2 (p a)x + ar = 0. Do c b a nên ta phải có 8 < f 0 f (a) 0 : b+c p a 2 = 2 8 0 Do đó, ta chỉ cần xét bài toán trong 2 trường hợp sau là đủ Trường hợp 1. a = 0; ta cần chứng minh 2k 2k k+1 b k+1 + c k+1 2k bk ck min 2; 3 2k Bất đẳng thức này hiển nhiên đúng theo bất đẳng thức AM-GM. 1 Trường hợp 2. b = c; chuẩn hóa cho b = c = 1 và đặt t = a k+1 ; ta cần chứng minh (t2k + 2)k+1 2k (2tk+1 + 1)k , g(t) = Ta có g 0 (t) = 2kt2k 1 (t2k + 2)k+1 (2tk+1 + 1)k min 2; 3 2k min 2k+1 ; 3 (k + 1)(t2k + 2)k (tk+1 (2tk+1 + 1)k+1 http://boxtailieu.net 2t1 k + 1) 1.3. KỸ THUẬT P QR 53 g 0 (t) = 0 , tk+1 2t1 k + 1 = 0 , h(t) = h0 (t) = 2ktk+1 + k t2 k tk+1 + 1 t1 k 2=0 1 Từ đây dễ thấy h0 (t) có tối đa một nghiệm thuộc (0; 1], suy ra có tối đa 2 nghiệm thuộc (0; 1], trong đó luôn có một nghiệm là 1. Bằng cách cách lập bảng biến thiên, dễ thấy g(t) min fg(0); g(1)g = min 2k+1 ; 3 : Bài toán được giải quyết xong. Ví dụ 1.33 Cho các số không âm a; b; c thỏa mãn a + b + c + abc = 1: Chứng minh rằng (2 + abc)(1 + 2abc) ab + bc + ca : 7 abc Lời giải. Giống như các bài trước, bài này ta cũng chỉ cần xét a = b là đủ. Khi đó, ta có c = 11+a2a2 ) a 12 ; bất đẳng thức trở thành (2 + a2 c)(1 + 2a2 c) 7 a2 c a2 + 2ac , , a(1 a(1 a)(2 + a) a2 + 1 (2 + 3a2 2a3 )(1 a)(4a2 + a + 1) (2a3 + 6a2 + 7)(a2 + 1) a)(2 + a)(2a3 + 6a2 + 7) , 2(a3 + 3a (2 + 3a2 1)2 2a3 )(4a2 + a + 1) 0: Vậy ta có đpcm. Ví dụ 1.34 Cho các số dương x; y; z thỏa mãn xyz = 8: Chứng minh rằng y2 z2 +p +p (x3 + 1)(y 3 + 1) (y 3 + 1)(z 3 + 1) (z 3 + 1)(x3 + 1) p x2 4 : 3 (APMO 2005) q p p Lời giải. Đặt x = 2 3 ab ; y = 2 3 ac ; z = 2 3 cb ; bất đẳng thức trở thành X cyc b1=6 p a7=6 (8a + b)(8c + a) http://boxtailieu.net 1 3 54 CHƯƠNG 1. TÌM TÒI MỘT SỐ KỸ THUẬT GIẢI TOÁN Sử dụng bất đẳng thức Holder, ta có " #6 " #3 X X a7=6 p (8a + b)(8c + a) 1=6 (8a + b)(8c + a) cyc b cyc Do đó ta chỉ cần chứng minh !10 X 6 4=5 3 a X cyc !" ab cyc X = ! ab cyc X X ! ab cyc 4=5 a cyc !10 #3 (8a + b)(8c + a) cyc X 8 X a2 + 73 cyc X !3 ab cyc Do X cyc ! ab 8 X 2 a + 73 cyc X cyc !3 X ab ! ab cyc 6 = 3 X 9 ! ab cyc X 2 a + 72 cyc X cyc X 2 a +8 cyc X cyc !3 ab !3 ab nên ta chỉ cần chứng minh X 4=5 a cyc !10 X ! ab cyc X 2 a +8 cyc X cyc !3 ab Đến đây, sử dụng kết quả của ta, dễ dàng suy ra được ta chỉ cần xét các trường hợp a = 0 hoặc b = c. Trường hợp 1. a = 0; giả sử b c = 1; bất đẳng thức trở thành (b4=5 + 1)10 , f (b) = f 0 (b) = = b(b2 + 8b + 1)3 (b4=5 + 1)10 b(b2 + 8b + 1)3 (b4=5 + 1)9 (b14=5 1 7b2 + 32b9=5 32b + 7b4=5 b2 (b2 + 8b + 1)4 (b4=5 + 1)9 (b2=5 1)m(b) b2 (b2 + 8b + 1)4 1) 0 trong đó m(b) = b12=5 + b2 6b8=5 + 32b7=5 6b6=5 + 32b http://boxtailieu.net 6b4=5 + b2=5 + 1 > 0 1.3. KỸ THUẬT P QR 55 ) f (b) f (1) = 128 >1 125 Trường hợp 2. b = c; giả sử b = c = 1; bất đẳng thức trở thành (a4=5 + 2)10 , g(a) = g 0 (a) = = 2(a4=5 + 2)9 (a3 (2a + 1)(a2 + 16a + 10)3 (a4=5 + 2)10 (2a + 1)(a2 + 16a + 10)3 1 14a11=5 + 65a2 134a6=5 + 110a a1=5 (2a + 1)2 (a2 + 16a + 10)4 68a1=5 + 40) 2(a4=5 + 2)9 (a1=5 1)h(a1=5 ) a1=5 (2a + 1)2 (a2 + 16a + 10)4 Trong đó h(x) = x14 + x13 + x12 + x11 13x10 + 52x9 + 52x8 + 52x7 + 52x6 +28x3 + 28x2 + 28x 40 82x5 + 28x4 Dễ thấy h(x) đồng biến, và h(0) h(1) < 0 nên tồn tại duy nhất nghiệm x0 2 (0; 1) của h(x), suy ra g 0 (a) có đúng 2 nghiệm là 1 và x50 2 (0; 1). Từ đây, bằng cách lập bảng biến thiên dễ thấy g(a) min fg(0); g(1)g = min 128 ;1 125 = 1: Bất đẳng thức được chứng minh xong. Đẳng thức xảy ra khi a = b = c , x = y = z = 1: Nhận xét 4 Đây là bài toán thi Olympic toán Châu Á-Thái Bình Dương 2005 (APMO). 1 Cách giải ở đáp án rất hay và đẹp mắt nhờ sử dụng kết quả px13 +1 x2 +2 . Hiện nay, ngoài lời giải ở đáp án và lời giải của chúng tôi ra chưa có một lời giải nào khác cho bất đẳng thức này. 1.3.6 pqr hoán vị Với các kiến thức bổ sung ở trên, ta đã giải được khá nhiều các bài toán dạng đối xứng. Nhưng còn các dạng hoán vị thì sao? Kỹ thuật này liệu có dùng được cho nó không? Câu trả lời là được. Điều khó khăn lớn nhất khi gặp phải các dạng này là ta không biết làm sao để biểu diễn các biểu thức dạng hoán vị sang pqr: Có một cách làm rất thú vị để chuyển các dạng này sang pqr là dùng tam thức bậc 2. Chúng ta có kết quả http://boxtailieu.net 56 CHƯƠNG 1. TÌM TÒI MỘT SỐ KỸ THUẬT GIẢI TOÁN cơ bản sau (định lý đảo của định lý Viet): Với 2 số thực u; v thỏa u + v = S; uv = P (S 2 4P ) thì u; v là nghiệm của phương trình bậc 2: X 2 SX + P = 0: Dựa trên cơ sở này, ta có thể dễ dàng biểu diễn các biểu thức hoán vị vòng quanh cho 3 biến a; b; c theo p; q; r. Và sau khi biểu diễn về dạng này, ta chỉ việc xét một hàm một biến theo r (hoặc q) khi đã cố định p = const. Như vậy, có thể nói bản chất của kỹ thuật này chẳng qua chỉ là tam thức bậc 2 và khảo sát hàm số. Ví dụ 1.35 Biểu diễn a2 b + b2 c + c2 a; ab2 + bc2 + ca2 theo p; q; r: x = a2 b + b2 c + c2 a . Khi đó, ta có y = ab2 + bc2 + ca2 Lời giải. Đặt x+y = X ab(a + b) = cyc xy = X 2 a b cyc = q 3 a X a3 b3 + cyc ! X 2 ab cyc 2 ! 3pqr + 3r + r(p = ! X X ab cyc X cyc 3 ! 3abc = pq 3r a4 bc + 3a2 b2 c2 cyc 3pq + 3r) + 3r2 = 9r2 + p(p2 6q)r + q 3 Vậy nên x; y là các nghiệm của phương trình X 2 (pq 3r)X+9r2 +p(p2 6q)r+q 3 = 0: Giải phương trình này, ta được 8 p pq 3r+ p2 q 2 4q 3 +2p(9q 2p2 )r 27r 2 < X1 = 2 p : X = pq 3r p2 q2 4q3 +2p(9q 2p2 )r 27r2 2 2 Các nghiệm này luôn tồn tại vì ta luôn có p2 q 2 4q 3 + 2p(9q p2 )r 27r2 0 (bởi x = X1 ; y = X2 vì nó bằng (a b)2 (b c)2 (c a)2 !). Do đó : Tùy theo trường hợp x = X2 ; y = X1 mà ta có thể lựa chọn đáp số, chẳng hạn như trong trường hợp a b c thì x y nên ta phải có x = X1 ; y = X2 : Ví dụ 1.36 Biểu diễn a3 b + b3 c + c3 a; ab3 + bc3 + ca3 theo p; q; r: Lời giải. Đặt x+y = X cyc x = a3 b + b3 c + c3 a : Khi đó, ta có y = ab3 + bc3 + ca3 2 2 ab(a + b ) = X cyc 2 a ! X cyc ! ab abc http://boxtailieu.net X cyc a = (p2 2q)q pr 1.3. KỸ THUẬT P QR xy = X 3 a b cyc = X cyc +abc = ! a2 b2 " 57 !2 X X ab cyc ! = a ! 7p2 r2 + p(p4 + q 2 X a4 b4 + abc cyc 2a2 b2 c2 3 cyc 3 X 2 a2 + a2 b2 c2 a cyc ! a5 + a2 b2 c2 cyc cyc X X X cyc ! X ab X 2 2 a b cyc 5p2 q)r + q 4 ab cyc cyc a X ! + abc X # ab cyc Do đó x; y là các nghiệm của phương trình X2 [(p2 2q)q pr]X + 7p2 r2 + p(p4 + q 2 Giải phương trình này, ta được 8 p2 q 2q 2 < X1 = : X = p2 q 2q2 2 Do đó, ta được p pr+p 5p2 q)r + q 4 = 0 p2 q 2 4q 3 +2p(9q 2p2 )r 27r 2 2 p2 q 2 4q 3 +2p(9q 2p2 )r 27r 2 2 p pr p : x = X1 ; y = X2 : x = X2 ; y = X1 Ví dụ 1.37 Biểu diễn a4 b + b4 c + c4 a; ab4 + bc4 + ca4 theo p; q; r: Lời giải. Thực hiện tương tự như trên, ta dễ dàng tìm được 8 p (5q p2 )r+pq(p2 3q) (p2 q) p2 q 2 4q 3 +2p(9q < 4 a b + b4 c + c4 a = 2 p : ab4 + bc4 + ca4 = (5q p2 )r+pq(p2 3q) (p2 q) p2 q2 4q3 +2p(9q 2p2 )r 27r 2 2p2 )r 27r 2 : 2 Ví dụ 1.38 Biểu diễn a3 b2 + b3 c2 + c3 a2 ; a2 b3 + b2 c3 + c2 a3 theo p; q; r: Lời giải. Thực hiện tương tự như trên, ta dễ dàng tìm được 8 p pq 2 (2p2 +q)r q p2 q 2 4q 3 +2p(9q < 3 2 a b + b3 c2 + c3 a2 = 2 p : a2 b3 + b2 c3 + c2 a3 = pq2 (2p2 +q)r q p2 q2 4q3 +2p(9q 2p2 )r 27r 2 2p2 )r 27r 2 : 2 Ví dụ 1.39 Cho các số không âm a; b; c thỏa mãn a + b + c = 1: Tìm giá trị lớn nhất của biểu thức P = a2 b + b2 c + c2 a + abc: (Vasile Cirtoaje) http://boxtailieu.net 58 CHƯƠNG 1. TÌM TÒI MỘT SỐ KỸ THUẬT GIẢI TOÁN Lời giải. Giả sử a 2 P b c; ta có 2 q 2 3r + = a b + b c + c a + abc = p q r + q 2 4q 3 + 2(9q = 2 Ta có 0 f (r) = 9q 2 q2 f (r) = 0 ) r = r0 = 2) p 3q) 7(1 (1 7(9q 2)r 4q 3 + 2(9q 3q); ta có f 0 (r) +r 27r2 2)r 27r2 2)r p (1 3q) 7(1 189 2) 27r2 = f (r) 4q 3 + 2(9q 0 Nếu 7(9q 4q 3 + 2(9q 2 27r2 2)r p 27r p 2 q2 p q2 3q) 0; suy ra 2 p 1 x2 p + x(1 4 x ) q + q 1 4q (1 + x)(1 x2 ) 4 f (r) f (0) = = = x= 1 2 2 8 (3x 1)2 (3x + 5) 4 4 = + 216 27 27 p Nếu 7(9q 2) (1 3q) 7(1 3q); bằng cách lập bảng biến thiên, ta có f (r) f (r0 ) = = q q r0 + r0 + (9q 2 h2 2 7(9q = = Do 7(9q 5q 3(1 2) 1 p q2 4q 3 + 2(9q 2 27r0 ) 2) = 5q (1 14r0 p 3q) 7(1 27 p 2 7t3 2)r0 27r02 1 i 3q) = 9q + 1 + 2(1 p p t2 ) + 1 + 2 7t3 3t2 + 4 t= 1 = 27 27 p p 7(1 3t2 ) ) t (1 3q) 7(1 3q) ) t3 p 2 7t3 Tóm lại, ta có max P = 4 27 4q 3q 3 p ; 2 7 p t2 2 7t 3 + 4 3t2 + 4 4 = 27 27 27 2 a = b = c = 31 6 a = 2; b = 1; c = 0 3 3 đạt được khi 6 4 a = 1 ; b = 0; c = 2 : 3 3 a = 0; b = 32 ; c = 13 http://boxtailieu.net p 3q) 7(1 27 do đó 3q) 1.3. KỸ THUẬT P QR 59 Nhận xét 5 Chúng ta có một vài điểm cần chú ý 7(9q 2) (1 3q) 189 Thật ra, khi giải phương trình f 0 (r) = 0 ta được đến 2 nghiệm là nhưng các nghiệm phải thỏa mãn điều kiện là r 0 và 9q 2 p27r 7(9q 2) (1 3q) 7(1 so lại với hệ điều kiện này thì chỉ có nghiệm r0 = 189 p 7(9q 2) (1 3q) 7(1 3q) p 7(1 3q) 0: Nhưng khi 3q) thỏa khi Do đó ta phải xét 2 trường hợp như ở lời giảiptrên. 7(9q 2) (1 3q) 7(1 3q) Trong trường hợp nghiệm r0 = thỏa thì chắc hẳn các bạn cũng 189 rất ngại khi thay vào biểu thức ban đầu, bởi lẽ toàn là căn thức (căn trong căn), tính p toán rất phức tạp. Nhưng chúng ta có một mẹo nhỏ ở đây là 9q 2 27r0 = 2 4q 3 + 2(9q 2)r0 27r02 , do đó khi thay r0 vào biểu thức f (r); ta hãy thay pq q 2 4q 3 + 2(9q 2)r0 27r02 bởi 9q 2 27r0 rồi hãy thay trực tiếp giá trị của r0 vào, tính toán sẽ trở nên đơn giản rất nhiều! Ví dụ 1.40 Cho các số thực a; b; c: Tìm giá trị lớn nhất của biểu thức P = ab(a2 b2 ) + bc(b2 c2 ) + ca(c2 (a2 + b2 + c2 )2 a2 ) : (IMO 2006) Lời giải. Chuẩn hóa cho p = 1; khi đó, ta có p q 2 4q 3 + 2(9q 2)r 27r2 j(a b)(b c)(c a)j = P = (1 2q)2 (1 2q)2 q q 3 p 2 4(1 3q)3 27 r 9q27 2 + 4(1 273q) 2(1 3q) 3(1 3q) 27 = = = f (q) (1 2q)2 (1 2q)2 9(1 2q)2 Ta có 0 f (q) = p (6q + 1) 3(1 3q) 9(1 2q)3 f 0 (q) = 0 , q= q= 1 3 1 6 Bằng cách lập bảng biến thiên, ta thấy f (q) 8 1 > < a= 3 1 b= 3+ Mặt khác, cho > : c= 1 3 p 2 p2 2 2 f 1 6 thì P = p 9 2 = 8q 32 p 9 2 32 1 : 3 nên max P = http://boxtailieu.net p 9 2 32 : 60 CHƯƠNG 1. TÌM TÒI MỘT SỐ KỸ THUẬT GIẢI TOÁN Nhận xét 6 Bài toán này là bài toán trong đề thi toán quốc tế năm 2006, cách giải trên ngắn gọn hơn cách giải ở đáp án rất nhiều. Ví dụ 1.41 Cho các số không âm a; b; c; d: Chứng minh rằng (a b)(a c)(a d)(b c)(b (a + b + c + d)6 d)(c 1 : 1728 d) (Võ Quốc Bá Cẩn) Lời giải. Không mất tính tổng quát, giả sử d = min fa; b; c; dg ; đặt a y; c d = z (x; y; z 0); khi đó ta có (a b)(a c)(a d)(b c)(b (a + b + c + d)6 1 27 ; Chuẩn hóa cho p = 1 ) r xyz j(x y)(x z)(y (x + y + z)6 d)(c d) = d = x; b xyz(x y)(x z)(y z) (x + y + z + 3d)6 xyz j(x y)(x z)(y z)j (x + y + z + 3d)6 xyz j(x y)(x z)(y z)j (x + y + z)6 ta có z)j =r p q2 4q 3 + 18qr 27r2 = r 4r Ta lại có p f (q) f 0 (q) = 2(q 6q 2 + 9r) p 1 + 216r + 1 0 f (q) = 0 , q = 12 Bằng cách lập bảng biến thiên, ta thấy p 1 + 216r + 1 (216r + 1)3=2 f (q) f = 27r2 12 216 5 1 r+ 2 216 Do đó r p q2 4q 3 + 18qr 27r2 4r r r (216r + 1)3=2 216 27r2 5 1 r+ 2 216 2 Đặt t2 = 216r + 1 1 ) r = t2161 , từ đây ta có thể thấy r p (t2 1)(3 t) (t + 1)(3 (216r + 1)3=2 5 1 2 p r 27r r+ = 216 2 216 5184 3 http://boxtailieu.net t) = h(t) d= 1.3. KỸ THUẬT P QR 61 Ta có 0 h (t) = p t) (t + 1)(3 p 1296 3 t(2 t) h0 (t) = 0 , t = 2 nên bằng cách lập bảng biến thiên, ta thấy h(t) h(2) = 1 : 1728 8 a = 2t cos2 18 > > < b = 1 sin 18 t và Từ đây ta có đpcm. Đẳng thức xảy ra khi và chỉ khi c > = sin 18 2 sin 18 + 1 t > : d=0 các hoán vị tương ứng. Ví dụ 1.42 Cho các số thực a; b; c. Chứng minh rằng (a2 + b2 + c2 )2 3(a3 b + b3 c + c3 a): (Vasile Cirtoaje) Lời giải. Chuẩn hóa cho p = 1; khi đó ta chỉ cần xét bất đẳng thức trong trường hợp a b c là đủ, suy ra p X q 2q 2 r + q 2 4q 3 + 2(9q 2)r 27r2 = f (r) a3 b = 2 cyc Ta có 0 f (r) = 9q 2 27r p 2 q2 f 0 (r) = 0 , r = r0 = Lập bảng biến thiên, ta có f (r) f (r0 ) = = = = q 2q 2 q 2q 2 5q q2 1 + 9q p q2 4q 3 + 2(9q 4q 3 + 2(9q 7(9q 2) 2)r 27r2 p (1 3q) 7(1 189 27r2 2)r 3q) f (r0 ) 8r: Mặt khác, ta lại có p r0 + q 2 4q 3 + 2(9q 2)r0 27r02 2 r0 + 9q 2 27r0 = 5q q 2 1 14r0 2h i p 2 7(9q 2) (1 3q) 7(1 3q) 1 p 27 27q 2 + 2(1 3q) 7(1 3q) 27 http://boxtailieu.net 62 CHƯƠNG 1. TÌM TÒI MỘT SỐ KỸ THUẬT GIẢI TOÁN Ta cần chứng minh (1 2q)2 p , 1 + 9q 27q 2 + 2(1 3q) 7(1 3q) 9(1 2q)2 i h p 0 , (1 3q) 8 21q 2 7(1 3q) 3f (r0 ) , (1 21q)2 28(1 3q)] p 21q + 2 7(1 3q) 3q)[(8 8 , 3q)(2 7q)2 p 21q + 2 7(1 3q) 9(1 8 0 0: Bất đẳng thức cuối hiển nhiên đúng nên ta có đpcm. Ví dụ 1.43 Cho các số dương a; b; c thỏa mãn a + b + c = 1 và ab + bc + ca = q (1 3q): Tìm giá trị nhỏ nhất của biểu thức P = b2 c2 a2 + + : b c a (Võ Quốc Bá Cẩn) Lời giải. Không mất tính tổng quát, ta chỉ cần xét a b c. Ta có P 3 p ab 2 2 2 q 2q 2 r q 2 4q 3 + 2(9q 2)r 27r2 a b c cyc + + = = = f (r) b c a abc 2r p q 2 4q 3 + (9q 2)r + (2q 2 1) q 2 4q 3 + 2(9q 2)r 27r2 0 p f (r) = 2r2 q 2 4q 3 + 2(9q 2)r 27r2 i h p q 3 9q 2 2q + (1 3q) (1 2q)(1 3q) f 0 (r) = 0 ) r = r0 = 27q 4 27q 3 + 27q 2 9q + 1 Từ đây, bằng cách lập bảng biến thiên, dễ thấy f (r) f (r0 ) 8r, lại có p q 2q 2 r0 q 2 4q 3 + 2(9q 2)r0 27r02 f (r0 ) = 2r0 = = 2q 2 q (2q 2 r0 q 2 4q 3 +(9q 2)r0 1 2q 2 2r0 9q + 1)r0 + q + 2q 3 2r0 (1 2q 2 ) 3q 2 + 4q 4 Từ đây, dễ dàng đi đến kết luận bài toán. http://boxtailieu.net 1.3. KỸ THUẬT P QR 63 Nhận xét 7 Chúng ta có một vài điểm khá thú vị, không chỉ cho bài này nói riêng mà còn cho tất cả các bài khác nói chung. Xin phân tích rõ hơn ở bài này, các bài khác, ta có thể lấy ý tưởng tương tự. Sau khi thay xong biểu thức f (r0 ) = (2q 2 9q+1)r0 +q+2q 3 3q 2 +4q 4 = g(r0 ), chúng ta thấy được gì ở đây? g(r0 ) là một hàm 2r0 (1 2q 2 ) đơn điệu theo r0 ; cụ thể là nó nghịch biến, điều này có ý nghĩa rất lớn, các bạn ắt hẳn còn nhớ kết quả sau trong bất đẳng thức ba biến (xem ở bài viết trước) p p p(9q 2p2 ) 2(p2 3q) p2 3q p(9q 2p2 ) + 2(p2 3q) p2 3q r 27 27 p+ , p p 2 p2 2 p2 p 3q 27 p p2 p 3q r 2 3q 27 p p + 2 p2 3q Ở đây, ta cũng sẽ có p+ p p2 p 2 p2 2 3q p 27 p p2 p 3q r0 2 3q 27 p p + 2 p2 3q vì ta có q2 4q 3 + (9q ) [q 2 q 1) q 2 2)r0 +(2q 2 4q 3 + (9q 2)r0 ]2 = (1 4q 3 + 2(9q 2q 2 )2 [q 2 27r02 = 0 2)r0 4q 3 + 2(9q 2)r0 27r02 ] Do đó, ta phải có q2 p+ , p p2 2 3q p 27 4q 3 + 2(9q p 2 p2 2)r0 3q 27r02 p r0 p p2 0 2 3q 27 p p + 2 p2 3q Như vậy, chúng ta sẽ có f (r) f (r0 ) 0 B p g@ p p2 2 3q 27 p p + 2 p2 1 3q C A Cái lợi của kết quả cuối này ở chỗ đối với những bất đẳng thức không chặt lắm mà dùng cả một kết quả khổng ! lồ là f (r0 ) để giải thì quả là bất tiện, nhưng với p 2 p 2 p p 3q p+2 p2 3q g thì mọi việc sẽ trở nên đơn giản hơn rất nhiều, chúng 27 ta sẽ không phải tính toán với căn thức (vì nếu ta đặt 3q = p2 http://boxtailieu.net x2 (x 0) thì 64 g CHƯƠNG 1. TÌM TÒI MỘT SỐ KỸ THUẬT GIẢI TOÁN p p p2 3q 2 p+2 p p2 3q 27 ! là một biểu thức không chứa căn). Rõ hơn, chúng ta lấy ví dụ đơn giản sau a2 b2 c2 + + +a+b+c b c a 6(a2 + b2 + c2 ) a+b+c Bất đẳng thức này tương đương a2 b2 c2 + + 5 b c a n 2 Như vậy, nếu ta chứng minh f (r0 ) = min ab + 12q: b2 c + c2 a o 5 12q thì bài toán được giải quyết xong. Vì đây là bất đẳng thức 1 biến theo nên ta tin chắc là sẽ làm được nếu có một kỹ thuật tính toán tốt, nhưng với một bài toán không quá chặt như thế này thì việc dùng f (r0 ) thì có vẻ là “hơi quá tay”, chúng ta hãy thử dùng 0 p p2 B p g@ 27 x2 ) 1 Đặt 3q = 1 g 2 3q p 1 1 p p + 2 p2 x 2 3q 27 g (1 1 3q C A=g p 1 1 3q 2 p 1+2 1 3q 27 ! 0, khi đó ta có ! p 1 + 2 1 3q 2 x) (1 + 2x) 27 = 5 7 + 7x + 32x2 + 16x3 + 5x4 + 28x5 8x6 (1 x)(1 + 2x)(7 + 4x2 2x4 ) 6x7 12q = 1 + 4x2 Nên ta chỉ cần chứng minh 7 + 7x + 32x2 + 16x3 + 5x4 + 28x5 8x6 (1 x)(1 + 2x)(7 + 4x2 2x4 ) 6x7 1 + 4x2 Rất đơn giản, bằng biến đổi tương đương, bạn có thể bất đẳng thức trên tương đương với x2 (14 16x + 55x2 + 14x3 + 28x4 + 2x5 16x6 ) 0: Nhưng bất đẳng thức này lại hiển nhiên đúng do 1 x http://boxtailieu.net 0: 1.3. KỸ THUẬT P QR 65 Ví dụ 1.44 Cho các số dương a; b; c: Chứng minh rằng r a2 b2 c2 a4 + b4 + c4 : + + 3 b c a a2 + b2 + c2 (Nguyễn Văn Thạch) Lời giải. Theo bài trên, ta có X a2 cyc b 7 + 7x + 32x2 + 16x3 + 5x4 + 28x5 8x6 (1 x)(1 + 2x)(7 + 4x2 2x4 ) 6x7 Lại có r 3 a4 + b4 + c4 a2 + b2 + c2 = s 3 s 3 = r 4r + 2q 2 4q + 1 1 2q 4 27 (1 x)2 (1 + 2x) + 2q 2 1 2q 4q + 1 1 + 12x2 + 8x3 + 6x4 1 + 2x2 Do đó ta chỉ cần chứng minh 7 + 7x + 32x2 + 16x3 + 5x4 + 28x5 8x6 (1 x)(1 + 2x)(7 + 4x2 2x4 ) 6x7 r 1 + 12x2 + 8x3 + 6x4 1 + 2x2 Chú ý là 3 + 13x2 7x4 7 + 7x + 32x2 + 16x3 + 5x4 + 28x5 8x6 6x7 2 4 (1 x)(1 + 2x)(7 + 4x 2x ) 3(1 x2 ) x2 (14 62x + 63x2 + 99x3 + 101x4 + 40x5 25x6 14x7 ) = 0 3(1 x2 )(1 + 2x)(7 + 4x2 2x4 ) ) 7 + 7x + 32x2 + 16x3 + 5x4 + 28x5 8x6 (1 x)(1 + 2x)(7 + 4x2 2x4 ) 6x7 3 + 13x2 7x4 3(1 x2 ) Nên ta chỉ cần chứng minh 3 + 13x2 7x4 3(1 x2 ) , x2 (6 r 1 + 12x2 + 8x3 + 6x4 1 + 2x2 72x + 436x2 + 144x3 + 72x4 72x5 9(1 x2 )2 (1 + 2x2 ) Bất đẳng thức cuối hiển nhiên đúng do 1 ra khi a = b = c: x 369x6 + 98x8 ) 0 0: Vậy ta có đpcm. Đẳng thức xảy http://boxtailieu.net 66 CHƯƠNG 1. TÌM TÒI MỘT SỐ KỸ THUẬT GIẢI TOÁN Nhận xét 8 Bất đẳng thức a2 b2 c2 + + b c a 3 + 13x2 7x4 3(1 x2 ) Có dạng tương đương là P P P 2 2 3 x4 + 13 x3 (y + z) x y 2 Xx cyc cyc cyc ! ! y P P cyc 3 x xy cyc xyz P x cyc 8x; y; z 0 cyc Chúng ta có thể dùng kết quả này để chứng minh kết quả sau (hiện vẫn unsolved trên mathlinks) r 6 6 6 b2 c2 a2 6 a + b + c + + 3 : b c a 3 (Võ Quốc Bá Cẩn) Ví dụ 1.45 Cho các số dương a; b; c: Chứng minh rằng p a b c 3 + + + 3 4 b c a 2 ab + bc + ca a2 + b2 + c2 p 3 3 4 + 1: Lời giải. Rõ ràng ta chỉ cần xét bất đẳng thức trong trường hợp a Chuẩn hóa cho p = 1; khi đó, ta có p q 3r q 2 4q 3 + 2(9q 2)r 27r2 c a b + + = b c a 2r b q ab + bc + ca = 2 2 2 a +b +c 1 2q Nên bất đẳng thức tương đương với p q 3r q 2 4q 3 + 2(9q 2r Xét hàm số f (r) = Ta có 0 f (r) = q2 q 3r 2)r p q2 27r2 p 334 2 q + 1 2q 4q 3 + 2(9q 2r 2)r 27r2 p 4q 3 + (9q 2)r q q 2 4q 3 + 2(9q 2)r p 2r2 q 2 4q 3 + 2(9q 2)r 27r2 http://boxtailieu.net p 3 3 4+1 27r2 c là đủ. 1.3. KỸ THUẬT P QR 67 f 0 (r) = 0 , r = r0 = h q 2 9q 2 Lập bảng biến thiên, ta có thể thấy f (r) f (r0 ) = = = 2q + (1 27q 2 p 3q) q(1 9q + 1 f (r0 ) 8r > 0. Ta lại có p q2 q 4q 3 + 2(9q 2)r0 27r02 = 2r0 3 2q + (1 6q)r0 2q 2 1 = + 6 qr0 r0 q 1 2(27q 2 9q + 1) p + 6 9q 2 2q + (1 3q) q(1 3q) q q 3r0 i 3q) 3r0 q 2 4q 3 +(9q 2)r0 q 2r0 Như vậy, để hoàn tất yêu cầu của bài toán, ta chỉ cần chứng minh được p p 334 2 q 3 f (r0 ) + 3 4+1 1 2q p p 334 2 q 1 2(27q 2 9q + 1) 3 p + + 3 4+7 , 2 1 2q 9q 2q + (1 3q) q(1 3q) q Bằng khai triển trực tiếp, ta thấy 2(27q 2 9q 2 p p 334 2 q 1 3 3 4 + + 1 2q 3q) q (1 3q)A h i p 2q) 9q 2 2q + (1 3q) q(1 3q) 9q + 1) p 3q) q(1 2q + (1 = q(1 7 p p Với A = [3(k+6)q 2 (k+11)q +1] q(1 3q) q 2 [2k+9 9(k+2)q]; k = 3 3 4 2: Do 2k + 9 9(k + 2)q nên nếu 3(k + 6)q 2 (k + 11)q + 1 0 thì hiển nhiên A 0 nên bất đẳng thức của ta đúng. Ngược lại, nếu 3(k + 6)q 2 (k + 11)q + 1 0 thì ta có p A 0 , q 2 [2k + 9 9(k + 2)q] [3(k + 6)q 2 (k + 11)q + 1] q(1 3q) , q 4 [2k + 9 9(k + 2)q]2 [3(k + 6)q 2 (k + 11)q + 1]2 q(1 3q) p p p 2 p 9 3 16 + 6 3 4 + 4 q(27q 2 9q + 1) 20q 4 3 16 8q + 2 3 16 , 800 Do 3(k + 6)q 2 (k + 11)q + 1 q 0 nên k + 11 p k 2 + 10k + 49 6(k + 6) http://boxtailieu.net 0 68 CHƯƠNG 1. TÌM TÒI MỘT SỐ KỸ THUẬT GIẢI TOÁN Suy ra 8q + 2 p 3 16 = = = Do k > 5 2 p p k 2 + 10k + 49 3 16 +2 3(k + 6) p 4 k + 11 k 2 + 10k + 49 12 +2 3(k + 6) k+2 p 2 2 2 5k + 32k 28 2(k + 2) k + 10k + 49 3(k + 2)(k + 6) 3k(7k 2 + 46k 152) p 3(k + 2) 5k 2 + 32k 28 + 2(k + 2) k 2 + 10k + 49 4 k + 11 nên 7k 2 + 46k 152 > 7 5 2 2 5 2 + 46 152 = 27 > 0 ) A > 0: 4 Bài toán được giải quyết xong. Đẳng thức xảy ra khi và chỉ khi a = b = c hoặc 8 q pp p p p p 3 3 > 3 3 3 4 1 2 1 > +3 4 + 8 2 11 cos 3 arccos 17 203 4 > a= 3+ 2 3 > < c=t q > p p p p > p p 3 3 > 3 3 2 1 2 3 > 2 3 sin 3 arccos 27+27 202 27 4 t : b= 3 4+ 3 3 4 t 2 và các hoán vị tương ứng. p Ví dụ 1.46 Cho các số dương a; b; c: Chứng minh rằng với k = 3 3 2 a b c + + +k b c a 3; ta có 3(k + 3)(a2 + b2 + c2 ) : (a + b + c)2 (Võ Quốc Bá Cẩn, Bách Ngọc Thành Công) Lời giải. Chuẩn hóa cho p = 1; theo kết quả trên, ta có Xa cyc b 2(27q 2 9q 2 2q + (1 Ta phải chứng minh 2(27q 2 9q 2 2q + (1 9q + 1) p 3q) q(1 3q) 9q + 1) p 3q) q(1 + 1 q 6+k http://boxtailieu.net 3q) + 1 q 6 3(k + 3)(1 2q): 1.3. KỸ THUẬT P QR 69 Ta có VT VP = q Trong đó A = [6(k + 3)q 2 Do 4k + 21 15)q + 1 lại 2k+15 A h (1 9q 2 3q)A p 2q + (1 3q) q(1 p (2k + 15)q + 1] q(1 18(k + 3)q nên nếu ta có q i 3q) 3q) + q 2 [4k + 21 2k+15 p 4k2 +36k+153 12(k+3) 0 thì A p 18(k + 3)q]: , 6(k + 3)q 2 (2k + 0 nên bất đẳng thức hiển nhiên đúng, trong trường hợp ngược 4k2 +36k+153 q 13 : Ta có 12(k+3) p 0 , q 2 [4k + 21 18(k + 3)q] [6(k + 3)q 2 (2k + 15)q + 1] q(1 3q) , q 3 [4k + 21 18(k + 3)q]2 [6(k + 3)q 2 (2k + 15)q + 1]2 (1 3q) p p p 2 3 4(27q 2 9q + 1) 12q + 1 3 4 12q 2 3 4 , 0 12 Do q 2k+15 p 4k2 +36k+153 12(k+3) nên 12q + 1 p 3 4 > 0: Vậy bất đẳng thức cần chứng minh đúng. Ví dụ 1.47 Cho các số không âm a; b; c; không có 2 số nào đồng thời bằng 0: Chứng minh rằng r a + 2c b + 2a c + 2b 5(a2 + b2 + c2 ) + + + 4: a + 2b b + 2c c + 2a ab + bc + ca (Võ Quốc Bá Cẩn) Ví dụ 1.48 Cho các số không âm a; b; c; không có 2 số nào đồng thời bằng 0: Chứng p p 3(3 3 3+ 3 9+1) minh rằng với k = thì 8 ab2 + bc2 + ca2 a2 + b2 + c2 +k 2 ab + bc + ca a b + b2 c + c2 a k + 1: (Bách Ngọc Thành Công) Ví dụ 1.49 Cho các số không âm a; b; c; không có 2 số nào đồng thời bằng 0 thỏa mãn a + b + c = 1: Tìm giá trị lớn nhất và giá trị nhỏ nhất của biểu thức a b b c c a P (a; b; c) = p +p +p : c+a a+b b+c (Phạm Kim Hùng) http://boxtailieu.net 70 CHƯƠNG 1. TÌM TÒI MỘT SỐ KỸ THUẬT GIẢI TOÁN Một điều hạn chế của kỹ thuật này là mặc dù rất mạnh nhưng nó đòi hỏi chúng ta phải tính toán nhiều. Nhưng các bạn ạ, các bài toán ở đây chúng tôi đưa ra đều là những bài toán rất khó, đẳng thức của chúng hầu hết xảy ra tại những điểm lệch nhau. Điều này rất khó cho chúng ta tìm được 1 lời giải đẹp mắt cho nó ngoài những lời giải như thế này. Tuy nhiên, chúng ta có thể thấy một điều là nếu dùng kỹ thuật này để giải những bài toán thi quốc gia, quốc tế thì chúng ta lại thu được những lời giải gọn đẹp và rất nhẹ nhàng bởi lẽ các bài toán ấy đều là những bài toán "rất lỏng". Vì vậy, chúng tôi viết về kỹ thuật này với mong muốn thiết lập cho chúng ta một kỹ thuật, một phương pháp để chúng ta có thể giải được những bài toán ấy khi "chạm trán" chúng trong các kỳ thi. 1.4 1.4.1 The CYH techniques Lời nói đầu Ngay từ khi còn học ở mái trường THCS, chúng ta đã được làm quen với bất đẳng thức Cauchy Schwarz và khi bước sang THPT, chúng ta được làm quen thêm với bất đẳng thức Holder, cả 2 bất đẳng thức này đều rất thường được sử dụng, ngay cả trong các kỳ thi học sinh giỏi quốc gia, quốc tế. Có thể nói chúng và bất đẳng thức trung bình cộng-trung bình nhân (AM-GM) là những bất đẳng thức cổ điển thông dụng nhất hiện nay, nhưng việc sử dụng chúng như thế nào là hiệu quả? Bài viết nhỏ này, chúng tôi xin được chia sẻ với các bạn một vài kỹ thuật thông dụng, mong nhận được ý kiến đóng góp của các bạn. 1.4.2 Bất đẳng thức Cauchy Schwarz và Holder. Trước khi bắt đầu bài viết, chúng ta hãy nhắc lại vài nét về bất đẳng thức Cauchy Schwarz và Holder. Định lý 1.4 (Bất đẳng thức Cauchy Schwarz) Với mọi số thực (a1 ; a2 ; :::; an ) và (b1 ; b2 ; :::; bn ); ta có (a1 b1 + a2 b2 + + an bn )2 (a21 + a22 + + a2n )(b21 + b22 + + b2n ): Đẳng thức xảy ra khi và chỉ khi ai : aj = bi : bj 8i; j 2 f1; 2; :::; ng : Chứng minh bất đẳng thức trên có rất nhiều cách nhưng cách ngắn gọn nhất là sử dụng đẳng thức Lagrange (a21 + a22 + + a2n )(b21 + b22 + + b2n ) (a1 b1 + a2 b2 + + an bn )2 = X i6=j http://boxtailieu.net (ai bj aj bi )2 1.4. THE CYH TECHNIQUES 71 Hệ quả 1.5 Với mọi số thực (a1 ; a2 ; :::; an ) và (b1 ; b2 ; :::; bn ); bi > 0 8i = 1; 2; :::; n; ta có a21 a2 + 2+ b1 b2 + a2n bn (b1 + b2 + + bn ) (a1 + a2 + + an )2 Đẳng thức xảy ra khi và chỉ khi ai : aj = bi : bj 8i; j 2 f1; 2; :::; ng : Định lý 1.5 (Bất đẳng thức Holder) Cho các số dương xij (i = 1; m; j = 1; n): Khi đó với mọi ! 1 ; :::; ! n 0 thỏa ! 1 + + ! n = 1; ta có 0 1! j ! n m n m Y X Y X ! j @ xij : xij A i=1 i=1 j=1 j=1 Chứng minh bất đẳng thức này bằng cách dùng bất đẳng thức AM-GM tổng quát như sau m P Giả sử xij = 1 8i = 1; n (ta luôn có thể giả sử được điều này! Tại sao?), khi đó j=1 bất đẳng thức ở trên trở thành m n X Y 1 j=1 ! xijj i=1 ! Sử dụng bất đẳng thức AM-GM, ta có m n X Y j=1 i=1 ! xijj ! m n X X j=1 ! j xij i=1 ! = n X i=1 0 1 m X @ ! j xij A j=1 0 1 n m n X X X xij A = ! j = 1: !j @ = i=1 j=1 j=1 Bất đẳng thức Holder được chứng minh. Một trường hợp đặc biệt thường gặp của bất đẳng thức Holder là khi n = 3; ta có (a3 +b3 +c3 )(m3 +n3 +p3 )(x3 +y 3 +z 3 ) ( a b c m = n = p b c a x = y = z Đẳng thức xảy ra khi và chỉ khi (a3 + b3 + c3 )(m3 + n3 + p3 )2 Đẳng thức xảy ra khi và chỉ khi (amx+bny+cpz)3 8a; b; c; m; n; p; x; y; z a m = : Và khi (m; n; p) (x; y; z) thì (am2 + bn2 + cp2 )3 8a; b; c; m; n; p b n = pc : http://boxtailieu.net 0 0 72 CHƯƠNG 1. TÌM TÒI MỘT SỐ KỸ THUẬT GIẢI TOÁN 1.4.3 Một số kỹ thuật cần chú ý Tham số hóa Đây là kỹ thuật cơ bản nhất trong các kỹ thuật được trình bày ở đây, các bạn hãy xem xét kỹ nó trước khi sang phần khác. Ví dụ 1.50 Cho các số không âm x; y; z thỏa mãn 2x + 3y + z = 1: Tìm giá trị nhỏ nhất của biểu thức P = x3 + y 3 + z 3 : Lời giải. Sử dụng bất đẳng thức Holder, ta có (x3 + y 3 + z 3 )(a3 + b3 + c3 )(m3 + n3 + p3 ) ) P = x3 + y 3 + z 3 (a3 (xam + ybn + zcp)3 8a; b; c; m; n; p 0 (xam + ybn + zcp)3 + b3 + c3 )(m3 + n3 + p3 ) Ta hãy chọn a; b; c; m; n; p sao cho giả thiết 2x + 3y + z = 1 được tận dụng triệt để, cp bn từ đó theo lẽ tự nhiên ta có thể chọn a; b; c; m; n; p thỏa am 2 = 3 = 1 = 1: Hơn nữa, do ta cần tìm min P nên đẳng thức ở bất đẳng thức này phải xảy ra, tức là y x z a = b = c y x z m = n = p 2x+3y+z z 1 = 3y 3b = c = 2a+3b+c = 2a+3b+c 2ax = 3by = cz 8 a < x = 2a+3b+c b y = 2a+3b+c : Từ phương trình thứ 2 suy ra Từ phương trình thứ nhất suy ra : c z = 2a+3b+c , 2x 2a 2a2 3b2 c2 = = , 2a2 = 3b2 = c2 2a + 3b + c 2a + 3b + c 2a + 3b + c p p Từ đây, ta chọn được a = p12 ; b = p13 ; c = 1 ) m = 2 2; n = 3 3; p = 1, từ đó theo 2ax = 3by = cz , trên, ta có P p1 2 = 3 + p1 3 (2x + 3y + z)3 h p 3 3 p +1 2 2 + 3 3 36 p p 36 + 4 3 + 9 2 1 + 81 3 + 16 2 p p http://boxtailieu.net 3 i +1 1.4. THE CYH TECHNIQUES 8 < x= y= Đẳng thức xảy ra khi : z= min P = 73 a 2a+3b+c b 2a+3b+c c 2a+3b+c 8 1 < a = p2 b = p13 : Vậy nên với : c=1 36 p p : 36 + 4 3 + 9 2 1 + 81 3 + 16 2 p p Ví dụ 1.51 Cho các số không âm x; y thỏa mãn x3 + y 3 = 1: Tìm giá trị lớn nhất của biểu thức P = x + 2y: Lời giải. Sử dụng bất đẳng thức Holder, ta có (x3 + y 3 )(a3 + b3 )(m3 + n3 ) (xam + ybn)3 8a; b; m; n p 3 ) xam + ybn (x3 + y 3 )(a3 + b3 )(m3 + n3 ) 0 Lẽ tự nhiên, do yêu cầu của bài toán nên ta phải chọn a; b; m; n sao cho biểu thức bn xam + ybn P; tức là các số a; b; m; n phải thỏa am 1 = 2 = 1: Ngoài ra, cũng như ví dụ trên, ta cần tìm giá trị lớn nhất của P nên bắt buộc đẳng thức ở bất đẳng thức trên phải xảy ra, tức là y x a = b y x m = n , x+2y = 2y 2b = a+2b = ax = 2by 8 a < x = a+2b b , y = a+2b : ax = 2by 8 a > < x = a+2b b y = a+2b , > : a2 = 2b2 a+2b a+2b 8 a < x = a+2b , y= b : 2 a+2b a = 2b2 8 a x = a+2b > > < b y = a+2b p ) > a= 2 > : b=1 x a 1 a+2b http://boxtailieu.net 74 CHƯƠNG 1. TÌM TÒI MỘT SỐ KỸ THUẬT GIẢI TOÁN 8 a x = a+2b > > > b > y = a+2b > > p < a= 2 ) b=1 > > > 1 > > > m = p2 : n=2 Do đó theo trên, ta có P = x + 2y p 3 (x3 + y 3 )(a3 + b3 )(m3 + n3 ) = s 3 p 2 2+1 1 p +8 2 2 Đẳng thức luôn xảy ra nên max P = s 3 p 2 2+1 1 p +8 : 2 2 Đối với những bất đẳng thức mà không có đẳng thức xảy ra thì ta chọn tham số là những số mà đẳng thức của bất đẳng thức Cauchy Schwarz hoặc Holder để giải là “lân cận bằng” của bất đẳng thức ban đầu. Ví dụ 1.52 Cho các số dương a1 ; a2 ; :::; an : Chứng minh rằng 1 2 + + a1 a1 + a2 + n a1 + a2 + + an 1 1 + + a1 a2 0 8i = 1; n; ta có b21 b2 + 2 + a1 a2 ) k a1 + a2 + b2k ak + + ak ) n X k=1 (a1 + a2 + k (b1 + b2 + k a1 + a2 + + ak ) (b1 + b2 + b21 b2 + 2 + a1 a2 + bk )2 + ak + bk )2 + b2k ak n X ci a i=1 i Với ck = kb2k (k + 1)b2k + + 2 (b1 + b2 + + bk ) (b1 + b2 + + bk+1 )2 http://boxtailieu.net + nb2k (b1 + b2 + + bk+1 )2 1.4. THE CYH TECHNIQUES 75 Chọn bk = k; ta có ck k3 = k P 2 + k 2 (k + 1) k+1 P i i=1 = 4k 2 = 4k 2 < 4k 2 = 4k 2 = 4k 2 2 + + i k2 n n P i 2 i=1 i=1 1 1 1 + + + k(k + 1)2 (k + 1)(k + 2)2 n(n + 1)2 1 1 1 1 + + k(k + 1) n(n + 1) (k + 1)2 (n + 1)2 1 1 1 1 1 1 1 1 + + + + 2 k2 (k + 1)2 2 n2 (n + 1)2 (k + 1)2 (n + 1)2 1 1 1 1 1 1 + + + + 2 2k 2 2(n + 1)2 (k + 1)2 n (k + 1)2 (n + 1)2 1 1 < 2 8k = 1; n 2k 2 2(n + 1)2 Nên từ đây hiển nhiên ta có bất đẳng thức cần chứng minh đúng. Ví dụ 1.53 Cho các số thực a1 ; a2 ; :::; an : Chứng minh rằng 4(a21 + a22 + + a2n ) a21 + 2 a1 + a2 2 + + a1 + a2 + n + an 2 : Lời giải. Sử dụng bất đẳng thức Cauchy Schwarz, 8k = 1; n; bi > 0 8i = 1; n ta có a21 a2 + 2+ b1 b2 ) + a1 + a2 + k a2k bk + ak ) n X k=1 (b1 + b2 + 2 + bk ) b 1 + b2 + k2 a1 + a2 + k + ak + bk 2 (a1 + a2 + a21 a2 + 2+ b1 b2 n X + ak )2 + a2k bk ck a2k k=1 Với ck = b 1 + b2 + + bk b 1 + b2 + + bk+1 + + k 2 bk (k + 1)2 bk http://boxtailieu.net + b 1 + b2 + + bn n2 bk 76 CHƯƠNG 1. TÌM TÒI MỘT SỐ KỸ THUẬT GIẢI TOÁN Chọn bk = ck = = = = p k p k + 1; ta có b1 + b 2 + + bk b1 + b2 + + bk+1 b1 + b 2 + + bn + + + k 2 bk (k + 1)2 bk n 2 bk 1 1 1 1 p + + + 3=2 p (k + 1)3=2 n k k 1 k 3=2 2 0 1 0 13 1 1 1 1 1 42 @ q A+ A5 p q q + 2 @q p 1 1 1 1 k k 1 k 2 k+ 2 n 2 n+ 2 1 0 2 1 1 A< q @q 1 p q p p p k k 1 k 12 n + 12 k 12 k k 1 p p p 2 2 k+ k 1 p 4 2k 1 Nên bất đẳng thức hiển nhiên đúng. 0; x3 + y 3 = 1: Chứng minh rằng r 5 p p p 6 5 x+2 y 1+2 2 : Ví dụ 1.54 Cho các số x; y Ví dụ 1.55 Cho các số a; b; c biểu thức 0; a2 + b2 + c2 = 1: Tìm giá trị lớn nhỏ nhất của P = a3 + 3b3 + 2c3 : Hướng dẫn. Dùng bất đẳng thức Holder (a3 + 3b3 + 2c3 )2 (m3 + n3 + p3 ) Ví dụ 1.56 Cho các số a; b; c p p 3 3 a2 m + b2 n 9 + c2 p 4 3 : 0; a + b + c = 3: Tìm giá trị nhỏ nhất của biểu thức P = a4 + 2b4 + 4c4 : Hướng dẫn. Dùng bất đẳng thức Holder (a4 + 2b4 + 3c4 )3 (m4 + n4 + p4 ) p p 4 4 a3 m + b3 n 8 + c3 p 27 http://boxtailieu.net 4 : 1.4. THE CYH TECHNIQUES 77 Ví dụ 1.57 Cho các số thực x1 ; x2 ; :::; xn : Chứng minh rằng x21 + (x1 + x2 )2 + 1 + xn )2 + (x1 + x2 + 4 sin2 (x21 + x22 + + x2n ): 2(2n+1) Hướng dẫn. Dùng bất đẳng thức Cauchy Schwarz, n k X X k=1 i=1 xi !2 n X k=1 " k X ci i=1 ! k X x2 i ci i=1 !# = n X k=1 " Sk k X x2 i i=1 ci !# Chọn các số ci sao cho S1 + S2 + c1 + Sn = ) ci = sin S2 + i 2n + 1 + Sn c2 sin = = Sn =k cn (i 1) : 2n + 1 Ví dụ 1.58 Cho các số a; b; c r P = 0; a + b + c = 1: Tìm giá trị nhỏ nhất của biểu thức r r 1 1 1 a2 + 2 + b2 + 2 + c2 + 2 : b c a Hướng dẫn. Dự đoán đẳng thức Cauchy Schwarz, s 1 a2 + 2 b s 1 b2 + 2 c s 1 c2 + 2 a xảy ra tại a = b = c = 31 : Sử dụng bất đẳng thức (m2 + n2 ) ma + n b (m2 + n2 ) mb + n c (m2 + n2 ) mc + n : a "Số không âm" Đối với các bất đẳng thức dạng ab11 + ab22 + + abnn k mà ta chưa chắc ai 0 8i = 1; n hay không, chỉ biết rằng bi 0 8i = 1; n thôi thì việc thêm vào là một kỹ thuật cần http://boxtailieu.net 78 CHƯƠNG 1. TÌM TÒI MỘT SỐ KỸ THUẬT GIẢI TOÁN thiết. Ý tưởng đơn giản như sau, cộng các tham số mi vào tuong ?ng ai + mi bi 0 8i = 1; n; khi đó đưa bất đẳng thức về dạng n X a0 i i=1 k+ bi n X ai bi sao cho mi i=1 Với a0i ; bi 0 8i = 1; n: Từ đây ta có thể sử dụng bất đẳng thức Cauchy SchwarzHolder một cách tự nhiên mà không phải lo ngại nữa. Ví dụ 1.59 Cho các số thực a; b; c; d thỏa mãn a2 + b2 + c2 + d2 = 1: Chứng minh rằng 1 1 1 16 1 + + + : 1 ab 1 bc 1 cd 1 da 3 (Võ Quốc Bá Cẩn) Lời giải. Trước hết, ta hãy viết bất đẳng thức về dạng X 1 1 ab X k(1 1 ab) ab cyc , Vì 1 = a2 + b2 + c2 + d2 cyc a2 + b2 k(1 ab) 16 3 1 4k 16 3 2ab, suy ra 1 1 2 k 1 1= k 2 1 Ta cần chọn k > 0 sao cho k(1 ab) 1 0. Từ trên, ta chỉ cần chọn k 2, và với k = 2, bất đẳng thức k(1 ab) 1 0 có đẳng thức tại a = b = p12 ; c = d = 0. Do đó ta hãy thử sử dụng Cauchy Schwarz với k = 2. Bất đẳng thức trở thành X1 cyc 1 2ab ab 8 3 Sử dụng bất đẳng thức Cauchy Schwarz, ta có X1 cyc 2ab 1 ab " P P (1 #2 2ab) cyc (1 2ab)(1 ab) = 4 4[2 (a + c)(b + d)]2 3(a + c)(b + d) + 2(a2 + c2 )(b2 + d2 ) cyc http://boxtailieu.net 1.4. THE CYH TECHNIQUES 79 Do đó, ta chỉ cần chứng minh được 3[2 (a + c)(b + d)]2 , 3(a + c)2 (b + d)2 , 3[1 3(a + c)(b + d) + 2(a2 + c2 )(b2 + d2 )] 2[4 6(a + c)(b + d) + 4 2 (a + c)(b + d)] + 1 4(a2 + c2 )(b2 + d2 ) 2 2 2 2 4(a + c )(b + d ) 0 0 Sử dụng bát đẳng thức AM-GM, ta có 4(a2 + c2 )(b2 + d2 ) (a2 + b2 + c2 + d2 )2 = 1: Bất đẳng thức được chứng minh. Đẳng thức xảy ra khi và chỉ khi a = b = c = d = 1 2. Ví dụ 1.60 Cho các số thực a; b; c: Chứng minh rằng a2 bc b2 ca c2 ab + + a2 + 2b2 + 3c2 b2 + 2c2 + 3a2 c2 + 2a2 + 3b2 0: (Nguyễn Anh Tuấn) Lời giải. Viết lại bất đẳng thức như sau X 4(a2 bc) a2 + 2b2 + 3c2 cyc , ,2 X cyc a2 X cyc 0 4(a2 bc) +1 a2 + 2b2 + 3c2 3 X (b c)2 5a2 + c2 + 2 2 2 + 2b + 3c a + 2b2 + 3c2 cyc Vì 2 X cyc a2 (b c)2 + 2b2 + 3c2 3 0 Nên ta chỉ cần chứng minh được X cyc a2 5a2 + c2 + 2b2 + 3c2 3 Nhưng bất đẳng thức này hiển nhiên đúng theo bất đẳng thức Cauchy Schwarz ! ! " #2 X X X 5a2 + c2 2 2 2 2 2 2 2 (5a + c )(a + 2b + 3c ) (5a + c ) a2 + 2b2 + 3c2 cyc cyc cyc !2 X 2 = 36 a cyc http://boxtailieu.net 80 CHƯƠNG 1. TÌM TÒI MỘT SỐ KỸ THUẬT GIẢI TOÁN và 12 X cyc a2 !2 X (5a2 + c2 )(a2 + 2b2 + 3c2 ) = 4 cyc X a4 4 cyc X a2 b2 0: cyc Đẳng thức xảy ra khi và chỉ khi a = b = c: Ví dụ 1.61 Cho các số không âm a; b; c thỏa a + b + c = 3: Chứng minh rằng 5 3ab 5 3bc 5 3ca + + 1+c 1+a 1+b ab + bc + ca: (Vasile Cirtoaje) Lời giải. Bất đẳng thức tương đương với X 9(5 3ab) 1+c cyc , X 9(5 ,5 cyc X ab cyc 3ab) + 7ab(1 + c) 1+c cyc X9 9 16 X ab cyc X 1 4ab + 7abc 1+c 1+c cyc 16 X ab cyc Sử dụng bất dẳng thức AM-GM, ta có 9 = (a + b + c)2 (a + b)2 4ab Do đó, sử dụng bất đẳng thức Cauchy Schwarz, ta được !2 " #2 P P 27 4 ab (9 4ab) X 9 4ab cyc cyc P P = 1+c (9 4ab)(1 + c) 54 4 ab 12abc cyc cyc và cyc X cyc 1 1+c P cyc Ta cần chứng minh 5 27 54 4 4 P 9 3 = (1 + c) 2 P cyc ab !2 ab 12abc + 21 abc 2 cyc http://boxtailieu.net 16 X cyc ab 1.4. THE CYH TECHNIQUES 81 n max 0; (4q Đặt q = ab+bc+ca; r = abc, theo bất đẳng thức Schur bậc 4, ta có r Bất đẳng thức trở thành 5(27 4q)2 21 + r 54 4q 12r 2 Nếu 9 0 4q thì 21 5(27 4q)2 + r 54 4q 12r 2 Nếu 4q 16q 9)(9 q) 18 5(27 54 16q 4q)2 4q 16q = 9(9 4q)(45 4q) 2(27 2q) 0 9 thì 5(27 4q)2 21 + r 54 4q 12r 2 16q 21(4q 9)(9 5(27 4q)2 + 2 36 54 4q 3 (4q 9)(9 q) (4q 9)(3 q)(36 7q) = 0: 12(6 q) q) 16q Đẳng thức xảy ra khi và chỉ khi a = b = c = 1 hoặc a = b = 32 ; c = 0 hoặc các hoán vị tương ứng. Ví dụ 1.62 Cho a; b; c là độ dài 3 cạnh của một tam giác. Chứng minh rằng b(b c) c(c a) a(a b) + + a2 + 2bc b2 + 2ca c2 + 2ab 0: (Võ Quốc Bá Cẩn) Lời giải. Bất đẳng thức tương đương X a(a b) +1 a2 + 2bc cyc , X 2a2 cyc a2 ab + 2bc + 2bc Do a; b; c là độ dài 3 cạnh của một tam giác nên c 2a2 ab + 2bc 2a2 ab + 2b(b 3 3 b a) = 2(a http://boxtailieu.net a, từ đó b)2 + ab 0 o . 82 CHƯƠNG 1. TÌM TÒI MỘT SỐ KỸ THUẬT GIẢI TOÁN Suy ra, theo bất đẳng thức Cauchy Schwarz, ta có X 2a2 ab + 2bc 2 a + 2bc cyc P " P 2 (2a #2 ab + 2bc) cyc (2a2 ab + 2bc)(a2 + 2bc) cyc Ta cần chứng minh " X 2 (2a #2 cyc ,7 X a3 b + 4 cyc 3 ab + 2bc) X ab3 X (2a2 ab + 2bc)(a2 + 2bc) cyc 2 cyc X a4 + 3 cyc X a2 b2 + 6 cyc X a2 bc cyc Lại doa; b; c là độ dài 3 cạnh của một tam giác nên tồn tại x; y; z > 0 sao cho a = y + z; b = z + x; c = x + y. Bất đẳng thức trở thành X X X X X 2 x4 + 2 xy(x2 + y 2 ) + 3 xy 3 6 x2 y 2 + 3 x2 yz cyc cyc cyc cyc cyc Nhưng bất đẳng thức này hiển nhiên đúng theo bất đẳng thức AM-GM vì X X X X X 2 x4 + 2 xy(x2 + y 2 ) 6 x2 y 2 ; 3 xy 3 3 x2 yz: cyc cyc cyc cyc cyc Đẳng thức xảy ra khi và chỉ khi a = b = c: Ví dụ 1.63 Cho các số không âm a; b; c; không có 2 số nào đồng thời bằng 0: Chứng minh rằng 2a2 bc 2b2 ca 2c2 ab + + 3: b2 bc + c2 c2 ca + a2 a2 ab + b2 (Vasile Cirtoaje) Lời giải. Viết lại bất đẳng thức như sau X cyc , 2a2 bc +1 b2 bc + c2 X 2a2 + (b cyc b2 c)2 bc + c2 6 6 http://boxtailieu.net 1.4. THE CYH TECHNIQUES 83 Sử dụng bất đẳng thức Cauchy Schwarz, ta có X 2a2 + (b cyc c)2 bc + c2 b2 P " P 2 #2 2 [2a + (b c) ] cyc [2a2 + (b c)2 ](b2 bc + c2 ) cyc Ta cần chứng minh " X 2 [2a + (b 2 c) ] cyc ,2 X a4 + 2abc cyc ,2 #2 6 X a2 (a c)2 ](b2 bc + c2 ) cyc X a+ cyc X [2a2 + (b X ab(a2 + b2 ) 6 cyc b)(a c) + 3 cyc X a2 b2 cyc X ab(a b)2 0: cyc Bất đẳng thức cuối hiển nhiên đúng theo bất đẳng thức Schur bậc 4. Đẳng thức xảy ra khi và chỉ khi a = b = c hoặc a = b; c = 0 hoặc các hoán vị tương ứng. Ví dụ 1.64 Cho các số không âm a; b; c; tất cả không đồng thời bằng 0: Chứng minh rằng 3a2 bc 3b2 ca 3c2 ab + + 2a2 + b2 + c2 2b2 + c2 + a2 2c2 + a2 + b2 3 : 2 (Vasile Cirtoaje) Lời giải. Viết lại bất đẳng thức như sau X 3 cyc , ,3 X cyc 2(3a2 bc) 2a2 + b2 + c2 X 3b2 + 2bc + 3c2 cyc 2a2 + b2 + c2 6 6 X (b c)2 bc +8 2 2 2 + b2 + c2 +b +c 2a cyc 2a2 http://boxtailieu.net 6 84 CHƯƠNG 1. TÌM TÒI MỘT SỐ KỸ THUẬT GIẢI TOÁN Nếu (a b)2 + (b c)2 + (c a)2 = 0, bất đẳng thức là hiển nhiên. Nếu (a (b c)2 + (c a)2 > 0; khi đó theo bất đẳng thức Cauchy Schwarz, ta có !2 P 2 (b c) X (b c)2 cyc P 2a2 + b2 + c2 (b c)2 (2a2 + b2 + c2 ) cyc cyc P 4 = " P a2 cyc P c)2 (b cyc # !2 ab cyc P a2 cyc P 2 ! + P P a2 cyc ! ab cyc P a2 (b P a2 !2 ab cyc P a2 cyc c)2 cyc cyc = b)2 + ! + P P b2 c2 cyc a2 bc cyc và X cyc bc 2a2 + b2 + c2 P P !2 P bc cyc bc(2a2 + b2 + c2 ) = cyc P bc cyc ! P bc cyc !2 a2 cyc ! + P a2 bc cyc Ta cần chứng minh 6 P P 2 a cyc P cyc a2 P cyc ! ab !2 ab cyc P a2 cyc ! + P b2 c2 cyc P 8 P + a2 bc cyc P P bc cyc bc cyc ! !2 a2 cyc ! + Do tính thuần nhất, ta có thể chuẩn hóa cho a + b + c = 1. Đặt q = P cyc P cyc khi đó bất đẳng thức trở thành (1 3(1 3q)2 3q)(1 2q) + q 2 , (1 3r + 4q 2 q(1 2q) + r (3r + q 4q 2 )2 5q + 7q 2 3r)(q 2q 2 + r) http://boxtailieu.net 0: 3 6 a2 bc bc; r = abc, 1.4. THE CYH TECHNIQUES 85 hiển nhiên đúng. Đẳng thức xảy ra khi và chỉ khi a = b = c hoặc a b c 1 = 0 = 0 hoặc các hoán vị tương ứng. a 1 = b 1 = c 0 hoặc Ví dụ 1.65 Cho các số dương a; b; c: Chứng minh rằng p a2 bc b2 ca c2 ab +p +p a2 + 2b2 + 3c2 b2 + 2c2 + 3a2 c2 + 2a2 + 3b2 0: (Nguyễn Anh Tuấn) Lời giải. Sử dụng bất đẳng thức Cauchy Schwarz, ta có X cyc " 8(a2 # bc) p +b+c 6(a2 + 2b2 + 3c2 ) = X 8(a2 cyc X 8(a2 cyc p bc) + (b + c) 6(a2 + 2b2 + 3c2 ) p 6(a2 + 2b2 + 3c2 ) bc) + (b + c)(a + 2b + 3c) p 6(a2 + 2b2 + 3c2 ) X 8a2 + c2 + ab + bc + ca + 2(b p = 6(a2 + 2b2 + 3c2 ) cyc c)2 X 8a2 + c2 + ab + bc + ca p 6(a2 + 2b2 + 3c2 ) cyc Nên ta chỉ cần chứng minh được X 8a2 + c2 + ab + bc + ca p a2 + 2b2 + 3c2 cyc p X 2 6 a cyc Sử dụng bất đẳng thức Holder, ta có X 8a2 + c2 + ab + bc + ca p a2 + 2b2 + 3c2 cyc !2 " X 2 2 2 2 2 # (8a + c + ab + bc + ca)(a + 2b + 3c ) cyc 27 3 X cyc 2 a + X cyc http://boxtailieu.net !3 ab 86 CHƯƠNG 1. TÌM TÒI MỘT SỐ KỸ THUẬT GIẢI TOÁN Do tính thuần nhất, ta có thể chuẩn hóa cho a+b+c = 1. Đặt q = ab+bc+ca; r = abc, khi đó theo bất đẳng thức Schur bậc 3, ta có r 4q9 1 . Từ đây, ta được X (8a2 + c2 + ab + bc + ca)(a2 + 2b2 + 3c2 ) cyc X = 6 ! ab cyc X = 6 và 3 2 a cyc ! X ab cyc = 53q 2 X a2 cyc 38q + 11 X a2 + cyc X ! + X (8a2 + c2 )(a2 + 2b2 + 3c2 ) cyc ! + 11 X a2 cyc 42r ab = 3 !2 + 21 X a2 b2 cyc 5q cyc Ta phải chứng minh 5q)3 27(3 911q + 1601q 2 , 155 Ta có 155 24(53q 2 911q + 1601q 2 = 38q + 11 42r) 1125q 3 + 336r 0 1125q 3 + 336r 155 911q + 1601q 2 1 (1 3 3q)(1125q 2 4q 1125q 3 + 336 1226q + 353) 1 9 0: Đẳng thức xảy ra khi và chỉ khi a = b = c: Ví dụ 1.66 Cho a; b; c là độ dài 3 cạnh của một tam giác. Chứng minh rằng a(b + c) b(c + a) c(a + b) + + a2 + 2bc b2 + 2ca c2 + 2ab 2: (Võ Quốc Bá Cẩn) Lời giải. Bất đẳng thức tương đương với X 1 a(b + c) a2 + 2bc X a(a b) + (2b a2 + 2bc cyc , cyc 1 a)c http://boxtailieu.net 1 1.4. THE CYH TECHNIQUES 87 Ta sẽ chỉ ra rằng a(a Thật vậy, nếu 2b a(a Nếu a b) + (2b a)c 0 a, ta có b) + (2b a)c a(a b) + (2b a)(b b)2 a) = 2(a 0 2b, ta có a(a b) + (2b a)c a(a a)(a + b) = 2b2 b) + (2b 0 Từ đây, sử dụng bất đẳng thức Cauchy Schwarz, ta được #" " # X a(a b) + (2b a)c X 2 [a(a b) + (2b a)c](a + 2bc) a2 + 2bc cyc cyc " #2 X [a(a b) + (2b a)c] cyc Ta cần chứng minh " X [a(a b) + (2b #2 X a)c] cyc , X [a(a b) + (2b a)c](a2 + 2bc) cyc b)2 ab(a 0: cyc hiển nhiên đúng. Đẳng thức xảy ra khi và chỉ khi a = b = c hoặc a = b; c = 0 hoặc các hoán vị tương ứng. Ví dụ 1.67 Cho a; b; c là độ dài 3 cạnh của một tam giác. Chứng minh rằng b c ab + bc + ca a + + + b + c c + a a + b a2 + b2 + c2 5 : 2 (Phạm Kim Hùng) Lời giải. Bất đẳng thức tương đương với X 1 cyc , P ab 1 cyc +P 2 2 a a b+c cyc Xb+c cyc b+c a P a cyc 2 !2 P a2 cyc http://boxtailieu.net 88 CHƯƠNG 1. TÌM TÒI MỘT SỐ KỸ THUẬT GIẢI TOÁN Sử dụng bất đẳng thức Cauchy Schwarz, ta có ! ! " # ! X Xb+c a X Xb+c a 2 2 a = (b + c)(b + c a) b+c b+c cyc cyc cyc cyc " #2 !2 X X (b + c a) = a : cyc cyc Đẳng thức xảy ra khi và chỉ khi a = b = c hoặc a = b; c = 0 hoặc các hoán vị tương ứng. Ví dụ 1.68 Cho các số không âm a; b; c; d; không có 3 số nào đồng thời bằng 0: Chứng minh rằng ab bc cd da + + + a+b+c b+c+d c+d+a d+a+b 1 (a + b + c + d): 3 (Park Doo Sung) Lời giải. Bất đẳng thức tương đương với X 4ab a+b+c a+b cyc , X ac + bc + (a b)2 a+b+c cyc 2X a 3 cyc 2X a 3 cyc Sử dụng bất đẳng thức Cauchy Schwarz, ta có " #2 P [ac + bc + (a b)2 ] X ac + bc + (a b)2 cyc P a+b+c [ac + bc + (a b)2 ](a + b + c) cyc cyc Ta có X [ac + bc + (a cyc X [ac + bc + (a b)2 ] = X a2 + (a + c)2 + (b + d)2 (a + c)(b + d) cyc b)2 ](a + b + c) = 2 cyc X a3 + cyc 3 X X a2 b + 3ac(a + c) + 3bd(b + d) cyc 3 a + 3ac(a + c) + 3bd(b + d) cyc = 3(a + c)(a2 + c2 ) + 3(b + d)(b2 + d2 ) http://boxtailieu.net 1.4. THE CYH TECHNIQUES 89 Nên ta chỉ cần chứng minh được " P #2 a2 + (a + c)2 + (b + d)2 (a + c)(b + d) cyc 2 (a + c)(a2 + c2 ) + (b + d)(b2 + d2 ) Không mất tính tổng quát, ta có thể giả sử b + d X a cyc a + c. Đặt 2x = a + c 2y = b + d (a + c)2 = 2x2 2 t = a2 + c2 A = b2 + d2 + (a + c)2 + (b + d)2 (a + c)(b + d) = b2 + d2 + 4x2 + 4y 2 B = (b + d)(b2 + d2 ) = 2y(b2 + d2 ) Bất đẳng thức trở thành f (t) = (t + A)2 2xt + B Ta có f 0 (t) = và xt + B 2(2x + 2y) 2(t + A)(xt + B xA) (2xt + B)2 xA = xt + 2y(b2 + d2 ) 3 2 x(b2 + d2 + 4x2 + 4y 2 2 2x + 2y(b + d ) 2 2 = (b + d )(2y 2 2y (2y = 2(y Nên f 0 (t) 0 2 2 3 x) + 4x y x) + 4x y x)(2y 2 x(b + d + 4x + 4y 2 2 2 2x 3 2x 2 xy + x ) 4xy 4xy 4xy) 2 4xy) 2 2 0 0. Do đó f (t) đồng biến, và vì thế ta chỉ cần chứng minh được f (2x2 ) = Ta có g 0 (bd) = (3x2 (3x2 2xy + 4y 2 bd)2 x3 ybd + 2y 3 4(x + y) = g(bd) 2xy + 4y 2 bd)[ybd x(2x2 (x3 ybd + 2y 3 )2 g 0 (bd) = 0 , bd = x(2x2 3xy + 2y 2 )] 3xy + 2y 2 ) y http://boxtailieu.net 0 4xy 90 Do x CHƯƠNG 1. TÌM TÒI MỘT SỐ KỸ THUẬT GIẢI TOÁN y nên x(2x2 3xy+2y 2 ) y g(bd) g y 2 , khi đó ta dễ dàng kiểm tra được x(2x2 3xy + 2y 2 ) y = x)3 4(y y2 0: Bất đẳng thức được chứng minh. Đẳng thức xảy ra khi và chỉ khi a = b = c = d: 1 2 Ví dụ 1.69 Cho các số a1 ; a2 ; a3 thỏa a1 a2 a3 = 1: Chứng minh rằng 1 1 1 + + a1 + 2a2 a2 + 2a3 a3 + 2a1 Lời giải. Đặt a1 = xy ; a2 = xz ; a3 = y z 1: với x; y; z > 0 và do a1 ; a2 ; a3 8 1; x1 + p x 1+ p y 1 y 1+ + p 1 z z = 2: Chứng minh rằng p 1 x + y + z: Lời giải. Với bài toán này, thông thường chúng ta sẽ áp dụng Cauchy Schwarz theo lối tự nhiên là p p p p x 1+ y 1+ z 1 3(x + y + z 3) Rồi đi đến việc chứng minh p 3(x + y + z 3) ,x+y+z p x+y+z 9 : 2 Nhưng bất đẳng thức này lại ngược chiều vì cũng theo bất đẳng thức Cauchy Schwarz, ta có 9 9 x+y+z 1 1 1 = 2: + + x y z Do đó lối đi này không có hiệu quả, chúng ta nảy sinh ý tưởng thêm các tham số vào để sử dụng bất đẳng thức Cauchy Schwarz như sau r r r p p p x 1 y 1 z 1 x 1+ y 1+ z 1 = a + b + c a b c s x 1 y 1 z 1 (a + b + c) + + a b c Từ đây, nếu ta để ý đến điều kiện bài toán một tí, ta có thể chọn được y; c = z và khi đó s s x 1 y 1 z 1 x 1 y (a + b + c) + + = (x + y + z) + a b c x y s 1 1 = (x + y + z) 3 x y p = x+y+z http://boxtailieu.net a = x; b = 1 1 z + z 1 z 1.4. THE CYH TECHNIQUES 115 Bài toán được giải. Đẳng thức xảy ra khi và chỉ khi x = y = z = 23 : Do các biểu thức dạng tuyến tính ma + nb + pc; mb + nc + pa; mc + na + pb dễ dàng chọn được các giá trị của m; n; p hơn các biểu thức khác nên ta thường dùng chúng để giải, nhưng đôi khi trong một vài trường hợp việc sử dụng chúng không mang lại hiệu quả mà ta phải sử dụng các biểu thức phụ khác (việc chọn các biểu thức này không có mẫu mực mà phần lớn dựa vào kinh nghiệm của người làm toán) Ví dụ 1.95 Cho các số không âm a; b; c; không có 2 số nào đồng thời bằng 0: Chứng minh rằng p 1 1 1 +p +p a2 + bc b2 + ca c2 + ab p 2 1 1 1 + + b+c c+a a+b : (Võ Quốc Bá Cẩn) Lời giải. Sử dụng bất đẳng thức Cauchy Schwarz, ta có !2 " #" # X X (a + b)(a + c) X 1 1 p a2 + bc (a + b)(a + c) a2 + bc cyc cyc cyc ! P ! 2 a X a(b + c) cyc +3 = (a + b)(b + c)(c + a) cyc a2 + bc Ta cần chứng minh 2 P cyc ! a X a(b + c) (a + b)(b + c)(c + a) , , , X a(b + c) cyc a2 + bc X a(b + c) cyc X cyc (a a2 + bc b)(a cyc +3 a2 + bc P ! X +3 a2 + 3 cyc cyc P 1 a+b !2 ab cyc (a + b)(b + c)(c + a) P cyc 3 P cyc a4 P a2 b2 cyc (a + b)(b + c)(c + a) P cyc c) 1 1 + a2 + bc (b + c)(a + b + c) http://boxtailieu.net !2 ! a ! a 0 116 CHƯƠNG 1. TÌM TÒI MỘT SỐ KỸ THUẬT GIẢI TOÁN Không mất tính tổng quát, giả sử a (c a)(c b) b c 0)a a b (b c c) 1 1 + c2 + ab (a + b)(a + b + c) 0: Khi đó 0 và 1 1 + + bc (b + c)(a + b + c) 1 1 +(b a)(b c) 2 + b + ca (c + a)(a + b + c) (a b)(b c) a a b + 2 2 b a + bc (b + c)(a + b + c) b + ca c(a b)2 (a + b)(b c)[(a b)2 + ab + bc + ca] = 0: (a2 + bc)(b2 + ca)(b + c)(c + a)(a + b + c) (a b)(a c) a2 b (c + a)(a + b + c) Vậy ta có đpcm. Đẳng thức xảy ra khi a = b = c: Ví dụ 1.96 Cho các số không âm a; b; c; không có 2 số nào đồng thời bằng 0: Chứng minh rằng p p p 3(a + b + c) 2 a2 + bc + b2 + ca + c2 + ab : (Phạm Kim Hùng) Lời giải. Sử dụng bất đăng thức Cauchy Schwarz, ta có !2 " # X X Xp 2 2 a + bc (3a + 4bc + ab + ac) cyc cyc = X 3 a cyc Ta cần chứng minh ,8 X a4 (b2 + c2 ) cyc !2 X cyc 3a2 ! a2 + bc + 4bc + ab + ac X a2 + bc 3 2 + 4bc + ab + ac 3a 4 cyc X X X 16 a3 b3 11abc a3 + 43abc a2 (b + c) + 18a2 b2 c2 cyc Y , 8 (a cyc cyc a2 + bc 2 3a + 4bc + ab + ac ! 2 b) + abc cyc X cyc ! a cyc 5 X 2 a + 22 cyc X cyc ! ab 0 0: Bất đẳng thức cuối hiển nhiên đúng. Đẳng thức xảy ra khi và chỉ khi a = b; c = 0 hoặc các hoán vị tương ứng. http://boxtailieu.net 1.4. THE CYH TECHNIQUES 117 Nhận xét 14 Ý tưởng của lời giải này như sau Chúng ta thấy là đẳng thức xảy ra khi và chỉ khi a = b; c = 0, khi đó có một điểm đặc biệt là a2 + bc = b2 + ca = c2 + ab Do đó, khi ta dùng Cauchy Schwarz để khử căn !2 Xp a2 + bc " cyc X #" g(a; b; c) cyc X a2 + bc g(a; b; c) cyc # Ta cần chọn g(a; b; c); g(b; c; a); g(c; a; b) sao cho đẳng thức trong bất đẳng thức này cũng đạt tại a = b; c = 0. Với chú ý trên, ta thấy là nếu ta chọn g(a; b; c) có dạng a2 + bc + k(một đại lượng đối xứng với a; b; c) thì rõ ràng đẳng thức ban đầu vẫn được đảm bảo (các đại lượng đối xứng này càng đơn giản càng tốt, sẽ thuận lợi hơn cho chúng ta trong việc chứng minh bất đẳng thức sau, chúng ta có thể chọn các đại lượng như (a + b + c)2 ; a2 + b2 + c2 ; ab + bc + ca). Ngoài ra, ta thấy bên vế trái của bất 2 đẳng thức ban đầu có sự xuất hiện của (a + b + Pc) nên bất đẳng thức sau khi sử dụng2 g(a; b; c) cũng có dạng m(a + b + c) Cauchy Schwarz sẽ dễ chứng minh hơn nếu cyc từ đó, ta dễ dàng thấy được một trường hợp hiển nhiên thỏa là k = 31 và đại lượng đối xứng thêm vào là ab + bc + ca: Từ đó dẫn đến lời giải khá đặc sắc như trên. Ví dụ 1.97 Cho các số không âm a; b; c; không có 2 số nào đồng thời bằng 0: Chứng minh rằng (b + c)2 (c + a)2 (a + b)2 + + 6: a2 + bc b2 + ca c2 + ab (Darij Grinberg) Lời giải. Sử dụng bất đẳng thức Cauchy Schwarz, ta có " X (b + c)2 cyc a2 + bc #" X 2 # " !2 3 2 (b + c) (a + bc) cyc X 2 (b + c) cyc #2 =4 X 2 a + cyc cyc Ta cần chứng minh 2 X 2 a + cyc ,2 X cyc X cyc a4 + 2abc X cyc ab a+ X X (b + c)2 (a2 + bc) cyc ab(a2 + b2 ) cyc http://boxtailieu.net 6 X cyc a2 b2 X 0 !2 ab 118 CHƯƠNG 1. TÌM TÒI MỘT SỐ KỸ THUẬT GIẢI TOÁN Bất đẳng thức này được suy ra từ bất đẳng thức Schur bậc 4 X X X 2 a4 + 2abc a 2 ab(a2 + b2 ) cyc cyc và 3 X cyc ab(a2 + b2 ) 6 cyc X a2 b2 : cyc với bất đẳng thức cuối hiển nhiên đúng theo bất đẳng thức AM-GM. Đẳng thức xảy ra khi và chỉ khi a = b = c hoặc a = b; c = 0 hoặc các hoán vị tương ứng. Ví dụ 1.98 Cho các số không âm a; b; c thỏa a + b + c = 1: Chứng minh rằng a+b b+c c+a + + ab + 1 bc + 1 ca + 1 9 : 5 (Michael Rozenberg) Lời giải. Bất đẳng thức tương đương với X (1 + a)(1 + b) cyc 1 + ab , (1 + a)(1 + b)(1 + c) X cyc 24 5 1 (1 + c)(1 + ab) Sử dụng bất đẳng thức Cauchy Schwarz, ta có #" # " X X 1 2 (3 c) (1 + c)(1 + ab) (1 + c)(1 + ab) cyc cyc " 24 5 X (3 cyc Ta cần chứng minh 40(1 + a)(1 + b)(1 + c) 3 X (3 c)2 (1 + c)(1 + ab) cyc Đặt q = P ab; r = abc. Ta có cyc (1 + a)(1 + b)(1 + c) = 2 + q + r X (3 c)2 (1 + c)(1 + ab) = (8 2q)r + 26 + 16q cyc Bất đẳng thức trở thành (16 + 6q)r + 2 8q 0 http://boxtailieu.net #2 c) = 64 1.4. THE CYH TECHNIQUES 119 Nếu 1 4q, bất đẳng thức là hiển nhiên. Nếu 4q q) bậc 4, ta có r (4q 1)(1 , do đó 6 (16 + 6q)r + 2 8q (16 + 6q) = 1 (4q 3 (4q 1)(1 Đẳng thức xảy ra khi và chỉ khi a = b = c = vị tương ứng. 1 3 1, sử dụng bất đẳng thức Schur 1)(1 6 q) +2 3q)(q + 2) 8q 0: hoặc a = b = 12 ; c = 0 hoặc các hoán Ví dụ 1.99 Cho các số không âm a; b; c thỏa a + b + c = 1: Chứng minh rằng với p 3 k=1 , ta có 2 p p p p a + k(b c)2 + b + k(c a)2 + c + k(a b)2 3: (Phan Thành Nam) Lời giải. Sử dụng bất đẳng thức Cauchy Schwarz, ta có v 2 32 u p ! Xu 3 ta + 1 4 (b c)2 5 2 cyc " # #" p X Xa+ 1 32 (b c)2 1 a+ p a + p13 3 cyc cyc " # p X p a 2 3 X (b c)2 3+1 + = 2 a + p1 a + p1 cyc cyc 3 3 Ta cần chứng minh p 3+1 " X cyc # p a 2 3 X (b c)2 + 2 a + p13 a + p13 cyc 3 Đặt q = ab + bc + ca =; r = abc, khi đó ta có q 2 3r. Bất đẳng thức trở thành p p 9 2 + 3 r q 6q + 3 0 Ta có 9 2+ p 3 r q 6q + p 3 3 2+ p 3 q2 Đẳng thức xảy ra khi và chỉ khi a = b = c = vị tương ứng. 1 3 q 6q + p 3 = p 3q(3q 1) 0: hoặc a = 1; b = c = 0 hoặc các hoán http://boxtailieu.net 120 CHƯƠNG 1. TÌM TÒI MỘT SỐ KỸ THUẬT GIẢI TOÁN Ví dụ 1.100 Cho các số dương a; b; c; d thỏa (a + b + c + d) Chứng minh rằng 1 1 1 1 + 2+ 2+ 2 a2 b c d (a2 + b2 + c2 + d2 ) 1 a + 1 b + 1 c + 1 d = 20: 36: (Phạm Kim Hùng) Lời giải. Sử dụng bất đẳng thức Cauchy Schwarz, ta có X 1 a2 cyc !" X (b + c + d a)2 cyc Ta cần chứng minh 4 X a2 cyc # X Xb+c+d a a cyc !2 = 144 a)2 (b + c + d cyc ,0 0: hiển nhiên đúng. Ví dụ 1.101 Cho các số không âm a; b; c; không có 2 số nào đồng thời bằng 0: Chứng minh rằng b+c c+a a+b 6 + + : 2a2 + bc 2b2 + ca 2c2 + ab a+b+c (Vasile Cirtoaje) Lời giải. Sử dụng bất đẳng thức Cauchy Schwarz, ta có X cyc b+c 2a2 + bc !" X 3 # 2 (b + c) (2a + bc) cyc " X 2 (b + c) cyc #2 Ta cần chứng minh 2 X cyc 2 a + X !2 ab cyc X ! a cyc 3 X (b + c)3 (2a2 + bc) cyc Chuẩn hóa cho a + b + c = 1. Đặt q = ab + bc + ca; r = abc, khi đó theo bất đẳng thức Schur bậc 3, ta có r max 0; 4q9 1 . Ta có X cyc 2 a + X cyc !2 ab X cyc ! a = (1 http://boxtailieu.net q)2 1.4. THE CYH TECHNIQUES X 121 (b + c)3 (2a2 + bc) = q + 2q 2 + 12qr 11r cyc Bất đẳng thức trở thành q)2 2(1 , 3(11 Nếu 1 3(q + 2q 2 + 12qr 12q)r + (q + 2)(1 4q, bất đẳng thức là hiển nhiên. Nếu 4q 3(11 12q)r + (q + 2)(1 4q) 4q) 0 1, ta có 3(11 12q) 5 (4q 3 1)(1 = 11r) 4q 1 9 3q) + (q + 2)(1 4q) 0: Đẳng thức xảy ra khi và chỉ khi a = b = c hoặc a = b; c = 0 hoặc các hoán vị tương ứng. Ví dụ 1.102 Cho các số không âm a; b; c thỏa a + b + c = 1: Chứng minh rằng r s X (b c)2 1X a+ 3+ (b c)2 : 4 2 cyc cyc (Phan Thành Việt) Lời giải. Sử dụng bất đẳng thức Cauchy Schwarz, ta có " X cyc r a+ c)2 (b 4 #2 " = X #" (a + 1) cyc X 4a + (b Xa+ (b c)2 4 a+1 cyc c)2 a+1 cyc Ta cần chứng minh X 4a + (b a+1 cyc , X c)2 (b cyc , c)2 X cyc (b 1 2 c)2 3+ 1 a+1 1X (b 2 cyc +3 c)2 X 4a a+1 cyc X 1 3a a 1 + 2(a + 1) cyc 1 + a http://boxtailieu.net 0 0 # 122 CHƯƠNG 1. TÌM TÒI MỘT SỐ KỸ THUẬT GIẢI TOÁN Ta có X1 cyc 3a X b + c 2a X = = (b 1+a 1+a cyc cyc 1 1+c c) 1 1+b = X cyc (b c)2 (b + 1)(c + 1) Do đó, bất đẳng thức tương đương với X cyc , X (b c)2 + (b (b + 1)(c + 1) cyc X c)2 a 1 2(a + 1) 0 1 a 1 + (b + 1)(c + 1) 2(a + 1) X , x(b c)2 0 c)2 (b cyc 0 cyc trong đó x = 2(a + 1) + (a Do tính đối xứng, giả sử a x 2(a + 1) + (a 1 3 (a = 4 1)(b + 1)(c + 1) và y; z tương tự. b c, khi đó a 31 c; 21 b, ta có 1) 1 + 7a2 + 23a 1) 2 b+c 2 = 2(a + 1) + (a 1) 1 + 1 a 2 2 0 y = 2(b + 1) + (b2 1)(a + c b + 1) + (1 2(b + 1) + (b 2 1)(a + c b + 1) = 2(b + 1) + (b 2 1)(2 2b) = 2b(b + 1)(2 b)(a b) b)(b c) 0 y + z = 2(b + c + 2) + (a + 1)[(b 1)(c + 1) + (c 1)(b + 1)] = 2(3 a) + 2(a + 1)(bc 1) 2(3 a) 2(a + 1) = 4(1 X ) x(b c)2 (y + z)(a b)2 0: a) 0 cyc Đẳng thức xảy ra khi và chỉ khi a = b = c = vị tương ứng. 1 3 hoặc a = 1; b = c = 0 hoặc các hoán Nhận xét 15 Chúng ta có một kết quả tương tự như sau r r r (b c)2 (c a)2 (a b)2 a+ + b+ + c+ 4 4 4 Không mất tính tổng quát, giả sử a b c: Xét 2 trường hợp sau Trường hợp 1. b 61 ) a = 1 b c 1 2b 23 ; khi đó ta có r r p (b c)2 2 a+ a 4 3 http://boxtailieu.net 3 2 1.4. THE CYH TECHNIQUES r b+ a)2 (c 4 + r c+ 123 r b)2 (a 4 ) Trường hợp 2. b cyc r a+ b+ r 1 = X p r 4 2 c + (2a 1)2 (3a a+ c)2 (b p 2 + 3 4 r = b 4 r r c)2 (9a b+c+ 4)(3a 12 2) + 7 3 7 > 12 2 1 6; bình phương 2 vế, ta được bất đẳng thức tương đương là s X 1X (b c)2 (a c)2 5 2 (a b) + 2 a+ b+ 4 cyc 4 4 4 cyc Sử dụng bất đẳng thức Cauchy Schwarz, ta có s X p X (a c)2 1 (b c)2 b+ 2 2 ab + j(a a+ 4 4 4 cyc cyc b)(a Nên ta chỉ cần chứng minh 1X (a 4 cyc b)2 + 2 Xp ab + cyc , (a c)2 + 2 1X j(a 2 cyc Xp cyc 2 X 5 4 c)j 5 4 ab cyc Sử dụng bất đẳng thức AM-GM, ta có X Xp p ab = 2 2 (a + b + c) ab cyc b)(a p (a + b) ab 4 cyc X ab cyc Nên ta chỉ cần chứng minh được (a , (a , 1 (a 4 c)2 + 4(ab + bc + ca) b + c)(5b 5 4 c)2 + 4(ab + bc + ca) a c) 0, 1 (a 4 5 (a + b + c)2 4 b + c)(6b Bất đẳng thức cuối hiển nhiên đúng. Vậy ta có đpcm. http://boxtailieu.net 1) 0: c)j (2a b 4 r c)2 7 12 124 CHƯƠNG 1. TÌM TÒI MỘT SỐ KỸ THUẬT GIẢI TOÁN Ví dụ 1.103 Cho các số không âm a; b; c; không có 2 số nào đồng thời bằng 0: Chứng minh rằng p 1 1 1 +p +p 4a2 + bc 4b2 + ca 4c2 + ab p 2 : ab + bc + ca (Võ Quốc Bá Cẩn) Lời giải. Sử dụng bất đẳng thức Holder, ta có X cyc p 1 4a2 + bc !2 " X # (b + c)3 (4a2 + bc) cyc " X #3 (b + c) cyc =8 X !3 a cyc Ta cần chứng minh X 2 X X a cyc , !3 cyc , ab cyc ab(a3 + b3 ) ! X X (b + c)3 (4a2 + bc) cyc a2 b2 (a + b) + 14abc cyc X ab(a cyc X a2 0 cyc b)2 (a + b) + 14abc X a2 0: cyc hiển nhiên đúng. Đẳng thức xảy ra khi và chỉ khi a = b; c = 0 hoặc các hoán vị tương ứng. Ví dụ 1.104 Cho các số không âm a; b; c; tất cả không đồng thời bằng 0: Chứng minh rằng (a + b)2 (b + c)2 (c + a)2 3 + + : a2 + 2b2 + 3c2 b2 + 2c2 + 3a2 c2 + 2a2 + 3b2 2 (Dương Đức Lâm, Võ Quốc Bá Cẩn) Lời giải. Sử dụng bất đẳng thức Cauchy Schwarz, ta có #" # " X X (a + b)2 2 2 2 2 (2a + b) (a + 2b + 3c ) a2 + 2b2 + 3c2 cyc cyc " #2 !2 X X X 2 (a + b)(2a + b) = 9 a + ab cyc http://boxtailieu.net cyc cyc 1.4. THE CYH TECHNIQUES 125 Chú ý rằng X (2a + b)2 (a2 + 2b2 + 3c2 ) cyc = 6 X a4 + 4 cyc 6 X X a3 b + 8 cyc 4 a +8 cyc X X ab3 + 24 cyc 2 2 ab(a + b ) + 24 cyc X X a2 b2 + 12 cyc 2 2 a b + 12 cyc X X a2 bc cyc a2 bc cyc Ta cần chứng minh 3 X cyc 2 a + X cyc !2 ab ,2 X 3 X a4 + 4 cyc ab(a2 + b2 ) X ab(a2 + b2 ) + 12 cyc cyc ,2 X 3 b)2 + cyc X a2 b2 + 6 cyc X a2 b2 + 6 cyc ab(a X X a2 bc X a2 bc cyc 0 cyc a2 b2 + 6 cyc X a2 bc = = 0: cyc hiển nhiên đúng. Đẳng thức xảy ra khi và chỉ khi ứng. a 1 b 0 c 0 hoặc các hoán vị tương Ví dụ 1.105 Cho các số không âm a; b; c; tất cả không đồng thời bằng 0: Chứng minh rằng b2 c2 2 a2 + + : 2a2 + (b + c)2 2b2 + (c + a)2 2c2 + (a + b)2 3 (Darij Grinberg) Lời giải. Bất đẳng thức tương đương với X 1 2a2 2a2 + (b + c)2 X (b + c)2 2a2 + (b + c)2 cyc , cyc 5 3 5 3 Sử dụng bất đẳng thức Cauchy Schwarz, ta có " #" # X X (b + c)2 2 2 2 (a + 3b + 3c) [2a + (b + c) ] 2a2 + (b + c)2 cyc cyc " #2 X (b + c)(a + 3b + 3c) cyc http://boxtailieu.net 126 CHƯƠNG 1. TÌM TÒI MỘT SỐ KỸ THUẬT GIẢI TOÁN Ta cần chứngm inh 3 " X #2 (b + c)(a + 3b + 3c) cyc 2 X (a + 3b + 3c)2 [2a2 + (b + c)2 ] cyc Chuẩn hóa cho a + b + c = 1. Đặt q = ab + bc + ca; r = abc, ta có X (b + c)(a + 3b + 3c) = 6 4q cyc X X (a + 3b + 3c)2 [2a2 + (b + c)2 ] = cyc 2a)2 (3a2 (3 cyc = 12 X a4 44 cyc a3 + 55 cyc 84r + 24q 2 = X 2a + 1) X a2 3 cyc 26q + 20 Bất đẳng thức trở thành 3(6 4q)2 5( 84r + 24q 2 , 210r + (1 4q)(4 + 9q) Nếu 1 4q, bất đẳng thức là hiển nhiên. Nếu 4q bậc 3 ta có r 4q9 1 , do đó 210r + (1 4q)(4 + 9q) 4q 210 1 9 26q + 20) + (1 0 1, sử dụng bất đẳng thức Schur 4q)(4 + 9q) = 1 (4q 3 1)(58 27q) 0: Đẳng thức xảy ra khi và chỉ khi a = b; c = 0 hoặc các hoán vị tương ứng. Ví dụ 1.106 Cho các số không âm a; b; c; tất cả không đồng thời bằng 0: Chứng minh rằng a2 b2 c2 1 + 2 + 2 : 2 2 2 5a + (b + c) 5b + (c + a) 5c + (a + b)2 3 (Võ Quốc Bá Cẩn) Lời giải. Bất đẳng thức tương đương X cyc , X cyc 2 a !" X cyc 1 4a2 5a2 + (b + c)2 5 3 # X 1 bc +2 2 2 2 + (b + c)2 5a + (b + c) 5a cyc http://boxtailieu.net 5 3 1.4. THE CYH TECHNIQUES 127 Sử dụng bất đẳng thức Cauchy Schwarz, ta có " X 1 2 5a + (b + c)2 cyc # P " P #2 (a + 3b + 3c) cyc (a + 3b + 3c)2 [5a2 + (b + c)2 ] cyc P 49 = P !2 a cyc (a + 3b + 3c)2 [5a2 + (b + c)2 ] cyc X cyc bc 2 5a + (b + c)2 P P cyc !2 bc bc[5a2 + (b + c)2 ] = P P cyc bc cyc !2 a cyc Ta cần chứng minh 49 P P cyc a2 ! P !2 a cyc (a + 3b + 3c)2 [5a2 + (b + c)2 ] 2 P bc cyc + P cyc !2 5 3 a cyc Chuẩn hóa cho a + b + c = 1. Đặt q = ab + bc + ca; r = abc, ta có X (a + 3b + 3c)2 [5a2 + (b + c)2 ] = 23 20q + 48q 2 144r cyc Bất đẳng thức trở thành 23 Nếu 1 49(1 2q) + 2q 20q + 48q 2 144r 5 3 4q, ta có 23 49(1 2q) + 2q 20q + 48q 2 144r 5 3 49(1 2q) 5 + 2q 23 20q + 48q 2 3 8(4 3q)(1 4q)(1 + 3q) = 3(23 20q + 48q 2 ) http://boxtailieu.net 0 128 CHƯƠNG 1. TÌM TÒI MỘT SỐ KỸ THUẬT GIẢI TOÁN Nếu 4q 23 4q 1 9 . 1, sử dụng bất đẳng thức Schur bậc 3 ta có r 5 3 49(1 2q) + 2q 20q + 48q 2 144r Suy ra 49(1 2q) 23 20q + 48q 2 16(4q 8(1 3q)(4q 1)(2 q) = 3(13 28q + 16q 2 ) 1) + 2q 5 3 0: Đẳng thức xảy ra khi và chỉ khi a = b = c hoặc a = b; c = 0 hoặc các hoán vị tương ứng. Ví dụ 1.107 Cho các số không âm a; b; c; không có 2 số nào đồng thời bằng 0: Chứng minh rằng r r r p 3 6 a2 + ab + b2 b2 + bc + c2 c2 + ca + a2 + + : c2 + ab a2 + bc b2 + ca 2 (Nguyễn Văn Thạch) Lời giải. Sử dụng bất đẳng thức Holder, ta có X cyc r !" a2 + ab + b2 c2 + ab Ta cần chứng minh X 16 27 X 16 , 27 Do a2 + ab + b2 cyc 3 4 (a 16 27 16 , 27 a X cyc 8 X !3 a cyc cyc a2 + ab + b2 X (a + b)(a2 + 2ab + b2 )(c2 + ab) a !3 a2 + ab + b2 # X (a + b)3 (c2 + ab) a !3 cyc cyc !3 cyc X 16 , 27 X (a + b)3 (c2 + ab) a2 + ab + b2 cyc 2 X ab(a + b) + cyc X ab(a + b)(c2 + ab) cyc a2 + ab + b2 + b)2 , nên ta chỉ cần chứng minh được X cyc !3 a !3 2 2 X a X ab(a + b) + cyc cyc ab(a + b) + 4 X ab(c2 + ab) 3 cyc a+b X c 4 X a2 b2 4 + abc 3 cyc a + b 3 a+b cyc http://boxtailieu.net 1.4. THE CYH TECHNIQUES Lại do 4a2 b2 a+b 16 27 129 ab(a + b), nên ta chỉ cần chứng minh !3 X X X c 1X 4 a 2 ab(a + b) + ab(a + b) + abc 3 cyc 3 a+b cyc cyc cyc 16 , 9 X cyc !3 a 7 X ab(a + b) + 4abc cyc X cyc c a+b Chuẩn hóa cho a + b + c = 1. Đặt q = ab + bc + ca; r = abc, khi đó theo bất đẳng thức 2 q) r (4q 1)(1 . Bất đẳng thức trở thành Schur và bất đẳng thức Newton, ta có q3 6 16 9 7(q 1+q q r 3r) + 4r , f (r) = 297r2 + (52 3 16q + 63q 2 324q)r 0 Vì f (r) là hàm lồi nên f (r) max f q2 3 ;f (4q 1)(1 6 q) Mặt khác, ta có f f (4q 1)(1 6 q2 3 = q) 1 q(3q 3 = 1)(33q 2 1 (3q 12 1)(528q 3 vì 528q 3 280q 2 + 29q + 5 = q 3 = q3 q 3 97q + 48) " 5 29 + 2 q3 q 1 3 q 1 q = q 3 29 3 1 q 0 280q 2 + 29q + 5) 0 280 + 528 q 5 44 + 148 + 84 q2 q ! # 5 44 148 + 84 2 + 1 1 3 3 + 84 3 0: Bất đẳng thức được chứng minh. Đẳng thức xảy ra khi và chỉ khi a = b = c. Ví dụ 1.108 Cho các số không âm a; b; c: Chứng minh rằng a2 b2 c2 + + a2 + b2 + ab + ca b2 + c2 + bc + ab c2 + a2 + ca + bc 3 : 4 (Michael Rozenberg) http://boxtailieu.net 130 CHƯƠNG 1. TÌM TÒI MỘT SỐ KỸ THUẬT GIẢI TOÁN Lời giải. Sử dụng bất đẳng thức Cauchy Schwarz, ta có !" # X X a2 2 2 2 (2a + c) (a + b + ab + ca) a2 + b2 + ab + ca cyc cyc " #2 !2 X X X 2 a(2a + c) = 2 a + ab cyc cyc cyc Nên ta chỉ cần chứng minh 4 2 X 2 a + cyc ,4 X X !2 3 a2 b2 + X ab cyc a4 + 6 cyc X X 2 (2a + c) (a2 + b2 + ab + ca) cyc cyc a3 b 8 cyc X ab3 3abc cyc X a 0 cyc Sử dụng bất đẳng thức Vasile, ta có 8 X 3 ab cyc 8 3 X 8 3 X 2 a cyc !2 Nên ta chỉ cần chứng minh 4 X 4 a +6 cyc X 2 2 a b + cyc , X 3 a b cyc 2 a cyc 4X 4 2X 2 2 X 3 a + a b + a b 3 cyc 3 cyc cyc !2 3abc 3abc X a 0 cyc X a 0 cyc Sử dụng bất đẳng thức AM-GM, ta có 4X 4 a 3 cyc 2 X a2 b2 4X 2 2 a b 3 cyc 2abc cyc X cyc X a cyc a3 b abc X a: cyc Cộng tương ứng vế với vế 3 bất đẳng thức trên, ta thu được bất đẳng thức ở trên. Vậy ta có đpcm. Đẳng thức xảy ra khi và chỉ khi a = b = c: http://boxtailieu.net 1.4. THE CYH TECHNIQUES 131 Ví dụ 1.109 Cho các số dương a; b; c: Chứng minh rằng 2b3 b c a + 3 + 3 2 2 + c a 2c + a b 2a + b2 c a2 3 : + b2 + c2 (Dương Đức Lâm) Lời giải. Sử dụng bất đẳng thức Cauchy Schwarz, ta có !" # " #2 X X X a a(a + c)2 (2b3 + c2 a) a(a + c) 2b3 + c2 a cyc cyc cyc X = 2 a + cyc Nên ta chỉ cần chứng minh được !2 X X 2 a + ab cyc X cyc a cyc Bằng khai triển trực tiếp, ta dễ thấy X a(a + c)2 (2b3 + c2 a) cyc = X a4 (b2 + c2 ) + 4 cyc X = ! X cyc a4 (b2 + c2 ) + 4 cyc X 2 X cyc a4 (b2 + c2 ) + 4 cyc X X !2 ab cyc X 3 a(a + c)2 (2b3 + c2 a) cyc X X a3 b3 + 2 a3 b2 c + 4 a2 b3 c cyc cyc X X a3 b3 + 2abc a2 (b + c) + 2abc a3 cyc a3 b3 + 2abc cyc X cyc ! a X a2 cyc ! cyc Không mất tính tổng quát giả sử a + b + c = 1 và đặt q = ab + bc + ca; r = abc thì ta có ! ! X X X X 4 2 2 3 3 2 a (b + c ) + 4 a b + 2abc a a cyc = X cyc 2 2 a b ! 2 cyc X cyc 2 a ! cyc cyc ! ! X X X 3 3 2 + 4 a b + 2abc a a = (1 2q)(q 2r) + 4(q 2 3 = q + 2q 12qr + 9r2 cyc 3 cyc 2 3qr + 3r ) + 2r(1 http://boxtailieu.net cyc 2q) 3r2 3a2 b2 c2 132 CHƯƠNG 1. TÌM TÒI MỘT SỐ KỸ THUẬT GIẢI TOÁN Ta cần chứng minh (1 q)2 2q)(1 3(q 2 + 2q 3 4q + 2q 2 ,1 8q 3 + 36qr 2 , (1 12qr + 9r2 ) 4q)(1 + q ) + 2(9q + 2)r + [q 2 27r2 0 3 4q + 2(9q 27r2 ] 2)r 0 Ta có q2 4q 3 + 2(9q 2)r 27r2 = (a (4q 1)(1 q) 6 và theo bất đẳng thức Schur bậc 4 thì r (1 4q)(1 + q 2 ) + 2(9q + 2)r c)2 (c a)2 1 (4q 3 1)2 (1 0 nên 4q)(1 + q 2 ) + 2(9q + 2) (1 = b)2 (b 3q) (4q 1)(1 6 q) 0: Bất đẳng thức được chứng minh xong. Đẳng thức xảy ra khi và chỉ khi a = b = c hoặc a = b; c = 0 và các hoán vị. Ví dụ 1.110 Cho a; b; c là độ dài 3 cạnh của một tam giác. Chứng minh rằng a b c + + 2b2 + ca 2c2 + ab 2a2 + bc 3 : a+b+c (Võ Quốc Bá Cẩn) Lời giải. Sử dụng bất đẳng thức Cauchy Schwarz, ta có X a 2 2b + ca cyc !" X 2 2 # a(a + c) (2b + ca) cyc X cyc 2 a + X !2 ab cyc Bằng khai triển trực tiếp, ta được X cyc 2 2 a(a + c) (2b + ca) = " # X X 1 X 4 3 2 2 a (b + c) + 5 a (b + c ) + 12abc ab 2 cyc cyc cyc X 1 + (a b)(b c)(c a) a2 2 cyc Do a; b; c là độ dài 3 cạnh tam giác nên ta dễ dàng chứng minh được X X (a b)(b c)(c a) a3 + 3abc a2 (b + c) cyc http://boxtailieu.net cyc 1.4. THE CYH TECHNIQUES ) X 133 a(a + c)2 (2b2 + ca) cyc " # X X 1 X 4 3 2 2 a (b + c) + 5 a (b + c ) + 12abc ab 2 cyc cyc cyc " # ! X X 1 X 3 a + 3abc a2 (b + c) a2 + 2 cyc cyc cyc 1 = 2 2 X cyc ! X X X 3 2 2 2 a + 5 a (b + c ) + 3abc a + 10abc ab 5 cyc 14 X 3 a = 2 cyc ! cyc X cyc a2 ! +4 X a2 b2 cyc cyc ! X ! a cyc + 3abc cyc Chuẩn hóa cho a + b + c = 1; và đặt ab + bc + ca = q; r = abc ) X cyc 3 a ! X 2 a cyc ! +4 X 2 2 a b cyc ! X cyc ! a + 3abc X 1 3 X q !2 3 a 5 1 4, ta có !2 a cyc = (1 2q)(1 3q + 3r) + 4(q 2 = 1 5q + 10q 2 2(1 + 3q)r 2r) + 3r Ta cần chứng minh 2(1 q)2 3[1 5q + 10q 2 , 6(1 + 3q)r + (4q Theo bất đẳng thức Schur bậc 4, ta có 6r 6(1 + 3q)r + (4q 1)(1 7q) 1)(1 (4q (1 + 3q)(4q = (1 3q)(4q 2(1 + 3q)r] 7q) 1)(1 0 q) nên 1)(1 q) + (4q 1)(q + 2) 0: 1)(1 7q) Bất đẳng thức được chứng minh xong. Đẳng thức xảy ra khi và chỉ khi a = b = c hoặc a = b; c = 0 và các hoán vị. Ví dụ 1.111 Cho các số dương a; b; c. Chứng minh rằng với mọi k s X Xp X 2 2 a + kab + b 4 a2 + (3k + 2) ab: cyc cyc 2, ta có cyc (Michael Rozenberg) http://boxtailieu.net 134 CHƯƠNG 1. TÌM TÒI MỘT SỐ KỸ THUẬT GIẢI TOÁN Lời giải. Sử dụng bất đẳng thức Cauchy Schwarz, ta có !2 " # ! X X a2 + kab + b2 Xp a2 + kab + b2 (a + b) a+b cyc cyc cyc ! ! X X a2 + kab + b2 = 2 a a+b cyc cyc Ta cần chứng minh 2 a+b cyc ,2 X 4 X a2 + kab + b2 (a + b) + 2(k ,2 P a2 + (3k + 2) ab cyc cyc P a cyc X ab 2) a+b cyc cyc ,2 P 4 P P X cyc cyc ab a cyc X 3 a+b , 2abc a+ P 2) ab cyc P a cyc X ab(a + b + c) cyc 3(k cyc 3 X ab a+b cyc X ab cyc X 1 a+b ab cyc Sử dụng bất đẳng thức Cauchy Schwarz, ta có 2abc X cyc 1 a+b 2abc X1 cyc 4 1 1 + a b = X ab: cyc Bất đẳng thức được chứng minh. Đẳng thức xảy ra khi và chỉ khi a = b = c: Ví dụ 1.112 Cho các số không âm a; b; c thỏa mãn a + b + c = 1: Chứng minh rằng r p p p 2 2 2 2 a b + 3c + b c + 3a + c a + 3b : 3 Lời giải. Ta có bổ đề sau 2 X cyc ab + 5 X cyc a2 b 11 9 http://boxtailieu.net 1.4. THE CYH TECHNIQUES 135 Thật vậy, bất đẳng thức tương đương với X X 11 a3 + 15 ab2 + 12abc cyc X 30 cyc xy + y 2 )c + 11x3 0 cyc Giả sử c = minfa; b; cg, đặt a = c + x; b = c + y (x; y 18(x2 a2 b 0), bất đẳng thức trở thành 30x2 y + 15xy 2 + 11y 3 0 Ta cần chứng minh f (x) = 11x3 30x2 y + 15xy 2 + 11y 3 0 Ta có f 0 (x) = 33x2 60xy + 15y 2 p p 10 + 3 5 y 10 3 5 y f (x) = 0 , x = _x= 11 11 0 Từ đây, ta dễ dàng kiểm tra được f (x) f ! p p 10 + 3 5 y 9 109 30 5 y 3 = 11 121 0 Trở lại bài toán của ta, sử dụng bất đẳng thức Cauchy Schwarz, ta được X p a b + 3c2 cyc !2 " = X #" a(1 + c) cyc 1+ X ! ab cyc 1+ X cyc và cyc cyc 1+c # X X ab +3 ab 1+c cyc cyc !6 6X ab X 6 ab 6 +3 ab 6 cyc 1 + c cyc 4 X ab X = ab 1+c cyc cyc X 2 X a(b + 3c2 ) a2 b abc X cyc 11 45 1 c+1 2X 5 X cyc ab cyc http://boxtailieu.net ab ! X ca 3 1+c cyc !2 3 P 7 3 ab 7 cyc P P 2 7 7 ab + a b7 5 cyc cyc 9 abc 4 136 CHƯƠNG 1. TÌM TÒI MỘT SỐ KỸ THUẬT GIẢI TOÁN Ta cần chứng minh 1+ X cyc 2 !6 6 X 6 ab ab 64 6 cyc 4 3 9 abc 4 P 7 7 7 P 7 3 +5 ab 7 5 cyc ab cyc 11 45 !2 3 2 3 Đặt q = ab + bc + ca; r = abc, thì bất đẳng thức này trở thành 4q , 9 r 4 2 3(q + 1) 3q 2 3 11 5 q + 45 2 3(q + 1) 9 135q 2 4q + r + 4 27q + 11 (4q 1)(1 q) . 6 Theo bất đẳng thức Schur bậc 4, ta có r 0 Suy ra 9 135q 2 2 4q + r + 3(q + 1) 4 27q + 11 2 9 (4q 1)(1 q) 135q 2 4q + + 3(q + 1) 4 6 27q + 11 2 3 2 7 60q 87q 36q 135q = + 24(q + 1) 27q + 11 (1 3q)(324q 3 57q 2 240q + 77) = 24(q + 1)(27q + 11) Do đó, ta chỉ cần chứng minh được 324q 3 57q 2 240q + 77 0 Đây là một hàm giảm theo q nên 324q 3 57q 2 240q + 77 324 1 33 57 1 32 240 1 8 + 77 = > 0: 3 3 Bất đăng thức được chứng minh. Đẳng thức xảy ra khi và chỉ khi a = b = c = 31 : Ví dụ 1.113 Cho các số dương a; b; c: Chứng minh rằng a b c + + 2 2 b+c c+a a + b2 9 : 3+a+b+c (Trần Quốc Luật) http://boxtailieu.net 1.4. THE CYH TECHNIQUES 137 Lời giải. Sử dụng bất đẳng thức Cauchy Schwarz, ta có X cyc a b + c2 !" X 2 2 a(b + c )(2a + 2b + c) cyc # = 2 X 2 a +3 cyc X !2 ab cyc Nên ta chỉ cần chứng minh 2 X 2 a +3 cyc X !2 ab cyc 3+ ! X a cyc 9 X a(b + c2 )(2a + 2b + c)2 cyc Bất đẳng thức này được suy ra từ các bất đẳng thức sau 2 X 2 a +3 cyc 2 X 2 ab cyc a +3 cyc !2 X X !2 X ab cyc 3 X ab(2a + 2b + c)2 cyc ! 9 3 X a cyc X c2 a(2a + 2b + c)2 cyc a) Trước hết, ta sẽ chứng minh 2 X 2 a +3 cyc !2 X ab cyc ab(2a + 2b + c)2 cyc Do tính thuần nhất, ta có thể giả sử a + b + c = 1: Đặt q = ab + bc + ca; r = abc thì ta có !2 X X 2 2 a +3 ab = (2 q)2 cyc X ab(2a + 2b + c)2 = cyc cyc X ab(2 (2 q)2 c)2 = cyc X ab(4 4c + c2 ) = 4q 11r cyc Bất đẳng thức trở thành 3(4q , 33r + 4 11r) 16q + q 2 0 Theo bất đẳng thức Schur, ta có nên 33r + 4 16q + q 2 33 4q 1 9 +4 16q + q 2 = http://boxtailieu.net 1 (1 3 q)(1 3q) 0 138 CHƯƠNG 1. TÌM TÒI MỘT SỐ KỸ THUẬT GIẢI TOÁN b) Tiếp theo, ta sẽ chứng minh 2 X X 2 a +3 cyc !2 ab cyc ! X a cyc 9 X c2 a(2a + 2b + c)2 cyc Đây cũng là một bất đẳng thức thuần nhất nên ta cũng có thể chuẩn hóa cho a+b+c = 1 và đặt q = ab + bc + ca; r = abc tương tự như trên, khi đó ta có !2 ! X X X 2 2 a +3 ab a = (2 q)2 cyc cyc cyc Và X c2 a(2a + 2b + c)2 X = cyc c2 a(2 c)2 = cyc = 4 X 2 a b 4 cyc Mà X X a3 b = cyc X a2 b cyc 4 a b = cyc ! X X 3 a b (1 q) ! X X a cyc cyc = ! X a cyc a2 b c2 a(4 cyc X a b+ abc X 3 cyc a2 b2 cyc ! X X ! a b ! + (1 2 cyc a4 b cyc a= cyc X 4c + c2 ) q 2 + (1 + q)r X a2 b q2 + r cyc X ! ab cyc + abc X ab cyc cyc Nên X c2 a(2a + 2b + c)2 cyc = 4 X 2 a b cyc = (1 q) X 4 X 2 2 a b q +r cyc 2 2 a b + 3q + (q q) X a2 b q 2 + (1 + q)r cyc 3)r cyc Sử dụng kết quả quen thuộc P 4 27 a2 b cyc (2 q)2 9 (1 q) r; ta chỉ cần chứng minh được 4 27 r + 3q 2 + (q http://boxtailieu.net 3)r 1.4. THE CYH TECHNIQUES 139 , 9(2 Nếu 1 q)r + 8 q 3 26q 2 0 4q thì ta có 8 3 Nếu 4q 9(2 8 3 8 q 3 26q 2 0 1 thì theo bất đẳng thức Schur, ta có 9r q)r + 8 3 8 q 3 26q 2 (2 q)(4q 1) + 8 3 8 q 3 4q 1 nên 26q 2 = 2 (17q 3 2)(1 3q) 0: Bất đẳng thức được chứng minh xong. Đẳng thức xảy ra khi và chỉ khi a = b = c: Ví dụ 1.114 Cho các số không âm a; b; c thỏa mãn a + b + c = 1: Chứng minh rằng a2 b2 c2 + + b + 27c3 c + 27a3 a + 27b3 1 : 4 (Võ Quốc Bá Cẩn) Lời giải. Sử dụng bất đẳng thức Cauchy Schwarz, ta có X a2 b + 27c3 hX i (2a + b)2 (b + 27c3 ) 2 X a2 + X 2 ab Đặt q = ab + bc + ca; r = abc thì ta có 2 27 X b(2a + b)2 = X X c3 (2a + b)2 = X a3 + 4 a2 + X X 2 ab a2 (b + c) = 1 = (2 3q)2 3q + 3r + 4(q 9r i X X a2 b2 (a + b) + 4abc a2 + 3 a3 b2 h X i X X X X = 27 a a2 b2 + 4abc a2 abc ab + 3 a3 b2 h i X = 27 q 2 2r + 4(1 2q)r qr + 3 a3 b2 h i X = 27 q 2 + (2 9q)r + 3 a3 b2 27 hX 3r) = 1 + q http://boxtailieu.net 140 CHƯƠNG 1. TÌM TÒI MỘT SỐ KỸ THUẬT GIẢI TOÁN và X a3 b2 = X X ab X a2 b abc X a2 X a2 b (1 q)r = q a X X X = q a3 b + a2 b2 + abc a = q X abc ab X a2 b (1 (1 q)r X 1 X 2 2 X 2 2 + a b + abc a (1 a 3 1 1 = q (1 2q)2 + q 2 r (1 q)r = q(1 3 3 q)r q ) 27 X c3 (2a + b)2 27 q 2 + (2 = 9q)r + q(1 q)r 4q + 7q 2 ) 4q + 7q 2 ) r 3r 3q + 7q 2 )] 27[ (9q + 1)r + q(1 Do đó ta chỉ cần chứng minh được 3q)2 4(2 1+q 9r + 27[ (9q + 1)r + q(1 , 9(4 + 37q)r + 15 76q + 117q 2 Theo bất đẳng thức Schur bậc 3, ta có 9r 4q 3q + 7q 2 )] 189q 3 0 1 nên 9(4 + 37q)r + 15 76q + 117q 2 189q 3 (4 + 27q)(4q 1) + 15 76q + 117q 2 = (1 3q)2 (11 21q) 0: 189q 3 Bất đẳng thức được chứng minh xong. Đẳng thức xảy ra khi và chỉ khi a = b = c = 31 : Ví dụ 1.115 Cho các số a; b; c 0; a + b + c = 3: Chứng minh rằng 1 1 1 + + ab2 + 8 bc2 + 8 ca2 + 8 1 : 3 (Vasile Cirtoaje) Hướng dẫn. Sử dụng bất đẳng thức Cauchy Schwarz, X cyc 1 ab2 + 8 ! X cyc 2 2 (ab + 8)(a + 2c + 5) ! " http://boxtailieu.net X cyc #2 (a + 2c + 5) = 576: 1.4. THE CYH TECHNIQUES Ví dụ 1.116 Cho các số a; b; c 141 0: Chứng minh rằng 1 1 1 + + 2a2 + bc 2b2 + ca 2c2 + ab 6 : a2 + b2 + c2 + ab + bc + ca (Vasile Cirtoaje) Hướng dẫn. Sử dụng bất đẳng thức Cauchy Schwarz X 1 2a2 + bc cyc 1 4a2 P #2 (b + c) cyc (b + c)2 (2a2 + bc) : cyc Ví dụ 1.117 Cho các số a; b; c p P " + bc +p 0: Chứng minh rằng 1 1 +p 2 + ca 4c + ab 4 : a+b+c 4b2 (Phạm Kim Hùng) Hướng dẫn. Sử dụng bất đẳng thức Holder X cyc p 1 4a2 + bc Ví dụ 1.118 Cho các số a; b; c p 2a2 !2 P " P #3 (b + c) cyc (b + c)3 (4a2 + bc) : cyc 0: Chứng minh rằng 1 1 1 +p +p 2 2 + ab + bc 2b + ca + ab 2c + ca + ab 9 : 2(a + b + c) Hướng dẫn. Sử dụng bất đẳng thức Holder X cyc 1 p 2 2a + ab + bc !2 P 27(a + b + c)3 : (b + 2c)3 (2a2 + ab + bc) cyc http://boxtailieu.net 142 CHƯƠNG 1. TÌM TÒI MỘT SỐ KỸ THUẬT GIẢI TOÁN Ví dụ 1.119 Cho a; b; c là độ dài 3 cạnh của một tam giác. Chứng minh rằng r b c a 3 p +p +p (a + b + c) 2 c+a a+b b+c (Võ Quốc Bá Cẩn) Hướng dẫn. Sử dụng bất đẳng thức Cauchy Schwarz, X cyc a p a+b !2 " X b2 a(a + b + 2c) cyc Ví dụ 1.120 Cho các số a; b; c p #" X cyc # a : (a + b)(a + b + 2c) 0: Chứng minh rằng a b c +p +p 2 2 + 15ca c + 15ab a + 15bc 3 : 4 (Park Doo Sung) Hướng dẫn. Sử dụng bất đẳng thức Holder X cyc a p b2 + 15ca Ví dụ 1.121 Cho các số a; b; c p b2 !2 P " P #3 a(a + 2b) cyc a(a + 2b)3 (b2 + 15ca) : cyc 0: Chứng minh rằng a b c +p +p 2 2 2 2 + 3c c + 3a a + 3b2 3 : 2 (Vasile Cirtoaje) Hướng dẫn. Sử dụng bất đẳng thức Holder X cyc a p b2 + 3c2 !2 P " P #3 a(2a + b) cyc a(2a + b)3 (b2 + 3c2 ) cyc http://boxtailieu.net : 1.5. THE HYBERBOLIC FUNCTIONAL TECHNIQUE Ví dụ 1.122 Cho các số a; b; c p 143 0: Chứng minh rằng b c a +p +p 2 2 + ca 3c + ab 3a + bc 3b2 3 : 2 (Vasile Cirtoaje) Hướng dẫn. Sử dụng bất đẳng thức Holder X cyc 1.5 1.5.1 a p 3b2 + ca !2 P " P #3 a(a + c) cyc a(a + c)3 (3b2 + ca) : cyc The Hyberbolic functional technique Lời nói đầu Kỹ thuật này có vẻ là khá mới mẻ nếu các bạn chỉ xem tên của kỹ thuật thôi nhưng thực ra nó đã từng được giới thiệu rất nhiều lần trên các diễn đàn, các tạp chí với cái tên phương pháp tiếp tuyến để chứng minh bất đẳng thức (chẳng hạn như ở [5]). Nhưng, trong các bài viết đó, các tác giả đều chưa khai thác thật triệt để các tính chất của tiếp tuyến để kỹ thuật trở nên mạnh hơn và được sử dụng nhiều hơn trong chứng minh bất đẳng thức. Ở đây, trong bài viết này, chúng tôi xin được giới thiệu cùng các bạn một số tìm tòi của mình trong việc làm mạnh kỹ thuật trên. 1.5.2 Một số ví dụ mở đầu Để chứng minh một bất đẳng thức f (x1 ) + f (x2 ) + + f (xn ) 0; mà việc đánh giá f (x) gặp nhiều khó khăn thì chúng ta sẽ tìm một hàm g(x) dễ đánh giá hơn sao cho f (x) g(x) và ta chỉ còn việc phải chứng minh bất đẳng thức còn lại chặt hơn nhưng dễ hơn là g(x1 ) + g(x2 ) + + g(xn ) 0: Ví dụ 1.123 Cho các số không âm a; b; c; không có 2 số nào đồng thời bằng 0: Chứng minh rằng b c 3 a + + : b+c c+a a+b 2 (Nesbitt) http://boxtailieu.net 144 CHƯƠNG 1. TÌM TÒI MỘT SỐ KỸ THUẬT GIẢI TOÁN Lời giải. Chuẩn hóa cho a + b + c = 3; bất đẳng thức trở thành a 3 Với mọi x a + b 3 b + c 3 3 2 c 3; ta có 4x 3 x 3(x , 3 Do đó 4a 3 a + 4b 3 b + 4c 3 c (3a 3x 1 1)2 x 0: 1) + (3b 1) + (3c 1) = 6 Bất đẳng thức được chứng minh. Ví dụ 1.124 Cho các số không âm a; b; c; tất cả không đời bằng 0: Chứng minh rằng (2a + b + c)2 (2b + c + a)2 (2c + a + b)2 + + 2a2 + (b + c)2 2b2 + (c + a)2 2c2 + (a + b)2 8: (USAMO 2003) Lời giải. Chuẩn hóa cho a + b + c = 3; bất đẳng thức trở thành (3 + a)2 (3 + b)2 (3 + c)2 + 2 + 2 2 2 + (3 a) 2b + (3 b) 2c + (3 c)2 2a2 , Với mọi x X a2 + 6a + 9 cyc a2 2a + 3 8 24 3; ta có x2 + 6x + 9 4x + 4 x2 2x + 3 (4x + 3)(x 1)2 , 0 x2 2x + 3 Do đó X a2 + 6a + 9 cyc a2 2a + 3 X (4a + 4) = 24 cyc Bất đẳng thức được chứng minh. Ví dụ 1.125 Cho các số dương a; b; c thỏa mãn abc = 1: Chứng minh răng p p p p a2 + 1 + b2 + 1 + c2 + 1 2(a + b + c): (Gabriel Dospinescu) http://boxtailieu.net 1.5. THE HYBERBOLIC FUNCTIONAL TECHNIQUE Lời giải. Xét hàm số f (x) = f 0 (x) = p x2 + 1 p x+ p1 2 145 ln x với x > 0; ta có i h p x) 1 x + 2x2 + 2x2 2(1 + x2 ) p p p x 2(x2 + 1) 2x2 + x2 + 1 (1 f 0 (x) = 0 , x = 1 Từ đây dễ thấy f (x) f (1) = 0 8x > 0 p p 1 ) x2 + 1 2x p ln x 2 Do đó p p p a2 + 1 + b2 + 1 + c2 + 1 p 2(a + b + c) p 1 p (ln a + ln b + ln c) = 2(a + b + c) 2 Bất đẳng thức được chứng minh. Câu hỏi đặt ra là làm sao để chúng ta có thể chọn được các hàm g(x) thích hợp? Thật ra, ở đây hàm g(x) được lựa chọn dựa vào điều kiện ràng buộc các biến của bài toán, chẳng hạn như nếu điều kiện là x1 + x2 + + xn = n thì g(x) = k(x 1); nếu xn = 1 thì g(x) = k ln x x21 + x22 + + x2n = n thì g(x) = k(x2 1), và nếu x1 x2 với k là hằng số mà ta sẽ chọn sau. (Ở đây ta giả sử bất đẳng thức có đẳng thức xảy ra tại x1 = x2 = = xn = 1). Ở đây nếu f có đạo hàm và liên tục lại x = 1 thì k = f 0 (1): Nhưng trong một vài trường hợp, ta không cần phải tính đạo hàm làm gì mà vẫn có thể dễ dàng chọn bằng phép biến đổi tương đương, chẳng hạn như ở bất đẳng thức Nesbitt, chúng ta cần chọn sao cho x 3 x , (x k(x 1) 1) + 1 2 3 2(3 x) 8x 2 (0; 3) k 0 Để bất đẳng thức này không đổi dấu khi x chạy qua giá trị 1 thì ta phải chọn k sao cho 3 k=0 2(3 x) có nghiệm x = 1 (nếu không thì bất đẳng thức sẽ không đúng), từ đó suy ra k = 43 : Ví dụ 1.126 Cho các số thực a; b; c thỏa mãn a + b + c = 6: Chứng minh rằng a4 + b4 + c4 2(a3 + b3 + c3 ): http://boxtailieu.net 146 CHƯƠNG 1. TÌM TÒI MỘT SỐ KỸ THUẬT GIẢI TOÁN Ví dụ 1.127 Cho các số không âm a; b; c thỏa mãn a + b + c = 3: Chứng minh rằng p p p a + b + c ab + bc + ca: (Russia MO 2002) Hướng dẫn. 2(ab + bc + ca) = 9 a2 b2 c2 : Ví dụ 1.128 Cho các số dương a; b; c thỏa mãn a + b + c = 1: Chứng minh rằng a b c + + 1 + bc 1 + ca 1 + ab Hướng dẫn. bc 9 : 10 (b + c)2 (1 a)2 = : 4 4 Ví dụ 1.129 Cho các số dương a; b; c thỏa mãn abc = 1: Chứng minh rằng a2 + b2 + c2 + 9(ab + bc + ca) 10(a + b + c): (Vasile Cirtoaje) Ví dụ 1.130 Cho các số dương a; b; c. Chứng minh rằng (b + c a)2 (c + a b)2 (a + b c)2 + 2 + 2 2 2 2 a + (b + c) b + (c + a) c + (a + b)2 3 : 5 (Japan MO 1997) 1.5.3 Đặt vấn đề Đối với những bài toán không chặt, cách làm trên tỏ ra rất hiệu quả nhưng đối với những bài toán tương đối chặt thì chúng ta khó lòng dùng nó để giải quyết, bởi vì bất đẳng thức f (x) g(x) lúc này không phải luôn đúng nữa mà nó chỉ đúng trong một số trường hợp. Chẳng hạn như với bài toán sau Ví dụ 1.131 Cho các số thực a; b; c thỏa mãn a + b + c = 1: Chứng minh rằng a b c + + a2 + 1 b2 + 1 c2 + 1 http://boxtailieu.net 9 : 10 1.5. THE HYBERBOLIC FUNCTIONAL TECHNIQUE 147 Bằng cách tương tự, chúng ta thiết lập được bất đẳng thức 36x + 3 50 x x2 + 1 3 4 Nhưng tiếc thay bất đẳng thức này chỉ đúng trong trường hợp x 36x + 3 50 vì x (4x + 3)(3x 1)2 = x2 + 1 50(x2 + 1) trong khi giả thiết lại yêu cầu chúng ta chứng minh bài toán với các số thực tùy ý. Vì thế, cách làm này không phát huy được tác dụng. Nhưng chúng ta cũng có thể giải quyết bài toán bằng cách chia thành từng trường hợp nhỏ để giải. Với bài toán trên, chúng ta có thể giải quyết nó như sau 3 Lời giải. Trường hợp 1. Nếu min fa; b; cg 4 ; khi đó sử dụng bất đẳng thức 36x + 3 50 x x2 + 1 3 4 8x Ta dễ dàng suy ra kết quả bài toán. Trường hợp 2. Giả sử tồn tại một số trong ba số a; b; c nhỏ hơn Khi đó, sử dụng bất đẳng thức AM-GM, ta có b +1 1 2 a a2 + 1 2 5 b2 3 4; chẳng hạn c < 3 4: Nên bất đẳng thức của ta sẽ đúng nếu ,a Do đó, ta chỉ cần xét trường hợp 2 trường hợp 2 a; b 12 ; khi đó ta có 1 _a 2 2 1 a 2 : Hoàn toàn tương tự, ta chỉ cần xét 3 >c=1 4 ) Do đó a2 c2 c +1 a b c + 2 + 2 +1 b +1 c +1 a b 3 3 10 1 1 + 2 2 3 7 9 = < : 10 10 10 Bất đẳng thức được chứng minh xong. Đẳng thức xảy ra khi và chỉ khi a = b = c = 31 : http://boxtailieu.net 148 CHƯƠNG 1. TÌM TÒI MỘT SỐ KỸ THUẬT GIẢI TOÁN Ví dụ 1.132 Cho các số dương a; b; c thỏa mãn abc = 1: Chứng minh rằng 3a2 1 + (a 1)2 + 3b2 1 + (b 1)2 + 3c2 1 + (c 1: 1)2 (Lê Hữu Điền Khuê) Lời giải. Xét hàm số f (x) = 1 3x2 +(x 1)2 f 0 (x) = 1 3 + 2 3 ln x; ta có 2(x 1)(16x3 1) 3x(4x2 2x + 1)2 Từ đây dễ thấy f 0 (x) = 0 không phải chỉ có một nghiệm x = 1 mà còn có thêm một 1 nghiệm nữa là x = 2 p nên chắc chắn rằng bất đẳng thức f (x) 0 mà ta đang mong 3 2 đợi sẽ không phải luôn đúng. Vậy ta hãy xem xét xem nó đúng trong trường hợp nào? Bằng kiểm tra trực tiếp, ta thấy được f (x) 0 8x 21 : Từ đây, dẫn đến lời giải như sau Nếu min fa; b; cg 12 khi đó ta có X cyc Bây giờ, ta giả sử a X cyc 3a2 3a2 1 2; 1 + (a 1 + (a 1)2 X cyc 1 3 2 ln a 3 = 1: khi đó ta có 1)2 3a2 1 + (a 1)2 = 1 2a(2a 1: 1) + 1 nên bất đẳng thức đã cho luôn đúng. Ta có đpcm. Ví dụ 1.133 Cho các số không âm a; b; c; không có 2 số nào đồng thời bằng 0: Chứng minh rằng Ví dụ 1.134 a2 b2 c2 + + 5a2 + (b + c)2 5b2 + (c + a)2 5c2 + (a + b)2 1 : 3 (Võ Quốc Bá Cẩn) Lời giải. Giả sử a+b+c=1 a b c)a 1 3 X 6a2 cyc c , bất đẳng thức trở thành a2 2a + 1 1 3 http://boxtailieu.net 1.5. THE HYBERBOLIC FUNCTIONAL TECHNIQUE 1 8; Nếu c khi đó ta có 27 1 3 X cyc 6a2 a2 2a + 1 ! = X X (1 cyc 1 8; 1 6 6 12q 27a2 6a2 2a + 1 1 cyc = Nếu c 149 3a)2 (8a 1) 6a2 2a + 1 ta có X cyc 6a2 a2 2a + 1 ! 1 2b 6c2 2a + 2 2 2a + 1 6b 2b + 1 6c 2c + 1 b+c a a+c b 6c2 = + 6a2 2a + 1 6b2 2b + 1 6c2 2c + 1 1 2(a b)2 (2 3c) +c + = (6a2 2a + 1)(6b2 2b + 1) 6a2 2a + 1 6b2 1 1 6c c + 6a2 2a + 1 6b2 2b + 1 6c2 2c + 1 1 1 c + 1 6a2 2a + 1 6b2 2b + 1 = 1 0 6a2 1 2b + 1 6c2 6c 2c + 1 Do đó, ta chỉ cần chứng minh 6a2 1 + 2a + 1 6b2 1 2b + 1 1 Nếu b 13 thì bất đẳng thức này hiển nhiên đúng, nếu b Cauchy Schwarz 6a2 1 + 2a + 1 6b2 1 2b + 1 = thì theo bất đẳng thức 4 + 2(a + b) + 2 2 3(a2 + b2 ) (a + b) + 1 2(a + b + c)2 3(a2 + b2 ) + c(a + b + c) 6(a2 = 1 3 b2 ) Ta cần chứng minh 2(a + b + c)2 3(a2 + b2 ) + c(a + b + c) http://boxtailieu.net 150 Do b CHƯƠNG 1. TÌM TÒI MỘT SỐ KỸ THUẬT GIẢI TOÁN 1 3 3(a2 + b2 ) , (a + b + c)(2a + 2b + c) nên 3b a; suy ra 2(a + b)2 = 3(a2 + b2 ) + b(a (a + b + c)(2a + 2b + c) b) + a(3b a) 3(a2 + b2 ): Bất đẳng thức được chứng minh xong. Đẳng thức xảy ra khi a = b = c hoặc a = b; c = 0 và các hoán vị. Ví dụ 1.135 Cho các số không âm x; y; z thỏa mãn x2 + y 2 + z 2 = 3: Chứng minh rằng x5 x2 y5 y2 z5 z2 + + 0: x5 + y 2 + z 2 y 5 + z 2 + x2 z 5 + x2 + y 2 (Vasile Cirtoaje) Lời giải. Bất đẳng thức tương đương X cyc 1 x5 + y 2 + z 2 , X cyc 3 x2 + y 2 + z 2 1 x2 + 3 x5 1: Sử dụng bất đẳng thức AM-GM, ta có x5 = 2x6 x2 + 1 x6 x Đặt a = x2 ; b = y 2 ; c = z 2 ) a + b + c = 3 thì ta chỉ cần chứng minh X 1 2a3 cyc a+1 , , X 2a3 cyc X (a cyc 1 a+3 a+1 a2 + 2a + 3 1 1)2 (3 + 3a 2a2 ) a2 + 2a + 3 2a3 0 Giả sử a b c ) a 1 c: Xét 2 trường hợp Trường hợp 1. b + c 1 ) a 2; khi đó ta có 3 + 3a 2a2 0; 3 + 3b 2b2 0; 3 + 3c Nên bất đẳng thức đúng. http://boxtailieu.net 2c2 0 1.5. THE HYBERBOLIC FUNCTIONAL TECHNIQUE Trường hợp 2. b + c 2a3 a2 + 2a + 3 1)a 151 2; ta có = 2a3 5(a + 1) a2 a3 2 ) 2a3 2 = a3 2 3a 1 2 2 23 = c+1 c2 + 2c + 3 4 5 3 22 1 a 3 a2 2 a3 1 3 a >0 2 1 5 a+1 a2 + 2a + 3 Do đó ta chỉ cần chứng minh b+1 + b2 + 2b + 3 2c3 2b3 Nhưng bất đẳng thức này hiển nhiên đúng vì với mọi ta có Nếu x 4x3 1 2 2 5 x+1 x2 + 2x + 3 2x3 , 4x3 (x + 1)(2x 1) 1 2; thì bất đẳng thức hiển nhiên đúng, nếu x (x + 1)(2x 4x3 1) 2(2x 1) 2x2 ta có 2(2x 1) = 2(x 1)2 0: Bất đẳng thức được chứng minh. Ví dụ 1.136 Cho các số không âm a; b; c thỏa mãn a + b + c = 3: Chứng minh rằng p a2 1 1 1 +p +p 2 2 3a + 3 b 3b + 3 c 3c + 3 3: (Nguyễn Văn Thạch) Lời giải. Chúng ta thiết lập được bất đẳng thức sau p Giả sử a b c)a x2 2 3x + 3 1 c: Nếu c X p cyc Xét trường hợp ngược lại, c a2 p x + 1 8x p 5 1 2 2 3a + 3 5 1 2 ; p 5 1 2 thì ta có X (a + 1) = 6 cyc ta xét 2 trường hợp nhỏ http://boxtailieu.net 152 CHƯƠNG 1. TÌM TÒI MỘT SỐ KỸ THUẬT GIẢI TOÁN Nếu b 1; ta có a2 3a + 3 = b2 c2 c X p 1 3a + 3 Nếu b 1)2 a b 1; xét hàm số f (x) = cyc a2 f "(x) = 8x2 4(x2 1 + 3 4 3 4 b)(1 b) + 1 1 !2 p p 5 1 5 1 16 +2 = p 2 2 5+1 p 5+1 2 p +1+ 0 8a 2. Bất đẳng thức được chứng minh. Đẳng thức xảy ra khi và chỉ khi a = b = c = d = 1. Ví dụ 1.142 Cho các số không âm a; b; c; không có 2 số nào đồng thời bằng 0: Chứng minh rằng r r r 48a 48b 48c 1+ + 1+ + 1+ 15: b+c c+a a+b (Vasile Cirtoaje) Lời giải. Chuẩn hóa cho a + b + c = 1 và giả sử a b thành r X 1 + 47a 15 1 a cyc Ta xét 2 trường hợp Trường hợp 1. Nếu c 1 + 47x 1 x Trường hợp 2. Nếu c 1 + 47x 1 x 2 27 , c, khi đó bất đẳng thức trở thì 2 54 7 12(27x 2)(3x 1)2 x+ = 0 81 5 5 25(1 x) r 1 + 47x 54 7 2 ) x+ 81 x 1 x 5 5 27 r X 1 + 47a X 54 7 ) x+ = 15 1 a 5 5 cyc cyc 2 27 , x 2 27 thì 2 96 1 48(48x + 1)(2x 1)2 x+ = 7 7 49(1 x) r 1 + 47x 96 1 ) x+ 81 x 1 x 7 7 http://boxtailieu.net 0 81 0 x 0 1.5. THE HYBERBOLIC FUNCTIONAL TECHNIQUE ) r 1 + 47a + 1 a r Ta cần chứng minh Đặt 1+47c 1 c = t2 (t 1 + 47b 1 b r 11 5 0) ) t 96 2 (a + b) + = 14 7 7 1 + 47c 1 c 1+ 96 c 7 96 c 7 1, bất đẳng thức trở thành t 96(t2 1) 7(t2 + 47) 1+ 1)(7t2 , (t 159 96t + 233) 0: hiển nhiên đúng do 11 t 1: 5 Bất đẳng thức được chứng minh. Đẳng thức xảy ra khi và chỉ khi a = b = c hoặc a = b; c = 0 hoặc các hoán vị tương ứng. Ví dụ 1.143 Cho các số không âm a; b; c thỏa mãn a + b + c = 1. Chứng minh rằng 1 4a2 1 4b2 1 4c2 + + 2 2 1 + 3a 3a 1 + 3b 3b 1 + 3c 3c2 1: (Michael Rozenberg) Lời giải. Không mất tính tổng quát, giả sử a Trường hợp 1. Nếu c 91 , thì 1 4x2 1 + 3x 3x2 1 3 3 (1 5 ) ) X cyc Trường hợp 2. Nếu c 1 4x2 1 + 3x 3x2 1 4a2 1 + 3a 3a2 1 9, (1 9x)(1 15(1 + 3x 1 4x2 1 + 3x 3x2 c. Ta xét 2 trường hợp 3x)2 3x2 ) 1 3 + (1 3 5 0 8x 2 1 ;1 9 3x) X 1 3 + (1 3 5 cyc 3a) = 1 thì 8 (1 7 ) ) 3x) = b 2x) = (12x + 1)(2x 1)2 7(1 + 3x 3x2 ) 1 4x2 1 + 3x 3x2 1 4a2 1 4b2 + 1 + 3a 3a2 1 + 3b 3b2 8 (1 7 0 8x 2 [0; 1] 2x) 8 (2 7 http://boxtailieu.net 2a 2b) = 16 c 7 160 CHƯƠNG 1. TÌM TÒI MỘT SỐ KỸ THUẬT GIẢI TOÁN Ta cần chứng minh 1 4c2 16 1 c+ 7 1 + 3c 3c2 c(48c2 41c + 5) , 0: 7(1 + 3c 3c2 ) hiển nhiên đúng do c 91 : Bất đẳng thức được chứng minh. Đẳng thức xảy ra khi và chỉ khi a = b = c = a = b = 12 ; c = 0 hoặc các hoán vị tương ứng. 1 3 hoặc Ví dụ 1.144 Cho các số không âm a; b; c; d thỏa mãn a + b + c + d = 1. Chứng minh rằng (1 + 2a)(1 + 2b)(1 + 2c)(1 + 2d) 125 : (1 a)(1 b)(1 c)(1 d) 8 (Vasile Cirtoaje) Lời giải. Không mất tính tổng quát, giả sử a b tương đương với X [ln(1 + 2a) ln(1 a)] c d)d 3 ln 5 2 cyc 1 4. Bất đẳng thức Ta xét 2 trường hợp Trường hợp 1. Nếu c x0 với x0 là nghiệm thuộc (0; 0:09) của phương trình ln(1 + 9 2x) ln(1 x) 10 (3x 1) ln 52 = 0, xét hàm số f (x) = ln(1 + 2x) ln(1 x) 9 5 1) ln 2 with x x0 , ta có 10 (3x f 0 (x) = 3(6x 1)(3x 10(1 + 2x)(1 1 _ 6 min f 1) x) f 0 (x) = 0 , x = Từ đây, ta dễ dàng kiểm tra được f (x) 1 3 ; f (x0 ) = 0 8x x= 1 3 [ln(1 + 2a) ln(1 a)] + [ln(1 + 2b) ln(1 b)] + [ln(1 + 2c) 9 5 27 5 (3a + 3b + 3c 3) + 3 ln = c + 3 ln 10 2 10 2 Ta cần chứng minh g(d) = ln(1 + 2d) Ta có g 0 (d) = ln(1 27 d 10 d) 3(6d 1)(3d 10(1 + 2d)(1 1) d) http://boxtailieu.net 0 x0 . Suy ra ln(1 c)] 1.5. THE HYBERBOLIC FUNCTIONAL TECHNIQUE 161 1 6 Từ đây, ta dễ dàng kiểm tra được g(d) min g(0); g 41 = 0. Trường hợp 2. Nếu x0 c d, xét hàm số h(x) = ln(1 + 2x) ln(1 1) 2 ln 2 với x 0, ta có g 0 (d) = 0 , d = h0 (x) = x) 3 2 (2x 3x(2x 1) (1 + 2x)(1 x) h0 (x) = 0 , x = 0 Từ đây, ta dễ dàng kiểm tra được h(x) h _ 1 2 x= 1 2 = 0 8x 0. Suy ra [ln(1 + 2a) ln(1 a)] + [ln(1 + 2b) ln(1 b)] 3 (2a + 2b 2) + 4 ln 2 = 3c 3d + 4 ln 2 2 Ta cần chứng minh k(c) + k(d) 0 3 2 trong đó k(x) = ln(1 + 2x) ln(1 x) 3x + 2 ln 2 chứng minh được k(x) 0 8x x0 . Ta có k 0 (x) = Suy ra k(x) nghịch biến với mọi x k(x) 3x(2x 1) (1 + 2x)(1 x) = > 0 x0 , do đo k(x0 ) = ln(1 + 2x) = ln 52 . Và vì thế, ta chỉ cần ln(1 x) 3x + 2 ln 2 3 5 ln 2 2 9 5 3 5 (3x0 1) + ln ln 3x0 + 2 ln 2 10 2 2 2 3 9 1 5 x0 + 2 ln 2 ln 10 10 2 2 3 9 1 5 0:09 + 2 ln 2 ln 0:0011 > 0: 10 10 2 2 Bất đẳng thức được chứng minh. Đẳng thức xảy ra khi và chỉ khi a = b = c = 31 ; d = 0 hoặc các hoán vị tương ứng. Ví dụ 1.145 Let a; b; c; d be nonnegative real numbers such that a + b + c + d = 2. Prove that a2 b2 c2 d2 16 + + + : 2 2 2 2 2 2 2 2 (a + 1) (b + 1) (c + 1) (d + 1) 25 (Ji Chen) http://boxtailieu.net 162 CHƯƠNG 1. TÌM TÒI MỘT SỐ KỸ THUẬT GIẢI TOÁN Lời giải. Không mất tính tổng quát, giả sử a Trường hợp 1. Nếu 12d3 + 11d2 + 32d 4, thì X cyc a2 (a2 + 1)2 24 (2a 125 = b Trường hợp 2. Nếu 4 x2 (x2 + 1)2 X (12a3 + 11a2 + 32a 125(a2 cyc 24 X (2a 125 cyc a2 (a2 + 1)2 12d3 + 11d2 + 32d + 1) + x2 (x2 + 1)2 540 2197 1 8, 32d ) d b2 c2 d2 + + (b2 + 1)2 (c2 + 1)2 (d2 + 1)2 x 2 3 + 1)2 0 ta có 2)2 36 169 540 (b + c + d 2197 4)(2a 1)2 16 16 = 25 25 540 2 36 x 2197 3 169 (60x3 + 92x2 + 216x + 27)(3x = 2197(x2 + 1)2 ) ) X d. Ta xét 2 trường hợp 4 25 1) cyc ) c 8x 2) + 0 8x 0 0 108 = 169 540 108 a+ 2197 169 Ta cần chứng minh (a2 a2 + 1)2 , 540 108 a+ 2197 169 169a2 (a2 + 1)2 540 a 13 16 25 4 25 Ta có 169a2 (a2 + 1)2 540 a 13 4 25 540 4 169a2 4 169a2 a = 36a (a2 + 1)2 15 25 (a2 + 1)2 25 4 + 900a 4217a2 + 1800a3 + 4a4 + 900a5 = 25(a2 + 1)2 2 4 + 8 900a 4217a2 + 1800a3 + 4a4 + 900a5 25(a2 + 1)2 2 4 + 2983a + 1800a3 + 4a4 + 900a5 = < 0: 25(a2 + 1)2 Bất đẳng thức được chứng minh. Đẳng thức xảy ra khi và chỉ khi a = b = c = d = 12 : http://boxtailieu.net 1.5. THE HYBERBOLIC FUNCTIONAL TECHNIQUE Ví dụ 1.146 Cho các số không âm a1 ; a2 ; :::; an (n n. Chứng minh rằng 1)(a31 + a32 + (n + a3n ) + n2 163 2) thỏa mãn a1 +a2 + 1)(a21 + a22 + (2n +an = + a2n ): (Vasile Cirtoaje) Lời giải. Nếu n = 2, bất đẳng thức trở thành đẳng thức. Nếu n = 3, đặt q = a1 a2 + a2 a3 + a3 a1 ; r = a1 a2 a3 , bất đẳng thức trở thành 2(27 9q + 3r) + 9 , 3r + 9 5(9 4q 2q) 0 Đây chính là bất đẳng thức Schur bậc 3. Suy ra, ta chỉ cần xét trường hợp n 4. Không mất tính tổng quát, giả sử a1 an . Ta xét 2 trường hợp 1 Trường hợp 1. Nếu an n 1 , ta có (n 1)x3 1)x2 + (n + 1)(x (2n 1)x3 + n ) (n ) n X ) 1)x3 n X1 n X 1)a3i + n] [(n i=1 (n [(2n 1)a2i 1)x 1] 1 0 8x n 1 1) 8x n (n + 1)(ai 1 1)] (2n n2 n 1)a2i + i=1 n X1 i=1 i=1 n(n 2) n2 [(n 1)x n] + (n 1)2 n 1 2 [(n 1)x n] [(n 1)x + 1] = (n 1)2 1)x2 + 1) n X 1) a2i ta có 1)x2 + 1)a3i (2n i=1 n 1, (2n , (n (n + 1)(x n X 1) a3i + n2 1 (2n 1)x3 1)2 [(n i=1 Trường hợp 2. Nếu an ) (n 1)x2 (2n , (n (n 1) + n = (x a2 a3i (2n n(n 2) [n (n 1)2 1 n2 n 1 n X1 1) (n 1)x] 8x n 1 n(n 2) X [n (n 1)2 i=1 a2i + n2 i=1 http://boxtailieu.net 0 8x (n n(n 2) an n 1 0 0 1)ai ] 1 164 CHƯƠNG 1. TÌM TÒI MỘT SỐ KỸ THUẬT GIẢI TOÁN Ta cần chứng minh (n , an [(n 1)a3n 1)a2n + (2n 1)(2 an )[1 1)an ] + n2 (n n n(n 2) an n 1 0 4n + 2] 1 0: hiển nhiên đúng do n 4: Bất đẳng thức được chứng minh. 1.5.5 Một số mở rộng Mở rộng thứ nhất Ví dụ 1.147 Cho các số không âm a; b; c thỏa mãn a + b + c = 3: Chứng minh rằng 1 9 ab + 1 9 bc + 1 9 3 : 8 ca Lời giải. Bài toán này có đặc điểm gần giống dạng của các ví dụ trên nhưng chúng ta không thể dùng cách của các bài trên để giải nó vì các biến trong bất đẳng thức dạng f (x1 ) + f (x2 ) + f (x3 ) 0 không phải là a; b; c liên quan đến giả thiết của bài toán mà là ab; bc; ca. Một lí do nữa là khi ta cố gắng thiết lập bất đẳng thức f (x) g(x) trong đó g(x) có dạng k(x 1) thì bất đẳng thức này bị ngược chiều (các bạn hãy kiểm tra). Tuy nhiên, chúng ta có thể dùng dạng sau để giải bài toán này, đó là thiết lập g(x) có dạng m(x2 1) + n(x 1), chúng ta chỉ việc thiết tìm các tham số m; n sao cho f (x) g(x) được thỏa mãn. Cụ thể, ở bài này, chúng ta sẽ tìm m; n sao cho 8 9 x , (x m(x2 1) + n(x 1) m(x + 1) + n 1) + 1 1 9 x 0 Giống như các bài trước, ta sẽ chọn m; n sao cho phương trình m(x + 1) + n 9 1 x = 0 có một nghiệm là 1, từ đó ta suy ra được n = 18 2m, thay vào bất đẳng thức trên và phân tích ra, ta được bất đẳng thức tương đương là (x 1)2 (72m 1 8mx) 0 Bây giờ hãy chú ý rằng 3 > max fab; bc; cag và a; b; c 0 nên ta chỉ cần tìm m sao cho bất đẳng thức trên đúng với mọi x 2 [0; 3] là đủ (nếu cần ta có thể dùng đánh giá mạnh hơn là 94 max fab; bc; cag, tức là tìm để bất đẳng thức đúng với mọi x 2 0; 49 nhưng vì bài này không chặt nên ta không cần phải đánh giá quá chặt như thế). Cho x = 0 ) m > 0, do đó 72m 1 8mx 72m 1 24m = 48m http://boxtailieu.net 1 1.5. THE HYBERBOLIC FUNCTIONAL TECHNIQUE 165 1 1 Từ đây, rõ ràng nếu ta chọn m = 48 ) n = 12 thì bất đẳng thức đúng. Vậy 1 1 m = 48 ; n = 12 và ta thiết lập được bất đẳng thức 8 9 1 2 (x 48 x 1) + Và lời giải của ta như sau Dễ dàng chứng minh được 9 8 x này với chú ý là max fab; bc; cag X cyc X 1 48 ab 1 2 (x + 4x + 3) 48 1) + 1 = 1 2 48 (x + 4x + 3) 9 4 < 3; ta có 8 9 1 (x 12 8x 2 [0; 3]. Sử dụng bất đẳng thức a2 b2 + 4 cyc X ! ab cyc + 43 16 Do đó ta chỉ cần chứng minh X a2 b2 + 4 cyc Đặt x = ab + bc + ca ) x Nếu 4x 9; ta có X cyc Nếu 4x a2 b2 + 4 max 0; 4x3 ab = x2 + 4x 15 9 : x2 + 4x 6abc cyc 9; ta có X X a2 b2 + 4 ab = x2 + 4x cyc ab cyc 3; abc X X x2 + 4x 6abc 225 < 15 16 2(4x 9) cyc = (x 1)(x 3) + 15 15 Bất đẳng thức được chứng minh. Ví dụ 1.148 Cho các số không âm a; b; c thỏa a4 + b4 + c4 = 3: Chứng minh rằng 1 4 ab + 1 4 bc + 1 4 ca 1: (Moldova TST 2005) Lời giải. Dễ dàng chứng minh được với mọi x 3 4 x 3 2 thì 1 (2x2 + x + 12) 15 http://boxtailieu.net 166 CHƯƠNG 1. TÌM TÒI MỘT SỐ KỸ THUẬT GIẢI TOÁN q Chú ý là max fab; bc; cag X cyc 3 2 < nên 1 15 3 4 3 2 ab 2 X 2 2 a b + cyc X ! ab + 36 cyc Mặt khác, ta có X X a2 b2 cyc a4 = 3; cyc X s X 3 a2 b2 ab 3 cyc cyc Từ đây dễ dàng suy ra đpcm. Ví dụ 1.149 Cho các số không âm a; b; c; d thỏa mãn a2 + b2 + c2 + d2 = 4: Chứng minh rằng 1 1 1 1 + + + 2: 3 abc 3 bcd 3 cda 3 dab (Phạm Kim Hùng) 8 p 3 3 Lời giải. Dễ thấy max fabc; bcd; cdag thức 2 3 Dễ thấy n = 1 2 m(x2 x 1)2 (6m 1 2mx Ta cần có 6m 1 )m p 3> 5 3 1) + 1 1 2mx) 0 6m 1 16 p m 3 3 0; suy ra 6m Do 1) + n(x 2m; khi đó bất đẳng thức tương đương (x Dễ thấy m nên chúng ta cần thiết lập bất đẳng 16 p m 0 3 3 1 p 6 316 3 nên ta chỉ cần chọn sao cho m 1 6 16 5 )m= = 5 14 5 14 http://boxtailieu.net 1.5. THE HYBERBOLIC FUNCTIONAL TECHNIQUE )n= 167 3 14 Như vậy, ta thiết lập được bất đẳng thức 5x2 2 3 x 3x + 12 14 8x 8 p 3 3 Sử dụng các bất đẳng thức này lần lượt, ta suy ra ta chỉ cần chứng minh X X 5 a2 b2 c2 3 abc 8 cyc cyc Chứng minh bất đẳng thức này bằng cách dùng kỹ thuật hàm lồi. Mở rộng thứ hai Đối với những bất đẳng thức đối xứng, chúng ta có thể làm như các cách ở trên, phần lớn đều giúp chúng ta đi đến kết quả. Nhưng đối với các bất đẳng thức hoán vị, lại chứa căn thức thì mọi chuyện lại không đơn giản như thế. Chúng ta không thể thiết lập những hàm số trung gian bậc nhất hay bậc hai rồi dựa vào chúng để chứng minh bài toán ban đầu được. Tuy nhiên, trong một số trường hợp, ta có thể tìm các hàm phân thức trung gian (trong một số trường hợp, ta cũng có thể thiết lập hàm bậc 2) để đánh giá các biểu thức trong căn nhằm giúp ta loại bỏ căn thức, điều này giúp ta dễ dàng hơn trong việc giải bài toán. Để thiết lập được các hàm phân thức này, chúng ta có rất nhiều cách, nhưng tốt hơn hết là ta hãy đi từ bất đẳng thức để suy ra bất đẳng thức, chẳng hạn từ bất đẳng thức Cauchy Schwarz, ta có p 2x2 + 2y 2 x + y 8x; y 0 Như vậy, ta có p 2x2 + 2y 2 x (x y)2 y=p 2x2 + 2y 2 + x + y (x y)2 2(x + y) Và như vậy, ta thiết lập được bất đẳng thức p 2x2 + 2y 2 x+y+ (x y)2 3x2 + 2xy + 3y 2 = 2(x + y) 2(x + y) Và nó đã giúp ta giải được bài toán rất khó sau Ví dụ 1.150 Cho các số dương x; y; z: Chứng minh rằng p p p x+ y+ z x y z p p +p +p : x+y y+z z+x 2 (Walther Janous) http://boxtailieu.net 168 CHƯƠNG 1. TÌM TÒI MỘT SỐ KỸ THUẬT GIẢI TOÁN Lời giải. Chú ý rằng với mọi x; y 0, ta có p 2(x2 + y 2 ) 2 3x + 2xy + 3y 2 2(x + y) Thật vậy, ta có (3x2 + 2xy + 3y 2 )2 8(x + y)2 (x2 + y 2 ) = (x y)4 0 p p p Trở lại bài toán, đặt a = x; b = y; c = z. Bất đẳng thức trở thành X cyc 1X a 2 cyc a2 p 2(a2 + b2 ) Sử dụng bất đẳng thức trên, ta chỉ cần chứng minh được X cyc , X cyc , Đặt x = 3c b 3b2 +2bc+3c2 X 4a2 (a + b) 3a2 + 2ab + 3b2 8a2 (a + b) 3a2 + 2ab + 3b2 X (a cyc 3a2 a cyc 3a + b b)2 (3b a) + 2ab + 3b2 0 0 và y; z tương tự. Ta phải chứng minh X x(b c)2 0 cyc Trường hợp 1. Nếu a b c, khi đó ta có y 2 0. Ta sẽ chứng minh 2 a y + 2b x 0 và x + 2z 0 Thật vậy, bất đẳng thức thứ nhất tương đương với a2 (3a c) 2b2 (3c b) + 2 2 + 2ac + 3c 3b + 2bc + 3c2 3a2 Nếu 3c b, bất đẳng thức là hiển nhiên. Nếu b một hàm nghịch biến theo b; do đó a2 (3a c) 2b2 (3c b) + 2 2 + 2ac + 3c 3b + 2bc + 3c2 3a2 0 3c, ta có thể kiểm tra được đây là a2 (3a c) 2a2 (3c a) + 2 2 + 2ac + 3c 3a + 2ac + 3c2 2 a (a + 5c) = 0 2 3a + 2ac + 3c2 = 3a2 http://boxtailieu.net 1.5. THE HYBERBOLIC FUNCTIONAL TECHNIQUE 169 Bất đẳng thức thứ 2 tương đương với 2(3b a) 3a c + 2 3a2 + 2ac + 3c2 3a + 2ab + 3b2 0 Đây là một hàm nghịch biến theo c nên 3a c 2(3b a) + 2 3a2 + 2ac + 3c2 3a + 2ab + 3b2 3a b 2(3b a) + 2 3a2 + 2ab + 3b2 3a + 2ab + 3b2 a + 5b = 0 2 3a + 2ab + 3b2 Chú ý rằng (a a2 (b b2 c)2 c)2 ; c)2 (a (a b)2 Nên 2 X x(b c)2 = [2x(b c)2 + y(a c)2 ] + [y(a c)2 + 2z(a b)2 ] cyc 2x(b = a2 (b b2 c)2 + y a2 y + 2b2 x (b b2 c)2 + [y(a c)2 + (a b)2 + 2z(a b)2 (y + 2z) b)2 ] 0 Trường hợp 2. Nếu c b a, thì ta có x; z 0. Nếu y 0, bất đẳng thức là hiển nhiên. Nếu y 0, tức là c 3a, xét nhứng trường hợp nhỏ sau i) Nếu 2b c + a, ta sẽ chứng minh z(a , b)2 + y(a c)2 0 (a b)2 (3b a) (a c)2 (3a c) + 2 3a2 + 2ab + 3b2 3a + 2ac + 3c2 0 Đây là một hàm đồng biêns theo b nên (a b)2 (3b a) 3a2 + 2ab + 3b2 a a+c 2 2 3 a+c 2 3a2 + a(a + c) + 3 a a+c 2 2 = (a c)2 (a + 3c) 2(19a2 + 10ac + 3c2 ) Mặt khác (a c)2 (3a c) (a c)2 (a + 3c) + 2 2 2 2(19a + 10ac + 3c ) 3a + 2ac + 3c2 (a c)2 (3a + c)(39a2 2ac + 3c2 ) = 2(3a2 + 2ac + 3c2 )(19a2 + 10ac + 3c2 ) http://boxtailieu.net 0 170 CHƯƠNG 1. TÌM TÒI MỘT SỐ KỸ THUẬT GIẢI TOÁN ) ii) Nếu a + c X b)2 + y(a ) z(a 2 x(b 2b và c 2 c) = x(b cyc 2b a c)2 c) + [z(a 0 2 c)2 ] b) + y(a 0 p 3 + 2 3 a, ta sẽ chứng minh z + 3y 0 và 3 x+ y 2 Bất đẳng thức thứ nhất tương đương với 3a2 0 3(3a c) 3b a + 2 2 + 2ab + 3b 3a + 2ac + 3c2 0 Đây là một hàm đồng biến theo c nên 3a c 3a2 + 2ac + 3c2 3a (2b a) 3a2 + 2a(2b a) + 3(2b a)2 = 2(a2 2a b 2ab + 3b2 ) Mặt khác, ta có 3(2a b) 3b a + 2(a2 2ab + 3b2 ) 3a2 + 2ab + 3b2 16a3 + 13a2 b 6ab2 + 9b3 = 2(a2 2ab + 3b2 )(3a2 + 2ab + 3b2 ) 0 Bất đẳng thức thứ hai tương đương với 3(3a c) 2(3c b) + 2 3b2 + 2bc + 3c2 3a + 2ac + 3c2 0 Đây là một hàm đồng biến theo a nên Nếu c 2b, ta có 2(3c b) 3(3a c) + 2 3b2 + 2bc + 3c2 3a + 2ac + 3c2 Nếu 2b 2(3c b) 3(3 0 c) + 3b2 + 2bc + 3c2 3 02 + 2 0 c + 3c2 3c2 4bc 3b2 0 = c(3b2 + 2bc + 3c2 ) c, ta có 3b2 2(3c b) 3(3a c) + 2 2 + 2bc + 3c 3a + 2ac + 3c2 2(3c b) 3[3(2b c) c] + 3b2 + 2bc + 3c2 3(2b c)2 + 2(2b c)c + 3c2 15b3 + 44b2 c 13bc2 6c3 = 0 2(3b2 2bc + c2 )(3b2 + 2bc + 3c2 ) http://boxtailieu.net 1.5. THE HYBERBOLIC FUNCTIONAL TECHNIQUE 171 Tiếp theo, sử dụng bất đẳng thức AM-GM, ta có c)2 (a = (a b)2 + (b = (a b)2 + (b (a b)2 + (b = Ta có X x(b 3 b)2 + (b 2 3(a c)2 (b cyc b)2 + z(b c)2 3 c)2 x + y + (a 2 p iii) Nếu a + c 2b và 3 + 2 3 a Schwarz, ta có x(c c)2 + 2(a b)(b c) p 1 2(a b) p (b c) c)2 + 2 2 1 c)2 + 2(a b)2 + (b c)2 2 a)2 2b [(c b 4, a)a b) + (b 1 1 x + z b)2 (z + 3y) 0 sử dụng bất đẳng thức Cauchy a)]2 = xz(c a)2 x+z Ta cần chứng minh xz +y 0 x+z , xy + yz + zx 0 X X X 3 , f (c) = 9 ab + 22 a2 bc 12 a2 b2 cyc cyc 3 cyc X a3 b 0 cyc Dễ dàng kiểm tra được f (c) là hàm đồng biến, suy ra f (c) f (b) = 2b(3a3 a2 b + 25ab2 3b3 ) 0: Bất đẳng thức được chứng minh. Đẳng thức xảy ra khi và chỉ khi x = y = z: Nhận xét ta p cũng có một lời giải khác rất hay của Peter Scholze như sau p 16 Chúng p Đặt a = x; b = y; c = z và bình phương 2 vế, ta có bất đẳng thức tương đương X cyc X a4 a2 b2 p + 2 a2 + b2 (a2 + b2 )(b2 + c2 ) cyc Sử dụng bất đẳng thức sắp xếp lại cho 2 dãy p a2 b2 b2 c2 c2 a2 ;p ;p a2 + b2 b2 + c2 c2 + a2 http://boxtailieu.net 1 2 X cyc !2 a 172 CHƯƠNG 1. TÌM TÒI MỘT SỐ KỸ THUẬT GIẢI TOÁN và p ta có X cyc p a2 b2 a2 + b2 1 1 1 ;p ;p a2 + b2 b2 + c2 c2 + a2 p 1 2 b + c2 X cyc p a2 b2 a2 + b2 p 1 + b2 a2 X a2 b2 = a2 + b2 cyc Ta cần chứng minh X cyc X 2a2 b2 a4 + a2 + b2 a2 + b2 cyc a X 4a2 b2 + a2 + b2 a2 + b2 cyc X a , X a4 + b4 ,2 , !2 X , cyc a cyc X 4a2 b2 2a4 + a2 + b2 a2 + b2 cyc X cyc !2 X 1 2 X cyc X cyc X 2a2 b2 a + a2 + b2 cyc 2 r a2 + b2 2 r cyc cyc X cyc 2a2 b2 a2 + b2 !2 !2 !2 a 0: hiển nhiên đúng. Một cách khác để thiết lập hàm phân thức trung gian là sử dụng bất đẳng thức AM-GM, chẳng hạn ta có p p 2(Ax + By) 8x2 + y 2 (Ax + By)2 + 8x2 + y 2 2 8x2 + y 2 = 8A; B; x; y 0 Ax + By Ax + By Chúng ta sẽ thiết lập một bất đẳng thức có đẳng thức xảy ra khi x = y; khi đó ta phải có A + B = 3. Ngoài ra, để bất đẳng thức này có độ chặt thì chúng ta nên chọn A; B sao cho bất đẳng thức này có đẳng thức tại 2 điểm. Vì ta cần dùng bất đẳng thức này để giải các bài toán hoán vị chứa căn thức (chú ý là các bất đẳng thức này thường có những điểm "nhạy cảm" là (x; y; 0)) nên tốt hơn hết là chúng ta sẽ chọn http://boxtailieu.net 1.5. THE HYBERBOLIC FUNCTIONAL TECHNIQUE 173 A; B sao cho bất đẳng thức trên có đẳng thức tại x = 1; y = 0 hoặc x = 0; y = 1: Nếu p ta chọn A; Bpsao cho bất đẳng thức có đẳng thức tại x = 1; y = 0 thì ta có A = 2 2 và B = 3 2 2; những giá trị này lẻ và sẽ gây trở ngại cho các tính toán của ta. Nếu ta chọn A; B sao cho bất đẳng thức tại x = 0; y = 1 thì ta được A = 2; B = 1 và ta thiết lập được bất đẳng thức p 8x2 + y 2 (2x + y)2 + 8x2 + y 2 6x2 + 2xy + y 2 = 2(2x + y) 2x + y Và ta giải được bài toán sau (cũng rất khó) Ví dụ 1.151 Cho các số không âm a; b; c; không có 2 số nào đồng thời bằng 0: Chứng minh rằng p p p (a + b + c)2 a 8b2 + c2 + b 8c2 + a2 + c 8a2 + b2 : (Võ Quốc Bá Cẩn) Lời giải. Chú ý rằng (6b2 + 2bc + c2 )2 (2b + c)2 ) p (8b2 + c2 ) = 4b2 (b c)2 (2b + c)2 6b2 + 2bc + c2 = 3b + c 2b + c 8b2 + c2 3bc 2b + c Do đó, ta chỉ cần chứng minh được X !2 cyc , 3abc X a X cyc 3bc 2b + c a 3b + c cyc X 1 + a2 2b + c cyc 2 X cyc Sử dụng bất đẳng thức Cauchy Schwarz, ta có X cyc 1 2b + c 3 P a X bc cyc Do đó, ta chỉ cần chứng minh 9abc X 2 P + a a cyc cyc 2 cyc http://boxtailieu.net 0 bc 0 0 174 CHƯƠNG 1. TÌM TÒI MỘT SỐ KỸ THUẬT GIẢI TOÁN , X a3 + 3abc cyc X bc(b + c): cyc Đây chính là bất đẳng thức Schur bậc 3. Vậy ta có đpcm. Đẳng thức xảy ra khi và chỉ khi a = b = c. Đôi khi chúng ta cũng có thể bắt đầu từ việc sử dụng liên phân số, chẳng hạn xuất phát từ bất đẳng thức hiển nhiên sau p 4x2 + y 2 2x + y 8x; y 0 Ta có p 4x2 + y 2 2x y = = 4xy p 2 4x + y 2 + 2x + y 4xy 2(2x + y) p 2 4xy 2 4x +y +2x+y = Chẳng hạn, ta sẽ sử dụng đẳng thức p 4x2 + y 2 2x y = kết hợp với 2(2x + y) 4xy p 4xy 4x2 +y 2 +2x+y p 4x2 + y 2 2x + y; ta thiết lập được p 4xy 4x2 + y 2 2x y = 2(2x + y) p 4xy 4x2 +y 2 +2x+y 4xy 2(2x + y) = ) p 4x2 + y 2 4xy 2(2x+y) 2xy(2x + y) 4x2 + 3xy + y 2 2xy(2x + y) 4x2 + 3xy + y 2 (2x + y)(4x2 + xy + y 2 ) 4x2 + 3xy + y 2 2x + y = Ta giải được bài toán sau Ví dụ 1.152 Cho các số không âm a; b; c; không có 2 số nào đồng thời bằng 0: Chứng minh rằng p p p 3 (a + b + c)2 a 4b2 + c2 + b 4c2 + a2 + c 4a2 + b2 : 4 (Võ Quốc Bá Cẩn) http://boxtailieu.net 1.5. THE HYBERBOLIC FUNCTIONAL TECHNIQUE 175 Lời giải. Chú ý rằng (2b + c)2 (4b2 + bc + c2 )2 (4b2 + 3bc + c2 )2 p 4b2 + c2 ) 4b2 (2b + c)(4b2 + bc + c2 ) = 2b + c 4b2 + 3bc + c2 Do đó, ta chỉ cần chứng minh được !2 X 3 X a 2b + c a 4 cyc cyc , 8abc 4b3 c3 (4b2 + 3bc + c2 )2 c2 = X 2bc(2b + c) 4b2 + 3bc + c2 2bc(2b + c) 4b2 + 3bc + c2 X 2b + c + 3 a2 4b2 + 3bc + c2 cyc cyc 0 6 X bc 0 cyc Để chứng minh bất đẳng thức này, ta chỉ cần chứng minh được 8 X cyc 4b2 27 P a 2b + c + 3bc + c2 cyc khi đó, bất đẳng thức trên là một hệ quả của bất đẳng thức Schur vì X 27abc P +3 a2 a cyc 6 X bc 0 cyc cyc , X X a3 + 3abc cyc bc(b + c) cyc Do đó, tất cả chúng ta phải làm bây giờ là chứng minh 8 X cyc ,64 X a5 b + 32 cyc X 4b2 ab5 + 68 X X ab(16a2 a2 b4 128 cyc X 3 a + 147 cyc ,4 27 P a cyc cyc + abc 132 2b + c + 3bc + c2 X cyc X 2 ab 243 cyc ab + 8b2 )(a a4 b2 + 60 X cyc 2 a b cyc X b)2 + 4 cyc a2 b2 (a2 + abc 132 cyc 3 a + 147 X 2 ab 243 cyc http://boxtailieu.net X cyc a3 b3 ! 396abc 0 11ab + 34b2 ) cyc X X 2 a b ! 396abc 0 176 CHƯƠNG 1. TÌM TÒI MỘT SỐ KỸ THUẬT GIẢI TOÁN ,4 X ab(16a2 ab + 8b2 )(a b)2 + cyc X a2 b2 (2a X 11b)2 + 15 cyc + abc 132 X 3 a + 147 cyc X a2 b4 cyc 2 ab 243 cyc X ! 2 a b 396abc cyc 0 Sử dụng bất đẳng thức AM-GM, ta có X a2 b4 abc cyc X a2 b cyc và X a2 b2 (2a 11b)2 = cyc 1X 2 2 [a b (2a 11b)2 + b2 c2 (2b 2 cyc X ab2 c(2a 11b)(2b 11c) 11c)2 ] cyc = abc 121 X 2 a b+4 cyc X 2 ab 22 cyc X ! 3 a 66abc cyc Ta cần chứng minh 121 X a2 b + 4 cyc X ab2 22 cyc + 132 X X , 107 X cyc X a + 147 a X 2 ab a3 + 151 X ab2 cyc 2 a b ! + X X a2 b cyc a2 b 396abc 0 cyc 107 cyc X 243 cyc cyc 3 66abc + 15 cyc 3 cyc , 110 a3 X a2 b 462abc 0 cyc 3 X cyc 3 a + 151 X cyc 2 ab ! 462abc 0: hiển nhiên đúng theo bất đẳng thức AM-GM. Vậy ta có đpcm. Đẳng thức xảy ra khi và chỉ khi a = b; c = 0 hoặc các hoán vị tương ứng. Ví dụ 1.153 Cho các số không âm a; b; c; không có 2 số nào đồng thời bằng 0: Chứng minh rằng p p p a2 + b2 + c2 + ab + bc + ca a 3b2 + c2 + b 3c2 + a2 + c 3a2 + b2 : http://boxtailieu.net 1.5. THE HYBERBOLIC FUNCTIONAL TECHNIQUE 177 Lời giải. Chú ý rằng (2b2 + bc + c2 )2 (b + c)2 ) Ta cần chứng minh p a2 + cyc X X ab cyc , 2abc c2 = b2 (b c)2 (b + c)2 2b2 + bc + c2 = 2b + c b+c 3b2 + c2 X 3b2 cyc cyc 2 2bc b+c 2bc b+c a 2b + c X 1 + a2 b + c cyc X 0 X ab 0 cyc Sử dụng bất đẳng thức Cauchy Schwarz, ta có X cyc Ta cần chứng minh 1 b+c 2 ab 0 cyc a3 + 3abc cyc X cyc cyc X a 2 9abc X 2 P + a a cyc , 9 P X bc(b + c): cyc hiển nhiên đúng vì đây chính là bất đẳng thức Schur bậc 3. Vậy ta có đpcm. Đẳng thức xảy ra khi và chỉ khi a = b = c: Nhận xét 17 Một cách tổng quát, ta có kết quả sau với mọi k > 0 2 2 2 a + b + c + ab + bc + ca p k+1 2 p p p a kb2 + c2 + b kc2 + a2 + c ka2 + b2 (Võ Quốc Bá Cẩn, Vasile Cirtoaje) Thật vậy, sử dụng bất đẳng thức Cauchy Schwarz, ta có X p a kb2 + c2 cyc !2 X cyc !" a X cyc http://boxtailieu.net 2 2 # a(kb + c ) 178 CHƯƠNG 1. TÌM TÒI MỘT SỐ KỸ THUẬT GIẢI TOÁN Từ đây, ta thấy bất đẳng thức trên được suy ra từ 2 bất đẳng thức sau !2 ! ! X X X X 2 2 a + ab 4 a a b cyc X cyc 2 a + cyc X cyc !2 4 ab cyc X cyc cyc ! X a 2 ab cyc ! Ta sẽ chứng minh bất đẳng thức thứ nhất, bất đẳng thức thức 2 được chứng minh tương tự. Ta có bất đẳng thức tương đương X X X X a4 a2 b2 + 2 ab3 2 a3 b 0 cyc cyc cyc cyc Giả sử c = min fa; b; cg ; đặt a = c + x; b = c + y với x; y trở thành 4(x2 xy + y 2 )c2 + 4[x(x y)2 + y 3 ]c + (x2 0 thì bất đẳng thức này xy y 2 )2 0: Bất đẳng thức này hiển nhiên đúng nên ta có đpcm. Chúng ta có kết quả tổng quát của bất đẳng thức !2 ! ! X X X X a2 + ab 4 a a2 b cyc cyc cyc cyc là [(q + r)a + (r + p)b + (p + q)c]2 4(p + q + r)(pbc + qca + rab) với p; q; r là các số không âm và a; b; c là các số thực tùy ý. Chứng minh bất đẳng thức này như sau Giả sử a = maxfa; b; cg, khi đó ta có [(q + r)a + (r + p)b + (p + q)c]2 = [(q r)a + (r + p)b Cho a; b; c 4(p + q + r)(pbc + qca + rab) (p + q)c]2 + 4qr(a b)(a c) 0 và p = bk ; q = ck ; r = ak ; ta được " #2 ! ! X X X k k k+1 a (a + b) 4 a a b cyc cyc Với k = 1; ta thu được bất đẳng thức ở trên. Với k = 1; ta được s c a b + + + 3 2 (a + b + c) a b c cyc 1 1 1 + + : a b c http://boxtailieu.net 0 1.6. CÁC DẠNG TỔNG BÌNH PHƯƠNG 179 Ví dụ 1.154 Cho các số không âm a; b; c; không có 2 số nào đồng thời bằng 0: Chứng minh rằng r r r p p p a3 b3 c3 a+ b+ c p + + : a2 + ab + b2 b2 + bc + c2 c2 + ca + a2 3 (Lê Trung Kiên) Hướng dẫn. Sử dụng bất đẳng thức AM-GM, ta có p p 2 3(x4 + x2 y 2 + y 4 ) = 2 (x2 + xy + y 2 ) 3(x2 xy + y 2 ) (x2 + xy + y 2 ) + 3(x2 xy + y 2 ) = 2(2x2 xy + 2y 2 ) và ta thiết lập được bất đẳng thức 2x2 1.6 xy + 2y 2 p 3(x4 + x2 y 2 + y 4 ): Các dạng tổng bình phương Kỹ thuật này dựa trên một kết quả hiển nhiên của bất đẳng thức là x2 0 8x 2 R; nó có thể giúp ta giải được những bài toán mà nếu dùng kỹ thuật thông thường thì rất khó (thông thường đây là các bất đẳng thức bậc 4). Chúng ta có định lý sau Định lý 1.6 Xét bất đẳng thức sau với các biến thực a; b; c X X X X X m a4 + n a2 b2 + p a3 b + g ab3 (m + n + p + g) a2 bc cyc cyc cyc cyc khi đó bất đẳng thức này đúng nếu m>0 3m(m + n) Chứng minh. Viết lại bất đẳng thức như sau ! X X X m a4 a2 b2 + (m + n) a2 b2 cyc cyc cyc +g X ab3 cyc Chú ý rằng X cyc a4 X X cyc ! a2 bc cyc a2 b2 = 0 cyc p2 + pg + g 2 X cyc ! a2 bc 0 1X 2 (a 2 cyc http://boxtailieu.net b2 )2 +p . X cyc a3 b X cyc ! a2 bc 180 CHƯƠNG 1. TÌM TÒI MỘT SỐ KỸ THUẬT GIẢI TOÁN X X a3 b cyc a2 bc = cyc X X b3 c cyc = a2 bc = cyc X X ab3 cyc X a2 bc = cyc bc(a cyc cyc = X b2 )(ab + ac X ca3 ab2 c = cyc ca(a2 X ca(a2 b2 ) cyc b2 ) 1X 2 (a 3 cyc b2 ) 2bc) X 1 (ab + bc + ca) (a2 3 cyc cyc = b2 ) 2 1X 2 (a 3 cyc X bc(a2 X 1 (a2 b ) + (ab + bc + ca) 3 cyc 2 cyc = X b2 )(ab + bc b2 ) 2ca) Do đó, bất đẳng thức trên tương đương với mX 2 (a 2 cyc b2 )2 + 1X 2 (a 3 cyc b2 )[(p +(m + n) X g)ab (2p + g)bc + (p + 2g)ca] X 2 2 a b cyc cyc 2 ! a bc 0 Mặt khác X a2 b2 cyc X a2 bc = cyc 6(p2 X 1 [(p 2 + pg + g ) cyc g)ab (2p + g)bc + (p + 2g)ca]2 Bất đẳng thức được viết lại thành mX 2 (a 2 cyc b2 )2 + + , 6(p2 1 X [3m(a2 18m cyc + 1X 2 (a 3 cyc b2 )[(p X m+n [(p 2 + pg + g ) cyc b2 ) + (p g)ab g)ab g)ab (2p + g)bc + (p + 2g)ca] (2p + g)bc + (p + 2g)ca]2 0 (2p + g)bc + (p + 2g)ca]2 3m(m + n) p2 pg g 2 X [(p 18m(p2 + pg + g 2 ) cyc g)ab http://boxtailieu.net (2p + g)bc + (p + 2g)ca]2 0 1.6. CÁC DẠNG TỔNG BÌNH PHƯƠNG 181 m>0 3m(m + n) đẳng thức trên hiển nhiên đúng. Định lý được chứng minh. Từ đây, ta có thể dễ dàng kiểm tra được với thì bất p2 + pg + g 2 Ví dụ 1.155 Cho các số thực a; b; c: Chứng minh rằng (a2 + b2 + c2 )2 3(a3 b + b3 c + c3 a): (Vasile Cirtoaje) Lời giải. Bất đẳng thức tương đương X X a4 + 2 a2 b2 cyc cyc Từ đây, ta được m = 1 > 0; n = 2; p = 3m(m + n) p2 X a3 b 3; g = 0, ta có g 2 = 3 1 (1 + 2) pg 0 cyc ( 3)2 ( 3) 0 02 = 0: Do đó, theo định lý của ta, bất đẳng thức được chứng minh. Ví dụ 1.156 Cho các số thực a; b; c: Chứng minh rằng p 3 p 3 1 abc(a + b + c) 3(a b + b3 c + c3 a): a4 + b4 + c4 + (Võ Quốc Bá Cẩn) p Lời giải. Ta có m = 1 > 0; n = 0; p = 3m(m + n) p2 3; g = 0 và p g 2 = 3 1 (1 + 0) pg p 2 3 3 0 02 = 0: Do đó bất đẳng thức cần chứng minh đúng. Ví dụ 1.157 Cho các số thực a; b; c: Chứng minh rằng 7(a4 + b4 + c4 ) + 10(a3 b + b3 c + c3 a) 0: (Phạm Văn Thuận) Lời giải. Ta sẽ chứng minh kết quả mạnh hơn là 7 X cyc , 86 X cyc a4 51 X cyc 4 a + 10 X 3 a b cyc a2 b2 + 101 X cyc a3 b 17 27 34 X cyc X cyc http://boxtailieu.net !4 a ab3 102 X cyc a2 bc 0 182 CHƯƠNG 1. TÌM TÒI MỘT SỐ KỸ THUẬT GIẢI TOÁN 8 m = 86 > 0 > > < n = 51 ) p = 101 > > : g = 34 Mặt khác, ta có 3m(m + n) p2 g 2 = 3 86 (86 pg 1012 51) ( 34)2 = 1107 > 0: 101 ( 34) Bất đẳng thức được chứng minh. Ví dụ 1.158 Cho các số thực a; b; c thỏa mãn abc = 1: Chứng minh rằng a2 1 + a + 1 b2 1 + b + 1 c2 1 c+1 3: (Vũ Đình Quý) Lời giải. Do abc = 1 nên tồn tại các số x; y; z sao cho a = xy ; b = yz ; c = thức trở thành X x2 3 2 x xy + y 2 cyc , , X X cyc 4 X cyc (x x2 2y)2 xy + y 2 bất đẳng 9 3x2 xy + y 2 x2 cyc , 3x2 xy + y 2 x2 x z 3 3 Sử dụng bất đẳng thức Cauchy Schwarz, ta có " X cyc (x x2 2y)2 xy + y 2 #" X (x 2y)2 #2 2 2 2y) (x 2 # " xy + y ) cyc X (x 2 2y) cyc #2 Ta cần chứng minh " , 10 X cyc X (x cyc x4 + 39 X cyc x2 y 2 3 X (x 2y)2 (x2 xy + y 2 ) cyc 25 X cyc x3 y 16 X cyc http://boxtailieu.net xy 3 8 X cyc x2 yz 0 1.6. CÁC DẠNG TỔNG BÌNH PHƯƠNG Từ đây, ta được m = 10 > 0; n = 39; p = 3m(m+n) p 2 183 25; g = 2 16 và 2 ( 25) ( 16) ( 16)2 = 189 > 0: pg g = 3 10 (10+39) ( 25) Bất đẳng thức được chứng minh. Ví dụ 1.159 Cho các số thực a; b; c: Chứng minh rằng (c + a b)2 (a + b c)2 (b + c a)2 + 2 + 2 2 2 2 a + (b + c) b + (c + a) c + (a + b)2 3 : 5 (Japan MO 1997) Lời giải. Đặt x = b + c a; y = c + a X b; z = a + b c; bất đẳng thức trở thành 2 (y + cyc ,2 z)2 X cyc x2 4x + (2x + y + z)2 3 5 x2 + (x + y + z)2 3 5 Sử dụng bất đẳng thức Cauchy Schwarz, ta có X cyc x2 x2 + (x + y + z)2 P X ,4 x4 + 14 cyc 2 x cyc X X !2 x2 y 2 cyc Từ đây, ta có m = 4 > 0; n = 14; p = 3m(m + n) p2 pg 2 x cyc !2 x2 [x2 + (x + y + z)2 ] cyc Ta cần chứng minh 10 P 3 X x2 [x2 + (x + y + z)2 ] cyc 6 X x3 y cyc 6; g = g 2 = 3 4 (4 + 14) 6 X xy 3 6xyz cyc X x 0 cyc 6 và ( 6)2 ( 6) ( 6) ( 6)2 = 108 > 0: Bất đẳng thức được chứng minh. Đẳng thức xảy ra khi và chỉ khi a = b = c: Ví dụ 1.160 Cho các số thực a; b; c: Chứng minh rằng (2a + b + c)2 (2b + c + a)2 (2c + a + b)2 + 2 + 2 2 2 2 2a + (b + c) 2b + (c + a) 2c + (a + b)2 8: (USA MO 2003) http://boxtailieu.net 184 CHƯƠNG 1. TÌM TÒI MỘT SỐ KỸ THUẬT GIẢI TOÁN Lời giải. Bất đẳng thức tương đương X (2a + b + c)2 2a2 + (b + c)2 3 cyc , Đặt x = b + c a; y = c + a X cyc X (b + c a)2 2a2 + (b + c)2 cyc b; z = a + b 2(y + z)2 1 1 2 c; khi đó bất đẳng thức trở thành x2 + (2x + y + z)2 1 8 Sử dụng bất đẳng thức Cauchy Schwarz, ta có X cyc x2 2 2(y + z) + (2x + y + z)2 P Ta cần chứng minh X 8 cyc ,2 X x4 + 5 cyc 2 x !2 X X p2 x2 y 2 x !2 x2 [2(y + z)2 + (2x + y + z)2 ] x2 [2(y + z)2 + (2x + y + z)2 ] 2 cyc pg cyc 2 cyc X x3 y cyc Từ đây, ta có m = 2 > 0; n = 5; p = 3m(m + n) P 2; g = g 2 = 3 2 (2 + 5) 2 X xy 3 cyc 3xyz X x 0 cyc 2 và ( 2)2 ( 2) ( 2) ( 2)2 = 30 > 0: Bất đẳng thức được chứng minh. Đẳng thức xảy ra khi và chỉ khi a = b = c: Ví dụ 1.161 Cho các số a; b; c p 0; a + b + c = 1: Chứng minh rằng a b c +p +p 2 2 4a + 5b 4b + 5c 4c + 5a2 3 p : 17 (Võ Quốc Bá Cẩn) Lời giải. Sử dụng bất đẳng thức Cauchy Schwarz, !2 ! ! X X X X a a a p a = 2 2 4a + 5b 4a + 5b2 4a + 5b cyc cyc cyc cyc http://boxtailieu.net 1.6. CÁC DẠNG TỔNG BÌNH PHƯƠNG nên ta chỉ cần chứng minh X cyc , 185 9 17 a 4a + 5b2 X cyc b2 4a + 5b2 3 17 Lại theo bất đẳng thức Cauchy Schwarz, ta có !2 !2 P 2 P 2 b a X cyc cyc b2 ! ! P 2 = b (4a + 5b2 ) 4a + 5b2 P P 2 P cyc cyc 4 a ab + 5 a4 cyc cyc cyc Ta cần chứng minh 17 X cyc , 2 a !2 X a4 + 11 p2 pg cyc 12 X cyc X cyc a2 b2 ! X a 6 cyc X ab3 2 ab ! 6 cyc + 15 X a4 cyc X a2 bc 0 cyc Từ đây, m=1>0 3m(m + n) g 2 = 3 1 (1 + 11) 02 ( 6)2 = 0 0 ( 6) nên bất đẳng thức trên đúng. Đẳng thức xảy ra khi a = b = c = 31 : Ví dụ 1.162 Cho các số thực a; b; c: Chứng minh rằng a4 + b4 + c4 + a3 b + b3 c + c3 a 2(a3 b + b3 c + c3 a): (Vasile Cirtoaje) Ví dụ 1.163 Cho các số thực a; b; c: Chứng minh rằng a(a + b)3 + b(b + c)3 + c(c + a)3 8 (a + b + c)4 : 27 (Phạm Văn Thuận, Võ Quốc Bá Cẩn) Ví dụ 1.164 Cho các số thực a; b; c: Chứng minh rằng 1 a4 + b4 + c4 + (ab + bc + ca)2 3 2(a3 b + b3 c + c3 a): (Phạm Kim Hùng) http://boxtailieu.net 186 CHƯƠNG 1. TÌM TÒI MỘT SỐ KỸ THUẬT GIẢI TOÁN 1.7 Hàm lồi, hàm bậc nhất Hàm lồi có những tính chất rất đặc biệt mà có thể giúp chúng ta giải toán một cách hiệu quả. Dưới đây là một số tính chất mà chúng tôi cho là cần thiết và phù hợp với chương trình toán THPT Định lý 1.7 Nếu f (x) lồi trên [a; b] thì f (x) max ff (a); f (b)g f (x) min ff (a); f (b)g : Nếu f (x) lõm trên [a; b] thì Tính chất này được suy ra từ định nghĩa của hàm lồi. Từ tính chất này, ta suy ra để chứng minh một bất đẳng thức f (x1 ; x2 ; :::; xn ) K Với f (x1 ; x2 ; :::; xn ) lồi trên [a; b] cho từng biến x1 ; x2 ; :::; xn và x1 ; x2 ; :::; xn 2 [a; b]; ta chỉ cần xét bất đẳng thức tại x1 = = xk = a xk+1 = = xn = b (k = 0; 1; :::n) Tương tự nếu f (x) là hàm lõm. Định lý 1.8 Nếu f (x) là hàm lồi và khả vi cấp 2 trên khoảng I thì f (x) f (y) + f 0 (y)(x y) 8x; y 2 I Nếu f (x) là hàm lõm và khả vi cấp 2 trên khoảng I thì f (x) f (y) + f 0 (y)(x y) 8x; y 2 I: Tính chất này ta có thể chứng minh dễ dàng bằng định lý Lagrange. Tính chất 2 được dùng để chứng minh các bất đẳng thức dạng f (x1 ) + f (x2 ) + + f (xn ) f (y1 ) + f (y2 ) + + f (yn ) kết hợp với kỹ thuật nhóm Abel, hoặc các bất đẳng thức dạng m1 f (x1 ) + m2 f (x2 ) + + mn f (xn ) m1 f (y1 ) + m2 f (y2 ) + trong đó ta có m1 f 0 (y1 ) = m2 f 0 (y2 ) = = mn f 0 (yn ) x1 + x2 + + xn = y1 + y2 + + yn Từ tính chất thứ hai, chúng ta suy ra được hệ quả sau http://boxtailieu.net + mn f (yn ) 1.7. HÀM LỒI, HÀM BẬC NHẤT 187 Hệ quả 1.6 Nếu f (x) lồi và khả vi cấp 2 trên khoảng I thì với mọi x; y; z 2 I thỏa x y z; ta có f (x) + f (z) f (y) + f (x + z y) Nếu f (x) lõm và khả vi cấp 2 trên khoảng I thì với mọi x; y; z 2 I thỏa x có f (x) + f (z) f (y) + f (x + z y): y z; ta Chứng minh. Ta sẽ chứng minh kết quả này trong trường hợp f (x) là hàm lồi (trường hợp hàm lõm chứng minh tương tự). Nếu y x + z y; theo tính chất 2, ta có f (x) f (z) ) f (x) + f (z) Nếu x + z y f (x + z = f (x + z f (y) + f 0 (y)(x y) + f 0 (x + z y) f 0 (x + z f (y) + f (x + z f (y) + f (x + z y) y)[z y)(x y) + [f 0 (y) y) (x + z y) y)] f 0 (x + z y)](x y) y; theo tính chất 2, ta có f (x) f (x + z = f (x + z f (z) ) f (x) + f (z) y) + f 0 (x + z y) + f 0 (x + z y)[x y)(y f (y) + f 0 (y)(z f (y) + f (x + z f (y) + f (x + z (x + z z) y)] y) y) + [f 0 (x + z y) y) f 0 (y)](y z) Hệ quả của ta được chứng minh xong. Có thể thấy những tính chất này được phát biểu rất đơn giản và nhẹ nhàng, nhưng ứng dụng của chúng thì lại rất lớn. Chúng ta xét 1 vài ví dụ Ví dụ 1.165 Cho các số a; b; c 2 [1; 2]: Chứng minh rằng a3 + b3 + c3 5abc: (Toán học tuổi trẻ) http://boxtailieu.net 188 CHƯƠNG 1. TÌM TÒI MỘT SỐ KỸ THUẬT GIẢI TOÁN Lời giải. Xét P (a; b; c) = a3 + b3 + c3 5abc, rõ ràng P (a; b; c) là hàm lồi lần lượt theo các biến a; b; c; do đó ta chỉ cần xét các trường hợp sau là đủ Trường hợp 1. a = b = c = 2; ta có P (a; b; c) = 16 < 0: Trường hợp 2. a = 1; b = c = 2; ta có P (a; b; c) = 3 < 0: Trường hợp 3. a = b = 1; c = 2; ta có P (a; b; c) = 0: Trường hợp 4. a = b = c = 1; ta có P (a; b; c) = 2 < 0: Do đó bất đẳng thức cần chứng minh đúng. Đẳng thức xảy ra khi a = b = 1; c = 2 và các hoán vị. k P Ví dụ 1.166 Cho dãy dương x1 ; x2 ; :::; xn thỏa p xi i=1 minh rằng x21 + x22 + 1 4 + x2n 1+ 1 + 2 + k 8k = 1; 2; :::; n: Chứng 1 n : (Titu Andreescu) Lời giải. Rõ ràng hàm số f (x) = x2 là hàm lồi, nên theo tính chất 2, ta có h i p p p p p p i i 1 + f0 i i 1 xi i i 1 8i = 1; 2; :::; n f (xi ) f Do đó n X n h X x2i i=1 p f p i p 1 + f0 i p i i h 1 i=1 = n X p f p i i n X 1 + i=1 p f0 p i i p xi 1 i=1 h p i p xi i i 1 p i ii 1 i Sử dụng kỹ thuật nhóm Abel, ta có n X f0 p p i i h 1 i=1 = n X1 h p f0 i p i +f 0 = n X1 h p p n p f0 i n p f0 1 i=1 1 i " p xi p i p +f 0 n p n i 2 i p i X xj i 4 j=1 n X xj i=1 n X n X i=1 p i=1 f0 1 1 1 i+1 p i+1 i=1 p i xj p n ! i p i 1 # 0 i p i X i @ xj j=1 http://boxtailieu.net i X p j=1 p 1 iA j p j 3 1 5 1.7. HÀM LỒI, HÀM BẬC NHẤT Do p i p i 1> n X1 h p p f0 p i+1 p i i i=1 n X x2i i=1 i=1 > i 8i = 1; 2; :::; n và f (x) lồi nên ta có 0 1 i i X p p p 1 f0 i+1 i @ iA xj 0 j=1 Từ đây, ta có n X 189 n X f p p i i+ p p i i 1 = 1 2 p n X p p i i 1 = i=1 i p i i=1 1 2 n = 4i i=1 n X 4i 2 p i p i 1 2 4i 1X1 : 4 i=1 i Bất đẳng thức được chứng minh xong. Ví dụ 1.167 Cho các số không âm a; b; c; không có 2 số nào đồng thời bằng 0: Chứng minh rằng p p p 3(a + b + c) 2 a2 + bc + b2 + ca + c2 + ab : (Phạm Kim Hùng) Lời giải. Nếu abc = 0; giả sử c = 0 thì dễ thấy bất đẳng thức hiển nhiên đúng. Nếu abc > 0; ta chuẩn hóa cho abc = 1 và giả sử a b c; khi đó đó tồn tại các số thực x y z sao cho a = ex ; b = ey ; c = ez thỏa p x + y + z = 0; bất đẳng thức trở thành f (x) + f (y) + f (z) 0 với f (t) = 3et 2 et + e t : Ta có f "(t) = 6e3t=2 (e3t + 1)3=2 4e6t 14e3t 2e2t (e2t + e t )3=2 1 f "(t) = 0 , 6e3t=2 (e3t + 1)3=2 = 4e6t + 14e3t + 1 , 36e3t (e3t + 1)3 = (4e6t + 14e3t + 1)2 , 36(e 3t + 1)3 = (4 + 14e 3 2 2 , 36(u + 1) = (4 + 14u + u ) , g(u) = u4 3t +e 6t 2 (u = e 3t 9u3 + 96u2 + 4u ) > 0) 20 = 0 Rõ ràng g(u) là hàm đồng biến, lại có g(0) = 20 < 0; g(1) = 73 > 0; nên tồn tại duy nhất u0 2 (0; 1) thỏa mãn g(u0 ) = 0; suy ra f "(t) có đúng một nghiệm t0 , từ đây dễ thấy f (t) lồi trên [t0 ; +1) và lõm trên ( 1; t0 ]: Trường hợp 1. Nếu y t0 ; khi đó sử dụng bất đẳng thức Jensen, ta có f (x) + f (y) 2f x+y 2 http://boxtailieu.net 190 CHƯƠNG 1. TÌM TÒI MỘT SỐ KỸ THUẬT GIẢI TOÁN Ta cần chứng minh x+y + f (z) 0 2 q p p p , 6 ab + 3c 4 ab + c ab + 2 c2 + ab q 2 p 2 p p , 6 ab + 3c 4 ab + c ab + 2 c2 + ab r p p 2 , 15ab + 20c ab + 5c ab + c ab (c2 + ab) 16 2f q 2 p p c2 + ab + 3c 4 ab c 0 p Bất đẳng thức cuối hiển nhiên đúng do ab c: Trường hợp 2. Nếu t0 y; khi đó ta có t0 y y + z t0 nên theo hệ quả của ta ,8 p ab + c ab f (y) + f (z) f (t0 ) + f (y + z t0 ) Mặt khác, theo bất đẳng thức Jensen thì f (x) + f (t0 ) 2f x + t0 2 N ên ta chỉ cần chứng minh 2f x + t0 2 + f (y + z t0 ) 0: Đây chính là trường hợp 1 mà ta đã xét ở trên. Bài toán được giải quyết xong. Đẳng thức xảy ra khi và chỉ khi 2 trong 3 số bằng nhau, số còn lại bằng 0. Ví dụ 1.168 Cho các số không âm a; b; c; d thỏa mãn a + b + c + d = 4: Chứng minh rằng 1 1 1 1 4 + + + : 48 11abc 48 11bcd 48 11cda 48 11dab 37 (Võ Quốc Bá Cẩn) ( 2 ( 2 z z x 0 42 4 =x,a=b . Lời giải. Đặt x = ab; y = cd; z = a+b; t = c+d ) ; với t t2 y 0 4 4 =y ,c=d Bất đẳng thức tương đương f (x; y) = = = 96 11xt 96 11yz + 2304 528xt + 121x2 y 2304 528yz + 121xy 2 1 1 96 11yz + + 48 11xc 48 11xd 2304 528yz + 121xy 2 1 1 96 11xt + + 48 11ya 48 11yb 2304 528xt + 121x2 y http://boxtailieu.net 4 37 1.7. HÀM LỒI, HÀM BẬC NHẤT 191 Có thể kiểm tra được f (x; y) là hàm lồi cho từng biến x; y; chẳng hạn 242d2 2(96 11yz) 242c2 + + 0 (48 11xc)3 (48 11xd)3 (2304 528yz + 121xy 2 ) 2 2 t2 z 42 = x; 4 = y 6 z 6 = x; y = 0 Do đó, ta chỉ cần xét các trường hợp sau là đủ 6 4 . Có thể thấy việc 2 4 x = 0; t = y 4 x=y=0 xét các trường hợp này tương đương với việc xét các trường hợp dưới đây 4 4 Trường hợp 1. a = c = 0; bất đẳng thức trở thành 48 37 : Trường hợp 2. a = b; c = 0; bất đẳng thức trở thành f "x (x; y) = 1 + 16 48 1 11abd 4 37 Sử dụng bất đẳng thức AM-GM, ta có 1 + 16 48 1 11abd Trường hợp 3. a = b; c = d ) c = 2 1 + 16 48 1 11 = 4 : 37 a; bất đẳng thức trở thành 2 2 + 2 1 a c 1 ac2 Thay c = 2 a+b+d 3 3 4 37 a vào và thu gọn, ta có bất đẳng thức tương đương 44(a 1)2 (48 22a 33a2 + 44a3 11a4 ) 37(48 22a2 + 11a3 )(48 44a + 44a2 11a3 ) 0: Bất đẳng thức này hiển nhiên đúng do a 2: Ta có đpcm. Đẳng thức xảy ra khi a = b = c = d = 1 hoặc ba trong bốn số a; b; c; d bằng 34 ; số còn lại bằng 0. Ví dụ 1.169 Cho các số dương a; b; c thỏa mãn a + b + c = 3: Tìm giá trị nhỏ nhất của biểu thức P = a3 + 2b3 + 3c3 : Lời giải. Rõ ràng hàm f (x) = x3 là hàm lồi, do đó f (a) f (A) + f 0 (A)(a A) f (b) f (B) + f 0 (B)(b B) f (c) f (C) + f 0 (C)(c C) http://boxtailieu.net 192 CHƯƠNG 1. TÌM TÒI MỘT SỐ KỸ THUẬT GIẢI TOÁN f (A) + 2f (B) + 3f (C) + f 0 (A)(a )P A) + 2f 0 (B)(b B) + 3f 0 (C)(c 0 C) 0 0 f (A) = 2f (B) = 3f (C) : A+B+C =3 Ý tưởng của ta là chọn các số dương A; B; C sao cho Khi đó, ta sẽ có P f (A) + 2f (B) + 3f (C) Như vậy, việc của ta còn lại chỉ là giải hệ phương trình f 0 (A) = 2f 0 (B) = 3f 0 (C) : A+B+C =3 Hệ này rất dễ giải nên xin được dành cho các bạn. Ví dụ 1.170 Cho các số dương a1 ; a2 ; :::; an : Chứng minh rằng (a21 (a1 + a2 + + an )2 2 + 1)(a2 + 1) (a2n + 1) (n 1)n nn 2 1 : (Vasile Cirtoaje) Lời giải. Bổ đề. Cho f là một hàm lõm trên [a; b], khi đó với mọi x1 ; x2 ; :::; xn 2 [a; b] thỏa mãn x1 + x2 + + xn (n 1)a b ta có f (x1 ) + f (x2 ) + + f (xn ) (n 1)f (a) + f (x1 + x2 + + xn (n 1)a) Chứng minh. Ta sẽ chứng minh bằng quy nạp theo n: Nếu n = 1; bất đẳng thức là hiển nhiên. Giả sử khẳng định đúng với n; ta sẽ chứng minh nó cũng đúng với n + 1: Thật vậy, giả sử xn+1 = max fx1 ; xn ; :::; xn+1 g ; khi đó theo giả thiết quy nạp, ta có f (x1 ) + f (x2 ) + + f (xn ) (n 1)f (a) + f (x1 + x2 + + xn (n 1)a) Ta cần chứng minh f (xn+1 ) + f (x1 + x2 + + xn (n 1)a) f (a) + f (x1 + x2 + + xn+1 na) Ta có b x1 + x2 + + xn+1 na x1 + x2 + + xn (n 1)a a Nên theo hệ quả của ta f (xn+1 ) + f (x1 + x2 + + xn (n 1)a) f (a) + f (x1 + x2 + Bổ đề được chứng minh. Trở lại bài toán của ta, bất đẳng thức đã cho tương đương với ! n n X X f (ai ) 2 ln ai + (n 2) ln n (n 1) ln(n i=1 i=1 http://boxtailieu.net + xn+1 1) na) : 1.7. HÀM LỒI, HÀM BẬC NHẤT 193 với f (x) = ln(1 + x2 ): Ta có f 00 (x) = 2(1 x2 ) (1 + x2 )2 nên f (x) lồi trên (0; 1] và lõm trên [1; +1): Không mất tính tổng quát, giả sử an an 1 a1 : Nếu tồn tại m sao cho am > 1; khi đó gọi k là chỉ số nhỏ nhất sao cho ak > 1; ta có an an 1 ak > 1 ak 1 a1 : Theo bổ đề trên, ta có f (ak ) + f (ak+1 ) + + f (an ) (n k)f (1) + f (ak + ak+1 + + an Nên ta chỉ cần chứng minh bất đẳng thức đã cho trong trường hợp 1 khi đó theo bất đẳng thức Jensen, ta có f (a1 ) + f (a2 ) + + f (an 1) a1 + a2 + nf an + an (n 1 1)n nn 1 2 (x2 + 1)n 1 a1 ; 1 n Và từ đây, ta suy ra được ta chỉ cần chứng minh bất đẳng thức khi a1 = a2 = an 1 = x; tức là (n k)) (a2n + 1) = 1)x + an ]2 [(n Sử dụng bất đẳng thức AM-GM, ta có (n 1)n 1 (x2 + 1)n 1 = n n 2 1 1)2 x2 + 1] [(n [(n n 1 +n n n 2 1 n 1 2 2 1) x + 1] Lại có [(n 1)2 x2 + 1](a2n + 1) [(n 1)x + an ]2 : Nên bất đẳng thức cần chứng minh đúng. Đẳng thức xảy ra khi và chỉ khi a1 = a2 = = an = 1: Ví dụ 1.171 Cho x; y; z 2 [0; 1]: Tìm giá trị lớn nhất của biểu thức P = 2(x3 + y 3 + z 3 ) x2 y y2 z z 2 x: Ví dụ 1.172 Cho các số không âm a; b; c; d thỏa mãn a + b + c + d = 4: Chứng minh rằng abc + bcd + cda + dab + a2 b2 c2 + b2 c2 d2 + c2 d2 a2 + d2 a2 b2 8: (Phan Thành Nam) http://boxtailieu.net 194 CHƯƠNG 1. TÌM TÒI MỘT SỐ KỸ THUẬT GIẢI TOÁN Ví dụ 1.173 Cho tam giác nhọn ABC: Chứng minh rằng cos B C A + cos + cos 2 2 2 4 p 3 1 + sin A B C sin sin 2 2 2 : (Jack Garfunkel) Ví dụ 1.174 Cho các số dương a; b; c thỏa mãn a + 3b + 4c = 1: Tìm giá trị lớn nhất của biểu thức p p p 3 P = 3 a + 4 b + 3 c: Ví dụ 1.175 Cho tam giác nhọn ABC: Tìm giá trị nhỏ nhất của biểu thức P = tan A + 2 tan B + 5 tan C: (VMEO 2005) Ví dụ 1.176 Cho các số dương a; b; c thỏa mãn 21ab + 2bc + 8ca 12: Tìm giá trị nhỏ nhất của biểu thức 1 2 3 P = + + : a b c (Vietnam TST 2001) Ví dụ 1.177 Cho dãy dương a1 ; a2 ; :::; an thỏa mãn k P ai i=1 Chứng minh rằng 1 1 + + a1 a2 + 1 an n : n+1 k P i=1 i(i + 1) 8k = 1; 2; :::; n: (Toán học tuổi trẻ) Ví dụ 1.178 Cho các số dương a1 ; a2 ; :::; an : Chứng minh rằng a1 + a2 + 2 (a1 + 1)(a22 + 1) + an (a2n + 1) (2n 1)n 2n nn 1 1 2 : (Vasile Cirtoaje) Tiếp theo, ta sẽ đi đến một tính chất cơ bản của hàm tuyến tính bậc nhất, đó là mọi hàm bậc nhất đều đơn điệu. Ý nghĩa của điều này là gì? Chúng ta hãy xét một trường hợp cụ thể, xét hàm f (x) = ax + b với x 2 [c; d]; nếu a 0; thì rõ ràng f (x) là hàm đồng biến, do đó f (c) f (x) f (d): Nếu a 0 thì f (x) là hàm nghịch biến, cho nên f (d) f (x) f (c): Như vậy, ta được Định lý 1.9 Cho hàm f (x) = ax + b (với a; b là các số thực tùy ý); khi đó với mọi x 2 [c; d]; ta có min ff (c); f (d)g f (x) max ff (c); f (d)g : http://boxtailieu.net 1.7. HÀM LỒI, HÀM BẬC NHẤT 195 Tính chất này rất đơn giản nhưng lại khá hiệu quả trong việc giải toán. Nó giúp ta thu gọn khoảng xét từ[c; d] đến việc xét 2 điểm cực biên là x = c và x = d: Ví dụ 1.179 Cho các số không âm x; y; z thỏa mãn x + y + z = 1: Chứng minh rằng x3 + y 3 + z 3 + 1 : 4 15 xyz 4 Lời giải. Biến đổi bất đẳng thức về dạng tương đương f (yz) = 27 x 4 1 1 yz + (1 4 2x)2 0 Dễ thấy đây là một hàm bậc nhất theo yz, hơn nữa dễ thấy (y + z)2 4 yz 0 Dựa trên định lý trên, ta chỉ cần xét bất đẳng thức tại yz = yz = 0 là đủ. Ta có f (0) = 1 (1 4 2x)2 0; x)2 (1 f 4 = 3 x(1 16 (y+z)2 4 3x)2 = (1 x)2 4 và 0: Nên bất đẳng thức cần chứng minh hiển nhiên đúng. Ví dụ 1.180 Cho các số không âm x; y; z thỏa mãn x + y + z = 1: Chứng minh rằng xy + yz + zx 2xyz 7 : 27 Ví dụ 1.181 Cho các số không âm a; b; c; d: Chứng minh rằng r a + a+b+c r b + b+c+d r c + c+d+a r d d+a+b 4 p : 3 (Phạm Văn Thuận) Ví dụ 1.182 Cho các số không âm a; b; c thỏa mãn a + b + c = 1: Chứng minh rằng a b c + + 1 + bc 1 + ca 1 + ab http://boxtailieu.net 9 : 10 196 1.8 CHƯƠNG 1. TÌM TÒI MỘT SỐ KỸ THUẬT GIẢI TOÁN Quy nạp Quy nạp là một trong những kỹ thuật rất hay của bất đẳng thức. Ý tưởng rất đơn giản như sau, để chứng minh một bất đẳng thức cho n biến, chúng ta sẽ chứng minh bất đẳng thức cho một trường hợp cụ thể, rồi giả định rằng nó đúng trong trường hợp n = k; khi đó ta sẽ chứng minh nó đúng cho n = k + 1; dựa trên cơ sở này chúng ta có kết luận nó đúng với mọi n: Ví dụ 1.183 Cho các số dương x1 ; x2 ; :::; xn > 0 (n n X i=1 x2i x2i + xi+1 xi+2 3): Chứng minh rằng n 1 trong đó xn+1 = x1 ; xn+2 = x2 : Lời giải. Trước hết, ta sẽ chứng minh bất đẳng thức đúng khi n = 3; thật vậy, ta cần chứng minh X x2 2 x2 + yz cyc X yz 1 , x2 + yz cyc Sử dụng bất đẳng thức Cauchy Schwarz, ta có X cyc yz 2 x + yz X cyc yz 2 2x + yz P P yz cyc !2 yz(2x2 + yz) =1 cyc Tiếp theo, ta giả sử bất đẳng thức đúng khi n = k (k đúng khi n = k + 1; tức là k+1 X x2i k 2 x + xi+1 xi+2 i=1 i 3), ta sẽ chứng minh nó cũng Giả sử xk+1 = max fx1 ; x2 ; :::; xk+1 g : Sử dụng giả thiết quy nạp, ta có k X i=1 x2i x2i + xi+1 xi+2 k 1 Nên để chứng minh bất đẳng thức trên, ta chỉ cần chứng minh x2k+1 x2k + 2 + 2 2 xk+1 + x1 x2 xk + xk+1 x1 xk x2k 1 1 + xk xk+1 x2k 2 xk + x1 x2 http://boxtailieu.net x2k x2k 1 1 + xk x1 1 1.8. QUY NẠP , 197 x2k+1 x2k+1 + x1 x2 1 +x2k , 1 x2k + x2k x2k 1 + xk x1 1 1 + x1 x2 x2k 1 + xk+1 x1 x2k 1 + xk xk+1 1 x1 x2 x2k x1 (xk+1 x2 ) + + x2k+1 + x1 x2 (x2k + x1 x2 )(x2k + xk+1 x1 ) (x2k 0 x2k 1 xk (xk+1 x1 ) 2 1 + xk x1 )(xk 1 + xk xk+1 ) 0: Bất đẳng thức cuối hiển nhiên đúng. Từ đây ta có đpcm. Ví dụ 1.184 Cho các số dương a1 ; a2 ; :::; an thỏa mãn a1 a2 rằng 1 1 1 + + + n 1 + a1 n 1 + a2 n 1 + an an = 1: Chứng minh 1: (Vasile Cirtoaje) Lời giải. Ta chứng minh kết quả tổng quát hơn 1 1 + + mn + a1 mn + a2 + 1 mn + an 1 8mn n 1 Với n = 1 thì bất đẳng thức hiển nhiên đúng. Giả sử bất đẳng thức đúng với n = k; khi đó ta sẽ chứng minh bất đẳng thức cũng đúng cho n = k + 1: Thật vậy, giả sử p ak+1 = max fa1 ; a2 ; :::; ak+1 g ) b = k a1 a2 ak 1: Đặt bi = abi 8i = 1; 2; :::; k ) mk+1 b1 b2 bk = 1: Chú ý là b mk+1 n > n 1 nên theo giả thiết quy nạp, ta có 1 mk+1 + a1 1 = b 1 + + 1 + mk+1 + a2 mk+1 + ak 1 1 + mk+1 + + mk+1 + b1 + b2 b b k k = mk+1 mk+1 + b b +1 b 1 mk+1 b + bk Do đó ta chỉ cần chứng minh k 1 + mk+1 + b mk+1 + ak+1 , , (b k mk+1 + b 1)2 k X1 + bk bk mk+1 + 1 [mk+1 (k + 1) n mk+1 + 1 n mk+1 + 1 i(mk+1 + 1)]bk i 0 i=1 Bất đẳng thức cuối hiển nhiên đúng do mk+1 minh đúng. Ta có đpcm. n do đó bất đẳng thức cần chứng http://boxtailieu.net 198 CHƯƠNG 1. TÌM TÒI MỘT SỐ KỸ THUẬT GIẢI TOÁN Ví dụ 1.185 Cho các số dương a1 ; a2 ; :::; an (n minh rằng a21 + a22 + + a2n p n (n n 2) thỏa mãn a1 a2 2n 1)n 1 (a1 + a2 + an = 1: Chứng + an n): Lời giải. Ta sẽ chứng minh kết quả tổng quát hơn a21 + a22 + + a2n n mn (a1 + a2 + + an n) 8mn Với n = 2; bất đẳng thức trở thành a21 + a22 2 m2 (a1 + a2 1 a21 2 m2 a1 + , a21 + 1)2 [(a1 + 1)2 a21 (a1 , 1 a1 22 (2 1)2 2n 1)n 1 2) 2 m2 a1 ] p Bất đẳng thức này hiển nhiên đúng vì m2 p n (n 1 0 = 4: Giả sử bất đẳng thức đúng khi n = k (k 2); ta sẽ chứng minh bất đẳng thức đúng p khi n = k + 1: Thật vậy, giả sử ak+1 = min fa1 ; a2 ; :::; ak+1 g ) b = k a1 a2 ak 1: ai Đặt bi = b 8i = 1; 2; :::; k ) b1 b2 bk = 1: Ta có mk mk+1 ; thật vậy với k = 2; 3; bất đẳng thức hiển nhiên đúng, xét với k > 3; ta có 2k p k 1)k (k ,1+ , 1+ k2 kk 1 = k+1 (k 1) k 1 Từ đây, ta có mk+1 b 1+ 1 mk+1 b21 + b22 + p k + b2k k(k+1) 1 k2 2k (k 1)k k >2 1 1+ 1 k k 1 kk (k 1)k+1 1 1 k p k k(k+1) Do k > 3 nên 1+ p k+1 1 1 k2 mk+1 1 1 k 1 1 k 1 < e k 1 1+ nên theo giả thiết quy nạp mk+1 (b1 + b2 + b http://boxtailieu.net + bk k) 1 k 1 < mk+1 b 1 > : b(k + 1 mk+1 0 ak+1 bk ) kb)bk ] (k + 1 n(n + 1) kb)(1 + 2b + + kbk 1) 8k (k+1)2 : kb) (k+1)2 4k Vậy ta có đpcm. Ví dụ 1.187 Cho các số dương a1 ; a2 ; :::; an thỏa mãn a1 + a2 + + an = n: Chứng minh rằng 1 1 3 2 1 (a + a22 + + a2n n): + + + n a1 a2 an n 1 (Phạm Kim Hùng) Ví dụ 1.188 Cho các số dương a1 ; a2 ; :::; an thỏa mãn a1 + a2 + + an = n: Chứng minh rằng (n 1)(a21 + a22 + + a2n ) + na1 a2 an n2 : Ví dụ 1.189 Cho các số dương a1 ; a2 ; :::; an thỏa mãn a1 a2 2n 1 rằng với mọi 0 < kn (n 1)2 ; ta có p 1 1 +p + 1 + kn a1 1 + kn a2 +p 1 1 + kn an an = 1: Chứng minh p n : 1 + kn (Vasile Cirtoaje) http://boxtailieu.net Chương 2 Sáng tạo bất đẳng thức Bài toán 2.1 Cho các số dương a; b; c thỏa mãn a + b + c = 3: Chứng minh rằng 1 1 1 + + a3 + b2 + c b3 + c2 + a c3 + a2 + b 1: (Dương Đức Lâm) Lời giải. Sử dụng bất đẳng thức Cauchy Schwarz, ta có X 1 3 + b2 + c a cyc X a + b2 + c3 (a3 + b2 + c)(a + b2 + c3 ) cyc P P 3 + a2 + a3 2 3 X a+b +c cyc cyc = !2 2 2 2 2 (a + b + c ) P 2 cyc a = cyc Nên ta chỉ cần chứng minh được X cyc 2 a !2 3+ X a2 + cyc X a3 cyc Đặt q = ab + bc + ca; r = abc thì bất đẳng thức này tương đương với (9 2q)2 , 3(r 3 + (9 1) + (q 2q) + (27 3) 4(q 9q + 3r) 3)2 0: Bất đẳng thức cuối hiển nhiên đúng nên ta có đpcm. Đẳng thức xảy ra khi và chỉ khi a = b = c = 1: 201 http://boxtailieu.net 202 CHƯƠNG 2. SÁNG TẠO BẤT ĐẲNG THỨC Bài toán 2.2 Cho các số dương a; b; c thỏa mãn a + b + c = 3. Chứng minh rằng a b c + + 3a2 + abc + 27 3b2 + abc + 27 3c2 + abc + 27 3 : 31 Lời giải. Sử dụng bất đẳng thức Schur, ta có 3abc 4(ab + bc + ca) 9 Nên ta chỉ cần chứng minh được X cyc , , , với s = a 3a2 + X cyc X 9a2 1 cyc 4(ab+bc+ca) 9 3 + 27 3 31 3a + 4(ab + bc + ca) + 72 9a2 3 31 31a(a + b + c) + 4(ab + bc + ca) + 72 X 9a2 + 4(ab + bc + ca) + 8(a + b + c)2 0 31a(a + b + c) a2 + s cyc 0 4(ab+bc+ca)+72 : 9 , X (7a + 8c + 10b)(c cyc , , X (a b) cyc X cyc (a b)2 a) (7a + 8b + 10c)(a a2 + s 8a + 7b + 10c b2 + s 7a + 8b + 10c a2 + s b) 0 0 8a2 + 8b2 + 15ab + 10c(a + b) + s (a2 + s)(b2 + s) 0: Bất đẳng thức được chứng minh xong. Đẳng thức xảy ra khi và chỉ khi a = b = c = 1: Bài toán 2.3 Cho các số dương a; b; c: Chứng minh rằng a + 2b b + 2c c + 2a + + c + 2b a + 2c b + 2a 2 a b c + + b+c c+a a+b : (Dương Đức Lâm) http://boxtailieu.net 203 Lời giải. Bất đẳng thức tương đương với X a + 2b c + 2b cyc 2 , Do 2 a3 cyc P a2 b 2 cyc cyc ab(a + b) X " X 3 a2 b 0 và cyc 2 3 P X 2 a3 X cyc a + 3abc " # ab(a + b) X , 2 a3 2 cyc X ab2 + cyc X cyc X (2a + b)(2b + c)(2c + a) (a + b)(b + c)(c + a) 2 nên ta chỉ cần chứng minh được X 2 2 a3 cyc ab(a + b) cyc cyc X 3 a2 b cyc P a3 (a + b)(b + c)(c + a) (2a+b)(2b+c)(2c+a) (a+b)(b+c)(c+a) 3 3 cyc (2a + b)(2b + c)(2c + a) cyc P a3 + 3abc cyc X , 2 a3 + 3abc P P X a 2 b+c cyc 3 a2 b # ab(a + b) cyc 3abc 0 3abc 0: cyc Theo bất đẳng thức AM-GM, ta có X 2 a3 cyc X 2 ab2 cyc 0; X a2 b cyc Bất đẳng thức được chứng minh. Đẳng thức xảy ra khi và chỉ khi a = b = c: Bài toán 2.4 Cho các số dương a; b; c thỏa mãn (a + b + c) minh rằng p p p 7+8 2 5 5 a b c 7+5 5 + + 2 b c a 2 1 a + 1 b p 8 2 + 1 c = 10: Chứng : (Phạm Kim Hùng, Võ Quốc Bá Cẩn) Lời giải. Do tính thuần nhất, không mất tính tổng quát giả sử a + b + c = 1; đặt q = ab + bc + ca; r = abc thì ta có q = 10r: Ta có a b c ab2 + bc2 + ca2 + + = b c a abc http://boxtailieu.net 204 CHƯƠNG 2. SÁNG TẠO BẤT ĐẲNG THỨC Do đó, để chứng minh bất đẳng thức bên trái, ta chỉ cần xét nó trong trường hợp c b a là đủ, từ đó p q 3r + q 2 4q 3 + 2(9q 2)r 27r2 c ab2 + bc2 + ca2 a b + + = = b c a abc 2r v u q 2 q q 2 4q 3 + 2(9q 2) 10 27 10 7 1u = + t q 2 2 2 10 s q p 1 7 1 7 1 253 40 10q + 253 80 10 = + + 2 2 q 2 2 p p 7+8 2 5 5 = 2 Tiếp theo, ta sẽ chứng minh bất đẳng thức bên phải, rõ ràng ta chỉ cần xét nó trong trường hợp a b c là đủ, khi đó p q 3r q 2 4q 3 + 2(9q 2)r 27r2 c ab2 + bc2 + ca2 a b + + = = b c a abc 2r v u q 2 q 2 4q 3 + 2(9q 2) 10 27 10 7 1u tq = q 2 2 2 10 s q p 1 7 1 7 1 253 40 10q + 253 80 10 = 2 2 q 2 2 p p 7+5 5 8 2 = : 2 Bất đẳng thức được chứng minh xong. Đẳng thức ở bấtp đẳngp thức bên trái xảy p p p p p 2 2 5 10 10 10 5 2+2 5 ; b = ; a = và các hoán ra khi và chỉ khi c = 10 10+5 20 10 20 vị tương ứng. Đẳng thức ở bất đẳng thức bên phải xảy ra khi và chỉ khi a = p p p p p p p 10+5 2 2 5 10 10 5 2+2 5 10 10 ; b = 10 ; c = và các hoán vị tương ứng. 20 20 Bài toán 2.5 Cho các số dương a; b; c thỏa mãn a + b + c = 3: Chứng minh rằng p p p a4 + b4 + c4 + 4 ab + bc + ca 15: (Dương Đức Lâm) Lời giải. Trước hết, ta sẽ chứng minh rằng P P a a X 1 cyc cyc P + P 2 a+b ab 2 a cyc cyc cyc http://boxtailieu.net 205 , X X c(a + b) + ab a+b cyc P ! a cyc a+ 2 cyc P X ab , a+b cyc a P 2 ! ab cyc a2 cyc ! cyc P P P ! ab cyc a2 cyc Sử dụng bất đẳng thức Cauchy Schwarz, ta có X ab a+b cyc P P !2 ab cyc ab(a + b) cyc Nên ta chỉ cần chứng minh 2 X 2 a cyc ! X ! ab cyc , X X a cyc b)2 ab(a !" X # ab(a + b) cyc 0 cyc Trở lại bài toán của ta, sử dụng bất đẳng thức GM-HM, ta có Xp ab cyc 2 X ab a+b cyc Nên ta chỉ cần chứng minh được X X ab a4 + 8 a+b cyc cyc , , Theo trên, ta có X a4 + cyc X cyc a4 + 15 8 X ab(a + b + c) 3 cyc a+b X 1 8X 8 ab + abc 3 cyc 3 a+b cyc 1X 1 3 cyc a + b 1 1 P + P 2 ab 2 a cyc cyc http://boxtailieu.net 15 15 206 CHƯƠNG 2. SÁNG TẠO BẤT ĐẲNG THỨC Nên ta chỉ cần chứng minh được X 8X a4 + 3 cyc cyc 1 0 1 C B 1 ab + 8abc @ P + P 2 A ab 2 a Đặt q = ab + bc + ca; r = abc thì ta có X a4 = 81 15 cyc cyc 36q + 2q 2 + 12r cyc Nên bất đẳng thức tương đương với 50 12r(3 + 4q q 2 ) q + q2 + 3 q(9 2q) 33 Nếu 9 0 4q thì ta có 50 q + q2 3 33 0 nên bất đẳng thức đúng. Nếu 4q 9 thì theo bất đẳng thức Schur bậc 4, ta có r chứng minh 2(4q 9)(9 q)(3 + 4q 50 q + q2 + 33 3 3q(9 2q) , (99 50q + 3q 2 )q(9 , (q 2q) + 2(4q 9)(9 (4q 9)(9 q) 18 q2 ) q)(3 + 4q 3 3)(2q + 11q 2 117q + 162) 0 3 2 117q + 162 0 , f (q) = 2q + 11q nên ta chỉ cần 0 q2 ) 0 Dễ thấy f (q) là hàm lồi nên f (q) max f (3); f 9 4 = max 36; 729 32 < 0: Bất đẳng thức được chứng minh xong. Đẳng thức xảy ra khi và chỉ khi a = b = c = 1: Bài toán 2.6 Cho các số dương a; b; c thỏa mãn a b + c: Chứng minh rằng s a b c abc + + + 2: b+c c+a a+b (a + b)(b + c)(c + a) (Dương Đức Lâm) http://boxtailieu.net 207 Lời giải. Do a b + c nên (a + b)(a + c)(b + c) abc b+c bc a+b+c+ bc a b+c bc (a b c)(ab + ac bc) 2b + 2c + + bc b+c a(b + c) b+c bc 2(b + c)2 2b + 2c + = +1 9 bc b+c bc = = ) (a + b)(a + c)(b + c) 9abc Do đó VT X a + = b+c cyc p abc(a + b)(b + c)(c + a) (a + b)(b + c)(c + a) X a 3abc + b + c (a + b)(b + c)(c + a) cyc = (a b c)(a + b c)(a b + c) (a + b)(b + c)(c + a) 2 2 0: Bất đẳng thức được chứng minh xong. Đẳng thức xảy ra khi và chỉ khi a = b; c = 0 hoặc a = c; b = 0 hoặc a = 2b = 2c: Bài toán 2.7 Cho các số không âm a; b; c, không có 2 số nào đồng thời bằng 0: Chứng minh rằng r r r 2 2 2 1 3 a + bc 3 b + ca 3 c + ab + + 2+ p : 3 2 2 2 2 2 2 b +c c +a a +b 2 (Võ Quốc Bá Cẩn) Lời giải. Không mất tính tổng quát, giả sử a r 3 a2 + bc + b2 + c2 r 3 b2 + ca c2 + a2 b c; ta sẽ chứng minh ( r ) 2 2 3 4(a + b ) max 2; c2 + ab Chú ý rằng (a2 + bc)(b2 + ca) (a2 + c2 )(b2 + c2 ) nên r 3 a2 + bc + b2 + c2 r 3 b2 + ca c2 + a2 (a2 + c2 )(b2 + c2 ) =1 (a2 + c2 )(b2 + c2 ) s 26 (a2 + bc)(b2 + ca) (a2 + c2 )(b2 + c2 ) http://boxtailieu.net 2 208 CHƯƠNG 2. SÁNG TẠO BẤT ĐẲNG THỨC Ta cần chứng minh r r 2 2 2 a2 + bc 3 4(a + b ) 3 b + ca + 2 2 2 2 2 b +c c +a c + ab s ! r r 2 2 2 2 a2 + bc b2 + ca 3 b + ca 3 a + bc 3 (a + bc)(b + ca) + 2 +3 + , 2 b + c2 c + a2 (a2 + c2 )(b2 + c2 ) b2 + c2 c2 + a2 r 3 4(a2 + b2 ) c2 + ab Lại có b2 + ca b2 + c2 a2 + bc c(a = a2 + c2 b)(a2 + b2 + c2 + ab ac (a2 + c2 )(b2 + c2 ) b2 + ca a2 + bc 1 b2 + c2 a2 + c2 Do đó, theo bất đẳng thức AM-GM, s ! r r 2 2 2 2 3 a + bc 3 b + ca 3 (a + bc)(b + ca) 3 + (a2 + c2 )(b2 + c2 ) b2 + c2 c2 + a2 bc) 0 ) s 6 (a2 + bc)(b2 + ca) (a2 + c2 )(b2 + c2 ) 6(a2 + bc) a2 + c2 Từ đó, ta chỉ cần chứng minh được a2 + bc b2 + ca 6(a2 + bc) + 2 + 2 b2 + c2 c + a2 a + c2 , f (c) + g(c) trong đó f (c) = (a + 7b)c5 + 3(a2 3 g(c) = (a 2 2 b)(3b + 2ab + 4a b 4(a2 + b2 ) c2 + ab 0 b2 )c4 + 2(a + b)(a + 3b)bc3 3 2 2 3 4 0 2 3 3a )c + (b a + 6b a + a b )c + ab(a b)4 Ta sẽ chứng minh rằng g(c) 0. Nếu 3b3 + 2ab2 + 4a2 b 3a3 0, điều này là 3 2 2 hiển nhiên. Nếu 3b + 2ab + 4a b 3a3 0, khi đó do g(c) là hàm lõm theo c nên g(c) min fg(0); g(b)g, mà g(0) = ab(a b)4 0; g(b) = b 1 (a 4 b)[(2a2 6ab b2 )2 + 43b4 ] + 8b5 0 Khẳng định được chứng minh. Trở lại bài toán của ta, có 2 trường hợp xảy ra 2 2 +b2 1 Nếu ac2 +ab 2 , ac 2+ab +b2 2 , khi đó từ khẳng định trên, ta dễ dàng đi đến kết luận. Nếu a2 +b2 c2 +ab 2, khi đó từ khẳng định trên, ta chỉ cần chứng minh r r 2 2 2 1 3 4(a + b ) 3 c + ab + 2+ p 3 c2 + ab a2 + b2 2 http://boxtailieu.net 209 p p 3 2. Bất đẳng thức này hiển nhiên đúng do hàm 3 4x + x1 là hàm tăng với mọi x Bất đẳng thức được chứng minh xong. Đẳng thức xảy ra khi và chỉ khi a = b; c = 0 hoặc các hoán vị. Bài toán 2.8 Cho a; b; c là các số không âm thỏa mãn a + b + c = 3. Chứng minh rằng 3 a2 b + b2 c + abc 4: 2 (Vasile Cirtoaje, Võ Quốc Bá Cẩn) Lời giải. Nếu a 2b thì (a + c)2 b a2 b bc(a 3 abc = 2 b2 c 3 ) a2 b + b2 c + abc 2 (a + c)2 b = 4 a+c 2 4 Nếu 2b + a+c 2 3 2b + 2c) 2 0 a+c a+c b 2 2 3 +b =4 a, bất đẳng thức tương đương với f (c) = 4(a + b + c)3 Ta có 27a2 b 27b2 c 81 abc 2 0 3 [8(a + b + c)2 9b(3a + 2b)] 2 3 p f 0 (c) = 0 , c = p b(3a + 2b) a b 2 2 f 0 (c) = Do 2b f (c) 3 p a nên f 2 2 p b(3a + 2b) 3 p p b(3a + 2b) 2 2 a + b, và ta dễ dàng kiểm tra được a b = 2b)2 (2a + b) q 3 4 a2 + 5ab + 2b2 + b(3a+2b) 2 27ab(a 0: Bất đẳng thức được chứng minh. Đẳng thức xảy ra khi và chỉ khi a = 2; b = 1; c = 0 hoặc a = 0; b = 2; c = 1. Bài toán 2.9 Cho a; b; c là các số không âm, không có 2 số nào đồng thời bằng 0. Chứng minh rằng a2 1 1 1 + 2 + 2 2 2 + ab + b b + bc + c c + ca + a2 9 : (a + b + c)2 (Vasile Cirtoaje) http://boxtailieu.net 210 CHƯƠNG 2. SÁNG TẠO BẤT ĐẲNG THỨC Lời giải. Bất đẳng thức được viết lại như sau X (a + b + c)2 a2 + ab + b2 cyc , , X cyc ! a X cyc 1 6 X c(a + b + c) + ab + bc + ca cyc c 2 a + ab + b a2 + ab + b2 ! ! X X + ab 2 cyc cyc 6 1 2 a + ab + b2 ! 6 Sử dụng bất đẳng thức Cauchy Schwarz, ta có X cyc X cyc c 2 a + ab + b2 P P !2 cyc 1 a2 + ab + b2 P cyc c(a2 + ab + b2 ) cyc P cyc ab = P !2 P c c2 (a2 + ab + b2 ) cyc = P 2 !2 ab cyc P 2 P cyc !2 P a cyc + ab cyc cyc Nên ta chỉ cần chứng minh được !2 P a a cyc cyc P P c ! ab a 3abc P a cyc ab cyc !2 !2 3abc 6 P a cyc Do tính thuần nhất, ta có thể giả sử a + b + c = 1. Đặt q = P ab; r = abc, bất đẳng cyc thức trở thành q 1 + 2 q 2q 3r Sử dụng bất đẳng thức Schur bậc 4, ta có r 1 q + 2 q 2q 3r 6 1 + q 2q 2 6 (4q 1)(1 q) , 6 q (4q 1)(1 q) 2 6= nên (1 3q)(4q 1)2 q(8q 2 5q + 1) 0: Bất đẳng thức được chứng minh. Đẳng thức xảy ra khi và chỉ khi a = b = c: http://boxtailieu.net 211 Bài toán 2.10 Cho các số không âm a; b; c; không có 2 số nào đồng thời bằng 0: Chứng minh rằng b4 c4 a4 + + a2 + ab + b2 b2 + bc + c2 c2 + ca + a2 a3 + b3 + c3 : a+b+c (Phan Thành Việt) Lời giải. Ta có a3 + b3 + c3 = a2 + b2 + c2 a+b+c ab bc ca + 3abc a+b+c Nên bất đẳng thức đã cho tương đương với X a2 cyc , a4 + ab + ab + b2 X cyc a2 3abc a+b+c a2 ab3 + ab + b2 3abc a+b+c Sử dụng bất đẳng thức Cauchy Schwarz, ta có ! ! X X a2 + ab + b2 ab3 cyc a2 + ab + b2 X ab cyc cyc !2 a Nên ta chỉ cần chứng minh X cyc , , !2 3abc X a2 + ab + b2 a + b + c cyc ab a X cyc X cyc !3 a ! a 3 X c(a2 + ab + b2 ) cyc X cyc a2 X cyc ! ab 0: Điều này hiển nhiên đúng. Đẳng thức xảy ra khi và chỉ khi a = b = c hoặc c = 0; ab ! 0 hoặc các hoán vị tương ứng. Bài toán 2.11 Cho a; b; c; d là các số không âm thỏa mãn a + b + c + d = 1: Chứng minh rằng a b c d 3 p +p +p +p : 2 a+b b+c c+d d+a (Mircea Lascu) http://boxtailieu.net 212 CHƯƠNG 2. SÁNG TẠO BẤT ĐẲNG THỨC Lời giải. Không mất tính tổng quát, ta có thể giả sử a + c Sử dụng bất đẳng thức Jack Garfunkel, ta có p 5p 5p a+b+c= 1 4 4 b c a +p +p c+a a+b b+c a b +p a+b b+c c d a p +p +p a+c c+d d+a d c +p )p c+d d+a )p b+d ) x = a+c 5p 1 4 5p c c+a 5p a+c+d= 1 4 4 5p a 1 b p 4 a+c d 1 2: d p b Suy ra X cyc p 5 p 1 4 a a+b = b+ p 5p 2(2 b d) c+a 4 p p p ( x 1) (17 x 7) 3 3 p x= p + : p p 2 2 2 2 5 x + 1 + 2 (2 x + 3) p 5p 2(x + 1) 4 1 p d a+c Bất đẳng thức được chứng minh xong. Đẳng thức không xảy ra. Bài toán 2.12 Cho a; b; c; d là các số không âm, không có 3 số nào đồng thời bằng 0: Chứng minh rằng p a b c d +p +p +p a+b+c b+c+d c+d+a d+a+b 5p a + b + c + d: 4 Lời giải. Giả sử d = minfa; b; c; dg và đặt x = a + c; khi đó ta dễ thấy p a d +p a+b+c d+a+b p p a d x +p =p a+b+d d+a+b x+b b b+c+d p b b+c Nên ta chỉ cần chứng minh p x b c +p +p x+b b+c c+d+a ,p x b c +p +p c+x x+b b+c 5p a+b+c+d 4 5p x+b+d 4 Đây chính là bất đẳng thức Jack Garfunkel nên bất đẳng thức đã cho được chứng minh. Đẳng thức xảy ra khi và chỉ khi a3 = 1b = 0c = d0 hoặc các hoán vị tương ứng. http://boxtailieu.net 213 Bài toán 2.13 Cho a; b; c là các số dương. Chứng minh rằng 2(a + b + c)(bc + ca + ab) a2 + b2 + c2 + ab + bc + ca 1 ((2a + b)(2b + c)(2c + a)) 3 : (Sung Yoon Kim) Lời giải 1. Sử dụng bất đẳng thức Cauchy Schwarz, ta có a2 b + b2 c + c2 a (ab+bc+ca)2 . Suy ra a+b+c ) (2a + b)(2b + c)(2c + a) = 2(a + b + c)(ab + bc + ca) + 3abc + 2(a2 b + b2 c + c2 a) 2(a + b + c)(ab + bc + ca) + 3abc + 2(ab + bc + ca)2 a+b+c Ta cần chứng minh 2(a + b + c)(bc + ca + ab) a2 + b2 + c2 + ab + bc + ca 2(ab + bc + ca)2 2(a + b + c)(ab + bc + ca) + 3abc + a+b+c 1 3 Do tính thuần nhất, ta có thể chuẩn hóa cho a+b+c = 1: Đặt q = ab+bc+ca; r = abc; khi đó theo bất đẳng thức Schur bậc 3, ta được r max 0; 4q9 1 . Bất đẳng thức trở thành 1 2q (3r + 2q 2 + 2q) 3 1 q Nếu 1 4q; thì 1 1 (2q 2 + 2q) 3 (3r + 2q 2 + 2q) 3 và 2q 2 + 2q Nếu 4q 8q 3 (1 q)3 4q 2 + 2q 3 (1 q)3 2q(1 3q) 0 (1 q)3 2q(1 q4 ) 2q 2q(1 2q 4q 2 ) (1 q)3 1; thì 2 (3r + 2q + 2q) và = 6q 2 + 10q 3 1 1 3 2 2q + 2q + 8q 3 (1 = (1 q)3 4q 1 3 3q)(2q 4 1 3 6q 2 + 10q 3 1 2q 3 + 3q 2 + 10q (1 q)3 1) = http://boxtailieu.net 1 3 0: 214 CHƯƠNG 2. SÁNG TẠO BẤT ĐẲNG THỨC Đẳng thức xảy ra khi và chỉ khi a = b = c: Lời giải 2. Sử dụng bất đẳng thức Holder, ta có [(a + b + c)(bc + ca + ab)]3 = [(a + b)(b + c)(c + a) + abc]3 [(a + b)3 + a3 ][(b + c)3 + b3 ][(c + a)3 + c3 ] = (2a + b)(2b + c)(2c + a)(a2 + ab + b2 )(b2 + bc + c2 )(c2 + ca + a2 ) Ta cần chứng minh 8(a2 + ab + b2 )(b2 + bc + c2 )(c2 + ca + a2 ) (a2 + b2 + c2 + bc + ca + ab)3 Bất đẳng thức này có thể được chứng minh bằng phép khai triển. Lời giải 3. Ta sẽ chứng minh kết quả mạnh hơn là a + 2b b + 2c c + 2a + + a + 2c b + 2a c + 2b 3(a2 + b2 + c2 ) ab + bc + ca Cộng 3 vào hai vế, ta viết được bất đẳng thức trên ở dạng 2(a + b + c) 1 1 1 + + a + 2c b + 2a c + 2b 3(a2 + b2 + c2 ) +3 ab + bc + ca Từ đây, sử dụng bất đẳng thức AM-GM, ta có thể suy ra bất đẳng thức ban đầu của bài toán. Do (a + b + c)2 = (a + 2b)(a + 2c) + (b c)2 nên a+b+c a + 2b (b c)2 = + a + 2c a + b + c (a + 2c)(a + b + c) Xa+b+c ) cyc a + 2c =3+ X cyc (b c)2 (a + 2c)(a + b + c) Do đó, bất đẳng thức trên tương đương với 6+2 X cyc (b c)2 (a + 2c)(a + b + c) , x(b với x = 3(a+b+c) 4(ab+bc+ca) 1 a+2c c)2 + y(c 3(a2 + b2 + c2 ) +3 ab + bc + ca a)2 + z(a b)2 0 và y; z tương tự. Không mất tính tổng quát, giả sử b là số hạng nằm giữa a và c; tức là (b a)(b c) http://boxtailieu.net 0: 215 Từ đó, ta có (a b)(a c) 3(ab + bc + ca) + (a b)(a 3(ab + bc + ca) = a + 2b a + 2c nên x Nếu a 0 và (c a)(c c), ta có (a b)(a c) a + 2c b) 0: Chú ý rằng (a + 2b)(a + 2c) = a + 2b 2(a + b + c) 0: Tương tự, ta cũng có z 0: b c; khi đó dễ thấy y 0 do 1 b + 2a 1 a+b+c a+b+c 3(ab + bc + ca) 3(a + b + c) 4(ab + bc + ca) nên bất đẳng thức đúng. Nếu c b a và nếu y 0 nên bất đẳng thức đúng. Giả sử y x + 2y = do 9(a + b + c) 4(ab + bc + ca) 2 4a + b + 4c 1 + = a + 2c b + 2a (a + 2c)(b + 2a) và z + 2y = do 9(a + b + c) 4 9(a + b + c) 4(ab + bc + ca) 2 2a + 5b + 2c 1 + = c + 2b b + 2a (c + 2b)(b + 2a) c)2 + y(c 2 b + 2a 2(a + b + c) ab + bc + ca 1 c + 2b 2(a b)2 + 2(b a)2 + z(a b)2 (x + 2y)(b 0 9(a + b + c) 4(ab + bc + ca) 2 b + 2a 2(a + b + c) ab + bc + ca c)2 Từ đây, với chú ý rằng (a x(b 1 a + 2c 0; khi đó ta có 0 9(a + b + c) 4(ab + bc + ca) c)2 ; ta được c)2 + (z + 2y)(a b)2 0: Bất đẳng thức được chứng minh. Bài toán 2.14 Cho các số dương a; b; c: Chứng minh rằng 3(a2 + b2 + c2 ) a+b+c a(a + b) b(b + c) c(c + a) + + : a+c b+a c+b (Phạm Hữu Đức) Lời giải 1. Chú ý rằng X a(a + b) cyc a+c X ac a+c a+c cyc cyc ! ! X X a X ab = a a+c a+b cyc cyc cyc ! ! X X c X ab = a 3 a+c a+b cyc cyc cyc = X a(a + b + c) http://boxtailieu.net 216 CHƯƠNG 2. SÁNG TẠO BẤT ĐẲNG THỨC Ta có thể viết lại bất đẳng thức như sau P ! ! 3 a2 X X c X ab cyc P + a + a+b a+c a cyc cyc cyc X 3 a cyc cyc Sử dụng bất đẳng thức Cauchy Schwarz, ta có X ab a+b cyc P P !2 ab cyc ab(a + b) cyc = P cyc c a+c P !2 c(a + c) 3 a2 cyc P cyc a + P ! P P ! P P 3abc !3 a !2 P a cyc ab cyc cyc P 3abc !2 a + ab cyc ab a !2 ab cyc a cyc P cyc !2 ! cyc cyc P Ta cần chứng minh P a = cyc ab P c cyc P !2 cyc ! cyc và X P 3 ab X a cyc cyc Do tính thuần nhất, ta có thể chuẩn hóa cho a+b+c = 1: Đặt q = ab+bc+ca; r = abc; khi đó theo bất đẳng thức Schur bậc 3, ta có r 4q9 1 . Bất đẳng thức trở thành 3(1 , Ta có q2 1 + q 3r 1 q 6q 2q) + 1 q2 + q 3r 1 q q2 1 + q 3r 1 q q2 q 4q 1 3 + 6q 1 1 q 3 0 6q = (1 3q)2 1 q 0: Bất đẳng thức được chứng minh xong. Đẳng thức xảy ra khi và chỉ khi a = b = c: http://boxtailieu.net 217 Lời giải 2. Bất đẳng thức đã cho tương đương với X a(a + b)(a + b + c) 3(a2 + b2 + c2 ) a+c cyc X ab(a + b) , a+c cyc 2 X cyc Gọi x; y; z là một hoán vị của a; b; c sao cho x xy(x + y) a2 ab cyc y xz(x + z) X z; khi đó với chú ý rằng yz(y + z) Sử dụng bất đẳng thức sắp xếp lại, ta có X ab(a + b) cyc xy(x + y) yz(y + z) zx(z + x) + + y+z y+x z+x a+c = xy(x + y) yz(y + z) + + xz y+z y+x Bây giờ, ta thấy xy(x + y) yz(y + z) + y+z y+x xy(x z) yz(z x) + y+z x+y y(x z)2 (x + y + z) = (x (x + y)(y + z) (xy + yz) = z)2 Từ đây, ta có X ab(a + b) cyc a+c (x z)2 + (xy + yz + xz) 2(x2 + y 2 + z 2 ) (xy + yz + zx): Bất đẳng thức được chứng minh. Bài toán 2.15 Cho các số không âm a; b; c; không có 2 số nào cùng bằng 0: Chứng minh rằng r r r a b c 3 + + : b + 3c c + 3a a + 3b 2 (Vasile Cirtoaje) 2 2 b a c = z4 ; c+3a = y4 ; b+3c = Lời giải. Đặt a+3b đó, ta dễ dàng kiểm tra được đẳng thức sau x2 4 ; với x; y; z là các số không âm. Khi 16 = 7x2 y 2 z 2 + 3(x2 y 2 + x2 z 2 + y 2 z 2 ) http://boxtailieu.net 218 CHƯƠNG 2. SÁNG TẠO BẤT ĐẲNG THỨC và ta cần chứng minh x+y+z 3 Giả sử x + y + z < 3; khi đó tồn tại k > 1 sao cho kx + ky + kz = 3. Đặt kx = u; ky = v; kz = w thì u + v + w = 3 và 16 = 3(u2 v 2 + u2 w2 + v 2 w2 ) 7u2 v 2 w2 + < 7u2 v 2 w2 + 3(u2 v 2 + u2 w2 + v 2 w2 ) 6 k k4 Nhưng ta dễ dàng chứng minh được 16 7u2 v 2 w2 + 3(u2 v 2 + u2 w2 + v 2 w2 ) với mọi u; v; w 0 thỏa mãn u + v + w = 3; điều này dẫn đến mâu thuẫn nên bất đẳng thức cần chứng minh đúng. Đẳng thức xảy ra khi và chỉ khi a = b = c: Bài toán 2.16 Cho các số dương a; b; c: Chứng minh rằng p p p 3 3 3 (a + b)2 (b + c)2 (c + a)2 (a b)2 (b c)2 (c a)2 + 4 a2 b2 c2 : (Sung Yoon Kim) Lời giải. Sử dụng bất đẳng thức AM-GM, ta có s s b)2 (b c)2 (c a)2 a2 b2 c2 3 (a +43 2 2 2 2 (a + b) (b + c) (a + c) (a + b) (b + c)2 (a + c)2 s s 2 (b 2 (c 2 (a b) c) a) ab bc = 3 +43 (a + b)2 (b + c)2 (c + a)2 (a + b)2 (b + c)2 1 X (a b)2 4X ab + 2 3 cyc (a + b) 3 cyc (a + b)2 = ca (c + a)2 4ab 1 X (a b)2 + = 1: 3 cyc (a + b)2 (a + b)2 Bất đẳng thức được chứng minh. Đẳng thức xảy ra khi và chỉ khi a = b = c: Bài toán 2.17 Cho a; b; c là các số dương thỏa mãn a2 b2 + b2 c2 + c2 a2 = 3: Chứng minh rằng r r r a + bc2 b + ca2 c + ab2 3 + + : 2 2 2 abc (Sung Yoon Kim) http://boxtailieu.net 219 Lời giải. Sử dụng bất đẳng thứ AM-GM, ta có X cyc r a + bc2 2 = X cyc = r 1 4 2 X ab 1X 2 cyc ! 1 ab + b2 c2 b 2 1 ab + b2 c2 + b 2 X1 X + ab + 3 a cyc cyc và 1X 2 2 (a b + 1) = 3 2 cyc cyc P ab X1 cyc ) = a abc cyc Do đó X cyc r a + bc2 2 1 4 6 +6 abc 3 abc = 3 3 + 2abc 2 3 : abc Bất đẳng thức được chứng minh. Đẳng thức xảy ra khi và chỉ khi a = b = c = 1: Bài toán 2.18 Cho k > 0 là một hằng số cho trước. Tìm hằng số lớn nhất sao cho với mọi a b c d 0 thỏa a + b + c + d = 1; bất đẳng thức sau đúng a b b c c d d a + + + k+a+b k+b+c k+c+d k+d+a (a b)(b c)(c d): (Shalex) Lời giải. Cho a = trở thành 1 4 +3 ;b = 2 Cho ! 0+ , ta được 1 4 + ;c = 3 (2k + 1)3 1 4 ;d = 64 64(2k + 1) 1 4 2 64 (2k + 1)3 http://boxtailieu.net 3 với 0 ! 0+ , bất đẳng thức 220 CHƯƠNG 2. SÁNG TẠO BẤT ĐẲNG THỨC Mặt khác, đặt x = a b; y = b c; z = c d ) x; y; z 0, ta có a b b c c d d a + + + k+a+b k+b+c k+c+d k+d+a a b c d b c d a = + + + k+a+b k+c+d k+b+c k+d+a = [k(a b+c d) + (a2 b2 + c2 d2 )] 1 1 (k + a + b)(k + c + d) (k + b + c)(k + c + d) [(k + a + b)x + (k + c + d)z](x + y)(y + z) = (k + a + b)(k + b + c)(k + c + d)(k + d + a) p 8xyz (k + a + b)(k + c + d) (k + a + b)(k + b + c)(k + c + d)(k + d + a) 8xyz = p (k + a + b)(k + c + d)(k + b + c)2 (k + d + a)2 64 xyz: (2k + 1)3 với bất đẳng thức cuối đúng theo bất đẳng thức AM-GM và giả thiết a + b + c + d = 1: Từ đó, ta đi đến kết luận 64 : max = (2k + 1)3 n P Bài toán 2.19 Cho x1 ; x2 ; :::; xn là các số dương thỏa mãn xi = 1: Chứng minh i=1 rằng n q X x2i + x2i+1 2 p 2 2 i=1 + 1 n P i=1 x2i xi+1 : Lời giải. Bất đẳng thức đã cho tương đương n X xi + xi+1 n X x2i i=1 , q x2i + x2i+1 x2i i=1 x i+1 + xi + xi xi+1 q x2i + p 2 2 x2i+1 http://boxtailieu.net p + 1 n P i=1 x2i xi+1 1 n P x2i 2 + 2 xi+1 i=1 221 Sử dụng bất đẳng thức Cauchy Schwarz, ta có 1" 0 # n n q 2 2 X X x x x i i i 2 2 A @ q + xi + xi + xi+1 x2i xi xi+1 xi+1 2 + x2 + x + x i=1 i=1 x i i+1 i xi+1 i+1 !2 n X xi =1 i=1 Ta cần chứng minh n X i=1 xi q 2 xi + x2i+1 xi+1 , n X i=1 Do p xi xi+1 q p x2i xi+1 2 2 1 1 x2i + x2i+1 + xi q x2i + x2i+1 xi + xi+1 p 2 Nên ta chỉ cần chứng minh được n X i=1 , n X i=1 xx p i i+1 1 + 2 xi + xi+1 xx p i i+1 1 + 2 xi + xi+1 , n X i=1 p 2 n X i=1 3 1 p 2 2 ! p p 2 1 2 2 xi + xi+1 2 2 1 (xi xi+1 )2 p 1 + 2 xi + xi+1 2 0: Bất đẳng thức được chứng minh xong. Đẳng thức xảy ra khi và chỉ khi x1 = x2 = = xn = n1 : Bài toán 2.20 Cho các số thực a; b; c; d: Chứng minh rằng ! ! X X 5X 4 X 2 2 3 a + a b a a + (a b)(b c)(c 2 cyc sym cyc cyc d)(d a): (Phạm Minh Khoa) http://boxtailieu.net 222 CHƯƠNG 2. SÁNG TẠO BẤT ĐẲNG THỨC Lời giải. Do 5X 4 X 2 2 a + a b 2 cyc sym X 3 a cyc ! X ! a cyc X 2 a cyc X = b2 )2 !2 1X 2 (a 2 sym = X b)2 2 a cyc !2 Nên ta có thể viết lại bất đẳng thức như sau X 1X 2 b2 )2 (a ab(a b)2 + (a 2 sym sym , 1X (a 2 sym 1 2 (a Chú ý rằng a2 + b2 1X (a 2 sym b)2 (a2 + b2 ) (a ab(a sym b)(b b)(b c)(c c)(c d)(d d)(d a) a) b)2 , nên 1X (a 4 sym b)2 (a2 + b2 ) b)4 1 [(a b)4 + (b c)4 + (c d)4 + (d 4 j(a b)(b c)(c d)(d a)j (a b)(b c)(c d)(d a): a)4 ] Bất đẳng thức được chứng minh xong. Bài toán 2.21 Cho các số dương a; b; c thỏa a2 + b2 + c2 = 3: Chứng minh rằng r r r p a2 b2 c2 + + 3: 2 2 2 a +b+c b +c+a c +a+b Lời giải. Theo bất đẳng thức Cauchy Schwarz, ta có a + b + c 3. Từ đó !2 ! ! r X X X a2 a a a2 + b + c a2 + b + c cyc cyc cyc ! ! X X a a a2 + 31 (b + c)(a + b + c) cyc cyc Ta cần chứng minh X cyc ! a X cyc a a2 + 31 (b + c)(a + b + c) http://boxtailieu.net ! 3 223 Bất đẳng thức này thuần nhất nên ta có thể bỏ qua giả thiết a2 + b2 + c2 = 3 và chuẩn hóa cho a + b + c = 1; khi đó bất đẳng thức trở thành X cyc Ta có X cyc 3a 3a2 a + 1 3a2 3= 3a a+1 X cyc = 3a2 3 3a a+1 2a X (2a + 1)(3a cyc 3(3a2 1)2 a + 1) 1 3 0: Bất đẳng thức được chứng minh. Đẳng thức xảy ra khi và chỉ khi a = b = c = 1: Bài toán 2.22 Cho a; b; c là các số dương. Chứng minh rằng a b c + + b + c2 c + a2 a + b2 2 1 4 a b c + 2+ 2 c2 a b a b c + + b c a : Lời giải. Sử dụng bất đẳng thức AM-GM, ta có b c a + + b + c2 c + a2 a + b2 a c b p + p + p 2 ca 2 ab 2 bc p p p p p 1 a a b b c p = +p +p 2 c a a b c s 1 a b c a b + + + 2+ 2 2 2 c a b b c p ! c b c : a Bất đẳng thức được chứng minh. Đẳng thức xảy ra khi và chỉ khi a = b = c = 1: Bài toán 2.23 Cho các số dương a; b; c: Chứng minh rằng r r r a b c a2 + c2 b2 + a2 c2 + b2 + + + + : 2 2 2 2 b c a b +c c +a a2 + b2 (Võ Quốc Bá Cẩn) Lời giải 1. Bình phương 2 vế, ta có thể viết lại bất đẳng thức như sau ! r X a2 2b a2 + c2 b2 + c2 + 2 0 b2 a b2 + c2 a2 + c2 cyc http://boxtailieu.net 224 CHƯƠNG 2. SÁNG TẠO BẤT ĐẲNG THỨC Suy ra, ta chỉ cần chứng minh 2b a2 + b2 a , r a2 + c2 b2 + c2 2 b2 + c2 a2 + c2 r a3 c2 + 2b5 + 2b3 c2 ab2 c2 ab2 (b2 + c2 ) 2 0 b2 + c2 a2 + c2 Sử dụng bất đẳng thức AM-GM, ta có a3 c2 + 2b3 c2 ) a3 c2 + 2b5 + 2b3 c2 3ab2 c2 ab2 c2 Ta cần phải chứng minh r b3 + ac2 a(b2 + c2 ) 2b2 (b3 + ac2 ) b2 + c2 a2 + c2 Nhưng bất đẳng thức này hiển nhiên vì (b3 + ac2 )2 (a2 + c2 ) a2 (b2 + c2 )3 = c2 (a b)2 (a2 c2 + 2b3 a + 2c2 ab + b4 ) Đẳng thức xảy ra khi và chỉ khi a = b = c: Lời giải 2. Trước hết, ta sẽ chứng minh bất đẳng thức sau r r b a a2 + c2 b2 + c2 + + 2 2 b a b +c a2 + c2 r r a b a2 + c2 b2 + c2 , + 2 + 2 2 2 b a b +c a2 + c2 p p 2 a2 + c2 b2 + c2 (a b)2 p , ab (a2 + c2 )(b2 + c2 ) (a2 b2 )2 p p p 2 (a2 + c2 )(b2 + c2 ) a2 + c2 + b2 + c2 p p p 2 , (a2 + c2 )(b2 + c2 ) a2 + c2 + b2 + c2 ab(a + b)2 , (a b)2 ab Bất đẳng thức cuối hiển nhiên đúng vì p p a2 + c2 + b2 + c2 a + b; Từ đó, ta có Xa cyc Xb + b a cyc X cyc r (a2 + c2 )(b2 + c2 ) a2 + c2 X + b2 + c2 cyc http://boxtailieu.net r b2 + c2 a2 + c2 a2 b2 0: 225 Giả sử P cyc a b < P q a2 +c2 b2 +c2 cyc , khi đó ta có cyc Xa cyc , Do P cyc b a > b < X cyc b a > P q b2 +c2 cyc r a2 +c2 . cyc a2 +c2 Xb X X a2 + c2 + 2 + 2 < b2 a b2 + c2 cyc cyc cyc r b2 + c2 a2 + c2 nên X a2 cyc điều này mâu thuẫn vì Nhưng a2 + c2 b2 + c2 X a2 P q b2 +c2 cyc P b2 X a2 cyc b2 < X a2 + c2 cyc b2 + c2 X a2 + c2 cyc b2 + c2 Thật vậy, giả sử c = minfa; b; cg, thì bất đẳng thức này tương đương với 1 a2 b2 1 (a2 b2 )2 (a2 + c2 )(b2 + c2 ) 1 1 (a2 + 2 2 a c (a2 + b2 )(a2 + c2 ) c2 )(b2 c2 ) 0 hiển nhiên đúng. Vậy nên ta phải có Xa cyc b X cyc r a2 + c2 : b2 + c2 Bài toán 2.24 Cho các số không âm a; b; c thỏa mãn a + b + c = 1: Chứng minh rằng p p p p a + (b c)2 + b + (c a)2 + c + (a b)2 3: (Phan Thành Nam) Lời giải. Bình phương 2 vế, bất đẳng thức đã cho có thể được viết lại như sau X Xp (b c)2 + 2 [a + (b c)2 ][b + (a c)2 ] 2 cyc cyc http://boxtailieu.net 226 CHƯƠNG 2. SÁNG TẠO BẤT ĐẲNG THỨC Sử dụng bất đẳng thức Cauchy Schwarz, ta có Xp X Xp [a + (b c)2 ][b + (a c)2 ] ab + j(a cyc cyc Nên ta chỉ cần chứng minh được X Xp X (b c)2 + 2 ab + 2 j(a cyc cyc c)(b c)j cyc c)(b c)j 2 cyc Không mất tính tổng quát, giả sử a b c; khi đó X Xp X (b c)2 + 2 ab + 2 j(a c)(b c)j 2 cyc cyc cyc = 4(a 2 c) 2 1 Xp cyc ! ab p p 2 + b c h i p p 2 p 2 p 2 p p 4(a c)2 2 a c =2 a c 2 a+ c 1 p 2 p p 2 p c [2(a + c) 1] = 2 a c (a b + c) 0: 2 a = 4(a c)2 p a p c 2 p p a 2 b Bất đẳng thức được chứng minh xong. Đẳng thức xảy ra khi và chỉ khi a = b = c = hoặc a = b = 21 ; c = 0 hoặc các hoán vị. 1 3 Bài toán 2.25 Cho các số dương a; b; c thỏa a + b + c = 3: Chứng minh rằng a b c + + 2b + 1 2c + 1 2a + 1 1 : abc (Võ Quốc Bá Cẩn) Lời giải. Bất đẳng thức đã cho tương đương với 3 a 2b + 1 + 3 b 2c + 1 + 3 c 2a + 1 + 1 abc 9 7b + c 7c + a 7a + b 1 + + + 9 2b + 1 2c + 1 2a + 1 abc c 1 a 1 b 1 2 1 1 1 + + + +3 5 + + 2b + 1 2c + 1 2a + 1 abc 2a + 1 2b + 1 2c + 1 , ,2 Nên để chứng minh bất đẳng thức đã cho, ta chỉ cần chứng minh 2 bất đẳng thức sau a 1 b 1 c 1 + + 2b + 1 2c + 1 2a + 1 http://boxtailieu.net 0 227 2 +3 abc 1 1 1 + + 2a + 1 2b + 1 2c + 1 5 a) Trước hết, ta sẽ chứng minh c 1 a 1 b 1 + + 2b + 1 2c + 1 2a + 1 , 2(a2 + b2 + c2 ab bc ca) + 4(a2 b + b2 c + c2 a) , a3 + b3 + c3 + 12(a2 b + b2 c + c2 a) , 0 6(ab2 + bc2 + ca2 ) + 21abc a3 + b3 + c3 + 3(a2 b + b2 c + c2 a + ab2 + bc2 + ca2 ) 6(ab2 + bc2 + ca2 a2 b b2 c c2 a) , (a + b + 7c)(a b)2 + (b + c + 7a)(b 18(a b)(b c)(c a) 21abc c)2 + (c + a + 7b)(c Từ đây, giả sử a = min fa; b; cg và đặt b = a + x; c = a + y (x; y dàng biến đổi bất đẳng thức về dạng tương đương là 9(x2 12 xy + y 2 )a + x3 + 3x2 y + y(3x y)2 a)2 0); ta có thể dễ 0 Nên bất đẳng thức hiển nhiên đúng. b) Tiếp theo, ta sẽ chứng minh 2 +3 abc 5 1 1 1 + + 2a + 1 2b + 1 2c + 1 Đặt q = ab + bc + ca; r = abc thì ta có 1 1 1 4q + 15 + + = 2a + 1 2b + 1 2c + 1 8r + 4q + 7 Nên bất đẳng thức tương đương với 3r + 2 r , 2(1 5(4q + 15) 8r + 4q + 7 r)(4q + 7 12r) r(8r + 4q + 7) Bất đẳng thức cuối hiển nhiên đúng vì 3 ra khi và chỉ khi a = b = c = 1: q 0: 3r: Vậy ta có đpcm. Đẳng thức xảy http://boxtailieu.net 228 CHƯƠNG 2. SÁNG TẠO BẤT ĐẲNG THỨC Bài toán 2.26 Cho các số dương a; b; c thỏa a b c và abc = 1: Chứng minh rằng 1 1 1 + 3: + a b2 c a + b2 + c3 (Rachid) Lời giải. Từ giả thiết, ta suy ra c 1 c3 a , (c3 a Do c3 a c2 ba 1 1=c 1 1: Bất đẳng thức đã cho tương đương 1 a 1 b2 b2 1 1 + 3 a c 1 b4 + c3 1) 0 nên nếu b (c3 a Nếu 1 1; bc 1) b2 1 thì bất đẳng thức hiển nhiên đúng vì 1 1 + a c3 b4 1 0 b2 b; ta có c3 a + b4 1 a a(c3 + b3 ) 2 1 b2 2 abc(b + c) ) c3 a 2 1 1 2=b+c b4 2 2 p bc 1 0 0 ,b a thì bất đẳng thức hiển nhiên đúng. Xét trường hợp ngược Nên nếu lại a b2 ; khi đó ta có c3 a 1 c(1 b4 ) c3 b2 + cb4 c 1 = c3 b2 + cb4 c (abc)4=3 (abc)5=3 c3 b2 + cb4 c (b2 c)4=3 (b2 c)5=3 = b2 c(c2 + b2 b4=3 c2=3 b2=3 c4=3 ) 0 ) c3 a 1 c(1 b4 ) 0 Lại có c 1 b2 c a b a = 0: 2 2 a b ab ab2 Bất đẳng thức được chứng minh xong. Đẳng thức xảy ra khi và chỉ khi a = b = c = 1: Bài toán 2.27 Cho các số dương x; y; z thỏa x2 + y 2 + z 2 = 1: Chứng minh rằng 1 1 1 + 2+ 2 x2 y z 2 xyz 9 p 6 3: (Ji Chen) http://boxtailieu.net 229 Lời giải. Bất đẳng thức tương đương với yz zx xy + + x y z , 2 yz zx xy + + x y z zx Đặt a = yz x ;b = y ;c = đẳng thức trở thành xy z p 6 3 xyz 9 2+9 2 p 3 1 xyz 2 p 3 2+9 1 abc 2 p 3 1 abc = p 2+ (2 p) (2 p) 2 p 3 2 p 3 2 p 3 3; bất 0 Nếu p 2 thì bất đẳng thức hiển nhiên đúng. Nếu 2 thức Schur bậc 3, ta có 9abc p(4 p2 ); suy ra 2+9 p và p = a + b + c thì ta có ab + bc + ca = 1 và p p p 0 p p 3 thì theo bất đẳng p2 ) 1 p(4 1 p(p + 2) 1 p 3+2 1 3 p 1 = 0: Bất đẳng thức được chứng minh xong. Đẳng thức xảy ra khi và chỉ khi x = y = z = p1 : 3 Bài toán 2.28 Cho các số dương x; y; z: Tìm hằng số k > 0 lớn nhất sao cho 3yz + z)2 (y k 3zx + x)2 (z k 3xy + y)2 (x k xyz(x + y + z)3 : (Ji Chen) Lời giải. Đặt a = x1 ; b = y1 ; c = 3ab + b)2 (a k 1 z 3bc + thì ta có a; b; c > 0 và bất đẳng thức trên trở thành c)2 (b k 3ca + a)2 (c k (ab + bc + ca)3 Cho c ! 0; a = b = 1 thì bất đẳng thức trở thành 3 k2 1,k p 3 Tiếp theo, ta sẽ chứng minh rằng 27 ab + (a b)2 p 3 3 bc + (b c)2 p 3 3 ca + (c a)2 p 3 3 http://boxtailieu.net (ab + bc + ca)3 230 CHƯƠNG 2. SÁNG TẠO BẤT ĐẲNG THỨC 1 p 3 3 Đặt t = Y thì ta có ab + cyc = Y (a b)2 p 3 3 b)2 ab + t(a cyc = [ab + t(a b)2 ][t2 (b c)2 + tbc(c a)2 + tca(b c)2 + abc2 ] Y X X = t3 (a b)2 + t2 bc(a b)2 (a c)2 + tabc c(a b)2 + a2 b2 c2 cyc t2 X cyc b)2 (a bc(a c)2 + tabc cyc X cyc b)2 + a2 b2 c2 c(a cyc " 1 X bc(a = 27 cyc X p c) + 3 3abc c(a 2 2 b) (a 2 2 2 2 b) + 27a b c cyc # và X !3 27a2 b2 c2 ab cyc X = a3 b3 cyc X = 3a2 b2 c2 !" ab cyc X ! bc(a + 3abc " X ab(a + b) c) + 3abc cyc 6abc cyc # b)(a # X b)2 c(a cyc Nên ta chỉ cần chứng minh X bc(a b)2 (a cyc , X bc(a X p c(a c)2 + 3 3abc cyc !" c)(a2 2ab X b)(a ab b)2 cyc X bc(a # b)(a c) + 3abc cyc 2ac) + 3 p 3 X c(a b)2 cyc 1 abc X c(a b)2 0 b)(a c) cyc cyc Ta có 3 p 3 1 abc X cyc c(a b)2 2abc X c(a b)2 = 2 cyc http://boxtailieu.net X cyc (b + c)(a 231 Và X bc(a c)(a2 b)(a cyc abc X 2ab 2ac) + 2abc X (b + c)(a b)(a c) cyc a(a b)(a c) 0: cyc Bất đẳng thức được chứng minh xong. Ta đi đến kết luận p kmax = 3: Bài toán 2.29 Cho các số x; y; z > 0: Chứng minh rằng y z x + + x y z x2 y2 z2 + + x2 + 2yz y 2 + 2zx z 2 + 2xy 3 : (Dương Đức Lâm) Lời giải. Ta sẽ sử dụng kết quả sau Bổ đề. Với mọi a; b; c > 0; ta có 9(a2 + b2 + c2 ) (a + b + c)2 c a b + + b c a (Võ Quốc Bá Cẩn) Chứng minh. Bất đẳng thức tương đương X cyc , , X a2 cyc b , X a2 X cyc X cyc b cyc (a ! a ! a + Xa cyc X ab cyc + b)2 c b ! + X ab cyc c 1 c + b 2ab 9 P a2 cyc P a cyc X 9 a cyc a2 cyc cyc X P P a cyc ! 9 a 3 a+b+c http://boxtailieu.net P a2 cyc P 3 a X cyc cyc 0 a 232 CHƯƠNG 2. SÁNG TẠO BẤT ĐẲNG THỨC , 3c Đặt z = 2a b + b + đủ, khi đó ta có x= c a X + cyc c2 ab (a b)2 c c2 2a 3c + + + b b a ab 4 0 4 và x; y tương tự. Rõ ràng ta chỉ cần xét a 2b 3a a a2 + + + c c b bc 4 2+3+1+1 4=3>0 và y+z 2y + z b2 c2 2a 3b b 3c 3c + + + + + + 8 b a c b a ca ab 2a 3b b 3c 3c b2 b2 c2 = + + + + + + + b a c b a 2ca 2ca ab 3 b 2a 3b b 3c 3c + + + + + p 8 3 b a c b a 4 a 2a 9b b 3c 3c + + + + 8 b 2a c b a a b 3c a 9b 2b 3c = + + + + + + b 3c a b 2a 3c b p p p 3+3 2+2 2 8=5 2 5>0 = 2a 6b 2b 3c 5c 2b2 c2 + + + + + + 12 b a c b a ca ab 2a 6b 2b 3c 5c b2 b2 c2 = + + + + + + + b a c b a ca ca ab 2a 6b 2b 3c 5c 3b + + + + + 12 b a c b a a 2a 9b 2b 3c 5c = + + + + 12 b a c b a a b 5c a 9b 9b 3c = + + + + + + b 5c a b a 5c b r r 3 3 3+6+6 12 = 6 3>0 5 5 8 8 = http://boxtailieu.net 12 12 b c là 233 x + 2y Do đó Nếu y 0 thì ta có X z(a 2b2 a 6b 4b 3a 4c a2 + + + + + + 12 b a c c a bc ca a 6b 4b 3a 4c a2 2b2 + + + + + + 12 b a c c a b a b a 2a 8b 4b 3a 4c = + + + + 12 b a c c a c a 4b 4b a c + = 2 + + +3 + 12 b a c c a a 8 + 4 + 6 + 0 12 = 6 > 0 = b)2 y(a c)2 + z(a b)2 x(b c)2 + y[2(b (y + z)(a b)2 0 cyc Nếu y 0 thì ta có X z(a b)2 c)2 + 2(a b)2 ] + z(a b)2 cyc = c)2 + (z + 2y)(a (x + 2y)(b Bổ đề được chứng minh xong. Trở lại bài toán của ta, đặt a = x1 ; b = y1 ; c = Xa cyc , b Xa cyc 3 b X cyc +6 X 1 z bc 2a2 + bc 9 Sử dụng bất đẳng thức Cauchy Schwarz, ta có X cyc a2 2a2 + bc P a2 + cyc Mặt khác, theo bổ đề trên, ta có Xa cyc b 9 cyc cyc P cyc P P !2 a cyc P 2 0: thì bất đẳng thức trở thành a2 2a2 + bc cyc b)2 a2 !2 a http://boxtailieu.net bc 234 CHƯƠNG 2. SÁNG TẠO BẤT ĐẲNG THỨC Nên ta chỉ cần chứng minh được 3 P cyc P Đặt t = P P cyc bc !2 1 3 2 !2 + P 2 !2 a cyc a2 + cyc a cyc cyc a2 P P 3 bc cyc thì bất đẳng thức trở thành a 3(1 2t) + , 2 2t) + t 2(1 3t)2 3t 2(1 2 3 0: Bất đẳng thức cuối hiển nhiên đúng nên ta có đpcm. Đẳng thức xảy ra khi và chỉ khi x = y = z: Bài toán 2.30 Cho các số không âm a; b; c; không có 2 số nào đồng thời bằng 0: Chứng minh rằng a3 a4 b4 c4 + 3 + 3 3 3 +b b +c c + a3 a+b+c : 2 (Vasile Cirtoaje) Lời giải. Sử dụng bất đẳng thức Cauchy Schwarz, ta có X a4 a3 + b3 2 X cyc !" X 2 3 3 # X a (a + b ) cyc 3 a cyc !2 Ta cần chứng minh cyc 3 a !2 X cyc !" a X 2 3 3 # a (a + b ) cyc http://boxtailieu.net 235 Sử dụng bất đẳng thức Vasile P ab cyc X cyc P 1 3 3 2 a cyc P !2 , ta có ! ! ab X X X 3 2 2 2 a (a + b ) = a + P ab + a b abc a a cyc cyc cyc cyc cyc P 2 3 !2 ! ab X X X X 1 cyc 5 2 2 2 2 a + a b 5 abc a + P 4 a a 3 cyc cyc cyc cyc 2 3 X 3 cyc 5 cyc Do tính thuần nhất, ta có thể chuẩn hóa cho a+b+c = 1. Đặt q = ab+bc+ca; r = abc, khi đó ta có P 2 3 !2 ! ab X X X X 1 cyc a2 + a2 b2 5 abc a2 a5 + P 4 a 3 cyc cyc cyc cyc cyc 1 [3(4 3 = và X a3 cyc !2 14q + 11q 2 + 7q 3 ] 5q)r + 3 3q + 3r)2 = (1 Ta phải chứng minh 2(1 3q + 3r)2 , 54r2 + 3(8 1 [3(4 3 5q)r + 3 22q + 43q 2 28q)r + 3 4q 1 9 Sử dụng bất đẳng thức Schur bậc 3, ta có r 28q)r tăng với mọi r 4q9 1 , ta có f (r) + 3 22q + 43q 2 7q 3 9qr b)2 (b r c)2 (c 1 h 9q 27 a)2 7q 3 9qr 1 +3 9 22 49 q + q2 3 3 22q + 43q 2 7q 3 0, ta suy ra được 2 + 2(1 0 và chú ý rằng f (r) = 54r2 + 3(8 4q f = 1 Từ bất đẳng thức (a 14q + 11q 2 + 7q 3 ] p 3q) 1 http://boxtailieu.net 3q i 9qr 7q 3 9qr 236 CHƯƠNG 2. SÁNG TẠO BẤT ĐẲNG THỨC Do đó, ta chỉ cần chứng minh 1 22 49 q + q2 3 3 7q 3 1 h q 9q 3 2 + 2(1 p 3q) 1 3q p 1 0 (1 3q) 7q 2 11q + 3 2q 1 3q 3 Sử dụng bất đẳng thức AM-GM, ta có p 7q 2 11q + 3 2q 1 3q 7q 2 11q + 3 (q 2 + 1 3q) = 2(1 i 0 , q)(1 3q) 0: Bất đẳng thức được chứng minh. Đẳng thức xảy ra khi và chỉ khi a = b = c: Bài toán 2.31 Cho các số dương a; b; c: Chứng minh rằng p p p 3 3 3 (a + b)2 (b + c)2 (c + a)2 (a b)2 (b c)2 (c a)2 + 4 a2 b2 c2 : (Sung Yoon Kim) Lời giải. Sử dụng bất đẳng thức Cauchy Schwarz, ta có s s b)2 (b c)2 (c a)2 a2 b2 c2 3 (a +43 2 2 2 2 (a + b) (b + c) (a + c) (a + b) (b + c)2 (a + c)2 s s 2 (b 2 (c 2 (a b) ab c) a) bc = 3 +43 (a + b)2 (b + c)2 (c + a)2 (a + b)2 (b + c)2 ab 4X 1 X (a b)2 + 2 3 cyc (a + b) 3 cyc (a + b)2 = ca (c + a)2 1 X (a b)2 4ab + = 1: 3 cyc (a + b)2 (a + b)2 Đẳng thức xảy ra khi và chỉ khi a = b = c: Bài toán 2.32 Cho các số dương a; b; c thỏa mãn abc = 1: Tìm hằng số k lớn nhất sao cho bất đẳng thức sau đúng 1 1 1 + + a(1 + bc)2 b(1 + ca)2 c(1 + ab)2 k 3 + (1 + ab)(1 + bc)(1 + ca) 4 k 8 (Võ Quốc Bá Cẩn) Lời giải 1. Cho a = 2; b = 1; c = 21 ; ta được k 4: Ta sẽ chứng minh đây là giá trị mà ta cần tìm, tức là X 1 16 4 1+ 2 a(1 + bc) (1 + ab)(1 + bc)(1 + ca) cyc http://boxtailieu.net 237 ,4 Đặt x = 1 a 1+a ; y = X cyc 1 b 1+b ; z = (1 a (a + 1)2 1 c 1+c ; x)(1 1+ 16 (1 + a)(1 + b)(1 + c) thì ta có x; y; z 2 [ 1; 1] và y)(1 z) = (1 + x)(1 + y)(1 + z) ) x + y + z + xyz = 0 Bất đẳng thức trở thành X (1 x2 ) 1 + 2(1 + x)(1 + y)(1 + z) cyc , x2 + y 2 + z 2 + 2(xy + yz + zx) + 2(x + y + z + xyz) , (x + y + z)2 0 0 hiển nhiên đúng. Bất đẳng thức được chứng minh. Vậy ta đi đến kết luận kmax = 4: Lời giải 2. Tương tự như trên, ta cần phải chứng minh 4 X cyc 1 a(1 + bc)2 1+ 16 (1 + ab)(1 + bc)(1 + ca) Vì a; b; c > 0; abc = 1 nên tồn tại các số x; y; z > 0 sao cho a = đẳng thức trở thành X cyc xy (x + y)2 1 4xyz + 4 (x + y)(y + z)(z + x) http://boxtailieu.net x y;b = yz ; c = z x. Bất 238 CHƯƠNG 2. SÁNG TẠO BẤT ĐẲNG THỨC Ta có 4xyz (x + y)(y + z)(z + x) 2[(x + y)(y + z)(z + x) xy(x + y) yz(y + z) zx(z + x)] = (x + y)(y + z)(z + x) P 2 (x + yz)(y + z) cyc = 2 (x + y)(y + z)(z + x) ! X x X y x z = 2 + x+y x+z x+y x+z cyc cyc ! ! # " X y X x X x y = 2 x+y x+z x+y x+y cyc cyc cyc ! ! X x X y X xy = 2 + x+y x+z (x + y)2 cyc cyc cyc Bất đẳng thức tương đương với X cyc x x+y ! X cyc y x+z ! 9 4 Sử dụng bất đẳng thức AM-GM, ta có X cyc x x+y ! X cyc y x+z ! 1 4 X cyc X y x + x+y x+y cyc !2 = 9 : 4 Bài toán 2.33 Cho các số không âm a; b; c thỏa mãn a2 + b2 + c2 = 1: Tìm giá trị lớn nhất của biểu thức P = (a b)(b c)(c a)(a + b + c): (Võ Quốc Bá Cẩn) Lời giải. Nếu a P = (c b(c b b)(b c 0; thì P a)(c 0: Nếu c b a b)(b a)(c2 + bc a2 1 1 bc)(b2 + bc) (b2 + c2 )2 = 4 4 a)(a + b + c) = (c b)(c2 + bc) = (c2 0; thì http://boxtailieu.net ab) 239 Mặt khác, cho a = 0; b = sin 8 ; c = cos 8 ; ta có P = 14 : Vậy nên max P = 1 : 4 Bài toán 2.34 Cho các số dương a; b; c; d: Chứng minh rằng b(a + c) c(b + d) d(c + a) a(d + b) + + + c(a + b) d(b + c) a(c + d) b(d + a) 4: (Võ Quốc Bá Cẩn) Lời giải. Viết lại bất đẳng thức như sau b d + c(a + b) a(c + d) (a + c) , (abc + abd + acd + bcd) 1 1 1 1 + + + a b c d , c a + d(b + c) b(d + a) + (b + d) 4 a+c b+d + ac(a + b)(c + d) bd(b + c)(d + a) ! 1 1 1 1 a + c b + d 1 1 + 1 1 1 1 1 1 a + b c + d b + c d + a 4 4 Sử dụng bất đẳng thức AM-GM, ta có 1 a + 1 a 1 b + 1 c 1 c + 1 d + 1 b + 1 b 1 c + 1 d 1 d 1 a 4 + 1 a 1 a = 1 a + + 1 b + 1 c 1 b + 1c + d1 4 : + 1c + d1 2 + 1 b 4 1 a + 1 b + + 1 c 1 d + 1 2 d Bất đẳng thức được chứng minh. Đẳng thức xảy ra khi và chỉ khi a = c và b = d: Bài toán 2.35 Cho các số dương a; b; c: Chứng minh rằng a2 + bc b2 + ca c2 + ab + + a2 + (b + c)2 b2 + (c + a)2 c2 + (a + b)2 18 a2 + b2 + c2 : 5 (a + b + c)2 (Phạm Hữu Đức) Lời giải. Bất đẳng thức đã cho tương đương với X (b + c)2 bc 18 a2 + b2 + c2 + 2 2 a + (b + c) 5 (a + b + c)2 cyc http://boxtailieu.net 3 240 CHƯƠNG 2. SÁNG TẠO BẤT ĐẲNG THỨC Do (b + c)2 4bc, nên ta chỉ cần chứng minh được X cyc (b + c)2 6 a2 + b2 + c2 + 4[a2 + (b + c)2 ] 5 (a + b + c)2 1 Sử dụng bất đẳng thức Cauchy Schwarz, ta có X cyc (b + c)2 + (b + c)2 ] (a + b + c)2 (a + b + c)2 P 2 = 2 2 2 [a + (b + c) ] 2(a + b + c2 ) + (a + b + c)2 4[a2 cyc Chuẩn hóa cho a + b + c = 1. Đặt x = a2 + b2 + c2 ) 3x 1 6x + 2x + 1 5 1 , x(3x 0: 1) 1, ta phải chứng minh Đẳng thức xảy ra khi và chỉ khi a = b = c: Bài toán 2.36 Cho các số dương a; b; c; d: Chứng minh rằng (a + b)(a + c)(a + d)(b + c)(b + d)(c + d) 4(abc + bcd + cda + dab)2 : (Võ Quốc Bá Cẩn) Lời giải. Sử dụng bất đẳng thức Cauchy Schwarz, ta có (abc + bcd + cda + dab)2 (ac + bc + ad + bd)(ab2 c + bcd2 + ac2 d + a2 bd) = (a + b)(c + d)(ab2 c + bcd2 + ac2 d + a2 bd) (abc + bcd + cda + dab)2 (bc + bd + ac + ad)(a2 bc + bc2 d + acd2 + ab2 d) = (a + b)(c + d)(a2 bc + bc2 d + acd2 + ab2 d) Công tương ứng vế với vế 2 bất đẳng thức trên, ta được 2(abc + bcd + cda + dab)2 (a + b)(c + d)(ab2 c + bcd2 + ac2 d + a2 bd + a2 bc + bc2 d + acd2 + ab2 d) = (a + b)2 (c + d)2 (ab + cd) Tương tự, ta cũng có 2(abc + bcd + cda + dab)2 (a + c)2 (b + d)2 (ac + bd) 2(abc + bcd + cda + dab)2 (a + d)2 (b + c)2 (ad + bc) http://boxtailieu.net 241 Nhân tương ứng vế với vế, ta được 8(abc + bcd + cda + dab)6 (ab + cd)(ac + bd)(ad + bc) Y (a + b)2 sym Mặt khác, theo bất đẳng thức AM-GM thì 4(ab + cd)(ac + bd) (ab + cd + ac + bd)2 = (a + d)2 (b + c)2 Tương tự, 4(ab + cd)(ad + bc) (a + c)2 (b + d)2 4(ac + bd)(ad + bc) (a + b)2 (c + d)2 Do đó Y 64(ab + cd)2 (ac + bd)2 (ad + bc)2 (a + b)2 sym , (ab + cd)(ac + bd)(ad + bc) Kết hợp với bất đẳng thức 8(abc+bcd+cda+dab)6 1 Y (a + b) 8 sym (ab+cd)(ac+bd)(ad+bc) Q (a+ sym b)2 , ta suy ra được 8(abc + bcd + cda + dab)6 " #" # Y 1 Y 1 Y (a + b) (a + b)2 = (a + b)3 : 8 sym 8 sym sym Từ đây, ta suy ra đpcm. Đẳng thức xảy ra khi và chỉ khi a = b = c = d: Bài toán 2.37 Cho các số dương a; b; c: Chứng minh rằng s a b c 1 1 1 3+ + + 2 (a + b + c) + + : b c a a b c (Phạm Hữu Đức) Lời giải. Đặt x3 = a; y 3 = b; z 3 = c. Sử dụng bất đẳng thức Schur bậc 3, ta có 3+ c x y z x3 y3 z3 a b + + = 3 + 3 + 3 + 3 b c a y z x y z x = X x2 cyc yz + X xz cyc y2 http://boxtailieu.net X x2 cyc y2 y z + z x 242 CHƯƠNG 2. SÁNG TẠO BẤT ĐẲNG THỨC Mặt khác, theo bất đẳng thức AM-GM thì X x2 cyc yz + X xz cyc y2 x3 + y 3 + z 3 (xz)3 + (yx)3 + (zy)3 + xyz (xyz)2 s (x3 + y 3 + z 3 )(x3 z 3 + y 3 x3 + z 3 y 3 ) 2 x3 y 3 z 3 s 1 1 1 + 3+ 3 = 2 (x3 + y 3 + z 3 ) x3 y z s 1 1 1 = 2 (a + b + c) + + : a b c = Đẳng thức xảy ra khi và chỉ khi a = b = c: Bài toán 2.38 Cho các số dương a; b; c. Chứng minh rằng abc + 2(a2 + b2 + c2 ) + 8 5(a + b + c): (Trần Nam Dũng) Lời giải. Sử dụng bất đẳng thức AM-GM và bất đẳng thức Schur bậc 3, ta có 6abc + 12(a2 + b2 + c2 ) + 48 30(a + b + c) 2 = 3(2abc + 1) + 12(a + b2 + c2 ) + 45 5 2 (a + b + c) 3 p 3 9 a2 b2 c2 + 12(a2 + b2 + c2 ) + 45 5[(a + b + c)2 + 9] 9abc = p + 7(a2 + b2 + c2 ) 10(ab + bc + ca) 3 abc 27abc + 7(a2 + b2 + c2 ) 10(ab + bc + ca) a+b+c 3[4(ab + bc + ca) (a + b + c)2 ] + 7(a2 + b2 + c2 ) 10(ab + bc + ca) = 4(a2 + b2 + c2 ab bc ca) 0: Đẳng thức xảy ra khi và chỉ khi a = b = c = 1: Bài toán 2.39 Cho các số không âm a; b; c thỏa mãn ab + bc + ca + 6abc = 9: Tìm hằng số k nhỏ nhất sao cho bất đẳng thức sau đúng a + b + c + kabc k + 3: (Võ Quốc Bá Cẩn) http://boxtailieu.net 243 Lời giải. Cho a = b = 3; c = 0, ta được k 3. Ta sẽ chứng minh đây là giá trị mà ta cần tìm, tức là a + b + c + 3abc 6 Đặt p = a + b + c; q = ab + bc + ca; r = abc thì ta có q + 6r = 9. Sử dụng bất đẳng thức AM-GM, ta có p2 3q 9. Bất đẳng thức trở thành p + 3r , 2p Nếu p Nếu 6 6 q 3 6, bất đẳng thức là hiển nhiên. p 3 và nếu p2 4q, thì 2p Nếu6 p Do đó 3 và nếu p2 q (p p2 = 4 2p 2)(6 4 p) +3 3 4q, theo bất đẳng thức Schur bậc 3, ta có r 27 = 3q + 18r ) 2p 2p 0. p2 ) 3q + 2p(4q q p(4q p2 ) 9 2p3 + 27 8p + 3 Ta cần chứng minh 2p 2p3 + 27 8p + 3 , (p + 1)(p 3)(p 3 6) 0: hiển nhiên đúng. Vậy ta đi đến kết luận kmin = 3: Bài toán 2.40 Cho các số dương a; b; c thỏa mãn a + b + c = 3: Chứng minh rằng 3(a4 + b4 + c4 ) + a2 + b2 + c2 + 6 6(a3 + b3 + c3 ): (Vasile Cirtoaje) Lời giải. Bất đẳng thức tương đương với X 3a4 6a3 + a2 + 4a 2 0 cyc , X (a 1)2 (3a2 2) cyc http://boxtailieu.net 0 244 CHƯƠNG 2. SÁNG TẠO BẤT ĐẲNG THỨC X , , X c)2 (3a2 (2a b 2) b)(a c)(4a2 + b2 + c2 0 cyc (a 4) 0 cyc Không mất tính tổng quát, giả sử a 4a2 + b2 + c2 b c 0; khi đó ta dễ dàng kiểm tra được 4b2 + c2 + a2 4 4 4c2 + a2 + b2 4 Mặt khác 4c2 + a2 + b2 4c2 + 4 (a + b)2 2 4= 1)2 (3c 0 2 Do đó 4a2 + b2 + c2 4 4b2 + c2 + a2 4c2 + a2 + b2 4 4 0 Từ đây, viết lại bất đẳng thức như sau (a c)(4a2 + b2 + c2 b)[(a 4) + (c (b c)(4b2 + a2 + c2 2 a)(c 2 4)] b)(4c + a + b 2 4) 0: Ta thu được kết quả cần chứng. Đẳng thức xảy ra khi và chỉ khi a = b = c = 1 hoặc a = b = 34 ; c = 13 hoặc các hoán vị tương ứng. Bài toán 2.41 Cho a; b; c là độ dài 3 cạnh của một tam giác. Chứng minh rằng c+a a+b b+c + + a2 + bc b2 + ca c2 + ab 3(a + b + c) : ab + bc + ca (Võ Quốc Bá Cẩn) Lời giải. Viết lại bất đẳng thức như sau X cyc , , X b+c a2 + bc 1 a X (a (a cyc b)(a c) a(a2 + bc) b)(a c) cyc , X (a cyc X1 a cyc 3(a + b + c) ab + bc + ca X bc(a b)(a c) abc(ab + bc + ca) cyc 1 a(a2 + bc) 1 a(ab + bc + ca) b)(a c)(b + c a2 + bc a) http://boxtailieu.net 0 0 245 Giả sử a b c; do a; b; c là độ dài 3 cạnh của một tam giác nên a Do a b c b(a b) c c nên X (a b)(a c)(b + c a2 + bc cyc (b (b = (a (a = (a a) c)(b a)(c + a b) (c a)(c b)(a + b c) + b2 + ca c2 + ab c)(b a)(c + a b) b(a b)(b c)(a + b c) + b2 + ca c(c2 + ab) b(a + b c) c + a b b)(b c) c(c2 + ab) b2 + ca b(c + a b) c + a b b)(b c) c(c2 + ab) b2 + ca 3 3 b)(b c)(c + a b)(b c ) 0: c(c2 + ab)(b2 + ca) Đẳng thức xảy ra khi và chỉ khi a = b = c hoặc ứng. a 2 = b 1 = c 1 hoặc các hoán vị tương Bài toán 2.42 Chứng minh rằng với mọi a; b; c 2 R và a2 + b2 + c2 = 9; ta có 3 min fa; b; cg abc + 1: (Virgil Nicula) Lời giải. Giả sử c b a; bất đẳng thức trở thành abc + 1 Nếu a 3a 0; sử dụng bất đẳng thức AM-GM, ta có abc + 1 3a a(b2 + c2 ) +1 2 Nếu a 0; vì c b p nhiên. Nếu 3 a 3a = a2 ) a(9 2 +1 3a = (a + 1)2 (2 2 a) 0 p 1 a; ta được 3 a 0: Nêu a 3 ; bất đẳng thức là hiển 1 ; ta có thể dễ dàng kiểm tra được 3 p p bc a b2 + c2 a2 = a 9 2a2 Do đó, ta phải chứng minh a2 p 9 2a2 + 1 3a http://boxtailieu.net 246 Nếu 1 CHƯƠNG 2. SÁNG TẠO BẤT ĐẲNG THỨC a 1 3; , f (a) = 2a6 6a + 1 0 ta có 2f (a) = a2 (a 1) 4a(a p Nếu 3 a 1; thì f (a) 9a4 + 9a2 2a6 1)2 + 12a2 9a4 + 9a2 18a + 7 + (1 5 = (a2 1)(2a4 3a)(7a2 7a2 + 2) 6a + 3) 3 0 30 2 8t4 ) 4t, do đó (1 + 2t Nếu 2 2 1 a 1 9 = 2 2t) + 2t(1 6t t2 (7 6t 7t2 + 20t3 20t) t) 1 8t4 7 8t4 ) 4 sử dụng bất đẳng thức AM-GM, ta có 1 1 1 +p +p a + bc b + ca c + ab 3 p 6 (a + bc)(b + ca)(c + ab) http://boxtailieu.net p 7 t 0 248 CHƯƠNG 2. SÁNG TẠO BẤT ĐẲNG THỨC Do đó, ta chỉ cần chứng minh được 729 512 (a + bc)(b + ca)(c + ab) Đặt p = a + b + c; r = abc p 2 p p Mặt khác, lấy u = 3 2u + v = p; 0; khi đó từ a 2; ab + bc + ca = 1; ta suy ra 52 p 3 p+2 p2 3 và v = ; ta dễ dàng kiểm tra được 3 p 5 + 2 13 6 2 u + 2uv = 1; 1 p 3 v u 5 p 13 6 u2 v ; p p r 3: 0 Ta có (a + bc)(b + ca)(c + ab) = r2 + (p2 2p 1)r + 1 = f (r) Dễ thấy f (r) là hàm lồi nên f (r) max f (0); f (u2 v) = max 1; f (u2 v) Ta còn phải chứng minh f (u2 v) 729 512 , v 2 (u + 1)2 (v + u2 ) , g(u) = u2 )2 (2u3 u (1 + 2u 729 512 u2 + 1) 729 64 Ta có g 0 (u) = (1 ) g(u) 3u2 )(1 + 2u g 5 p 6 13 ! u2 )(4u3 u2 = 14141 7u2 + 2u 1) 0 p 559 13 729 < : 1458 512 Bất đẳng thức được chứng minh. Đẳng thức xảy ra khi và chỉ khi x = y = p1 ; z 2 =0 hoặc các hoán vị tương ứng. Bài toán 2.44 Cho các số không âm x; y; z thỏa mãn xy + yz + zx = 1: Chứng minh rằng p 1 1 1 2 6 p +p +p : 3 2x2 + 3yz 2y 2 + 3zx 2z 2 + 3xy http://boxtailieu.net 249 Lời giải. Đặt a = yz; b = zx; c = xy (a; b; c 0), khi đó từ xy + yz + zx = 1; ta được a + b + c = 1. Bất đẳng thức trở thành p Xr 2 2 a r P 3a2 + 2bc 3 a cyc cyc Ta sẽ chứng minh X cyc X cyc a 3a2 + 2bc 3 4 P a cyc ab (3a2 + 2bc)(3b2 + 2ca) 4 P 9 cyc Thật vậy, ta có X cyc X cyc a 3a2 + 2bc 3 4 P 18 a Q (b cyc = cyc (3a2 4 9 P cyc 18 = Q (b c)2 + 15 cyc P cyc 9 P cyc 0 cyc !2 a !3 a !2 a a Q Q c)2 + 21 a(b + c) + 7 a2 cyc cyc ! P Q 3 a (3a2 + 2bc) cyc ab + 2bc)(3b2 + 2ca) !2 Q a + 36 cyc Q Q a(b + c) + 22 cyc (3a2 Q a2 cyc 0 + 2bc) cyc Mặt khác, với mọi m; n; p 0; ta có p m+n+p = m2 + n2 + p2 + 2(mn + np + pm) q p = m2 + n2 + p2 + 2 m2 n2 + n2 p2 + p2 m2 + 2mnp(m + n + p) q p m2 + n2 + p2 + 2 m2 n2 + n2 p2 + p2 m2 q q q a b c Sử dụng bất đẳng thức này với m = 3a2 +2bc ; n = 3b2 +2ca ; p = 3c2 +2ab ; ta được Xr cyc a 2 3a + 2bc v uX u t cyc s X a ab + 2 2 2 3a + 2bc (3a + 2bc)(3b2 + 2ca) cyc http://boxtailieu.net 250 CHƯƠNG 2. SÁNG TẠO BẤT ĐẲNG THỨC Từ 2 bất đẳng thức đã chỉ ra ở trên, ta được v s uX Xr X u a a ab t + 2 2 2 2 3a + 2bc 3a + 2bc (3a + 2bc)(3b2 + 2ca) cyc cyc cyc v v p u u u 4 2 2 4 u P + 2u !2 = r P : u u3 a u u 3 a P t t cyc 9 a cyc cyc Bất đẳng thức được chứng minh xong. Bài toán 2.45 Cho các số không âm a; b; c thỏa mãn a + b + c = 3: Chứng minh rằng q q q q p p p p 2 2 2 2 2 2 a+ b +c + b+ c +a + c+ a +b 3 2 + 1: (Phan Hồng Sơn) Lời giải. Sau khi bình phương 2 vế và thu gọn, ta có thể viết lại bất đẳng thức như sau r p p Xp X p a + b2 + c2 b + a2 + c2 a2 + b2 + 2 9 2+6 cyc cyc Sử dụng bất đẳng thức Cauchy Schwarz, ta có X cyc r p a + b2 + c2 1 X = p 2 cyc 1 X p 2 cyc p b + a2 + c2 r p p 2 2 X cyc p 1 a+3 1 p ab + 3 = 2 p s b+c a+ p 2 a+c b+ p 2 1 b+3 2 1 Xp 9 p ab + p 2 cyc 2 Do đó, ta cần chứng minh Xp a2 + b2 + 2 cyc p 2 Xp ab 6 cyc Để chứng minh bất đẳng thức này, ta chỉ cần chứng minh bất đẳng thức sau với mọi x; y 0 p p x4 + y 4 + 2 2 xy x2 + y 2 http://boxtailieu.net 251 Thật vậy, ta có p 2(x4 + y 4 ) + 2 p 2 = (x y)2 (x 2 = 2 p 1 xy y) p 2 2(x2 + y 2 ) (x + y)2 p 2(x4 + y 4 ) + x2 + y 2 p p (x + y)2 p 2(x2 + y 2 )2 + x2 + y 2 y)2 1 xy(x x2 + y 2 2+1 ! 2+1 ! 0: Đẳng thức xảy ra khi và chỉ khi a = b = c = 1: Bài toán 2.46 Cho các số dương a; b; c: Chứng minh rằng bc2 ca2 ab2 + + +a+b+c c2 a2 b2 6(a2 + b2 + c2 ) : a+b+c Lời giải. Sử dụng bất đẳng thức Cauchy Schwarz, ta có " # " #" # #" X ab2 X X a(b2 + c2 ) X X 1 1 = + a c2 a(b2 + c2 ) c2 a(b2 + c2 ) cyc cyc cyc cyc cyc !2 X1 a cyc Suy ra, ta chỉ cần chứng minh được X ! a cyc X , Nếu 1 c a cyc , 2(a+b) ab ; ! X (a cyc X1 a cyc X1 a cyc 2 b) ab !2 !2 6 X cyc X1 9 a cyc a2 !" 3 X cyc X cyc 1 a(b2 + c2 ) 2a b2 + c2 1 1 1 3(a + b)(c2 ab) + + + 2 a b c (a + c2 )(b2 + c2 ) # X1 a cyc ! 0 thì 1 1 1 3(a + b)(c2 ab) + + + 2 a b c (a + c2 )(b2 + c2 ) 1 1 1 + + a b c http://boxtailieu.net 3(a + b) ab 0 252 CHƯƠNG 2. SÁNG TẠO BẤT ĐẲNG THỨC Nếu 2(a+b) ab 1 c ,c ab 2(a+b) ; abc(a2 + c2 )(b2 + c2 ) thì 1 1 1 3(a + b)(c2 ab) + + + 2 a b c (a + c2 )(b2 + c2 ) = c4 (ab + bc + ca) + abc2 (a2 + b2 ) + (a + b)(a2 + b2 + 3ab)c3 + a3 b3 2a2 b2 c(a + b) abc2 (a2 + b2 ) + (a + b)3 c3 + a3 b3 2a2 b2 c(a + b) a3 b3 (a2 + b2 ) + (a + b)3 c3 + a3 b3 2a2 b2 c(a + b) 4(a + b)2 9 9 (a + b)3 c3 + a3 b3 + a3 b3 2a2 b2 c(a + b) 16 16 r 81 2 2 3 3 a b c(a + b) 2a2 b2 c(a + b) 0: 256 Đẳng thức xảy ra khi và chỉ khi a = b = c: Bài toán 2.47 Cho các số thực a; b; c khác 0 thỏa mãn a2 + b2 + c2 = (a c)2 + (c a)2 : Chứng minh rằng P (a; b; c) = 1 12 Q(a; b; c) = c a b + + b c a b)2 + (b 5 a2 b + b2 c + c2 a (a + b + c)3 5 : 36 (Võ Quốc Bá Cẩn) Lời giải. Nếu tồn tai 2 trong 3 số a; b; c, chẳng hạn a; b < 0, ta có thể thay a; b; c bởi a; b; c; khi đó ta có thể thấy điều kiện bài toán và 2 biểu thức P Q vẫn không thay đổi, do đó ta chỉ cần xét bài toán trong trường hợp trong 3 số a; b; c, tồn tại ít nhất 2 số dương, chẳng hạn a; b > 0. Nếu c < 0 thì b)2 + (b (a c)2 + (c a)2 (a2 + b2 + c2 ) = (a b)2 + c2 2c(a + b) > 0 Do đó, ta phải có c > 0. Từ đây, với giả thiết a = maxfa; b; cg, ta có 0 = (a = b)2 + (b c)2 + (c a)2 (a2 + b2 + c2 ) p p p p p p p p a+ b+ c a+ b c b+ c ) p a= p b+ p c http://boxtailieu.net p a p c+ p a p b 253 Do đó, bất đẳng thức trở thành p b+ p 2 c + b p 1 12 Đặt t = q b c b+ p 4 c p b + p c c b+ b + b2 c + c2 b+ p p p c p b+ 2 c 5 36 3 2 c 5 2 +b+c > 0; ta phải chứng minh (t + 1)2 1 + t2 + 2 t (t + 1)2 5 (t + 1)4 t2 + t4 + (t + 1)2 (t2 + t + 1)3 2 3 10 9 Nhưng chúng hiển nhiên đúng vì (t + 1)2 1 + t2 + t2 (t + 1)2 2 3 5 , (t3 + t2 (t + 1)4 t2 + t4 + (t + 1)2 , (t3 + 3t2 (t2 + t + 1)3 (t + 1)4 t2 + t4 + (t + 1)2 (t2 + t + 1)3 10 , (t3 9 3t2 1)2 2t 0 1)2 6t 0 1)2 0 Bất đẳng thức được chứng minh xong. Bài toán 2.48 Cho các số dương a; b; c; n: Chứng minh rằng a2 + bc b+c n + b2 + ca c+a n + c2 + ab a+b n an + bn + cn : (Võ Quốc Bá Cẩn) Lời giải. Do tính đối xứng, ta có thể giả sử a a2 + bc a(b + c) b c > 0, khi đó ta có n 1 và (a2 + bc)(b2 + ca) ab(a + c)(b + c) = c(a http://boxtailieu.net b)2 (a + b) 0 254 CHƯƠNG 2. SÁNG TẠO BẤT ĐẲNG THỨC n a2 + bc a(b + c) ) n a2 + bc a(b + c) ) n b2 + ca b(c + a) + 2 n b2 + ca b(c + a) + (a2 + bc)(b2 + ca) ab(a + c)(b + c) 2 n c2 + ab c(a + b) + n=2 3 Do đó x với x = 2 a +bc a(b+c) 0; n 1; y = x+y 0; x+y+z n 2 b +ca b(c+a) c +ab c(a+b) 1; z = 0 n 2 1. Bất đẳng thức đã cho tương đương với an x + bn y + cn z 0 Bất đẳng thức này hiển nhiên đúng vì an x + bn y + cn z = (an bn )x + (bn cn )(x + y) + cn (x + y + z) 0: Đẳng thức xảy ra khi và chỉ khi a = b = c hoặc n ! 0: Bài toán 2.49 Tìm giá trị lớn nhất của biểu thức P = a b c d + + + c d a b abcd (ab + cd)2 a b trong đó a; b; c; d là các số thực phân biệt thỏa mãn (Phạm Văn Thuận) Lời giải. Đặt x = do đó a b = dc ; y = b c = t)2 = (4 a d b c + và t = y+ y1 , suy ra jtj x+ )t )t + 1 x 2 = x2 + 2_t c d + d a = 4 và ac = bd: 2; thì ta có x+ x1 +t = 4; 1 +2 x2 4 6 2_t=2_t 6 Ta có P = a b c d + + + c d a b = t(4 t) a2 d2 abcd = x+ (ab + cd)2 1 = t(4 t) 2 + 1 cb2 + 1 Nếu y > 0; thì t 2 và do đó t phải có t 6; suy ra P = (t 1 x y+ 1 y b2 d 2 (a2 + d2 )(b2 + c2 ) 1 t2 6 hoặc t = 2; nhưng do a; b; c; d phân biệt nên ta 6)(36t3 + 72t2 36t2 t 6) http://boxtailieu.net 433 36 433 36 255 Nếu y < 0; thì t P = 2; suy ra 49 49 433 < 4 4 36 p p 2 2; b = 3 + 2 2; c = 1; d = 1; ta được (t + 2)[ (t + 2)(9 2t)(7 4t2 433 Do đó P 36 : Mặt khác, cho a = 433 P = 36 : Vậy ta đi đến kết luận 2t) + 124] 3 433 : 36 max P = Bài toán 2.50 Cho các số không âm a; b; c; không có 2 số nào đồng thờ bằng 0: Chứng minh rằng p p p a2 + bc + b2 + ca + c2 + ab : 3(a + b + c) 2 (Võ Quốc Bá Cẩn) Lời giải 1. Bình phương 2 vế, bất đẳng thức tương đương với X Xp X a2 + 14 ab 8 5 (a2 + bc)(b2 + ca) cyc cyc cyc Sử dụng bất đẳng thức Cauchy Schwarz, ta có s X Xp (a2 + bc)(b2 + ca) 3 (a2 + bc)(b2 + ca) cyc cyc Do đó, ta chỉ cần chứng minh được 5 X 2 a + 14 cyc X !2 ab cyc 24 X (a2 + bc)(b2 + ca) cyc Do tính thuần nhất, ta có thể chuẩn hóa cho a + b + c = 1. Đặt q = P ab; r = abc, cyc khi đó theo bất đẳng thức Schur bậc 3, ta có r thành 384r + (25 52q)(1 Nếu 1 4q, bất đẳng thức là hiển nhiên. Nếu 4q 384r +(25 52q)(1 4q) 384 4q 1 9 max 0; 4q9 4q) 1 . Bất đẳng thức trở 0 1 thì +(25 52q)(1 4q) = http://boxtailieu.net 1 (4q 1)(156q +53) 3 0: 256 CHƯƠNG 2. SÁNG TẠO BẤT ĐẲNG THỨC Đẳng thức xảy ra khi và chỉ khi a = b; c = 0 hoặc các hoán vị tương ứng. Lời giải 2. Không mất tính tổng quát, ta có thể giả sử a b c, khi đó theo bất đẳng thức AM-GM, ta có p 2 a2 + bc p 2 b2 + ca và a+c+ a2 + bc c(b a) = 2a + c + a+c a+c b+c+ b2 + ca c(a b) = 2b + c + b+c a+c p p ) 2 a2 + bc + 2 b2 + ca p 2 ab + c2 2a + 2b + 2c + b+c+ c(a b)2 (a + c)(b + c) ab + c2 b+c Nên ta chỉ cần chứng minh được 3(a + b + c) 2a + 3b + 3c + ab + c2 b+c ,a , c(a b)2 (a + c)(b + c) c(a b)2 (a + c)(b + c) c(a c) b+c , (a ab + c2 c(a b)2 + b+c (a + c)(b + c) (a b)2 a b 0: a+ c 2 c)(a + c) Bất đẳng thức này hiển nhiên đúng vì a+c Lời giải 3. Giả sử a và b a c c. Vì p a2 + bc p p b2 + ca + c2 + ab p 2(b2 + c2 + ab + ac) Nên ta chỉ cần chứng minh được p 2 2(b2 + c2 + ab + ac) , a2 2(b + 2c)a + b2 a + 3b + 2c 4c2 + 12bc http://boxtailieu.net 0 257 , (a hiển nhiên đúng. Lời giải 4. Giả sử a b 2c)2 + 8c(b b Nên ta chỉ cần chứng minh được Vì (a b+c 2 2s)2 p 2(b2 + c2 + ab + ac) p p a2 + bc + 2(b2 + c2 + ab + ac) (s 0: c. Ta có p p b2 + ca + c2 + ab Đặt s = c) 3 (a + b + c) 2 a) và p = bc, bất đẳng thức trở thành p p 2 a2 + p + 4 2s2 p + as 3(a + 2s) p p , 4 2s2 p + as 3(a + 2s) 2 a2 + p h i2 p , 16(2s2 p + as) 3(a + 2s) 2 a2 + p p , 12(a + 2s) a2 + p 13a2 + 20as + 4s2 + 20p p , 12(a + 2s) a2 + p a (a 2s)2 + 20p 0 và p a2 + p INên ta chỉ cần chứng minh p a= p 2 a +p+a p 2a 6(a + 2s)p 20p a , p(6s 7a) 0: hiển nhiên đúng. Bài toán 2.51 Cho x; y; z 2 [ 1; 1] thỏa mãn x + y + z = 0. Chứng minh rằng p p p 1 + x + y 2 + 1 + y + z 2 + 1 + z + x2 3: (Phan Thành Nam) Lời giải. Bình phương 2 vế, ta có thể viết lại bất đẳng thức như sau X Xp x2 + 2 (1 + x + y 2 )(1 + y + z 2 ) 6 cyc cyc http://boxtailieu.net 258 CHƯƠNG 2. SÁNG TẠO BẤT ĐẲNG THỨC Sử dụng bất đẳng thức Cauchy Schwarz, ta có Xp X Xp (1 + x + y 2 )(1 + y + z 2 ) (1 + x)(1 + y) + jyzj cyc cyc Xp cyc X (1 + x)(1 + y) cyc yz cyc Ta cần chứng minh X x2 2 cyc X xy + 2 cyc Xp (1 + x)(1 + y) 6 cyc Đặt a2 = 1 + x; b2 = 1 + y; c2 = 1 + z (a; b; c 0) ) a2 + b2 + c2 = 3. Bất đẳng thức trở thành X X X X a4 2 a2 b2 + 2 a2 + 2 ab 9 0 cyc cyc X , ,3 X 4 a 2 cyc 4 a 2 2 6 X 2 2 a b + a2 bc + a2 (a b)(a cyc , cyc X X ab 2 X ab X a2 0 cyc X cyc X a4 + X cyc cyc X cyc a b +2 cyc cyc , cyc X 2 a cyc ab(a2 + b2 ) ! 4 cyc X 2 a cyc X a2 b2 b)2 0: ! 0 0 cyc c) + 2 cyc X ab(a cyc Bất đẳng thức cuối hiển nhiên đúng theo bất đẳng thức Schur bậc 4. Đẳng thức xảy ra khi và chỉ khi x = y = z = 0: Bài toán 2.52 Cho các số dương a; b; c thỏa mãn a + b + c = 3: Chứng minh rằng a2 p bc + (a b2 +p 1)2 ca + (b 1)2 c2 +p ab + (c 1)2 (Phan Hồng Sơn) Lời giải. Sử dụng bất đẳng thức Cauchy Schwarz, ta có X cyc p a2 bc + (a 1)2 P cyc P 2 a cyc a2 3: p !2 bc + (a http://boxtailieu.net 1)2 259 và X cyc a2 p 1)2 = bc + (a Xp p abc + a(a a3 cyc v !" u X u X 3 t a [abc + a(a cyc cyc cyc P q2 ab; r = abc thì ta có 9 2 # 1)2 ] cyc Do đó, ta chỉ cần chứng minh được !" !4 X X X 2 3 9 a [abc + a(a a Đặt q = 1)2 # 1) ] cyc 9r. Bất đẳng thức trở thành cyc (9 Ta có (9 2q)4 2q)4 9(27 9(27 9q + 3r)(12 9q + 3r)(12 4 = (9 2q) 4 (9 2q) (81 (81 5q + 6r) 27q + 9r)(36 2 27q + q )(36 q)2 [14q 2 + 135(3 = (3 5q + 6r) q)] 15q + 18r) 15q + 2q 2 ) 0: Đẳng thức xảy ra khi và chỉ khi a = b = c = 1: Bài toán 2.53 Cho các số không âm a; b; c thỏa mãn a2 + b2 + c2 = 1: Chứng minh rằng p a b c 1 + + 2: 1 + bc 1 + ca 1 + ab (Faruk Zejnulahi) Lời giải. Với mọi x 2 0; 21 , ta có 2 x 3 1 1 x(1 2x) = 1+x 3(1 + x) 2 x 3 1 1 1+x Từ đây, chú ý rằng maxfab; bc; cag 21 , ta có ) X cyc a 1 + bc X cyc a 1 2 bc 3 = X 0 a 2abc = a(1 cyc p [a2 + (b + c)2 ][(1 2bc)2 + 1] p = 2(2b2 c2 2bc + 1)(1 + 2bc) p p = 2[1 2b2 c2 (1 2bc)] 2 http://boxtailieu.net 2bc) + b + c 260 CHƯƠNG 2. SÁNG TẠO BẤT ĐẲNG THỨC Mặt khác, ta có X cyc a 1 + bc P !2 a cyc X a X 3abc cyc cyc X = cyc = vì p P 3 a !2 a cyc a + 3abc cyc và X P 1 6 !2 a !2 a X cyc ! X a cyc X a a cyc X a 1 3 X 1 6 X cyc ! 1 cyc cyc 5 ! X a ! ab cyc !2 !2 X a 4 a cyc X ! a cyc 3 15 0: 1: Bất đẳng thức được chứng minh xong. cyc Bài toán 2.54 Cho các số dương a; b; c; d thỏa mãn (a + c)(b + d) minh rằng 1 1 1 1 + + + ab(1 + c) bc(1 + d) cd(1 + a) da(1 + b) 4abcd. Chứng 32 : (1 + a)(1 + b)(1 + c)(1 + d) (Phạm Kim Hùng) Lời giải. Đặt x = a1 ; y = 1b ; z = 1c ; t = d1 , thì ta có x; y; z; t > 0 và (x + z)(y + t) Bất đẳng thức trở thành xz y t + 1+z 1+x + yt x z + 1+y 1+t 32xyzt (1 + x)(1 + y)(1 + z)(1 + t) Sử dụng bất đẳng thức AM-GM, ta có xz y t + 1+z 1+x x z + 1+y 1+t s y t x z 2 xyzt + + 1+z 1+x 1+y 1+t s xyzt(x + z + xt + yz)(y + t + xy + zt) = 2 (1 + x)(1 + y)(1 + z)(1 + t) + yt http://boxtailieu.net 4. 261 Nên ta chỉ cần chứng minh được (x + z + xt + yz)(y + t + xy + zt) Nếu u4 = xyzt 256xyzt (1 + x)(1 + y)(1 + z)(1 + t) 1 thì ta có (x + z + xt + yz)(y + t + xy + zt) = (x + z)(y + t) + (x + z)(xy + zt) + (y + t)(xt + yz) + (xy + zt)(xt + yz) p p 4 xyzt + 2 (x + z)(y + t)(xy + zt)(xt + yz) + 4xyzt p 4 xyzt + 8(xyzt)3=4 + 4xyzt = 4u2 + 8u3 + 4u4 = 4u2 (u + 1)2 (1 + x)(1 + y)(1 + z)(1 + t) 1 + (xyzt)1=4 4 = (1 + u)4 và 4u2 (1 + u)6 Nếu u4 = xyzt 256u4 = 4u2 [(u + 1)6 64u2 ] = 4u2 [(u + 1)3 8u][(u + 1)3 + 8u] = 4u2 (u 1)(u2 + 4u 1)[(u + 1)3 + 8u] 0 1 thì ta có (x + z + xt + yz)(y + t + xy + zt) = (x + z)(y + t) + (x + z)(xy + zt) + (y + t)(xt + yz) + (xy + zt)(xt + yz) p 4 + 2 (x + z)(y + t)(xy + zt)(xt + yz) + 4xyzt p 4 + 8 xyzt + 4xyzt = 4(u2 + 1)2 (1 + x)(1 + y)(1 + z)(1 + t) = 1 + (x + z) + (y + t) + (x + z)(y + t) + xz + yt + xz(y + t) + yt(x + z) + xyzt p p p 5 + 2 (x + z)(y + t) + 2 xyzt + 2 xyzt(x + z)(y + t) + xyzt p p 2 9 + 6 xyzt + xyzt = 3 + xyzt = (3 + u2 )2 và 4(u2 + 1)2 (u2 + 3)2 256u4 = 4[(u2 + 1)2 (u2 + 3)2 64u4 ] = 4[(u2 + 1)(u2 + 3) 8u2 ][(u2 + 1)(u2 + 3) + 8u2 ] = 4(3 u2 )(1 u2 )[(u2 + 1)(u2 + 3) + 8u2 ] 0: Đẳng thức xảy ra khi và chỉ khi a = b = c = d = 1: http://boxtailieu.net 262 CHƯƠNG 2. SÁNG TẠO BẤT ĐẲNG THỨC Bài toán 2.55 Cho các số không âm a; b; c; không có 2 số nào đồng thời bằng 0: Chứng minh rằng r r r r a2 + bc b2 + ca c2 + ab 2(a2 + b2 + c2 ) 1 + + +p : 2 2 2 2 2 2 b +c c +a a +b ab + bc + ca 2 (Võ Quốc Bá Cẩn) Lời giải. Sử dụng bất đẳng thức AM-GM, ta có vP 0v 12 P u 2 P a2 u a2 2 a2 u u 1 C 1 cyc cyc u cyc Bu P P p t tP + + 2 + = @ A ab ab ab 2 2 cyc cyc 1 2+ p 2 cyc P a2 cyc P ab + p cyc Do đó, ta chỉ cần chứng minh được X cyc r a2 + bc b2 + c2 !2 = X a2 + bc cyc b2 + c2 1 2+ p 2 +2 X cyc P (a2 + bc)(b2 + ca) (a2 + c2 )(b2 + c2 ) p 2+ 2 a cyc P s ab + 1 2 cyc Giả sử a b c, ta sẽ chứng minh rằng r r a2 + c2 b2 + c2 + 2 2 b +c c2 + a2 , , , a2 + c2 b2 + c2 + 2 +2 2 2 b +c a + c2 a2 + c2 b2 + c2 + b2 + c2 a2 + c2 , r (a2 b2 )2 (a2 + c2 )(b2 + c2 ) 4c2 + (a c2 + ab b)2 2 2(a2 + b2 ) c2 + ab 2(a2 + b2 ) c2 + ab 2(a2 + b2 ) c2 + ab 2(a 4 b)2 4c2 c2 + ab (a + b)2 (a2 + c2 )(b2 + c2 ) c2 2 + ab 0 4c2 (a b)2 (ab c2 )(a2 + b2 + 2c2 ) + 0 c2 + ab (a2 + c2 )(b2 + c2 )(ab + c2 ) r r r r r a2 + bc b2 + ca a2 + c2 b2 + c2 2(a2 + b2 ) ) + + b2 + c2 c2 + a2 b2 + c2 c2 + a2 c2 + ab , http://boxtailieu.net 2+ 1 2 263 Mặt khác s (a2 + bc)(b2 + ca) (a2 + c2 )(b2 + c2 ) 1 Do đó X cyc s r (a2 + bc)(b2 + ca) = (a2 + c2 )(b2 + c2 ) + s c2 + ab a2 + b2 r a2 + bc + b2 + c2 r p (a2 + bc)(b2 + ca) (a2 + c2 )(b2 + c2 ) b2 + ca c2 + a2 ! 2+1 Ta còn phải chứng minh X a2 + bc b2 + c2 cyc X a + bc b2 + c2 cyc 2 , Nếu P a2 2 cyc P P b2 cyc P 1 2+ p 2 a2 cyc P ab cyc 5 2 1 2+ p 2 P P a2 2 ab cyc cyc P ab cyc ab, bất đẳng thức là hiển nhiên vì + a2 + c2 b2 + c2 ab + 2 + 2 2 2 b +c a + c2 a + b2 5 2 c2 = Nếu 3 2+ 2 cyc X a2 + bc cyc + p a2 2 P (2a2 ab + 2b2 )(a 2ab(a2 + b2 ) ab thì ta có a p b+ p b)2 5 2 a b ab + + 2 b a a + b2 0 2 c b + 3c và do 2 + cyc cần chứng minh được , X a2 + bc cyc b2 + c2 5 2 3 5 2 P P a2 6 ab cyc cyc P ab cyc http://boxtailieu.net p1 2 < 3, nên ta chỉ 264 Ta có CHƯƠNG 2. SÁNG TẠO BẤT ĐẲNG THỨC X a2 + bc b2 cyc + b2 + c2 ab a2 + c2 + 2 + 2 2 2 b +c a + c2 a + b2 5 2 c2 5 2 (a2 b2 )2 (a b)2 (a2 + c2 )(b2 + c2 ) 2(a2 + b2 ) (a + b)2 1 = (a b)2 (a2 + c2 )(b2 + c2 ) 2(a2 + b2 ) 3(a b)2 (a + b)2 4(a2 + c2 )(b2 + c2 ) = và P P a2 2 ab (a cyc cyc P = ab b)2 + c(c 2a P ab cyc 2b) (a cyc Nên ta chỉ cần chứng minh được (a b)2 (a + b)2 4(a2 + c2 )(b2 + c2 ) (a b)2 2ac P ab cyc b)2 2ac P ab cyc b)2 , (a Nếu a (a + b)2 4(a2 + c2 )(b2 + c2 ) 1 2ac + ab + bc + ca ab + bc + ca b + 5c; ta sẽ chứng minh (a + b)2 4(a2 + c2 )(b2 + c2 ) 1 ab + bc + ca , f (c) = (a + b)2 (ab + bc + ca) b)2 f (0) = ab(a f a b 5 = 1 (a 625 b + 5c thì 2ac b)(21a3 + 1112a2 b 2 a(a 5 b) 0 4(a2 + c2 )(b2 + c2 ) Bất đẳng thức này đúng vì f (c) là hàm lõm trên 0; Nếu a 0 2 (a 5 a b 5 và 0 362ab2 + 229b3 ) b)2 Suy ra, ta chỉ cần chứng minh 4(a2 (a + b)2 + c2 )(b2 + c2 ) 0 3 5(ab + bc + ca) http://boxtailieu.net 0 0 265 , g(c) = 5(a + b)2 (ab + bc + ca) Bất đẳng thức vì g(c) là hàm lõm trên 0; g(0) = ab(5a2 a g b = 3 1 (5a4 + 313a3 b 27 12(a2 + c2 )(b2 + c2 ) a b 3 0 và 2ab + 5b2 ) 0 150a2 b2 + 133ab3 85b4 ) 0: Đẳng thức xảy ra khi và chỉ khi a = b; c = 0 hoặc các hoán vị tương ứng. Bài toán 2.56 Cho các số không âm a; b; c; không có 2 số nào đồng thời bằng 0: Chứng minh rằng a b c + 3 + 3 b3 + c3 c + a3 a + b3 18 5(a2 + b2 + c2 ) ab bc ca : (Michael Rozenberg) Lời giải. Bất đẳng thức tương đương với X a(a + b + c) cyc , X cyc b3 + c3 2 a + b3 + c3 X cyc P 18 a cyc P P 5 a2 ab cyc a bc + c2 b2 X cyc a2 b3 + c3 P cyc a2 !2 X cyc b2 = a2 (b3 + c3 ) cyc a bc + c2 P a2 b2 cyc và P cyc P cyc a(b2 P 18 a cyc P P 5 a2 ab cyc Sử dụng bất đẳng thức Cauchy Schwarz, ta có P cyc !2 ! P cyc a2 cyc P cyc a bc + c2 ) = P cyc http://boxtailieu.net ! a !2 ! a P abc P ab cyc !2 a cyc P cyc ! ab 6abc 266 CHƯƠNG 2. SÁNG TẠO BẤT ĐẲNG THỨC Suy ra, ta chỉ cần chứng minh được !2 P 2 a P a2 b2 cyc ! cyc P cyc ! a abc P + ab P cyc cyc ! P a cyc a !2 P ! ab cyc 6abc P 18 a cyc P P 5 a2 ab cyc cyc P Do tính thuần nhất, ta có thể chuẩn hóa cho a + b + c = 1. Đặt q = ab; r = abc, cyc o n q) . Bất đẳng thức trở theo bất đẳng thức Schur bậc 4, ta có r max 0; (4q 1)(1 6 thành (1 q2 Nếu 1 2q)2 1 + (q + 2)r q 6r 5 18 11q 4q thì (1 q2 Nếu 4q (1 q2 2q)2 1 + (q + 2)r q 6r 18 5 11q 2q)2 (1 1 18 + q2 q 5 11q (1 4q)(11q 2 6q + 5) = q 2 (5 11q) 0 1 thì 1 2q)2 + (q + 2)r q 6r q2 = 18 5 11q (1 2q)2 (q + 2) (4q 1)(1 q) 6 3 + q (6q 1 1)(1 q) (1 3q)(4q 1)(112q 117q 2 + 39q 4) 2 3 (1 2q) (5 11q)(4q + 9q 2 9q + 2) 5 18 11q 0: Đẳng thức xảy ra khi và chỉ khi a = b = c hoặc a = b; c = 0 các hoán vị tương ứng. Bài toán 2.57 Cho các số dương a; b; c: Chứng minh rằng r r r r a b c 3 ab + bc + ca p + + : a+b b+c c+a a2 + b2 + c2 2 (Võ Quốc Bá Cẩn, Nguyễn Văn Thạch) Lời giải. Trước hết, ta sẽ chứng minh P 3 a2 cyc P a cyc X 2a2 a+c cyc http://boxtailieu.net 267 Thật vậy, ta có 3 X 2 a cyc X ! X 2a2 a+c cyc a cyc ! = 3 X X 2 a cyc X = cyc cyc a+b ! X c(a2 + b2 ) a2 X X a2 + b2 a cyc cyc = ! a+b cyc ab(a b)2 (a + c)(b + c) 0 Bây giờ, sử dụng bất đẳng thức bất đẳng thức này và bất đẳng thức Holder, ta được P !2 " # 3 a2 Xr a X cyc P a(a + b)(a + c) a a+b cyc cyc cyc X a2 2 a+c cyc ! Xr a a+b cyc !2 " X # a(a + b)(a + c) cyc X 2 cyc !4 a Ta cần chứng minh 4 X cyc ,4 X cyc !4 a !4 a 27 X !2 " ab cyc 27 X !2 " ab cyc X # a(a + b)(a + c) cyc X cyc 2 a ! X ! a cyc Do tính thuần nhất, ta chuẩn hóa cho a + b + c = 1. Đặt q = # + 3abc P ab; r = abc thì ta có cyc q2 3r. Bất đẳng thức trở thành 4 27q(1 2q + 3r) Ta có 4 27q(1 2q + 3r) 4 27q(1 2q + q 2 ) = (4 3q)(1 3q)2 0: Đẳng thức xảy ra khi và chỉ a = b = c: Bài toán 2.58 Cho các số dương a; b; c: Chứng minh rằng X a2 + b2 + c2 a2 + 2b2 + 2c2 cyc X b2 + bc + c2 : a2 + (b + c)2 cyc (Phạm Hữu Đức) http://boxtailieu.net 268 CHƯƠNG 2. SÁNG TẠO BẤT ĐẲNG THỨC Lời giải. Bất đẳng thức tương đương với !" X X X 1 a2 bc 2 a 2 2 2 a + (b + c) a + (b + c)2 cyc cyc cyc , X cyc X a2 bc a2 + (b + c)2 cyc 2 a !" X cyc 1 2 a + 2b2 + 2c2 (b c)2 [a2 + (b + c)2 ](a2 + 2b2 + 2c2 ) # # Do tính thuần nhất, ta có thể chuẩn hóa cho a2 + b2 + c2 = 1. Bất đẳng thức trở thành X a2 bc X (b c)2 1 + 2bc (1 + 2bc)(1 + b2 + c2 ) cyc cyc Chú ý rằng X 2(a2 X 2a2 b2 c2 X (b c)2 bc) = + 1 + 2bc 1 + 2bc 1 + 2bc cyc cyc cyc X = (b2 c2 ) cyc X = (b c)2 cyc 1 1 + 2ab 1 1 + 2ca + X (b cyc c)2 1 + 2bc 1 2a(b + c) + (1 + 2ab)(1 + 2ac) 1 + 2bc Do đó, bất đẳng thức tương đương với X c)2 (b cyc , X (b 2a(b + c) 1 + (1 + 2ab)(1 + 2ac) 1 + 2bc c)2 cyc 2a(b + c) (1 + 2ab)(1 + 2ac) 2 (1 + 2bc)(1 + b2 + c2 ) a2 (1 + 2bc)(1 + b2 + c2 ) 0 0 Mặt khác, ta có 2(1 + 2bc)(1 + b2 + c2 ) 2 = [1 + (b + c) 2(b + c)(1 (1 + 2ab)(1 + 2ac) 2a(b + c)] + [(b + c)2 a) + 4bc(1 4a2 bc] + 4bc(b2 + c2 ) a2 ) + 4bc(b2 + c2 ) 0 Nên ta chỉ cần chứng minh được X cyc (b c)2 2a(b + c) (1 + 2ab)(1 + 2ac) 2a2 (1 + 2ab)(1 + 2ac) http://boxtailieu.net 0 269 , X c)2 x(b 0 cyc trong đó x = a(b + c a)(1 + 2bc) và y; z tương tự. Không mất tính tổng quát, giả sử a b c, khi đó ta có y; z 2 b(a c) c)2 (b + c a(b a(b c) (b + c 1 + 2ac c) (1 + 2bc) 0 và 1 + 2bc c)2 (a + c b)(1 + 2ac) 2 a)(1 + 2bc) + a(b 2 = 2ac(b c) ; a)(1 + 2bc) + b(a 2 a(b 2 c) (a + c b)(1 + 2bc) 0: Đẳng thức xảy ra khi và chỉ khi a = b = c: Bài toán 2.59 Cho các số dương x; y thỏa mãn x + y 2 3x2 + 2y 3 x2 + y 3 : Chứng minh rằng 5: (Ji Chen) Lời giải 1. Từ x + y 2 x2 + y 3 , ta có 5(x + y 2 ) (3x2 + 2y 3 ) + (2x2 + 3y 3 ) Ta sẽ chứng minh rằng 5 + 2x2 + 3y 3 , (2x 3)(x 1) + [3y 2 Chú ý rằng nếu x; y 1 thì ta có x + y bài toán. Nếu y 1 x thì ta có (3 5(x + y 2 ) 2x)(1 2 x) 2(y + 1)](y 2 1) 0 3 x + y , điều này mâu thuẫn với giả thiết x(1 y 2 (y x) 1) Do đó (2x 3)(x 1) + [3y 2 Nếu x 1 2(y + 1)](y [4y 2 1) 2(y + 1)](y 1) = 2(y 1)2 (2y + 1) y thì [2(y + 2) 3y 2 ](1 y 2 (1 y) y) x(x 1) Do đó (2x 3)(x 1) + [3y 2 2(y + 1)](y 1) (3x 3)(x 1) = 3(x 1)2 0: Đẳng thức xảy ra khi và chỉ khi x = y = 1: Lời giải 2. Ta có 5 3x2 2y 3 = 6(x x2 + y 2 y 3 ) + 3(1 x)2 + 2(1 http://boxtailieu.net y)2 (1 + 2y) 0: 0 270 CHƯƠNG 2. SÁNG TẠO BẤT ĐẲNG THỨC Bài toán 2.60 Cho các số không âm a; b; c thỏa mãn a + b + c = 3. Chứng minh rằng b c a + + 1 + a + b2 1 + b + c2 1 + c + a2 3 : 4 (Võ Quốc Bá Cẩn) Lời giải. Xét 2 trường sau P 2hợpP Trường hợp 1. Nếu a + ab2 cyc X cyc 9, sử dụng bất đẳng thức Cauchy Schwarz, ta có cyc a 1 + a + b2 P !2 P a cyc a(1 + a + b2 ) = 3+ cyc Trường hợp 2. Nếu P a2 + P P 9 a2 P + cyc ab2 3 4 ab2 cyc 9, tức là cyc cyc X ab2 2 cyc X a2 b + 6abc cyc Bất đẳng thức tương đương với X 4 a(1 + b + c2 )(1 + c + a2 ) 3(1 + a + b2 )(1 + b + c2 )(1 + c + a2 ) cyc , X a2 b3 + 5 cyc X a3 b + cyc X a3 + 9abc+ cyc X a2 b + 5 cyc 3 X X ab cyc a2 b2 + 3 cyc Do P cyc ab2 2 P X a2 + 3a2 b2 c2 cyc a2 b + 6abc nên ta có cyc ab , X cyc , ! X X cyc cyc a2 b3 X 2 ab cyc a2 b3 2 ! X cyc 2 X ! 2 a b + 3abc cyc a3 b2 + 7 X a2 b2 c + cyc X ab cyc X ! a3 bc cyc 2X 2 2 7X 2 2 1X 3 a b (a + b) + a b c+ a bc 3 cyc 3 cyc 3 cyc http://boxtailieu.net 271 Đặt q = ab + bc + ca; r = abc thì 2X 2 2 7X 2 2 1X 3 a b (a + b) + a b c+ a bc = 2q 2 + (q 3 cyc 3 cyc 3 cyc ) X a2 b3 2q 2 + (q 9)r 9)r cyc Mặt khác, ta có X cyc 3 a b X 2 a bc = 3r; cyc X P !2 ab cyc 2 P a b cyc a = 1 2 q 3 cyc Từ những bất đẳng thức này, ta chỉ cần chứng minh được [2q 2 + (q 9)r] + 15r + (27 , 1 9q + 3r) + 9r + q 2 + 5q 3 2 q(3 3 3r2 q) + (36 + q)r 3(q 2 6r) + 3(9 2q) + 3r2 0: hiển nhiên đúng do 3 q; 1 r. Bất đẳng thức được chứng minh. Đẳng thức xảy ra khi và chỉ khi a = 3; b = c = 0 hoặc các hoán vị tương ứng. Bài toán 2.61 Cho các số dương a; b; c: Chứng minh rằng a3 (b2 + 1) + b3 (c2 + 1) + c3 (a2 + 1) 8(a + b + c) 18: (Nguyễn Công Minh) Lời giải. Sử dụng bất đẳng thức Cauchy Schwarz, ta có !2 !2 P 2 P a b ab X X cyc cyc 5 2 2 P P a b ; a b a b cyc cyc cyc ) X cyc cyc 5 2 a b P !4 ab cyc P cyc !3 a http://boxtailieu.net 272 CHƯƠNG 2. SÁNG TẠO BẤT ĐẲNG THỨC Mặt khác, bất đẳng thức đã cho tương đương với ! X X X 5 2 a b + a + a3 + 18 cyc cyc 9 cyc X Do đó, theo trên và theo bất đẳng thức AM-GM, ta có !4 P 2 ab X X X cyc 5 2 a a b + a !3 + P cyc cyc cyc a cyc Mặt khác, X !3 a cyc ) 18 9 + 27 + 27 27 a 0 cyc X P !2 ab cyc P a cyc a cyc X X 1 3 a cyc !3 a cyc Nên ta chỉ cần chứng minh được 2 !2 P ab cyc P a + a3 cyc cyc , X 2q 2 X 3 + a p cyc 1 3 X cyc !3 a 1 3 p 3 trong đó p = a + b + c; q = ab + bc + ca. Sử dụng bất đẳng thức Schur bậc 3, ta có X cyc a3 1 p(2p2 3 5q) Suy ra 2q 2 X 3 + a p cyc 1 3 p 3 2q 2 1 + p(2p2 p 3 5q) 1 3 (p2 p = 3 vì p2 3q: Đẳng thức xảy ra khi và chỉ khi a = b = c = 1: http://boxtailieu.net 3q)(p2 3p 2q) 0: 273 Bài toán 2.62 Cho các số không âm a; b; c; d thỏa mãn a + b + c + d = 3: Chứng minh rằng p ab(a + 2b + 3c) + bc(b + 2c + 3d) + cd(c + 2d + 3a) + da(d + 2a + 3b) 6 3: (Phạm Kim Hùng) Lời giải. Đặt f (a; b; c; d) = X ab(a + 2b + 3c) cyc Ta có f (a; b; c; d) f (a + c; b; 0; d) = c(b d)(a + c b d) và f (a; b; c; d) f (0; b; a + c; d) = a(b d)(a + c b d) Suy ra, nếu (b d)(a + c b d) 0 thì f (a; b; c; d) f (a + c; b; 0; d) và nếu f (b d)(a + c b d) 0 thì f (a; b; c; d) f (0; b; a + c; d), tức là f (a; b; c; d) maxff (a + c; b; 0; d); f (0; b; a + c; d)g Tương tự, ta có f (a + c; b; 0; d) f (0; b; a + c; d) maxff (a + c; b + d; 0; 0); f (a + c; 0; 0; d)g maxff (0; b + d; a + c; 0); f (0; 0; a + c; b + d)g Mặt khác, ta dễ dàng kiểm tra được p 6 3 p maxff (0; b + d; a + c; 0); f (0; 0; a + c; b + d)g 6 3: maxff (a + c; b + d; 0; 0); f (a + c; 0; 0; d)g Bất p đẳng thức p được chứng minh. Đẳng thức xảy ra khi và chỉ khi a = d = 0; b = 3 3; c = 3 hoặc các hoán vị tương ứng. Bài toán 2.63 Cho các số dương a; b; c: Chứng minh rằng r r r r a b c 4 3 + + (a + b + c): a2 + b + 4 b2 + c + 4 c2 + a + 4 4 (Nguyễn Công Minh) Lời giải. Sử dụng bất đẳng thức Cauchy Schwarz, ta có !2 Xr X X a a a 3 3 2 2 a + b + 4 a + b + 4 2a + b+3 cyc cyc cyc http://boxtailieu.net 274 CHƯƠNG 2. SÁNG TẠO BẤT ĐẲNG THỨC Ta cần chứng minh X 2a 2a + b + 3 X b+3 + 2a + b + 3 cyc , cyc vP u u a t cyc 3 vP u u a t cyc 3 3 Sử dụng bất đẳng thức Cauchy Schwarz, X cyc b+3 2a + b + 3 P " P #2 (b + 3) cyc (b + 3)(2a + b + 3) P cyc a+9 !2 a+9 cyc = cyc P P !2 a + 12 P a + 27 cyc cyc = P a+3 cyc Đặt P a = 3t2 (t 0), ta phải chứng minh cyc 3t2 + 9 +t 3t2 + 3 , t(t 1)2 t2 + 1 3 0: hiển nhiên đúng. Đẳng thức xảy ra khi và chỉ khi a = b = c = 1 hoặc a = b = c ! 0: Bài toán 2.64 Cho các số không âm a; b; c; không có 2 số nào đồng thời bằng 0: Chứng minh rằng s s s p a(b + c) b(c + a) c(a + b) + + 2: (2b + c)(b + 2c) (2c + a)(c + 2a) (2a + b)(a + 2b) Lời giải. Sử dụng bất đẳng thức Holder, ta có s " #" # X X a2 (2b + c)(b + 2c) a(b + c) (2b + c)(b + 2c) b+c cyc cyc http://boxtailieu.net X cyc !3 a 275 Ta cần chứng minh X !3 a cyc X , , Chú ý rằng X X cyc và a cyc !3 X X a2 (2b + c)(b + 2c) b+c cyc 4 a cyc 3 2 X a2 (b + c) + 2abc cyc cyc X a (b + c) + 3abc a2 (b + c) + 3abc = X cyc X 2a b+c cyc a b+c X 2a abc b+c cyc 2 cyc a3 X a(a ! 3 b)(a c) cyc 3= X (2a + b + c)(a cyc b)(a c) (a + b)(b + c)(c + a) Bất đẳng thức tương đương X x(a b)(a c) 0 cyc trong đó x = a abc(2a+b+c) (a+b)(b+c)(c+a) a2 (ab+ac+b2 +c2 ) (a+b)(b+c)(c+a) = Không mất tính tổng quát, ta có thể giả sử a a c và x y = (a ) x(a b) 1 c) y(b b 0 và y; z tương tự. b c c, khi đó 0 abc (a + b)(b + c)(c + a) c) (b c)(x 0 y) 0 Mặt khác, ta có thể viết lại bất đẳng thức như sau (a b)[x(a c) y(b c)] + z(a c)(b c) 0: Bất đẳng thức được chứng minh. Đẳng thức xảy ra khi và chỉ khi a = b = c hoặc a = b; c = 0 hoặc các hoán vị tương ứng. Nhận xét 18 Chú ý rằng 2(b2 + c2 ) (2b + c)(b + 2c), nên s s X X p a(b + c) a(b + c) 2 2 2 2(b + c ) (2b + c)(b + 2c) cyc cyc http://boxtailieu.net 276 CHƯƠNG 2. SÁNG TẠO BẤT ĐẲNG THỨC và ta thu được kết quả đẹp sau r r r a(b + c) b(c + a) c(a + b) + + b2 + c2 c2 + a2 a2 + b2 2: (Võ Quốc Bá Cẩn) Đẳng thức đạt được tại a = b; c = 0 và các hoán vị của nó. Bài toán 2.65 Cho các số không âm a; b; c; không có 2 số nào đồng thời bằng 0: Chứng minh rằng 2 b3 c3 a3 + + b+c c+a a+b p 3abc(a + b + c): 2(a2 + b2 + c2 ) (Nguyễn Công Minh) Lời giải. Sử dụng bất đẳng thức Holder, ta có X a3 b+c cyc P 2 a cyc P cyc !" a P !3 # a2 (b + c) cyc Ta cần chứng minh 2 P a cyc P cyc !" a 2 P !3 #+ a2 (b + c) cyc s 3abc X a 2 cyc X a2 cyc Do tính thuần nhất, ta có thể chuẩn hóa cho a+b+c =n1. Đặt q = ab+bc+ca; r = abc, o (4q 1)(1 q) khi đó theo bất đẳng thức Schur bậc 4, ta có r max 0; . Bất đẳng thức 6 trở thành 2(1 q Nếu 1 2q)3 p + 3r 3r 2(1 2q) 4q thì 2(1 q 2q)3 p + 3r 3r 2(1 2q)3 2(1 2q) 2(1 q = 2(1 2q)(1 http://boxtailieu.net q)(1 q 2q) 4q) 0 277 Nếu 4q 1 thì 2(1 q 2q)3 p + 3r 3r 2(1 2(1 2q) q 2q)3 (4q 1)(1 q) 2 4(1 2q)3 = + 4q 2 3q + 1 + r r (4q (4q 1)(1 2 1)(1 2 q) q) Ta cần chứng minh r 4(1 2q)3 (4q 1)(1 q) + 2(1 2q) 2 4q 3q + 1 2 r (4q 1)(1 q) 4(1 2q)3 , 2(1 2q) 2 4q 2 3q + 1 r 2(1 2q)(4q 1)(1 q) (4q 1)(1 q) , 2 4q 2 3q + 1 p , 4q 2 3q + 1 2(1 2q) 2(4q 1)(1 q) Theo bất đẳng thức AM-GM, ta có p 4q 2 3q + 1 2(1 2q) 2(4q 4q 2 3q + 1 (1 1)(1 q) 2q)[2(4q 1) + 1 3q)2 2q] = 2(1 0: Bất đẳng thức được chứng minh. Đẳng thức xảy ra khi và chỉ khi a = b = c hoặc a = b; c = 0 hoặc các hoán vị tương ứng. Bài toán 2.66 Cho a; b; c là độ dài 3 cạnh của một tam giác. Chứng minh rằng (a2 b + b2 c + c2 a)2 abc(a + b + c)(a2 + b2 + c2 ): (Võ Quốc Bá Cẩn) Lời giải 1. Bất đẳng thức tương đương với X X a4 b2 a4 bc abc(a cyc , , X X a4 (b c)2 + cyc a4 (b b)(a c)(b c) cyc X a4 b2 cyc c)2 + (a b)(a X a2 b4 2abc(a b)(a c)(b c) cyc c)(b c)[(a + b)(b + c)(c + a) cyc http://boxtailieu.net 2abc] 0 278 CHƯƠNG 2. SÁNG TẠO BẤT ĐẲNG THỨC , X a4 (b c)2 + (a b)(a c)(b cyc c) X ab(a + b) 0 cyc Do a; b; c là độ dài 3 cạnh của một tam giác nên tồn tại các số không âm x; y; z sao cho a = y + z; b = z + x; c = x + y. Bất đẳng thức trở thành " # X X X 4 2 3 2 (y + z) (y z) + (x y)(y z)(z x) 2 x +5 x (y + z) + 12xyz cyc cyc 0 0 cyc Từ đây, giả sử x = minfx; y; zg và đặt y = x + p; z = x + q, ta có thể dễ dàng viết lại bất đẳng thức như sau Ax4 + Bx3 + Cx2 + Dx + E 0 trong đó A = 32(p2 B = 16(4p C = 48(p 4 pq + q 2 ) 3 5p q + pq + 4q 3 ) 3 pq 3 4 3 2 2 3p q 3 2 q ) 3 q)2 (p + q) + q 4 ] 0 4 p q + pq + 2q ) q) + pq(5p2 2 0 4 p q + pq + q ) = 48[p(p 4 = 8(p + q)[2(p E = 2(p 2 2 2 p q D = 8(p + q)(2p 3 0 2 13pq + 9q 2 )] 0 0: Bất đẳng thức được chứng minh. Đẳng thức xảy ra khi và chỉ khi a=b=c hoặc a = b + c; b = p p 3 100 + 12 69 2 + p p 3 6 3 100 + 12 69 1 3 ! c: hoặc các hoán vị tương ứng. Lời giải 2. Không mất tính tổng quát, ta có thể giả sử (a b)(a c) 0. Xét hàm số sau f (x) = (a2 + b2 + c2 )x2 2(a2 b + b2 c + c2 a)x + abc(a + b + c) Ta có f (a) = a(a b)(a c)(a b + c) 0 và lim f (x) = +1 x!1 Do đó, tồn tại x0 sao cho f (x0 ) = 0, và vì thế biệt thức của f (x) phải không âm, tức là (a2 b + b2 c + c2 a)2 abc(a + b + c)(a2 + b2 + c2 ): http://boxtailieu.net 279 Nhận xét 19 Sử dụng bất đẳng thức Cauchy Schwarz và bất đẳng thức trên, ta có !2 P 2 a b X a2 b X cyc P 2 a2 c a bc cyc cyc cyc và ta được a2 b b2 c + + c a b 1 + b2 c , a2 c2 a a2 + b2 + c2 b c a 1 + c2 1 a b 0: Đây chính là bất đẳng thức trong kỳ thi chọn đội tuyển đi thi toán quốc tế của Moldova 2006. Bài toán 2.67 Cho các số không âm a; b; c thỏa mãn a + b + c = 3. Chứng minh rằng p p p ab a + bc b + ca c 3: (Vasile Cirtoaje) Lời giải. Sử dụng bất đẳng thức Cauchy Schwarz, ta có !2 ! ! X X X p ab a2 b ab a cyc cyc Ta cần chứng minh X ! ab cyc , , X cyc X ! ab cyc 3 a b cyc cyc X !2 1 ab 4 3 X cyc a2 a b 1 3 !2 ! 9 2 a b cyc X ab cyc P ! !2 2 a cyc + ! X a cyc cyc P 2 cyc !2 X ab 4 a3 b + Sử dụng bất đẳng thức Vasile X X cyc X cyc !2 27 3 3abc5 27 . Ta cần phải chứng minh !2 ab http://boxtailieu.net 3 3abc5 27 280 CHƯƠNG 2. SÁNG TẠO BẤT ĐẲNG THỨC Đặt q = ab + bc + ca, ta phải chứng minh q[(9 2q)2 + 3q 2 9abc] Theo bất đẳng thức Schur bậc 3, ta có 3r 4q được q[(9 2q)2 + 3q 2 3(4q , (3 q)(7q 2 81 9. Do đó, ta chỉ cần chứng minh 9)] 27q + 27) 81 0: hiển nhiên đúng do q 3. Đẳng thức xảy ra khi và chỉ khi a = b = c = 1: Nhận xét 20 Từ bài này, ta suy ra kết quả sau ở phần trước p 3(a2 + b2 + c2 ) b3 c3 a3 + + 2 2 2 2 2 2 a +b b +c c +a 2 (Võ Quốc Bá Cẩn) Ở đây, chúng ta có một lời giải khác bằng cách sử dụng bất đẳng thức Holder như sau X a3 2 a + b2 2 hX (a2 + b2 )2 (a2 + c2 )3 i hX i3 i3 1 hX 2 a2 (a2 + c2 ) = (a + b2 )2 8 Nên ta chỉ cần chứng minh được i3 i 1 hX 2 3 X 2 hX 2 (a + b2 )2 a (a + b2 )2 (a2 + c2 )3 8 4 p p 2 2 p 2 2 Đặt x = a + b ; y = b + c ; z = c2 + a2 thì bất đẳng thức trở thành p p p p p p (x + y + z)3 3 x + y + z xy x + yz y + zx z Theo bất đẳng thức Cauchy Schwarz, ta có p p p p x+ y+ z 3(x + y + z) Nên ta chỉ cần chứng minh được (x + y + z)5=2 p p p p 3 3 xy x + yz y + zx z Do tính thuần nhất nên ta có thể chuẩn hóa cho x + y + z = 3: Khi đó bất đẳng thức trở thành p p p xy x + yz y + zx z 3: Đây chính là bất đẳng thức được chứng minh ở trên. Vậy nên bất đẳng thức cần chứng minh đúng. http://boxtailieu.net 281 Bài toán 2.68 Cho a; b; c là độ dài 3 cạnh của một tam giác. Chứng minh rằng a b c + + +3 b c a a+b b+c c+a + + b+c c+a a+b 2 : (Võ Quốc Bá Cẩn) Lời giải. Bất đẳng thức tương đương với Xa X c +2 b b+c cyc cyc , X cyc ! ab , Xa cyc b +2 cyc c b+c X cyc ! a +3 b+c X X cyc ! ab cyc X bc2 +2 b b+c cyc X ca2 cyc X 2 2 X cyc ! a +3 b+c X abc a2 + 2 b+c cyc Sử dụng bất đẳng thức Moldova, ta có X ca2 X b cyc a2 cyc Do đó, ta chỉ cần chứng minh được X bc2 b+c cyc , , X X bc(c cyc a) b+c bc(c2 cyc , X cyc X abc b+c cyc a2 b3 0 a2 )(a + b) X 0 a2 b2 c: cyc hiển nhiên đúng theo bất đẳng thức AM-GM. Đẳng thức xảy ra khi và chỉ khi a = b = c: Bài toán 2.69 Cho các số không âm a; b; c; không có 2 số nào đồng thời bằng 0: Chứng minh rằng a2 + b2 + c2 3(a3 b + b3 c + c3 a) + 2 2 4: ab + bc + ca a b + b2 c2 + c2 a2 (Bách Ngọc Thành Công) http://boxtailieu.net 282 CHƯƠNG 2. SÁNG TẠO BẤT ĐẲNG THỨC Lời giải. Bất đẳng thức tương đương với ! ! ! ! X X X X 2 2 2 3 a a b +3 a b ab cyc ,4 X cyc a4 b2 + cyc ,4 X cyc a2 b4 + 3 cyc X 4 2 a b + cyc X X a b 4 cyc , X 2 2 a4 bc + 3a2 b2 c2 a b (2a 3 3 a b abc 4 cyc b) X a3 b3 + X abc 4 X 2 2 ab + ab + cyc X X X 2 2 a b cyc a3 b2 c + 4 a b 3 X 2 a b ! X a2 b3 c cyc 2 3 cyc cyc Sử dụng bất đẳng thức AM-GM, ta có X X 2 a2 b2 (2a b)2 = [a2 b2 (2a ab X cyc cyc cyc cyc 4 ! cyc cyc X 2 4 cyc cyc 2 4 X X 3 a 3abc cyc 3 a ! 3abc cyc b)2 + a2 c2 (2c ! a)2 ] cyc 2 X a2 bc(2a b)(2c a) cyc = 2abc 4 X 2 ab + cyc 2abc 4 X cyc X 2 a b 2 cyc 2 ab + X X 3 a ! 6abc cyc 2 a b cyc 3 X 3 a ! 3abc : cyc Bất đẳng thức được chứng minh. Đẳng thức xảy ra khi và chỉ khi a = b = c hoặc b c a 1 = 2 = 0 hoặc các hoán vị tương ứng. Bài toán 2.70 Cho các số không âm a; b; c; không có 2 số nào đồng thờ bằng 0: Chứng minh rằng b2 (c + a) c2 (a + b) a2 (b + c) + + (b2 + c2 )(2a + b + c) (c2 + a2 )(2b + c + a) (a2 + b2 )(2c + a + b) 2 : 3 (Darij Grinberg) Lời giải. Sử dụng bất đẳng thức Cauchy Schwarz, ta có " X cyc a2 (b + c) 2 (b + c2 )(2a + b + c) #" X a2 (b2 + c2 )(2a + b + c) cyc b+c http://boxtailieu.net # X cyc 2 a !2 283 Ta cần chứng minh X 3 2 a cyc ,3 X a4 + 6 cyc ,3 X 2 X a2 b2 X a2 (b2 + c2 )(2a + b + c) b+c cyc 4 cyc ,3 ,3 !2 X X cyc X cyc X a2 b2 4 4 cyc X a2 b2 b+c X a3 [(b + c)2 cyc X 2bc] b+c cyc 4 b+c X a3 (b2 + c2 ) cyc a2 b2 X a3 (b2 + c2 ) cyc cyc a4 + 2 a4 + 2 a2 b2 + 4 cyc a4 + 2 cyc X a3 (b + c) + 8abc cyc X a2 b+c cyc 0 Sử dụng bất đẳng thức AM-GM, ta có X a2 b+c cyc Do đó, ta chỉ cần chứng minh được X X 3 a4 + 2 a2 b2 cyc 4 cyc 1X a 2 cyc X a3 (b + c) + 4abc cyc X a 0 cyc Do tính thuần nhất, ta có thể chuẩn hóa cho a+b+c = 1. Đặt q = ab+bc+ca; r = abc, khi đó bất đẳng thức trở thành 3(4r + 2q 2 4q + 1) + 2(q 2 2r) , 16r + (3 4q)(1 Sử dụng bất đẳng thức Schur bậc 4, ta có r 16r + (3 4q)(1 4q) 16(4q 1)(1 6 q) 2q 2 4(q 4q) 0 0 (4q 1)(1 q) , 6 + (3 r) + 4r 4q)(1 suy ra 4q) = 1 (1 3 4q)2 0: Đẳng thức xảy ra khi và chỉ khi a = b; c = 0 hoặc các hoán vị tương ứng. Nhận xét 21 Chúng ta có cũng có 1 cách khác (đòi hỏi chúng ta phải có sự khéo léo) để chứng minh bất đẳng thức X X X X 3 a4 + 2 a2 b2 4 a3 (b + c) + 4abc a 0 cyc cyc cyc http://boxtailieu.net cyc 284 CHƯƠNG 2. SÁNG TẠO BẤT ĐẲNG THỨC bằng cách viết lại nó như sau (sau khi đã giả sử c = minfa; b; cg) c4 + 2c2 (a + b c)2 + (a b)2 (3a2 + 3b2 + 2ab 4ac 4bc) 0 Bất đẳng thức này hiển nhiên đúng. Bài toán 2.71 Cho các số không âm a; b; c. Chứng minh rằng b c a + + b+c c+a a+b 3 7 maxf(a + 2 16 b)2 ; (b c)2 ; (c ab + bc + ca a)2 g : (Phạm Văn Thuận, Võ Quốc Bá Cẩn) Lời giải. Không mất tính tổng quát, ta có thể giả sử a b thành X a 3 7 (a c)2 + b+c 2 16 ab + bc + ca cyc , X a[a(b + c) + bc] b+c cyc , a2 + b2 + c2 + abc X cyc cyc 3 7 (ab + bc + ca) + (a 2 16 1 b+c Sử dụng bất đẳng thức AM-GM, ta có X 1 b+c c. Bất đẳng thức trở c)2 3 7 (ab + bc + ca) + (a 2 16 c)2 9 2(a + b + c) Nên ta chỉ cần chứng minh được a2 + b2 + c2 + Đặt a = c + x; b = c + y (x (11x2 9abc 2(a + b + c) y 3 7 (ab + bc + ca) + (a 2 16 c)2 0), khi đó bất đẳng thức này tương đương với 32xy + 32y 2 )c + (x + y)(3x Hiển nhiên đúng. Đẳng thức xảy ra khi và chỉ khi a = b = c hoặc ứng. a 4 = 4y)2 b 3 = c 0 0: hoặc các hoán vị tương Bài toán 2.72 Cho các số dương a; b; c thỏa mãn abc = 1. Tìm giá trị lớn nhất của biểu thức 1 1 1 P = 2 + 2 + 2 : (a a + 1)2 (b b + 1)2 (c c + 1)2 (Võ Quốc Bá Cẩn) http://boxtailieu.net 285 Lời giải. Không mất tính tổng quát, giả sử a trường hợp sau Trường hợp 1. Nếu b 1, ta sẽ chứng minh , 1 1 a + 1)2 (a2 , ab(a (a , ab(a (a2 b2 b + 1)2 b + 2)(b2 b + 2)(a2 b2 ab + 1)2 2 (b2 1)(a + b)[2a2 b2 2 2 1 c. Xét nhứng c4 c + 1)2 1 =1+ 2 ab + 1)2 (c (a2 1)(b a + 2)(b 1+ c ) a 1 b + 1)2 1 a + 1)2 (b2 1 1)(a2 1)(b 2 Do a; b 1 b + 1)2 1 + 2 a + 1)2 (b (a2 b (a2 b2 1 ab + 1)2 ab(a + b) + a2 + b2 a b + 2)(a b ab + 1) 2 2 ab(a + b) + a2 + b2 (a + b)[2a b a b + 2] 1, ta có (a2 ) ab(a2 a + 2)(b2 b + 2)(a2 b2 a + 2)(b2 b + 2)(a2 b2 ab + 1) ab + 1)2 4; 2ab a+b 2(a + b)(a2 b2 ab + 1) Ta cần chứng minh 2(a2 b2 2a2 b2 ab + 1) , (a ab(a + b) + a2 + b2 1)(b 1)(a + b) a b+2 0 hiển nhiên đúng. Từ đây, ta có P 1+ c4 + 2 c + 1)2 (c (c2 Trường hợp 2. Nếu a 1 b (b2 Do đó, nếu a b + 1)2 (c (c2 9 ; 16 (c2 c + 1)2 1 4 )b 1)2 c + 1)2 9 16 4, ta có a + 1)2 )P a ) bc 1 =3 c + 1)2 c. Chú ý rằng (a2 Nếu 4 b + 2] 2 1 2, (42 4 + 1)2 = 169 1 32 5417 + = 169 9 1521 ta có 2 trường hợp nhỏ http://boxtailieu.net 3 286 CHƯƠNG 2. SÁNG TẠO BẤT ĐẲNG THỨC i) Nếu c 1 2, ta có (c2 b2 b + 1 = b(1 c + 1)2 9 16 1) + (4b2 c2 2c)(b + 2bc 2bc + 1) 1 1 +2 1 + (4b2 c2 b(1 2c) 2 4 a2 2a + 4 = 4b2 c2 2bc + 1 = a2 )P ) max P ii) Nếu 4 a 1 b c a4 16 + = f (a) 2a + 4)2 9 1 + 2 a + 1)2 (a (a2 1 2, 2bc + 1) max f (a) 4 a 1 xét hàm số sau g(x) = 1 ex + 1)2 (e2x với x 2 [ ln 2; 0]. Ta có g 00 (x) = 2ex (ex 1)(8e2x + ex (e2x ex + 1)4 1) 0 8x 2 [ ln 2; 0] Do đó g(x) lõm trên [ ln 2; 0]. Bây giờ, ta hãy chú ý rằng ln b; ln c 2 [ ln 2; 0], nên theo bất đẳng thức Jensen, g(ln b) + g(ln c) , (b2 1 + 2 b + 1)2 (c )P (a2 1 c + 1)2 2g 2 p bc + 1 bc 1 + a + 1)2 (a ) max P ln b + ln c 2 2 = (a 2a2 p 2 a + 1) 2a2 p 2 = h(a) a + 1) max h(a) 4 a 1 Từ những trường hợp đã xét ở trên, ta đi đến kết luận (với chú ý là đẳng thức luôn xảy ra) 5417 max P = max ; max f (a); max h(a) : 4 a 1 1521 4 a 1 http://boxtailieu.net 287 Bài toán 2.73 Cho các số không âm a; b; c; d. Chứng minh rằng X a(a b)(a c)(a d) + abcd 0: cyc (Phạm Kim Hùng) Lời giải. Không mất tính tổng quát, ta có thể giả sử a tương đương với (a b)[a(a c)(a d) b c c)(a d) b(b c)(b d)] + c(c d)(a + d[abc + (d a)(d b)(d a)] 0 d. Bất đẳng thức c)(b c) Ta có a(a b(b c)(b d) = (a b)[a2 + ab + b2 (a + b)(c + d) + cd] và a2 + ab + b2 (a + b)(c + d) + cd = (a b)(a + 2b c d) + (b ) a(a c)(a d) b(b c)(3b + c 2d) + c(c c)(b 0 d) d) 0 Mặt khác c(c d)(a c)(b c) 0 và abc + (d a)(d b)(d a) = d[d2 + a(b d) + b(c d) + c(a d)] 0: Bất đăng thức được chứng minh. Đẳng thức xảy ra khi và chỉ khi a = b = c; d = 0 hoặc a = b; c = d = 0 hoặc các hoán vị tương ứng. Bài toán 2.74 Cho các số không âm a; b; c thỏa mãn a + b + c = 2. Chứng minh rằng bc ca ab + + a2 + 1 b2 + 1 c2 + 1 1: (Phạm Kim Hùng) Lời giải. Bất đẳng thức tương đương với X a abc +1 a2 + 1 cyc Với mọi x X bc 0 cyc 0, ta có 1 2 x +1 1 x(x 1)2 1+ x= 2 x2 + 1 http://boxtailieu.net 0 288 CHƯƠNG 2. SÁNG TẠO BẤT ĐẲNG THỨC 1 x2 + 1 ) 1 x 2 1 Sử dụng bất đẳng thức này, ta chỉ cần chứng minh được abc X 1 a +1 2 a 1 cyc , abc X cyc bc 0 cyc X bc + 1 X bc 0 cyc Đặt q = ab + bc + ca; r = abc. Bất đẳng thức trở thành qr + 1 q(q q 1)(4 3 q) 0 (q 1)(4 q) . 3 Theo bất đẳng thức Schur bậc 4, ta có r qr + 1 q +1 q= Do đó 1 (3 3 1)2 q)(q 0: do q 34 < 3. Bất đẳng thức được chứng minh. Đẳng thức xảy ra khi và chỉ khi a = b = 1; c = 0 hoặc các hoán vị. Bài toán 2.75 Cho a; b; c là các số thực phân biệt. Chứng minh rằng 1 + a2 b2 1 + b2 c2 1 + c2 a2 + + (a b)2 (b c)2 (c a)2 3 : 2 (Nguyễn Văn Thạch) Lời giải. Ta dễ dàng kiểm tra được các đẳng thức sau X 1 + ab 1 + bc = 1; a b b c cyc X1 a cyc ab 1 b b bc = c 1 Mặt khác, với mọi x; y; z 2 R, ta có x2 + y 2 + z 2 x2 + y 2 + z 2 xy + yz + zx; 2(xy + yz + zx) Suy ra X cyc X cyc 2 1 + ab a b 1 ab a b 2 X 1 + ab 1 + bc =1 a b b c cyc 2 X1 cyc a ab 1 b b http://boxtailieu.net bc =2 c 289 )2 X 1 + a2 b2 b)2 (a cyc = X cyc 2 1 + ab a b + X cyc 1 ab a b 2 3: Bất đẳng thức được chứng minh. Bài toán 2.76 Cho các số dương a; b; c: Chứng minh rằng ab+c + bc+a + ca+b 1: Lời giải. Nếu tồn tại ít nhất một trong 3 số a; b; c, chẳng hạn a 1, thì bất đẳng thức hiển nhiên đúng. Do đó, ta sẽ xét bài toán trong trường hợp 1 > a; b; c > 0. Trường hợp 1. Nếu a + b + c 1 ) a + b; b + c; c + a 1, sử dụng bất đẳng thức Bernoulli, ta có 1 ab+c = 1+ 1 b+c a 1+ a (1 a)(b + c) b+c a+b+c 1+ = a a a a a+b+c X a =1 a+b+c cyc ) ab+c X ) ab+c cyc Trường hợp 2. Nếu a + b + c 1, lại sử dụng bất đẳng thức Bernoulli, ta có a + b(1 a 1 ab ) ab+c Ta cần chứng minh X cyc a) ; [a + b(1 1 ac a + c(1 a a2 a)][a + c(1 a2 a)][a + c(1 [a + b(1 a)] a) a)] 1 Sử dụng bất đẳng thức Cauchy Schwarz, ta có X cyc [a + b(1 và X cyc a2 a)][a + c(1 P a)] P a cyc [a + b(1 !2 a)][a + c(1 a)] cyc !2 a X [a + b(1 cyc = X cyc a)][a + c(1 ! ab X cyc a a)] ! 1 + abc 3 http://boxtailieu.net X cyc ! a 0: 290 CHƯƠNG 2. SÁNG TẠO BẤT ĐẲNG THỨC Từ đây, ta suy ra được kết quả bài toán ban đầu. Bài toán 2.77 Cho các số dương a; b; c: Chứng minh rằng a(b + c) b(c + a) c(a + b) + 2 + 2 a2 + bc b + ca c + ab a2 + bc b2 + ca c2 + ab + + : a(b + c) b(c + a) c(a + b) a+b+c p 3 abc (Phạm Hữu Đức) Lời giải. Trước hết, ta sẽ chứng minh P X a(b + c) p 3 a2 + bc cyc a cyc abc Đặt a = x3 ; b = y 3 ; c = z 3 , và với chú ý rằng x3 + y 3 X a(b + c) a2 + bc cyc = X x3 (y 3 + z 3 ) x6 + y 3 z 3 cyc Ta cần chứng minh X x3 (y 3 + z 3 ) x2 yz(x2 + yz) cyc , X xy(x + y) 8x; y 0, ta có X x3 (y 3 + z 3 ) x2 yz(x2 + yz) cyc P x3 cyc xyz X x2 (y 3 + z 3 ) x3 cyc x2 + yz cyc Ta có X X x2 (y 3 + z 3 ) x3 cyc x2 + yz cyc = = X x3 (x X x2 (x3 + xyz cyc y)(x 2 x + yz cyc y3 x2 + yz z) + X xy(x cyc z3) y)2 (x + y)(xz + yz (x2 + yz)(y 2 + zx) xy) Do tính đối xứng, ta có thể giả sử z = minfx; y; zg, khi đó X x3 (x x2 cyc 3 y)(x + yz z) + X xy(x cyc y)2 (x + y)(xz + yz (x2 + yz)(y 2 + zx) xy) x (x y)(x z) y 3 (y z)(y x) xy(x y)2 (x + y)(xz + yz + + x2 + yz x2 + yz (x2 + yz)(y 2 + zx) z(x y)2 (x + y)(x3 + y 3 x2 z y 2 z) = 0 (x2 + yz)(y 2 + zx) http://boxtailieu.net xy) 291 Tiếp theo, ta sẽ chứng minh Nếu P P X a2 + bc a(b + c) cyc ab p 3 cyc P a a cyc p 3 abc abc, khi đó theo bất đẳng thức Cauchy Schwarz và bất đẳng thức cyc AM-GM, ta có X a2 + bc X a2 X b2 c2 = + a(b + c) a(b + c) cyc abc(b + c) cyc cyc !2 !2 P P a ab cyc 2 P cyc + ab 2abc cyc Nếu p 3 v ! u u P u a u t cyc ab cyc P abc a a cyc P ! ab cyc abc P P P a cyc p 3 abc , ta có cyc X cyc !" ab X a2 + bc a(b + c) cyc # P cyc ! a p 3 ab cyc abc X cyc = ! P !" ab X a2 + bc a(b + c) cyc X (ab + bc)(bc + ca) cyc ab + ca # X cyc X !2 a (ab + ca) 0: cyc Bất đẳng thức được chứng minh. Bài toán 2.78 Cho a; b; c là các số không âm, không có 2 số nào đồng thời bằng 0 thỏa mãn a + b + c = 1. Chứng minh rằng p p 4 27 3 1 a b c p p +p +p : c + 2a a + 2b b + 2c 2 (Võ Quốc Bá Cẩn, Phạm Kim Hùng) http://boxtailieu.net 292 CHƯƠNG 2. SÁNG TẠO BẤT ĐẲNG THỨC Lời giải. Nếu c b X cyc a, khi đó theo bất đẳng thức Cauchy Schwarz, ta có a p a + 2b !2 X ! X a cyc cyc a a + 2b ! = X cyc a a + 2b Nên ta chỉ cần chứng minh được X p 4 a a + 2b cyc , trong đó x = ab ; y = cb ; z = a c X cyc 27 p p 3 2 1 !2 p 6 3 1 1 + 2x ) x; y 1 p =6 3 9 9 z; xyz = 1. Ta có 1 1 1 1 2(x 1)(y 1)(4xy 1) + = + 2x + 1 2y + 1 3 2xy + 1 3(2x + 1)(2y + 1)(2xy + 1) 1 1 1 z + = + 3 2xy + 1 3 z+2 ) Nếu a X cyc b 1 2x + 1 1 z 1 4 + + = 3 z + 2 2z + 1 3 c, đặt f (a; b; c) = P cyc Trường hợp 1. Nếu a 4b, thì (a + c)2 a + c + 2b a a + 2b ) f (a; b; c) a + 2c a + 2b + p 0 a+c a + c + 2b p b c +p b b + 2c c + 8b p a+c p + b = f (a + c; b; 0) a + c + 2b b c +p c + 2a b + 2c Trường hợp 2. Nếu 4b xét 2 trường hợp a2 c(a2 + 4ab + ac + 2bc) = a + 2b (a + 2b)(a + 2b + c) )p p p a ; a+2b p 4 0 t2k 2tk+1 + 2tk 1 1 = 0 2 t=0 4 t>0 , 2k k 1 1 g(t) = t +2t 2=0 tk+1 (k 1)t2k 4tk 1 +k+1 tk+2 h0k 2 1; ta có (t2k + 1)k+1 1 = k 1 f (t) 2k+2 k+1 k (t +t + 1) 2 f 0 (t) = g 0 (t) = 1 k+1 (ktk+1 2) = 1) h(t) tk+2 0 Do h0 (t) 0, ta suy ra h(t) nghịch biến, do đó g 0 (t) có tối đa một nghiệm thuộc (0; 1], và từ đây, ta suy ra g(t) có tối đa 2 nghiệm thuộc (0; 1], trong đó ta đã biết trước một nghiệm luôn thỏa là 1. Từ đây, ta có thể dễ dàng kiểm tra được f (t) ) min ff (0); f (1)g = min 1; X cyc ak (a + b + c)k min 2k+1 3k 1 4 ; 2k 1 3k http://boxtailieu.net 304 CHƯƠNG 2. SÁNG TẠO BẤT ĐẲNG THỨC Và do đó X cyc Từ đây, trong trường hợp k P cyc a2 (a + b + c)2 4 9 2, sử dụng bất đẳng thức Holder, ta được 2P ak (a+b+c)k 6 cyc 4 4 ) X cyc a2 (a+b+c)2 4 ak (a + b + c)k 3 k2 7 5 1 3k 4 : 3k Bất đẳng thức được chứng minh xong. Bài toán 2.87 Cho a; b; c là độ dài 3 cạnh của một tam giác nhọn. Chứng minh rằng r r r b+c a c+a b a+b c + + 3: a b c (Virgil Nicula) Lời giải. Đặt p a(b + c p B = b + b(c + a p C = c + c(a + b A = a+ và y = 1 z = 1 Chú ý rằng B = (a = (a b) + " b) c) a p b) + c(a + b c+a b p p c(a + b c) + a(b + c a+b c p p a(b + c a) + b(c + a p b(a + c x = 1 A a) hp b) 1 a(b + c p a(b + c b+c a) p b(c + a a+b c p a) + b(c + a http://boxtailieu.net c) a) b) i b) b) # = z(a b) 305 Bất đẳng thức tương đương với r X cyc , X a Xb+c , , b+c a 2a (a b) cyc (a 1 B b)(A 0 0 A cyc X 1 ! 1 A 0 B)C 0 cyc , X b)2 zC(a 0 cyc Do tính đối xứng, ta có thể giả sử a b q c)b b2 +c2 2 pa 2 tam giác đã cho là tam giác nhọn), khi đó ta có p p p c(a + b c) b(a + c b) a(b + c p p p p Vì b(a + c b) bc; c(a + b c) bc, nên ta có b+c p x = 1 b(a + c p p b c p 2 bc a = a p b) + c(a + b c) 1 (vì theo giả thiết, a) b+c a p 2 bc 2 0 Ta sẽ chứng minh rằng y Thật vậy, ta có ) 0; by + cz 0 p p c(a + b c) a(b + c a) p p c) + a(b + c a) 3a(b + c a) + c(a + b p c(a + b Do đó, để chứng minh y 0, ta chỉ cần chỉ ra được 3a(b + c a) + c(a + b c) (a + c b)2 2c2 + (3b + 2a)c (a b)(4a b) 0 p Đây là hàm tăng theo c và do c a2 b2 , nên ta chỉ cần chứng minh p 2(a2 b2 ) + (2a + 3b) a2 b2 (a b)(4a b) 0 , http://boxtailieu.net c) 306 CHƯƠNG 2. SÁNG TẠO BẤT ĐẲNG THỨC Do b pa , 2 p , (2a + 3b) a2 b2 (a p p , (2a + 3b) a + b a ta có p a+b p 2 a b)(6a + b) b(6a + b) b và 2(2a + 3b) 6a b = 5b 2a 0 Tiếp theo, ta sẽ chứng minh by + cz ,b+c Vì b) p c) + a(b + c c(a + b p và b(c + a p p a(b + c a) + p a(b + c a) + Ta chỉ cần chứng minh b(c + a b) p b(c + a b) b+c , f (a) = b(c + a p b(c + a 0 +p a) a(b + c p c(a + b b) + c(a + b b) + 3a(b + c (b c) p a) + b(c + a p a(b + c a) b) + 3a(b + c a) c) + p b(c + a 4(b + c)2 a2 + (b + c)(6b2 + 3bc + 5c2 )a c(a + b b) c) a) c)(2b3 b2 c + 4bc2 c3 ) 0 Do f (a) là hàm lõm, ta dễ thấy f (a) Ta có n o p min f (b); f b2 + c2 f (b) = c(4b3 b2 c + 10bc2 c3 ) 0 p p b2 + c2 = (b + c)(6b2 + 3bc + 5c2 ) b2 + c2 6b4 5b3 c f và Do 13b2 c2 3bc3 p b2 + c2 0 f p , (b + c)(6b2 + 3bc + 5c2 ) b2 + c2 6b4 + 5b3 c + 13b2 c2 + 3bc3 + 5c4 1 b2 + c2 + bc (6b2 + 3bc + 5c2 ) (6b4 + 5b3 c + 13b2 c2 + 3bc3 + 5c4 ) 2 1 = bc(2b2 bc + 5c2 ) 0 2 http://boxtailieu.net 5c4 307 Ta chỉ cần chứng minh p (b + c) b2 + c2 hiển nhiên đúng vì 1 b2 + c2 + bc 2 2 1 b2 + c2 + bc 2 (b + c)2 (b2 + c2 ) = 1 bc(4b2 + 4c2 4 bc) 0 Từ đây, với chú ý rằng b3 (a + c ) bB và c3 (a + b c) = (b c)[b2 (a b) + c2 (a c) + abc] 0 p p ) b b(a + c b) c c(a + b c) h p i p cC = b2 c2 + b b(a + c b) c c(a + b c) 0 b) (a c)2 b2 (a c2 b)2 Ta có X zC(a b)2 c)2 + zC(a yB(a b)2 b)2 (a cyc (a b)2 ybcC + Cz c2 = yBb2 + Cz c2 b)2 (yb + cz) c C(a 0: Bất đẳng thức được chứng minh. Đẳng thức xảy ra khi và chỉ khi a = b = c: Bài toán 2.88 Cho các số dương a; b; c. Chứng minh rằng a2 + b2 + c2 5 ab3 bc3 ca3 + + 3a2 + 2b2 3b2 + 2c2 3c2 + 2a2 : (Võ Quốc Bá Cẩn, Nguyễn Huỳnh Bảo Trung) Lời giải. Bất đẳng thức tương đương với X 11b2 a2 cyc , X z(a 50ab3 3a2 + 2b2 b)2 0 cyc http://boxtailieu.net 0 308 CHƯƠNG 2. SÁNG TẠO BẤT ĐẲNG THỨC 2 2 3b và y; z tương tự. trong đó x = 22c3b26bc +2c2 Trường hợp 1. Nếu a b c, khi đó ta có y= 22a2 6ac 3c2 2a2 + 3c2 0 Vì 22a2 6ac 3c2 2 2a2 a + 3c2 22b2 2 2b2 b + 3c2 3c2 6bc 0 2 3b2 b + 2c2 a2 (22a2 6ac 3c2 ) 2b2 (22c2 6bc 3b2 ) + 2a2 + 3c2 3b2 + 2c2 2 2 2 2 b (22b 6bc 3c ) 2b (22c2 6bc 3b2 ) + 3b2 + 2c2 3b2 + 2c2 2 2 2 b (16b 18bc + 41c ) = 0 3b2 + 2c2 ) a2 y + 2b2 x = Tương tự, ta có 22a2 6ac 2a2 1 + 3c2 3c2 2a2 22a2 1 + 3b2 3b2 6ab 3a2 0 1 + 2b2 22a2 6ac 3c2 2(22b2 6ab 3a2 ) + 2 2 2a + 3c 3a2 + 2b2 2 2 2(22b2 6ab 3a2 ) 22a 6ab 3b + 3a2 + 2b2 3a2 + 2b2 2 2 16a 18ab + 41b 0 = 3a2 + 2b2 n 2 o c)2 max ab2 (b c)2 ; (a b)2 , ta có ) y + 2z = Khi đó, chú ý rằng (a 2 X z(a b)2 = [y(a c)2 + 2x(b c)2 ] + [y(a c)2 + 2z(a b)2 ] cyc a2 (b c)2 + 2x(b c)2 + [y(a b)2 + 2z(a b2 (b c)2 2 (a y + 2b2 x) + (a b)2 (y + 2z) 0 = b2 y Trường hợp 2. Nếu c b x= a, ta có 22c2 6bc 3b2 3b2 + 2c2 13c2 3b2 + 2c2 http://boxtailieu.net 13 5 b)2 ] 309 z= 13b2 3a2 + 2b2 22b2 6ab 3b2 3a2 + 2b2 13 5 Suy ra x(b c)2 + z(a 13 [(b 5 b)2 c)2 + (a b)2 ] 13 (a 10 c)2 Nên ta chỉ cần chứng minh được 13 +y 10 0 , 3c2 + 82a2 20ac 0: hiển nhiên đúng. Bất đẳng thức được chứng minh. Đẳng thức xảy ra khi và chỉ khi a = b = c: Bài toán 2.89 Cho các số không âm a; b; c, không có 2 số nào đồng thời bằng 0: Chứng minh rằng 4 a2 + bc b2 + ca c2 + ab + + 2 2 (b + c) (c + a) (a + b)2 3+ 3(a2 + b2 + c2 ) : ab + bc + ca (Ji Chen) Lời giải. Sử dụng bất đẳng thức Cauchy Schwarz, ta có 4 X cyc 2 )0 2 a b+c P cyc P !2 a ab cyc P cyc P cyc !2 a ab 4 X cyc a b+c http://boxtailieu.net 310 CHƯƠNG 2. SÁNG TẠO BẤT ĐẲNG THỨC Suy ra 2 X a2 + bc 4 + (b + c)2 cyc X a2 + bc 4 (b + c)2 cyc = 4 X (a cyc = 4 X (a cyc X = (a !2 P a cyc P 4 ab cyc cyc P 2 b)(a c) + (b + c)2 P cyc b)(a 4 (b + c)2 c) cyc !2 ab cyc ab c) cyc P + b)(a (b + c)2 a b+c a cyc P X P a2 cyc ab P 3 a2 cyc +3+ P ab cyc P 3 a2 1 cyc +3+ P ab ab + bc + ca cyc Ta cần chứng minh X (a , X b)(a c) cyc ,4 ,4 X a(a cyc b)(a c) X a(a b)(a b+c c) cyc cyc (a b)(a b+c X a(a cyc b)(a b+c 1 ab + bc + ca 4[bc + a(b + c)] (b + c)2 (a X (a cyc 2 b) (b 1 2 2 c) (c a) 0 0 b)(a c)(b (b + c)2 c) P c)2 2 a +3 cyc c) = b c, khi đó 4(a b) a(a c) b+c 4(a b)2 (a2 + b2 + ab (a + c)(b + c) P cyc (a + b)2 (b + c)2 (c + a)2 Do tính đối xứng, ta có thể giả sử a 4 4 (b + c)2 ! ab b(b c) b+c http://boxtailieu.net c2 ) 8ab(a b)2 (a + c)(b + c) 311 và 8 > :cyc (a + b)2 (b + c)2 (c + a)2 ab(a + c)(b + c)(a + b)2 ! P P a2 + 3 ab b)2 (b c)2 (c a)2 cyc cyc ab(a b)2 (a + 5b) (a + b)2 (b + c)2 (c + a)2 (a + b)(a + c)(b + c) (a ) Ta cần phải chứng minh 8ab(a b)2 (a + c)(b + c) ab(a b)2 (a + 5b) (a + b)(a + c)(b + c) , 8(a + b) a + 5b , 7a + 3b 0: hiển nhiên đúng. Bất đẳng thức được chứng minh. Đẳng thức xảy ra khi và chỉ khi a = b = c hoặc a = b; c = 0 hoặc các hoán vị tương ứng. Nhận xét 23 Ta có một kết quả tương tự là a2 + bc b2 + ca c2 + ab + 2 + 2 b2 + c2 c + a2 a + b2 2 + a2 + b2 + c2 ab + bc + ca 7 (Võ Quốc Bá Cẩn) Thật vậy, bất đẳng thức tương đương với X 2a2 b 2 cyc , X (a2 b2 ) cyc b2 2 c (b b2 + c2 P a2 c) cyc P + 2 P cyc ab ab 0 cyc 1 + c2 X (a b)2 X 1 (a b)2 + 2 2 2 +c a +b 2(ab + bc + ca) cyc cyc X , z(a b)2 0 a2 cyc trong đó x = 2 (b+c) (a2 +b2 )(a2 +c2 ) + 1 2(ab+bc+ca) Do tính đối xứng, ta có thể giả sử a (a2 (a + b)2 + c2 )(b2 + c2 ) (a2 b 1 b2 +c2 và y; z tương tự. c, khi đó ta có (a + c)2 + b2 )(b2 + c2 ) http://boxtailieu.net (a2 (b + c)2 + b2 )(a2 + c2 ) 0 312 CHƯƠNG 2. SÁNG TẠO BẤT ĐẲNG THỨC và 1 a2 + b2 1 a2 + c2 )z y 1 b2 + c2 x Ta sẽ chứng minh a2 y + b2 x , 0 a2 (a + c)2 b2 (b + c)2 a2 + b2 1 + + a2 + b2 b2 + c2 a2 + c2 2(ab + bc + ca) , a2 b2 a2 + a2 + c2 b2 + c2 1 a2 (a2 + c2 ) b2 (b2 + c2 ) a3 2c b3 + + 2 + 2 2 2 2 2 2 2 2 2 +b b +c a +c a +b b +c b + c2 a2 + b2 c2 c2 + + 2 + 2 2(ab + bc + ca) a + c2 b2 + c2 Ta có a2 (a2 + c2 ) b2 (b2 + c2 ) + b2 + c2 a2 + c2 ) và a2 a2 b2 = (a2 b2 )2 (a2 + b2 + c2 ) (a2 + c2 )(b2 + c2 ) a2 (a2 + c2 ) b2 (b2 + c2 ) 1 + 2 +b b2 + c2 a2 + c2 2a3 2b3 + b2 + c2 b2 + c2 2(a2 + b2 )2 (a + b)(ab + c2 ) 0 1 (a + b)(a2 + b2 ) ab + c2 Suy ra a3 b3 a2 + b2 + + b2 + c2 b2 + c2 2(ab + bc + ca) a2 + b2 c(a + b) + ab + c2 2(ab + bc + ca) c(a + b) (a b)2 2c(a + b) = 1+ + ab + c2 2(ab + bc + ca) (a b)2 c2 (a + b)(a + b c) = 1+ + 2(ab + bc + ca) (ab + c2 )(ab + bc + ca) 2c 2 a + b2 ) a2 y + b2 x Khi đó từ z y x, ta được z y (a 0 0. Tiếp theo, với chú ý rằng c)2 a2 (b b2 c)2 http://boxtailieu.net 1 313 Ta có X b)2 z(a x(b c)2 + y cyc a2 (b b2 c)2 = (b c)2 b2 (a2 y + b2 x) 0: Bất đẳng thức được chứng minh. Bài toán 2.90 Cho các số dương a; b; c; d thỏa mãn a+b+c+d = abc+bcd+cda+dab. Chứng minh rằng a+b+c+d+ 2a 2b 2c 2d + + + a+1 b+1 c+1 d+1 8: (Võ Quốc Bá Cẩn) Lời giải. Đặt x = a + b + c + d = abc + bcd + cda + dab, khi đó từ bất đẳng thức AM-GM, ta có x 4. Chú ý rằng v ! ! u X X u X t 2 ab 3 a abc = 3x sym cyc cyc Sử dụng bất đẳng thức Cauchy Schwarz, ta có X 2a a+1 cyc 2x2 2x2 P 2 = P 2 a ab x+ x+x 2 cyc x2 2x2 2x = + x 3x x 2 sym Ta cần chứng minh 2x x+ x , (x 2 4)2 8 0: hiển nhiên đúng. Đẳng thức xảy ra khi và chỉ khi a = b = c = d = 1: Bài toán 2.91 Cho các số dương a; b; c thỏa mãn a + b + c = 3. Chứng minh rằng 8 1 1 1 + + a b c +9 10(a2 + b2 + c2 ): (Vasile Cirtoaje) http://boxtailieu.net 314 CHƯƠNG 2. SÁNG TẠO BẤT ĐẲNG THỨC Lời giải. Không mất tính tổng quát, giả sử a )16 16 a 1; 23 c)a 16 b 20b2 69 + 84b = 16 b 5 (2b 1)2 16 c 20c2 69 + 84c = 16 c 5 (2c 1)2 1 1 + b c b; 1 c. Ta có 20(b2 + c2 ) = 138 và b 16 b 84(b + c) + 20a2 + 18 + 138 1 a 1 a = = 16 c 1)2 + 5 (2b 5 (2c 1)2 84(b + c) 1 a 5 (2a 2)2 = 5 [(2b 1)2 + (2c 5 [(2b 1) + (2c 1)2 + 2(2b 1)(2c 1)]2 1)] Khi đó, ta có thể viết lại bất đẳng thức như sau 16 1 + b a Do a b 16 1 + c a 1)2 + 10 (2b c > 0, ta có a = 3 b c 16 1 + c a 1)2 10 (2c 3 10 1 a 2 5 (2b b và 16 1 + b a 10 Mặt khác, ta có 16 1 + b a Suy ra, nếu (2b Nếu (2b 1)(2c 16 1 + b 3 b 10 10 = 5(3 2b)(3 b) + 3 >0 b(3 b) 1)(2c 1) 0, thì bất đẳng thức là hiển nhiên. 1) 0 và b 1, ta có 16 1 + b a ) 10 16 1 + c a 1 a 5 10 = 16 2 + b a 16 1 + b a 15 > 10 http://boxtailieu.net 5 16 b 15 > 0 1 >0 a 1)(2c 1) 315 Do đó 16 1 + b a 1 a 5 Nếu (2b 1)(2c 16 1 + b a 1) 1)2 + (2c 1)2 ] 0 và b 1, ta có c 1 2 1 a 5 = = Do b ) 16 1 + b a 1, ta cóa = 3 b c 2 16 1 + c a 7 6 5 1 a ) 16 1 + c a 10 5 2 100 7 4b)(56 7b(5 2b) 10 6 7 1 a 2 5 )a 16 13 + b 7a 2(5 1)2 10 (2c [(2b 6 7 10 16 1 + c a 1)2 + 10 (2b 5 (2b b; b 5 4. 1)(2c 1) Ta có 100 7 16 13 + b 7 25 b 25b) 0 1 a c, suy ra 16 13 95 16 13 + + c 6a 7 c 6(2 c) (1 c)(178 95c) + 14 = >0 6c(2 c) = 10 7 6 5 96 6 1 a Do đó 16 1 + b a 10 (2b 5 1 a 16 1 + c a 7 1)2 + (2c 6 1)2 + 6 (2b 7 1)2 10 (2c 1)2 2 5 1 a (2b 1)(2c 1): Bất đẳng thức được chứng minh. Đẳng thức xảy ra khi và chỉ khi a = 2; b = c = hoặc các hoán vị tương ứng. 1 2 p Bài toán 2.92 Cho a; b; c 2 0; 3 1 thỏa mãn a2 + b2 + c2 + 2abc = 1: Chứng minh rằng 3(a + b + c) 4(1 + abc): (Jack Garfunkel) http://boxtailieu.net 316 CHƯƠNG 2. SÁNG TẠO BẤT ĐẲNG THỨC p a Lời giải. Do tính đối xứng, giả sử a b c) 3 1 3(a + b + c) 4(1 + abc), ta sẽ chứng minh ! r r 1 a 1 a f (a; b; c) f a; ; 2 2 h ,3 b+c , i a) + 2a(1 p 2(1 3[(b + c)2 + 2a 2] p + 2a(1 b + c + 2(1 a) a a 1 2. 2bc) Đặt f (a; b; c) = 0 2bc) 0 c)2 + 2(2bc + a 1)] p + 2a(1 a 2bc) 0 b + c + 2(1 a) " # 3(b c)2 3 p p , + 2(1 a 2bc) a b + c + 2(1 a) b + c + 2(1 a) , 3[(b Chú ý rằng (1 + a)(1 , a c)2 , bất đẳng thức tương đương " # 2(b c)2 3 p + a 0 1+a a) b + c + 2(1 a) 2bc) = (b 3(b c)2 p b + c + 2(1 , 3(a + 1) p b + c + 2(1 + 2a a) 1 nên a2 + (b + c)2 Suy ra 3(1 a) p b + c + 2(1 2a a) 1 a) p a + 2(1 1 a p f a; a) 3(1 Tiếp theo, ta sẽ chứng minh rằng r 0 a) a2 + b2 + c2 + 2abc = 1 p p 1 a2 1 a )b+c 2a 6 p b + c + 2(1 3(1 a) p b + c + 2(1 , 2a Do a 0 h p , 3 a + 2(1 2 ; i a) r 1 a 2 2a(1 a) ! p = 2a 0 a) http://boxtailieu.net 4 0 3 p 1 2+1 a>0 317 Đặt t2 = 2(1 a) (t 0) ) 1 t 2 t2 3 2 p 3 1; a = bất đẳng thức trở thành t2 2 t2 +t 2 t2 2 , 4 2 0 1 (t 1)2 (t2 + 2t 2) 0: 2 hiển nhiên đúng. Đẳng thức xảy ra khi và chỉ khi a = b = c = p p 3 1 3 1 hoặc các hoán vị tương ứng. 2 ;a = , 1 2 hoặc b = c = Nhận xét 24 Đặt a = sin A2 ; b = sin B2 ; c = sin C2 thì A; B; C là 3 góc của một tam p giác và A; B; C 2 arcsin 3 1 , ta được bất đẳng thức Jack Garfunkel với một giả thiết "lỏng" hơn là sin A B C + sin + sin 2 2 2 4 3 1 + sin A B C sin sin 2 2 2 Bài toán 2.93 Cho các số dương a; b; c thỏa mãn abc 1. Chứng minh rằng với mọi k 1, ta có a+k b+k c+k a+b+c + + : b+k c+k a+k (Phạm Kim Hùng) Lời giải. Bất đẳng thức tương đương với , X k b+k (a + k) 1 cyc , X b(a + k) b+k cyc + (k 1) X a 3k cyc + (k 1) X a 3k cyc Sử dụng bất đẳng thức AM-GM, ta có X b(a + k) cyc b+k X p 3 3 abc p 3 3 abc a 3 3 cyc Do đó X b(a + k) cyc b+k + (k 1) X a 3 + 3(k 1) = 3k: cyc Bất đẳng thức được chứng minh. Đẳng thức xảy ra khi và chỉ khi a = b = c = 1: http://boxtailieu.net 318 CHƯƠNG 2. SÁNG TẠO BẤT ĐẲNG THỨC Bài toán 2.94 Cho các số không âm a; b; c; không có 2 số nào đồng thời bằng 0: Chứng minh rằng p p p p 2 2 2 2 (a + b + c)2 : a a + bc + b b + ca + c c + ab 3 (Võ Quốc Bá Cẩn) Lời giải. Trước hết, ta sẽ chứng minh rằng X p X p a 2(a2 + bc) a (a + b)(a + c) cyc , cyc X hp a 2(a2 + bc) cyc , X cyc i p (a + b)(a + c) a(a b)(a c) p p 2 2(a + bc) + (a + b)(a + c) 0 0 Do tính đối xứng, ta có thể giả sử a b c, khi đó ta có p p p a (b + c)(b + a) b (a + b)(a + c); a 2(b2 + ca) Từ đó a p p 2 2(a + bc) + (a + b)(a + c) Nên X cyc p a(a b)(a c) p p 2 2(a + bc) + (a + b)(a + c) " a p (a b) p 2 2(a + bc) + (a + b)(a + c) Tiếp theo, ta sẽ chứng minh rằng X p a (a + b)(a + c) cyc , X cyc 2 a (a + b)(a + c) + 2 X cyc 2(b2 b p 2(a2 + bc) b p + ca) + (b + c)(b + a) b p p 2 2(b + ca) + (b + c)(b + a) 2 3 X cyc !2 a p ab (a + b)2 (a + c)(b + c) http://boxtailieu.net 4 9 X cyc !4 a # 0 319 Sử dụng bất đẳng thức Cauchy Schwarz, ta có (a + b)2 (a + c)(b + c) = (a2 + ab + bc + ca)(b2 + ab + bc + ca) (ab + ab + bc + ca)2 Ta cần chứng minh X 2 a (a + b)(a + c) + 2 cyc X 4 9 ab(2ab + ac + bc) cyc X !4 a cyc Do tính thuần nhất, ta chuẩn hóa cho a + b + c = 1. Đặt q = ab + bc + ca; r = abc, khi đó ta có X X a2 (a + b)(a + c) = a2 (a + bc) = 1 3q + 4r cyc X cyc ab(2ab + ac + bc) = 2 cyc X !2 ab cyc 2 X a2 bc = 2q 2 2r cyc Bất đẳng thức trở thành 1 3q + 4r + 2(2q 2 , 1 (1 9 3q)(5 4 9 2r) 12q) 0: hiển nhiên đúng do q 31 : Bất đẳng thức được chứng minh xong. Đẳng thức xảy ra khi và chỉ khi a = b = c: Bài toán 2.95 Cho các số dương a; b; c thỏa mãn abc = 1. Chứng minh rằng a+3 b+3 c+3 + + 2 2 (a + 1) (b + 1) (c + 1)2 3: (Vasile Cirtoaje) Lời giải. Ta có một chú ý rằng luôn có ít nhất 2 trong 3 số a; b; c, chẳng hạn a; b sao cho (a 1)(b 1) 0 ) 1 + ab a + b. Ta có 1 1 + 2 (a + 1) (b + 1)2 ) và 1 ab(a b)2 + (ab 1)2 = 1 + ab (1 + ab)(1 + a)2 (1 + b)2 1 1 + (a + 1)2 (b + 1)2 1 1 + a+1 b+1 4 a+b+2 1 c = 1 + ab c+1 4 4c = ab + 3 1 + 3c http://boxtailieu.net 0 320 CHƯƠNG 2. SÁNG TẠO BẤT ĐẲNG THỨC Do đó b+3 a+3 + 2 (a + 1) (b + 1)2 1 1 1 1 + + + 2 2 (a + 1) (b + 1) a+1 b+1 2c 4c + c + 1 1 + 3c = 2 Ta cần chứng minh 2c 4c c+3 + + c + 1 1 + 3c (c + 1)2 , 4c 1 + 3c , 4(c + 1)2 3 c(c + 3) (c + 1)2 (c + 3)(3c + 1) 1)2 , (c 0: hiển nhiên đúng. Đẳng thức xảy ra khi và chỉ khi a = b = c = 1 hoặc a ! +1; b ! +1; c ! 0 hoặc các hoán vị tương ứng. Bài toán 2.96 Cho các số không âm a; b; c; d; k: Chứng minh rằng 1+ ka b+c 1+ kb c+d 1+ kc d+a 1+ kd a+b (1 + k)2 : (Vasile Cirtoaje) Lời giải. Không mất tính tổng quát, ta có thể giả sử (a c)(d nếu (a c)(d b) 0, ta lấy (a0 ; b0 ; c0 ; d0 ) = (b; c; d; a) khi đó P (a0 ; b0 ; c0 ; d0 ) = 1+ ka b+c 1+ kb c+d 1+ kc d+a c) 0 b) 1+ 0. Thật vậy, kd a+b = P (a; b; c; d) và (a0 c0 )(d0 b0 ) = (b d)(a Từ đây, ta có 1+ ka b+c 1+ kb c+d 1+k a b + b+c c+d 1+ k(a + b)2 ab + bc + ac + bd 1+ kc d+a 1+ kd a+b 1+k c d + d+a a+b 1+ k(c + d)2 cd + da + ac + bd http://boxtailieu.net 321 Suy ra P (a; b; c; d) 1+ " Mặt khác, ta có và k(a + b)2 ab + bc + ac + bd k(c + d)2 cd + da + ac + bd #2 k(a + b)(c + d) 1+ 1+ p (ab + bc + ac + bd)(cd + da + ac + bd) (a + b)(c + d) p (ab + bc + ac + bd)(cd + da + ac + bd) 2(a + b)(c + d) 2(a + b)(c + d) ab + bc + cd + da + 2ac + 2bd (ab + bc + cd + da + 2ac + 2bd) = (a c)(d b) 0: Do đó bất đẳng thức cần chứng minh đúng. Đẳng thức xảy ra khi và chỉ khi a = c; b = d = 0 hoặc a = c = 0; b = d. Bài toán 2.97 Cho các số không âm a; b; c; không có 2 số nào đồng thời bằng 0: Chứng minh rằng a a+b 2 + 2 b b+c + 2 c c+a 3 a2 + b2 + c2 : 4 ab + bc + ca (Nguyễn Văn Thạch) Lời giải. Bất đẳng thức tương đương với 4 X a2 [c(a + b) + ab] (a + b)2 cyc ,4 3 X X ca2 X a3 b +4 a+b (a + b)2 cyc cyc Sử dụng bất đẳng thức AM-GM, ta có 4 X cyc Ta cần chứng minh 2 a3 b (a + b)2 X ca2 a+b cyc a2 cyc X a2 cyc X a2 cyc http://boxtailieu.net 3 X cyc a2 322 CHƯƠNG 2. SÁNG TẠO BẤT ĐẲNG THỨC , X a2 2 cyc X ab + 2abc cyc X cyc 1 a+b 0 Sử dụng bất đẳng thức Cauchy Schwarz, ta có 2abc X cyc 9abc P a 1 a+b cyc Nên ta chỉ cần chứng minh được X a2 cyc , X 2 X cyc 9abc ab + P a a3 + 3abc cyc 0 cyc X ab(a + b): cyc Đây chính là bất đẳng thức Schur bậc 3. Vậy ta có đpcm. Đẳng thức xảy ra khi và chỉ khi a = b = c: Bài toán 2.98 Cho các số không âm a; b; c; không có 2 số nào đồng thời bằng 0: Chứng minh rằng a b c 3abc + + + b + c c + a a + b 2(a2 b + b2 c + c2 a) 2: (Bách Ngọc Thành Công) Lời giải. Trước hết, ta sẽ chứng minh kết quả sau 2(a2 + b2 + c2 ) abc + 2 (a + b + c)2 a b + b2 c + c2 a , a2 b abc + b2 c + c2 a , abc(a + b + c)2 , 2(ab + bc + ca) a2 b2 (a + b + c)2 [2(ab + bc + ca) X a2 b(a 1 b)2 abc cyc a2 X b2 a2 cyc c2 ](a2 b + b2 c + c2 a) ! X ab cyc Từ đây, giả sử c = min fa; b; cg và đặt a = c + x; b = c + y (x; y dàng biến đổi bất đẳng thức về dạng sau (x2 xy + y 2 )c3 + (2x3 3x2 y + 2y 3 )c2 + (x2 y 2 )(x2 http://boxtailieu.net c2 xy 0), ta có thể dễ y 2 )c + x2 y(x y)2 0 323 Nếu y x hoặc x chứng minh f (c) = (2x3 3y thì bất đẳng thức trên là hiển nhiên. Nếu 3y 3x2 y + 2y 3 )c2 + (x2 y 2 )(x2 y 2 )c + x2 y(x xy x y)2 y; ta sẽ 0 Ta có f = (x2 y 2 )2 (x2 xy y 2 )2 4x2 y(x y)2 (2x3 3x2 y + 2y 3 ) = (x y)2 [(x + y)2 (x2 xy y 2 )2 4x2 y(2x3 3x2 y + 2y 3 )] (x y)2 [(x + x)2 (x2 xy y 2 )2 4x2 y(2x3 3x2 y + 2y 3 )] = 4x2 (x y)2 [(x2 xy y 2 )2 y(2x3 3x2 y + 2y 3 )] = 4x2 (x y)3 [x2 (x 3y) + y 2 (y x)] 0 Vậy nên bất đẳng thức trên đúng. Trở lại bài toán của ta, sử dụng bất đẳng thức trên, ta chỉ cần chứng minh được P 3 a2 X a 1 cyc !2 b + c 2 P cyc a cyc , , X 1X 2 (a cyc (a b) (a + c)(b + c) X (a 0 (a + b + c)2 b)(a2 b2 )[(a + b + c)2 X b)(a2 2(a + c)(b + c)] 0 (a c2 a2 + c2 b)(a2 b2 )(a2 + b2 c2 ) 0 cyc Không mất tính tổng quát, giả sử a ) b)2 (a cyc cyc , a2 + b2 P 2 b2 b2 )(a2 + b2 b 0; c; khi đó ta có (a c)(a2 c2 ) (b c)(b2 c2 ) 0 c2 ) cyc (a c)(a2 c2 )(a2 + c2 (b c)(b2 c2 )(a2 + c2 = 2c2 (b c)(b2 c2 ) 0: b2 ) + (b b2 ) + (b c)(b2 c)(b2 c2 )(b2 + c2 c2 )(b2 + c2 a2 ) a2 ) Bất đẳng thức được chứng minh. Đẳng thức xảy ra khi và chỉ khi a = b = c hoặc a = b; c = 0 hoặc các hoán vị. http://boxtailieu.net 324 CHƯƠNG 2. SÁNG TẠO BẤT ĐẲNG THỨC Bài toán 2.99 Cho các số dương x; y; z thỏa mãn xyz = 1: Chứng minh rằng 1 1 1 2 + + + 2 2 2 (1 + x) (1 + y) (1 + z) (1 + x)(1 + y)(1 + z) 1: (Phạm Văn Thuận) Lời giải. Do x; y; z > 0; xyz = 1 nên tồn tại các số dương a; b; c sao cho x = ab ; y = a c b ; z = c : Khi đó bất đẳng thức trở thành X cyc a2 2abc + (a + b)2 (a + b)(b + c)(c + a) Sử dụng bất đẳng thức Cauchy Schwarz, ta có #" # " X X a2 (a + b)2 (a + c)2 (a + b)2 cyc cyc X 1 a2 + cyc X !2 ab cyc Nên ta chỉ cần chứng minh được P cyc P a2 + P cyc !2 ab (a + b)2 (a + c)2 + 2abc (a + b)(b + c)(c + a) 1 cyc 2abc , (a + b)(b + c)(c + a) 1 P a2 + cyc P P !2 ab cyc (a + b)2 (a + c)2 cyc 2abc , (a + b)(b + c)(c + a) P 4abc (a + P a cyc b)2 (a + c)2 cyc , X (a + b)2 (a + c)2 2(a + b)(b + c)(c + a) cyc X a cyc Đặt m = a + b; n = b + c; p = c + a thì bất đẳng thức trở thành m2 + n2 + n2 p2 + p2 m2 mnp(m + n + p): Bất đẳng thức này hiển nhiên đúng theo bất đẳng thức AM-GM. Vậy ta có đpcm. Đẳng thức xảy ra khi và chỉ khi x = y = z = 1 hoặc x ! 0; y ! +1; z ! +1 hoặc các hoán vị tương ứng. http://boxtailieu.net 325 Nhận xét 25 Chúng ta cũng có 2 lời giải khác cho bài toán này, xem ở [2]. Cả 2 lời giải đó, chúng đều sử dụng những công cụ đặc biệt và khá đặc sắc nhưng theo quan niệm bản thân, chúng tôi thấy rằng những lời giải sử dụng bất đẳng thức kinh điển để giải bất đẳng thức luôn là những lời giải đặc sắc và hấp dẫn nhất, chúng luôn dễ hiểu và không đòi hỏi chúng ta phải có một kiến thức gì cao xa cả. Chúng ta cũng có một kết quả tương tự là b2 c2 ab + bc + ca a2 + + + 2 2 2 (a + b) (b + c) (c + a) 4(a2 + b2 + c2 ) 1 (Võ Quốc Bá Cẩn) Để chứng minh kết quả này, chúng ta cần có bổ đề sau a4 b2 + b4 c2 + c4 a2 + 6a2 b2 c2 3 abc(a2 b + b2 c + c2 a + ab2 + bc2 + ca2 ) 2 Đặt x = a2 b; y = b2 c; z = c2 a thì bất đăng thức trên trở thành p 3 p x2 + y 2 + z 2 + 6 3 x2 y 2 z 2 [ 3 xyz (x + y + z) + xy + yz + zx] 2 Chuẩn hóa cho xyz = 1 thì bất đẳng thức trở thành f (x; y; z) = x2 + y 2 + z 2 + 6 Giả sử x = min fx; y; zg ) t = f (x; y; z) f (x; p p p yz; yz) yz = 3 2 x+y+z+ 1 1 1 + + x y z 0 1; ta có 1 p y 2 1 p y 2 1 p y 2 p z p z p z 2 2 y+ p 8 yz 2 2 p (8 3 p 3 3) 1 ; t; t t2 1)2 + t2 + 1] z 2 3 3 yz 3 yz 0 p p f (x; yz; yz) = f ) f (x; y; z) = (t 1)2 [(t2 2t 2t4 0 Bất đẳng thức vừa phát biểu ở trên được chứng minh. Trở lại với bất đẳng thức ban đầu, bằng biến đổi tương đương, ta thấy bất đẳng thức tương đương với P P 4 2 P ab a b 2abc ab(a + b) 7a2 b2 c2 cyc cyc cyc P + 0 4 a2 (a + b)2 (b + c)2 (c + a)2 cyc http://boxtailieu.net 326 CHƯƠNG 2. SÁNG TẠO BẤT ĐẲNG THỨC Theo kết trên, ta chỉ cần chứng minh được P P abc ab(a + b) + 26a2 b2 c2 ab cyc cyc P 4 a2 2(a + b)2 (b + c)2 (c + a)2 0 cyc Chuẩn hóa cho a + b + c = 1; đặt q = ab + bc + ca; r = abc; khi đó bất đẳng thức tương đương q r(q 3r) + 26r2 2(1 2q) (q r)2 , q(q , (93q r)2 2r(q + 23r)(1 2 46)r + 2q(q 1)r + q 2q) 3 0 Đây là một hàm lõm theo r; lại có f (0) = q 3 0 và khi b = c = 1 thì bất đẳng thức trở thành 2a + 1 a2 + 2 , , 2a(2a2 + 2a + 2 + 26a) 4(a + 1)4 2a + 1 a2 + 2 (a3 a(a2 + 14a + 1) (a + 1)4 3a + 6a + 1)(a 1)2 (a2 + 2)(a + 1)4 0: Bất đẳng thức cuối hiển nhiên đúng nên ta có đpcm. Bài toán 2.100 Cho các số không âm x; y; z; không có 2 số nào đồng thời bằng 0: Chứng minh rằng s s r p 8x2 + yz 8y 2 + zx 8z 2 + xy 9 2 + + : y2 + z2 z 2 + x2 x2 + y 2 2 (Võ Quốc Bá Cẩn) Lời giải. Sử dụng bất đẳng thức GM-HM, ta có s s p 2 2 X 8x2 + yz X 8x2 + y2y2 +zz 2 p X 4x2 (y 2 + z 2 ) + y 2 z 2 = 2 y2 + z2 y2 + z2 y2 + z2 cyc cyc cyc http://boxtailieu.net 327 Đặt a = x2 ; b = y 2 ; c = z 2 thì ta cần chứng minh p X bc + 4ab + 4ac b+c cyc 9 2 Bình phương 2 vế, ta được bất đẳng thức tương đương p X a X [4ab + c(4a + b)][4ab + c(a + 4b)] X ab + bc + ca +3 +2 (b + c)2 b+c (a + c)(b + c) cyc cyc cyc Với mọi x; y 81 4 0; ta có 4(x2 + 3xy + y 2 )2 xy(x y)2 = (x + y)2 (x + y)2 (4x + y)(x + 4y) ) p (4x + y)(x + 4y) 0 2(x2 + 3xy + y 2 ) x+y Từ đây, sử dụng bất đẳng thức Cauchy Schwarz, ta có p p X [4ab + c(4a + b)][4ab + c(a + 4b)] X 4ab + c (4a + b)(a + 4b) (a + c)(b + c) (a + c)(b + c) cyc cyc X 4ab + (a + c)(b + c) cyc = 2 2c(a2 +3ab+b2 ) a+b X 2ab(a + b) + c(a2 + 3ab + b2 ) (a + b)(b + c)(c + a) ! ! P P a ab cyc 6 = cyc cyc (a + b)(b + c)(c + a) Ta cần chứng minh X ab + bc + ca cyc (b + c)2 +3 X cyc P ! ! 12 a ab cyc cyc a + b + c (a + b)(b + c)(c + a) Trong bài viết “CYH technique”, chúng ta đã chứng minh X ab + bc + ca cyc P (b + c)2 9 4 http://boxtailieu.net 81 4 328 CHƯƠNG 2. SÁNG TẠO BẤT ĐẲNG THỨC Do đó ta chỉ cần chứng minh 3 X cyc P ! ! P a ab 12 cyc cyc a + b + c (a + b)(b + c)(c + a) , X a3 + 3abc cyc X 18 ab(a + b): cyc Bất đẳng thức cuối chính là bất đẳng thức Schur bậc 3. Vậy ta có đpcm. Đẳng thức xảy ra khi x = y = z hoặc x = y; c = 0 và các hoán vị tương ứng. Nhận xét 26 Ngoài ra, chúng ta còn có một kết quả khá kinh ngạc là s 47x2 y 2 y 2 + 2xy + 2xz + 14yz 2(8x2 + yz) y2 + z2 5(x2 + y 2 + z 2 ) + 2(xy + yz + zx) (Võ Quốc Bá Cẩn) Với đẳng thức xảy ra khi (x; y; z) (1; 1; 1); (0; 1; 1); (1; 1; 0): Chúng ta có thể dễ dàng kiểm tra được X 47x2 cyc 5(x2 y 2 y 2 + 2xy + 2xz + 14yz =9 + y 2 + z 2 ) + 2(xy + yz + zx) Nên bất đẳng thức này chặt hơn bất đẳng thức ở bài toán ban đầu rất nhiều. Hiện nay vẫn chưa có một lời giải đơn giản nào cho bất đẳng thức trên. Bài toán 2.101 Cho các số không âm a; b; c; d; không có 3 số nào đồng thời bằng 0: Chứng minh rằng a(2a + b) b(2b + c) c(2c + d) d(2d + a) + + + a+b+c b+c+d c+d+a d+a+b a + b + c + d: (Park Doo Sung) Lời giải. Bất đẳng thức tương đương với X a(2a + b) cyc , a+b+c X a(a c) a+b+c cyc X a cyc 0 http://boxtailieu.net 329 , (a , (a c) a a+b+c c + (b a+c+d c)2 (a + c) + (a c)(ad (a + b + c)(a + c + d) bc) + (b d) b b+c+d d a+b+d d)2 (b + d) + (b d)(ab (a + b + d)(b + c + d) 0 cd) 0 Chú ý là 2(ad 2(ab bc) = (a cd) = (b c)(b + d) d)(a + c) (b (a d)(a + c) c)(b + d) Từ đây, ta có thể biến đổi bất đẳng thức về (a , c)2 (2a + 2c + b + d) (a2 c2 )(b d) (a + b + c)(a + c + d) (b d)2 (2b + 2d + a + c) (a c)(b2 + (a + b + d)(b + c + d) d2 ) 0 (a c)2 (2a + 2c + b + d) (b d)2 (2b + 2d + a + c) + (a + b + c)(a + c + d) (a + b + d)(b + c + d) a+c b+d (a c)(b d) + (a + b + c)(a + c + d) (a + b + d)(b + c + d) Nếu (a c)(b d) 0 thì bất đẳng thức hiển nhiên. Nếu (a bất đẳng thức AM-GM c)(b d) 0; sử dụng (a c)2 (2a + 2c + b + d) (b d)2 (2b + 2d + a + c) + (a + b + c)(a + c + d) (a + b + d)(b + c + d) s (2a + 2c + b + d)(2b + 2d + a + c) 2(a c)(b d) (a + b + c)(a + c + d)(a + b + d)(b + c + d) Do đó, để chứng minh bất đẳng thức đã cho, ta chỉ cần chứng minh được 2 bất đẳng thức sau s (2a + 2c + b + d)(2b + 2d + a + c) a+c (a + b + c)(a + c + d)(a + b + d)(b + c + d) (a + b + c)(a + c + d) s (2a + 2c + b + d)(2b + 2d + a + c) (a + b + c)(a + c + d)(a + b + d)(b + c + d) b+d (a + b + d)(b + c + d) Chẳng hạn, ta sẽ chứng minh bất đẳng thức thứ nhất, bất đẳng thức tương đương (2a + 2c + b + d)(2b + 2d + a + c) (a + b + d)(b + c + d) (a + c)2 (a + b + c)(a + c + d) http://boxtailieu.net 330 CHƯƠNG 2. SÁNG TẠO BẤT ĐẲNG THỨC Đặt x = a + c; y = b + d; ta có (2a + 2c + b + d)(2b + 2d + a + c) (a + b + d)(b + c + d) = (a + c)2 (a + b + c)(a + c + d) Và 4(2x + y) x + 2y 4(2a + 2c + b + d)(2b + 2d + a + c) (a + c + 2b + 2d)2 4(2x + y) x + 2y (a + c)2 x = (a + c)(a + c + b + d) x+y x 7x2 + 10xy + 4y 2 = >0 x+y (x + y)(x + 2y) nên bất đẳng thức đúng. Tương tự, ta cũng có s (2a + 2c + b + d)(2b + 2d + a + c) (a + b + c)(a + c + d)(a + b + d)(b + c + d) b+d : (a + b + d)(b + c + d) Vậy ta có đpcm. Đẳng thức xảy ra khi a = c; b = d: Bài toán 2.102 Cho các số dương a; b; c: Chứng minh rằng s r r r p p p a(b + c) b(c + a) c(a + b) a+ b+ c + + 2 2 2 a + bc b + ca c + ab 1 1 1 p +p +p : a c b (Phạm Hữu Đức) Lời giải. Bất đẳng thức tương đương s X a(b + c) X ab(a + c)(b + c) +2 2 + bc 2 + bc)(b2 + ca) a (a cyc cyc Xp ! a cyc X 1 p a cyc Chú ý là (a2 + bc)(b2 + ca) ab(a + c)(b + c) = c(a + b)(a s X ab(a + c)(b + c) ) 3 2 + bc)(b2 + ca) (a cyc Do đó, ta chỉ cần chứng minh X a(b + c) cyc a2 + bc +6 Xp cyc ! a X 1 p a cyc http://boxtailieu.net ! b)2 0 ! 331 Xb+c p bc cyc , , X cyc b+c p bc cyc 2 p p b c X6 p , 4 2 bc cyc X a(b + c) 2 3 a2 + bc a(b + c) a2 + bc 1 p (b + c) a bc p + 2 bc(a2 + bc) 0 0 2 3 7 5 0: Bất đẳng thức được chứng minh. Đẳng thức xảy ra khi a = b = c: Bài toán 2.103 Cho các số dương a; b; c: Chứng minh rằng r p r r r b c a 3 3 (a + b)(b + c)(c + a) a +b +c : c+a a+b b+c 4 a+b+c (Mathnfriends contest) Lời giải. Đặt x = bc; y = ca; z = ab; biến đổi, ta được bất đẳng thức tương đương p s y z 3 3 (x + y)(y + z)(z + x) x p +p +p x+y y+z 4 xy + yz + zx z+x p X x 1 3 3 p p p , 4 xy + yz + zx (x + y)(x + z) (x + y)(y + z) cyc Sử dụng bất đẳng thức sắp xếp lại (với giả thiết y là số hạng nằm giữa), ta thấy x 1 y 1 p p +p (x + y)(x + z) (y + z)(z + x) (y + z)(y + x) (y + z)(y + x) 1 y z 1 p = +p +p (y + z)(y + x) (z + x)(z + y) (z + x)(x + y) (y + z)(y + x) " # r 1 y xy + yz + zx = p 1+ p xy + yz + zx (y + z)(y + x) (y + z)(y + x) q p xy+yz+zx Đặt u = p y 1 ) (y+z)(y+x) = 1 u2 . Theo bất đẳng thức AM-GM, VT p (y+z)(y+x) r xy + yz + zx (y + z)(y + x) y 1+ p (y + z)(y + x) ! = = p (1 + u) 1 r u2 = p (1 + u)3 (1 (1 + u) (1 + u) (1 + u) 3(1 3 s p 4 (3/2) 3 3 = : 3 4 http://boxtailieu.net u) u) 332 CHƯƠNG 2. SÁNG TẠO BẤT ĐẲNG THỨC Bất đẳng thức được chứng minh xong. Đẳng thức xảy ra khi và chỉ khi a = b = c: 3) thỏa mãn a21 + a22 + Bài toán 2.104 Cho các số không âm a1 ; a2 ; :::; an (n a2n = 1: Chứng minh rằng 1 p (a1 + a2 + 3 + an ) a1 a2 + a2 a3 + + + an a1 : Lời giải. Đặt 1 fn (a1 ; a2 ; :::; an ) = p (a1 + a2 + 3 + an ) a1 a2 a2 a3 an a1 Không mát tính tổng quát, giả sử a1 = max fa1 ; a2 ; :::; an g : p1 thì Nếu an 3 fn (a1 ; a2 ; :::; an ) 1 = p 3 an an fn 1 1 (an 2 a1 ; a2 ; :::; an 1 + an q + an ) an a1 a2n ) fn (a1 ; a2 ; :::; an ) Nếu an p1 3 p1 3 ) a1 fn (a1 ; a2 ; :::; an ) 1 = p 3 +an 1 p 3 an q a2n an ) an fn 2 1 + a2n 2 + an 1 1 q a2n an 1 p 3 ) fn (a1 ; a2 ; :::; an ) fn 2 an 1 = an 1 2 an 3; 2 2 + an an 2 2 + an an 1 + a2n q + a1 ) a2n an 1 2; Ta có 1 q a2n a2n a1 ; a2 ; :::; an 1 p1 : 3 1 2 an 1 p 3 fn q + (an a1 ; a2 ; :::; an 1 2 + an + a2n 1 2; 1 q a2n 1 q a2n + an 1 1 + a2n 0 + a2n 1 ; an q a2n 1 + a2n q a2n + an + a2n 2 1 2 + a2n an 1 2 2 an 1 1 q a2n 1 +p 3 2 + a2n an 1 1 0 a1 ; a2 ; :::; an 3; q a2n 2 + a2n 1 ; an : Từ đây, bằng phép quy nạp theo n; ta dễ dàng chứng minh được bất đẳng thức đã cho. Đẳng thức xảy ra khi và chỉ khi n = 3 và a1 = a2 = a3 = p13 : http://boxtailieu.net 333 Bài toán 2.105 Cho các số a; b; c 2 R: Chứng minh rằng với mọi p > 0; ta có b2 ca c2 ab a2 bc + + 2pa2 + p2 b2 + c2 2pb2 + p2 c2 + a2 2pc2 + p2 a2 + b2 0: (Vasile Cirtoaje) Lời giải. Bất đẳng thức tương đương X cyc 2p(a2 bc) 2pa2 + p2 b2 + c2 X ,3 1 cyc X ,3 cyc 0 2p(a2 bc) 2pa2 + p2 b2 + c2 (pb + c)2 2pa2 + p2 b2 + c2 Sử dụng bất đẳng thức Cauchy Schwarz, ta có 3 = X cyc = X cyc a2 pb2 c2 + 2 2 + pb c + pa2 2 (pb + c) : 2pa2 + p2 b2 + c2 X cyc p(a2 (pb + c)2 + pb2 ) + c2 + pa2 Nên ta có đpcm. Đẳng thức xảy ra khi và chỉ khi a(b2 +pc2 ) = b(c2 +pa2 ) = c(a2 +pb2 ): Bài toán 2.106 Cho các số không âm x1 ; x2 ; :::; xn thỏa mãn x1 + x2 + Chứng minh rằng ! ! n n X X p 1 n2 p p xi : 1 + xj n+1 i=1 i=1 + xn = 1: (China TST 2006) 8 < yi 1 (i = 1; 2; :::; n) n P Lời giải. Đặt y1 = x1 + 1 ) : Bất đẳng thức tương đương y1 = n + 1 : i=1 với n X p yi i=1 ! 1 n X 1 p yi i=1 ! p http://boxtailieu.net n2 n+1 334 CHƯƠNG 2. SÁNG TẠO BẤT ĐẲNG THỨC , n X i=1 n p P yj j=1 p 1 p yi n2 n+1 Sử dụng bất đẳng thức Cauchy Schwarz, ta có n p P i=1 yi p y1 1 p n p1 n p n q = r = p 1 n y1 1+ + (y2 p y2 1) + ny1 + 2(n + 1) 1 p1n + p y1 + p yn + (yn 1) (y1 p y1 p1 n 1 1) + 1 n + + 1 n 2n + 1 ny1 Tương tự, ta có n p P yj j=1 ) n X i=1 p yi n p P yj j=1 1 p r 1 nyi + 2(n + 1) n X yi i=1 r v u n u X tn 2n + 1 8i = 1; 2; :::; n nyi nyi + 2(n + 1) 2n + 1 nyi 2n + 1 nyi nyi + 2(n + 1) i=1 v u u = tn n(n + 1) v 0 u u u B u B un @n(n + 1) t = s n n(n + 1) n 2n + 1 X 1 n i=1 yi 2n + 1 n ! 1 n2 C C n A P yi i=1 2n + 1 n2 n n+1 =p n2 : n+1 Bất đẳng thức được chứng minh. Đẳng thức xảy ra khi và chỉ khi x1 = x2 = xn = n1 : http://boxtailieu.net = 335 Bài toán 2.107 Cho các số dương a; b; c: Chứng minh rằng a2 b2 c2 + + 2a + b + c 2b + c + a 2c + a + b 3 4 r a2 + b2 + c2 : 3 (Michael Rozenberg) Lời giải. Sử dụng bất đẳng thức AM-GM, ta có r a + b + c a2 + b2 + c2 + 3 a+b+c a2 + b2 + c2 3 Nên ta chỉ cần chứng minh được X a2 2a + b + c cyc a + b + c a2 + b2 + c2 + 3 a+b+c 3 8 Chuẩn hóa cho a + b + c = 1 thì bất đẳng thức trở thành X cyc , X cyc a2 a+1 a2 a+1 , với x = 1 2a 1+a 3 2 a 8 3 2 a 8 X x(3a 1 24 0 3 1 a+ 16 48 1)2 0 0 cyc và y; z tương tự. Ta có X x(3a cyc 1)2 = X (4x + y + z)(a b)(a c) cyc và 4x + y + z = = 4(1 2a) 1 2b 1 2c + + 1+a 1+b 1+c 1 1 4(1 2a) +3 + 4 1+a 1+b 1+c 4(1 2a) 12 12(1 a)2 + 4= 1+a 3 a (1 + a)(3 a) http://boxtailieu.net 0 336 CHƯƠNG 2. SÁNG TẠO BẤT ĐẲNG THỨC Ngoài ra, do hàm f (t) = 11+t2t nghịch biến nên nếu ta giả sử a Từ đó, ta có X (4x + y + z)(a b)(a c) b c thì x y z: cyc (4y + z + x)(b c)(b (4y + z + x)(b c)(b = (z y)(a b)(b c) a) + (4z + x + y)(a a) + (4z + x + y)(a 0: c)(b b)(b c) c) Bất đẳng thức được chứng minh xong. Đẳng thức xảy ra khi và chỉ khi a = b = c: Ví dụ 2.1 Cho các số không âm x1 ; x2 ; :::; xn (n Tìm giá trị lớn nhất của biểu thức P (x1 ; x2 ; :::; xn ) = x31 x22 + x32 x23 + 3) thỏa mãn x1 +x2 + + x3n x21 + n2(n 1) 3 3 x1 x2 +xn = 1: x3n : (Võ Quốc Bá Cẩn) Lời giải. Giả sử x1 = max fx1 ; x2 ; :::; xn g : Ta chứng minh P (x1 ; x2 ; :::; xn ) P (x1 ; x2 + x3 + + xn ; 0; :::; 0) Thật vậy, P (x1 ; x2 + x3 + + xn ; 0; :::; 0) 3 = x1 (x2 + x3 + + xn )2 3 3 2(x1 x2 x3 + x1 x3 x4 + + x31 xn 1 xn ) + x31 x22 + x31 x2n (x31 x2 x3 + x31 x3 x4 + + x31 xn 1 xn ) + (x32 x23 + x33 x24 + +x3n 1 x2n ) + x31 x22 + x31 x2n x31 x22 + x32 x23 + + x3n x21 + x31 x2 x3 Ta chứng minh x31 x2 x3 n2(n , x22 x23 x34 1) 3 3 x1 x2 x3n x3n 1 n2(n 1) Ta có x22 x23 x34 x3n (x2 x3 xn )2 x1 + x2 + x3 + n x2 + x3 + + xn n 1 + xn http://boxtailieu.net 2(n 1) = 2(n 1) 1 n2(n 1) 337 Do đó P (x1 ; x2 ; :::; xn ) P (x1 ; x2 + x3 + + xn ; 0; :::; 0) Lại có P (x1 ; x2 + x3 + = x31 (x2 + x3 + + xn ; 0; :::; 0) x1 3 = 108 = 108 : 3125 Đẳng thức xảy ra chẳng hạn khi x1 = 108 max P = 3125 : 3 5 ; x2 3 + xn )2 = x31 (1 1 x1 2 108 2 = 2 5 ; x3 = x1 )2 x1 + 1 5 x1 5 = xn = 0: Vậy nên Bài toán 2.108 Cho các số không âm a; b; c; không có 2 số nào Chứng minh rằng r r r r a b c 3(ab + bc + ca) + + +3 b+c c+a a+b a2 + b2 + c2 đồng thời bằng 0: p 7 2 : 2 (Võ Quốc Bá Cẩn) Lời giải. Giả sử a = max fa; b; cg : Ta chứng minh r r r b c b+c + c+a a+b a s b c bc b+c , + +2 c+a a+b (a + b)(a + c) a s bc bc bc ,2 + (a + b)(a + c) a(a + b) a(a + c) 2a ,p bc , , 2a p 2a b c p + bc 2 p b p p 2 c 2a + b + c p (a + b)(a + c) (b c)2 p p p (a + b)(a + c) a + b + a + c bc 1 b c 6 +(b c)2 4 p p p bc bc b+ c 2 p (a + b)(a + c) http://boxtailieu.net 1 p a+b+ p 2 a+c 3 7 25 0 338 CHƯƠNG 2. SÁNG TẠO BẤT ĐẲNG THỨC Ta có 1 p bc p b+ p 2 c 1 p bc p b+ 1 p p p (a + b)(a + c) a + b + a + c p (b + b)(c + c) c 3 p p p 4 bc b+ c = p 2 2 1 p b+b+ 2 p c+c 2 >0 Lại có ab + bc + ca a2 + b2 + c2 ) Suy ra VT với x = q a b+c + r q a + b+c b+c a a(b + c) a2 + (b + c)2 a(b + c) a2 + (b + c)2 ab + bc + ca a2 + b2 + c2 r s p b+c 3a(b + c) 3 3 p =x+ +3 a a2 + (b + c)2 x2 2 2:Mặt khác ta dễ thấy x + p p3 3 x2 2 p 7 2 2 đẳng thức đã cho đúng. Đẳng thức xảy ra khi và chỉ khi (a; b; c) 8x p 2 nên bất 3 + 2 2; 1; 0 : Bài toán 2.109 Cho các số dương a; b; c; d thỏa mãn a2 + b2 + c2 + d2 = 4: Chứng minh rằng s 1 1 1 1 1 3 (abcd + 1) (a + b + c + d) + + + : 2 a b c d (Phạm Hữu Đức) Lời giải. Trước hết, ta sẽ chứng minh rằng 9(a + b + c + d) Thật vậy, giả sử d = min fa; b; c; dg ) 1 4abcd + 32 d > 0; đặt P (a; b; c; d) = 9(a + b + c + d) và 4abcd r a2 + b2 + c2 ; p=a+b+c 3 p p ) 2 3 3x p x 3; x 1 x= http://boxtailieu.net 32 339 Ta chứng minh P (a; b; c; d) , 9(3x P (x; x; x; d) 4d(x3 p) abc) Từ bất đẳng thức Schur bậc 4 X a2 (a b)(a c) 0 cyc (p2 ) abc 6x2 )(p2 + 3x2 ) 12p Ta cần chứng minh p(3x , (3x Do 3x p 81 (p2 4d x3 p) 6x2 )(p2 + 3x2 ) 12p d(p3 + 3p2 x + 6px2 + 6x3 ) p p) 27 0 p x 3 nên 3d(p3 + 3p2 x + 6px2 + 6x3 ) p 78x2 d = 81 81 = 3 + 26(1 d)(3 26d(4 d d2 ) Ta còn phải chứng minh P (x; x; x; d) , 9(3x + d) 0 4x3 d 32 4x3 )d 32 27x p 4x3 ) 4 3x2 32 27x , (9 , (9 , f (x) = f 0 (x) = 12(x (9 32 27x p 4x3 ) 4 3x2 1)(81x4 47x3 (9 4x3 )2 (4 ) f (x) 1 119x2 + 9x + 81) 3x2 )3=2 f (1) = 1 Trở lại bài toán, sử dung bất đẳng thức AM-GM, ta có x4 = abcd 1; 1 1 1 1 + + + a b c d http://boxtailieu.net p 4 4 4 = x abcd 0 d2 ) 0 340 CHƯƠNG 2. SÁNG TẠO BẤT ĐẲNG THỨC Do đó, kết hợp với bất đẳng thức ở trên, ta chỉ cần chứng minh được r 4 3 4(x + 1) 4 2(x + 8) 9 x , g(x) = x(x4 + 8)3 x4 + 1 729 2 Ta có (x4 + 8)2 (9x8 11x4 + 8) >0 (x4 + 1)2 729 ) g(x) g(1) = : 2 Bất đẳng thức được chứng minh. Đẳng thức xảy ra khi a = b = c = d = 1: g 0 (x) = Bài toán 2.110 Cho các số không âm a; b; c; không có 2 số nào đồng thời bằng 0: Chứng minh rằng 1 1 1 + + (a + 2b)2 (b + 2c)2 (c + 2a)2 1 : ab + bc + ca (Phạm Kim Hùng) Lời giải 1. Không mất tính tổng quát giả sử a = max fa; b; cg : Sử dụng bất đẳng thức AM-GM, ta có 1 1 + (a + 2b)2 (c + 2a)2 2 (a + 2b)(c + 2a) 1 1 + (a + 2b)2 (b + 2c)2 2 (a + 2b)(b + 2c) Nên ta chỉ cần chứng minh được 1 2 + (b + 2c)2 (a + 2b)(c + 2a) , f (a) = 2(b + c)a3 + (2b2 bc 1 ab + bc + ca 7c2 )a2 + (4c3 b2 c bc2 2b3 )a + 2b2 c2 0 hoặc 1 2 + (c + 2a)2 (a + 2b)(b + 2c) , g(a) = 4a3 b + (2c2 bc 7b2 )a2 + (2b3 1 ab + bc + ca b2 c bc2 )a + 2b3 c + 2b2 c2 2bc3 0 Ta sẽ chứng minh rằng trong 2 bất đẳng thức trên, có ít nhất 1 bất đẳng thức đúng. Để làm được điều này, ta chỉ cần chứng minh được f (a) + g(a) 0 http://boxtailieu.net 341 , h(a) = 2(3b+c)a3 (5b2 +2bc+5c2 )a2 +2(2c3 b2 c bc2 )a+2b3 c+4b2 c2 2bc3 0 Ta có h0 (a) = 6(3b + c)a2 2(5b2 + 2bc + 5c2 )a + 4c3 2b2 c 2bc2 = 10ab(a b) + 4ab(a c) + 2c(a c)(3a 2c) + 2b(a2 bc) + 2b(a2 = 10ab(a b) + 2b(a2 bc) + 2(a c)(3ab + 3ac + bc 2c2 ) 0 Nên h(a) đồng biến. Do đó Nếu b c thì h(a) Nếu c h(b) = b(b + 2c)(b c)2 0 h(c) = c(2b + c)(b c)2 0: c2 ) b thì h(a) Bất đẳng thức được chứng minh xong. Đẳng thức xảy ra khi và chỉ khi a = b = c: Lời giải 2. Giả sử a = max fa; b; cg và xét 2 trường hợp Trường hợp 1. Nếu a 3b + c: Đặt a + 2b = x + y; b + 2c = y + z; c + 2a = z + x; ta được 3a + b c 3b + c a 3c + a b x= 0; y = 0; z = 0 2 2 2 và a= 5x y + 2z ; 9 b= 5y z + 2x ; 9 c= 5z x + 2y 9 Khi đó, bất đẳng thức được viết lại là 1 1 1 + + (x + y)2 (y + z)2 (z + x)2 27 x2 + y 2 + z 2 + 11(xy + yz + zx) Sử dụng bất đẳng thức Iran 1996, ta có 1 1 1 + + 2 2 (x + y) (y + z) (z + x)2 Trường hợp 2. Nếu a 3b + c; do c + 2a 1 (c + 2a)2 9 4(xy + yz + zx) 27 x2 + y 2 + z 2 + 11(xy + yz + zx) 3a 3(a + 2b) nên 1 9(a + 2b)2 Do đó, ta chỉ cần chứng minh được 1 10 + (b + 2c)2 9(a + 2b)2 1 ab + bc + ca http://boxtailieu.net 342 CHƯƠNG 2. SÁNG TẠO BẤT ĐẲNG THỨC Sử dụng bất đẳng thức AM-GM, ta có 10 1 + (b + 2c)2 9(a + 2b)2 p 2 10 2:1 > 3(a + 2b)(b + 2c) (a + 2b)(b + 2c) Ta cần chứng minh 2:1 (a + 2b)(b + 2c) , 2:1(ab + bc + ca) , a(1:1b + 0:1c) 1 ab + bc + ca (a + 2b)(b + 2c) 2b2 1:9bc 0 Ta có a(1:1b + 0:1c) 2b2 1:9bc a(1:1b + 0:1c) 2b2 2bc (3b + c)(1:1b + 0:1c) 2b2 1 [(3b c)2 + 4b2 ] 0: = 10 Bất đẳng thức được chứng minh. http://boxtailieu.net 2bc Phụ lục A Một số bất đẳng thức thông dụng A.1 Bất đẳng thức trung bình cộng-trung bình nhântrung bình điều hòa (AM-GM-HM) Với mọi số dương a1 ; a2 ; :::; an ; ta có p a1 + a2 + + an n a1 a2 n an Đẳng thức xảy ra khi và chỉ khi a1 = a2 = A.2 + 1 a2 n + + 1 an : = an : Bất đẳng thức AM-GM suy rộng Cho các số dương 1 ; 2 ; :::; không âm a1 ; a2 ; :::; an ; ta có 1 a1 + n thỏa mãn 2 a2 + + 1 + n an Đẳng thức xảy ra khi và chỉ khi a1 = a2 = A.3 1 a1 2 + + n a1 1 a2 2 = 1: Khi đó với mọi số ann : = an : Bất đẳng thức trung bình lũy thừa Cho các số dương a1 ; a2 ; :::; an : Với mọi số thực r; đặt 8 < ar1 +ar2 + +arn r1 ; r 6= 0 n Mr = : p n a1 a2 an ;r = 0 343 http://boxtailieu.net 344 PHỤ LỤC A. MỘT SỐ BẤT ĐẲNG THỨC THÔNG DỤNG Khi đó Mr là hàm tăng theo r với mọi r: Chẳng hạn M2 M1 M0 ; tức là r p + a2n a21 + a22 + a1 + a2 + + an n a1 a2 an : n n A.4 Bất đẳng thức trung bình lũy thừa suy rộng Cho các số dương p1 ; p2 ; :::; pn thỏa mãn p1 + p2 + a1 ; a2 ; :::; an : Với mọi số thực r; đặt Mr = (p1 ar1 + p2 ar2 + ap11 ap22 + pn = 1 và các số dương 1 + pn arn ) r apnn ;r = 6 0 ;r = 0 Khi đó Mr là hàm tăng theo r với mọi r: A.5 Bất đẳng thức Bernoulli Với mọi số thực x 1; ta có (1 + x)r (1 + x)r 1 + rx ; r 1 + rx ; 0 1_r r 1 0 Ngoài ra, với mọi số thực a1 ; a2 ; :::; an thỏa mãn a1 ; a2 ; :::; an 0 hoặc 1 0 thì ta có (1 + a1 )(1 + a2 ) (1 + an ) 1 + a1 + a2 + + an : A.6 a1 ; a2 ; :::; an Bất đẳng thức Cauchy Schwarz Với mọi số thực (a1 ; a2 ; :::; an ) và (b1 ; b2 ; :::; bn ); ta có (a1 b1 + a2 b2 + + an bn )2 (a21 + a22 + + a2n )(b21 + b22 + + b2n ): Đẳng thức xảy ra khi và chỉ khi ai : aj = bi : bj 8i; j 2 f1; 2; :::; ng : A.7 Bất đẳng thức Holder Cho các số dương xij (i = 1; m; j = 1; n): Khi đó với mọi ! 1 ; :::; ! n + ! n = 1; ta có 0 1! j ! n m m n Y X X Y ! @ xij A xijj : i=1 j=1 j=1 i=1 http://boxtailieu.net 0 thỏa ! 1 + A.8. BẤT ĐẲNG THỨC MINKOWSKI A.8 Bất đẳng thức Minkowski Với mọi số thực r 1 và với mọi số dương a1 ; a2 ; :::; an ; b1 ; b2 ; :::; bn ; ta có " A.9 n X r (ai + bi ) i=1 # r1 n X ari i=1 ! r1 n X + bri i=1 ! r1 Bất đẳng thức Chebyshev Cho các số thực a1 i) Nếu b1 b2 a2 bn thì n an : Khi đó n X n X ai bi i=1 ii) Nếu b1 b2 ai i=1 ! n X bi ! : bi ! : i=1 bn thì n n X n X ai bi i=1 A.10 345 ai i=1 ! n X i=1 Khai triển Abel Giả sử x1 ; x2 ; :::; xn ; y1 ; y2 ; :::; yn là các số thực tùy ý. Đặt ck = y1 + y2 + + yk 8k = 1; 2; :::; n Khi đó, ta có n X i=1 A.11 xi yi = n X1 (xi xi+1 )ci + xn cn : i=1 Bất đẳng thức Maclaurin Với mọi số không âm a1 ; a2 ; :::; an ; ta có S1 S2 Sk = sP trong đó k Sn a1 a2 n k ak : http://boxtailieu.net : 346 A.12 PHỤ LỤC A. MỘT SỐ BẤT ĐẲNG THỨC THÔNG DỤNG Bất đẳng thức Schur Cho các số không âm a; b; c: Khi đó, với mọi r > 0; ta có bất đẳng thức sau ar (a b)(a c) + br (b c)(b a) + cr (c a)(c b) 0 Đẳng thức xảy ra khi và chỉ khi a = b = c hoặc a = b; c = 0 hoặc các hoán vị tương ứng. A.13 Hàm lồi, hàm lõm i) Một hàm số f được gọi là lồi trên khoảng I nếu và chỉ với mọi x; y 2 I và với mọi số không âm ; thỏa mãn + = 1; ta có f ( x + y) f (x) + f (y): ii) Một hàm số f được gọi là lõm trên khoảng I nếu và chỉ với mọi x; y 2 I và với mọi số không âm ; thỏa mãn + = 1; ta có f ( x + y) f (x) + f (y): Trong trường hợp f (x) khả vi cấp 2 trên [a; b] thì f (x) lồi trên [a; b] nếu và chỉ nếu f 00 (x) 0 8x 2 [a; b] và f (x) lõm trên [a; b] nếu và chỉ nếu f 00 (x) 0 8x 2 [a; b]: A.14 Bất đẳng thức Jensen Cho p1 ; p2 ; :::; pn là các số dương. i) Nếu f là một hàm lồi trên khoảng I; khi đó với mọi a1 ; a2 ; :::; an 2 I; ta có p1 f (a1 ) + p2 f (a2 ) + + pn f (an ) p 1 + p2 + + pn f p1 a1 + p2 a2 + p 1 + p2 + + pn an + pn : ii) Nếu f là một hàm lõm trên khoảng I; khi đó với mọi a1 ; a2 ; :::; an 2 I; ta có p1 f (a1 ) + p2 f (a2 ) + + pn f (an ) p 1 + p2 + + pn A.15 f p1 a1 + p2 a2 + p 1 + p2 + Tổng, tích hoán vị-đối xứng Ta P kí hiệu - Tổng hoán vị. Chẳng hạn cyc X a2 b = a2 b + b2 c + c2 a cyc http://boxtailieu.net + pn an + pn : A.15. TỔNG, TÍCH HOÁN VỊ-ĐỐI XỨNG a;b;c;d X 347 a2 bc = a2 bc + b2 cd + c2 da + d2 ab cyc P - Tổng đối xứng. Chẳng hạn sym X a2 b = a2 b + b2 c + c2 a + ab2 + bc2 + ca2 cyc a;b;c;d X ab = ab + ac + ad + bc + bd + cd sym Q - Tích hoán vị. Chẳng hạn cyc Y (a b) = (a b)(b c)(c a) cyc a;b;c;d Y (a b) = (a b)(b c)(c d)(d a) cyc Q - Tích đối xứng (dành cho 4 biến). Chẳng hạn cyc a;b;c;d Y (a b)2 = (a b)2 (a c)2 (a d)2 (b cyc http://boxtailieu.net c)2 (b d)2 (c d)2 : 348 PHỤ LỤC A. MỘT SỐ BẤT ĐẲNG THỨC THÔNG DỤNG http://boxtailieu.net Tài liệu tham khảo [1] Andreescu T., Cirtoaje V., Dospinescu G., Lascu M., Old and New Inequalities, GIL Publishing House, 2004 [2] Cirtoaje V., Algebraic Inequalities, GIL Publishing House, 2006 [3] Phạm Kim Hùng, Secret in Inequality, nhà xuất bản Tri Thức, 2006 [4] Hojoo Lee, Topic in Inequalities, online electronic book, 2006 [5] Kim-Yin Li, Using tangent lines to prove inequalities, Mathematical Excalibur, volume 10, number 5, 2005 349 http://boxtailieu.net [...]... đó, ta viết bất đẳng thức lại như sau (a b)[ar (a c) br (b c)] + cr (a c)(b c) 0 Ta có a c b c 0; ar br Nên bất đẳng thức đúng Bất đẳng thức Schur được chứng minh Chúng ta có 2 trường hợp đặc biệt thường hay được ứng dụng để giải toán là r = 1 và r = 2: Khi đó, chúng ta được những bất đẳng thức tương ứng là Hệ quả 1.1 (Bất đẳng thức Schur bậc 3) Cho các số không âm a; b; c: Khi đó, bất đẳng thức sau đúng... 1 2 b+c p 2 2 1 p 3 2 1 : Vậy ta có đpcm Đẳng thức xảy ra khi a = b = c = 1: 1.3.4 Đại lượng (a b)2 (b c)2 (c a)2 Đối với những bất đẳng thức rất chặt và đẳng thức xảy ra tại những điểm không đặc biệt như bất đăng thức Schur (chẳng hạn đẳng thức xảy ra tại a = 3; b = 2; c = 2) thì việc sử dụng bất đẳng thức Schur để giải chúng là điều hiển nhiên không thực hiện được, do đó chúng ta cần tìm một đánh... : Chúng ta thường dùng bất đẳng thức Schur để giải bất đẳng thức trong trường hợp bất đẳng thức có những đẳng thức tại các điểm a = b = c hoặc a = b; c = 0 hoặc trong trường hợp a; b; c là độ dài 3 cạnh tam giác thì là a = 2; b = c = 1: Ví dụ 1.17 Cho các số không âm a; b; c thỏa mãn ab + bc + ca = 3: Chứng minh rằng a3 + b3 + c3 + 7abc 10: (Vasile Cirtoaje) Lời giải Bất đẳng thức tương đương với 10r... (5q p2 )r 5p3 q + 5pq 2 + 5(p2 3pr): q)r Còn rất nhiều những đẳng thức khác nữa, các bạn hãy tự xây dựng cho mình thêm nhé, chúng sẽ rất có ứng dụng về sau 1.3.3 Bất đẳng thức Schur Định lý 1.1 (Bất đẳng thức Schur) Cho các số không âm a; b; c: Khi đó, với mọi r > 0; ta có bất đẳng thức sau ar (a b)(a c) + br (b c)(b a) + cr (c a)(c b) 0 Đẳng thức xảy ra khi và chỉ khi a = b = c hoặc a = b; c = 0 hoặc... a)(c + a b): Đẳng thức xảy ra khi và chỉ khi a = b = c hoặc a = b; c = 0 hoặc các hoán vị tương ứng Hệ quả 1.2 (Bất đẳng thức Schur bậc 4) Cho các số không âm a; b; c: Khi đó, bất đẳng thức sau đúng a4 + b4 + c4 + abc(a + b + c) a3 (b + c) + b3 (c + a) + c3 (a + b): Đẳng thức xảy ra khi và chỉ khi a = b = c hoặc a = b; c = 0 hoặc các hoán vị tương ứng Dạng pqr tương ứng của 2 bất đẳng thức trên là... vế với vế các bất đẳng thức này, ta thu được bất đẳng thức ở trên Đẳng thức xảy ra khi và chỉ khi a = b = c hoặc a = b; c = 0 và các hoán vị http://boxtailieu.net 1.1 ĐẠI LƯỢNG (A B)(B C)(C A) 5 Ví dụ 1.3 Cho các số không âm a; b; c; không có 2 số nào cùng bằng 0: Chứng minh rằng p 3(a2 + b2 + c2 ) b3 c3 a3 + 2 + 2 : 2 2 2 2 a +b b +c c +a 2 (Võ Quốc Bá Cẩn) Lời giải Viết lại bất đẳng thức như sau s... 12 ab + bc + ca 5: (Vasile Cirtoaje) http://boxtailieu.net 26 CHƯƠNG 1 TÌM TÒI MỘT SỐ KỸ THUẬT GIẢI TOÁN Lời giải Bất đẳng thức tương đương với r+ 12 q 5 0 Sử dụng bất đẳng thức Schur bậc 3, ta có 4q r 9 3 Do đó r+ 12 q 5 4q 9 3 12 q + 3)2 4(q 5= 0: 3q Bất đẳng thức được chứng minh Đẳng thức xảy ra khi và chỉ khi a = b = c = 1: Ví dụ 1.19 Cho các số không âm a; b; c; không có 2 số nào đồng thời bằng... cyc 2 2 a b cyc X ab(a b)2 cyc X 8abc a: cyc Bất đẳng thức cuối hiển nhiên đúng theo bất đẳng thức AM-GM nên ta có đpcm Đẳng thức xảy ra khi và chỉ khi a = b = c = 31 : Ví dụ 1.16 Cho các số không âm a; b; c; không có 2 số nào đồng thời bằng 0: Chứng minh rằng a b c 3 p : +p +p 2 2 2 2 ab + 3c bc + 3a ca + 3b (Vasile Cirtoaje) Lời giải Sử dụng bất đẳng thức AM-GM, ta có X cyc p a ab + 3c2 = = Ta cần... + bc + ca; r = abc ) q = 1 p 2 2 Nếu p p 1 p 2 2 p p 7p2 + 6 2p p 5 + 27r = Bất đẳng thức tương đương 5 + 27r p(4q p2 ) 9 2 theo bất dẳng thức Schur, ta có r p 7p2 + 6 2p p2 1 2 : 0 = p(p2 2) ; 9 do đó p 1 p 2 p 7p2 + 6 2p 2 p 1 p 2 (5 p2 ) 0 2 5 + 3p(p2 2) p Nếu 2 p thì bất đẳng thức trên hiển nhiên đúng Vậy ta có đpcm Đẳng thức xảy ra khia = b = p12 ; c = 0 và các hoán vị tương ứng Ví dụ 1.20 Cho... a)(13c + 5a) c2 + a2 0: (Võ Quốc Bá Cẩn) 1 Đây chính là bất đẳng thức Schur bậc 3 http://boxtailieu.net 1.1 ĐẠI LƯỢNG (A B)(B C)(C A) 11 Lời giải Bất đẳng thức tương đương với X 4(a b)2 + 9(a2 a2 + b2 cyc ,4 ,4 X (a cyc a2 X (a b)2 a2 + b2 cyc 2 b) + b2 9 b2 ) X b2 cyc 2 0 a2 a2 + b2 2 9(a b )(b2 c2 )(c2 a2 ) 2 (a + b2 )(b2 + c2 )(c2 + a2 ) Theo bất đẳng thức AM-GM, 4 X (a cyc b)2 2 a + b2 s 12 3 (a b)2 ... 343 iii http://boxtailieu.net iv MỤC LỤC A.2 A.3 A.4 A.5 A.6 Bất Bất Bất Bất Bất đẳng đẳng đẳng đẳng đẳng thức thức thức thức thức AM-GM suy rộng trung bình lũy thừa trung bình lũy... Schwarz A.7 Bất đẳng thức Holder A.8 Bất đẳng thức Minkowski A.9 Bất đẳng thức Chebyshev A.10 Khai triển Abel A.11 Bất đẳng thức Maclaurin A.12 Bất đẳng thức Schur A.13... nhiều đau đầu trước bất đẳng thức khó có cảm giác tự hào phấn khích mà chứng minh bất đẳng thức Nhằm “kích hoạt” niềm say mê bất đẳng thức bạn, thực sách “Chuyên đề bất đẳng thức đại Sách gồm chương

Ngày đăng: 09/10/2015, 16:18

Xem thêm: Bất đẳng thức hiện đại

TỪ KHÓA LIÊN QUAN

w